You are on page 1of 377

1.

THE APPLICABLE LAWS


2.BASIC PRINCIPLES
1. Jao vs. BCC Products Sales Inc. G.R. No. 163700, April 18, 2012
PRINCIPLE: The existence of an employer-employee relationship is a question of fact.
Generally, a re-examination of factual findings cannot be done by the Court acting on a petition
for review on certiorari because the Court is not a trier of facts but reviews only questions of
law. Nor may the Court be bound to analyze and weigh again the evidence adduced and
considered in the proceedings below. This rule is not absolute, however, and admits of
exceptions. For one, the Court may look into factual issues in labor cases when the factual
findings of the Labor Arbiter, the NLRC, and the CA are conflicting.
FACTS:
Petitioner maintained that respondent BCC Product Sales Inc. (BCC) and its President,
respondent Terrance Ty (Ty), employed him as comptroller starting from September 1995 with a
monthly salary of P20,000.00 to handle the financial aspect of BCCs business; that on October
19,1995, the security guards of BCC, acting upon the instruction of Ty, barred him from entering
the premises of BCC where he then worked. He filed a complaint dated December 28, 1995 for
illegal dismissal, reinstatement with full backwages, non-payment of wages, damages and
attorneys fees.
Respondents countered that petitioner was not their employee but the employee of Sobien Food
Corporation (SFC), the major creditor and supplier of BCC
ISSUE:
The sole issue is whether or not an employer-employee relationship existed between petitioner
and BCC. A finding on the existence of an employer-employee relationship will automatically
warrant a finding of illegal dismissal, considering that respondents did not state any valid
grounds to dismiss petitioner.
RULING:
The existence of an employer-employee relationship is a question of fact. Generally, a reexamination of factual findings cannot be done by the Court acting on a petition for review
on certiorari because the Court is not a trier of facts but reviews only questions of law. Nor may
the Court be bound to analyze and weigh again the evidence adduced and considered in the
proceedings below. This rule is not absolute, however, and admits of exceptions. For one, the
Court may look into factual issues in labor cases when the factual findings of the Labor Arbiter,
the NLRC, and the CA are conflicting.

ATHENA M. SALAS | LABOR CASE DIGEST 2015 1

Here, the findings of the NLRC differed from those of the Labor Arbiter and the CA. This conflict
among such adjudicating offices compels the Courts exercise of its authority to review and pass
upon the evidence presented and to draw its own conclusions therefrom.
Our perusal of the affidavit of petitioner compels a conclusion similar to that reached by the CA
and the Labor Arbiter to the effect that the affidavit actually supported the contention that
petitioner had really worked in BCC as SFCs representative. It does seem more natural and
more believable that petitioners affidavit was referring to his employment by SFC even while he
was reporting to BCC as a comptroller in behalf of SFC. As respondents pointed out, it was
implausible for SFC to still post him to oversee and supervise the collections of accounts
receivables due from BCC beyond December 1995 if, as he insisted, BCC had already illegally
dismissed him and had even prevented him from entering the premises of BCC. Given the
patent animosity and strained relations between him and respondents in such circumstances,
indeed, how could he still efficiently perform in behalf of SFC the essential responsibility to
oversee and supervise collections at BCC? Surely, respondents would have vigorously
objected to any arrangement with SFC involving him.
Moreover, in determining the presence or absence of an employer-employee relationship, the
Court has consistently looked for the following incidents, to wit: (a) the selection and
engagement of the employee; (b) the payment of wages; (c) the power of dismissal; and (d) the
employers power to control the employee on the means and methods by which the work is
accomplished. The last element, the so-called control test, is the most important element.
Hereunder are some of the circumstances and incidents occurring while petitioner was
supposedly employed by BCC that debunked his claim against respondents. It can be deduced
from the March 1996 affidavit of petitioner that respondents challenged his authority to deliver
some 158 checks to SFC. Considering that he contested respondents challenge by pointing to
the existing arrangements between BCC and SFC, it should be clear that respondents did not
exercise the power of control over him, because he thereby acted for the benefit and in the
interest of SFC more than of BCC.
In addition, petitioner presented no document setting forth the terms of his employment by BCC.
The failure to present such agreement on terms of employment may be understandable and
expected if he was a common or ordinary laborer who would not jeopardize his employment by
demanding such document from the employer, but may not square well with his actual status as
a highly educated professional.

2. Legend Hotel (Manila) vs. Realuyo, G.R. No. 153511, July 18, 2012
PRINCIPLE: Generally, the Court does not review factual questions, primarily because the
Court is not a trier of facts. However, where, like here, there is a conflict between the factual
findings of the Labor Arbiter and the NLRC, on the one hand, and those of the CA, on the other

ATHENA M. SALAS | LABOR CASE DIGEST 2015 2

hand, it becomes proper for the Court, in the exercise of its equity jurisdiction, to review and reevaluate the factual issues and to look into the records of the case and re-examine the
questioned findings.
FACTS:
This labor case for illegal dismissal involves a pianist employed to perform in the restaurant of a
hotel. On August 9, 1999, respondent, whose stage name was Joey R. Roa, filed a complaint
for
alleged
unfair
labor
practice,
constructive
illegal
dismissal,
and
the
underpayment/nonpayment of his premium pay for holidays, separation pay, service incentive
leave pay, and 13th month pay. He prayed for attorney's fees, moral damages off P100,000.00
and exemplary damages for P100,000.00.
Respondent averred that he had worked as a pianist at the Legend Hotels Tanglaw Restaurant
from September 1992 with an initial rate of P400.00/night that was given to him after each
nights performance; that his rate had increased to P750.00/night; and that during his
employment, he could not choose the time of performance, which had been fixed from 7:00 pm
to 10:00 pm for three to six times/week. He added that the Legend Hotels restaurant manager
had required him to conform with the venues motif; that he had been subjected to the rules on
employees representation checks and chits, a privilege granted to other employees; that on
July 9, 1999, the management had notified him that as a cost-cutting measure his services as a
pianist would no longer be required effective July 30, 1999; that he disputed the excuse,
insisting that Legend Hotel had been lucratively operating as of the filing of his complaint; and
that the loss of his employment made him bring his complaint.
ISSUES:
(a) whether or not respondent was an employee of petitioner; and
(b) if respondent was petitioners employee, whether he was validly terminated.
RULING:
Substantive Issue No. 1: Employer-employee relationship existed between the parties
We next ascertain if the CA correctly found that an employer-employee relationship existed
between the parties.
The issue of whether or not an employer-employee relationship existed between petitioner and
respondent is essentially a question of fact. The factors that determine the issue include who
has the power to select the employee, who pays the employees wages, who has the power to
dismiss the employee, and who exercises control of the methods and results by which the work
of the employee is accomplished. Although no particular form of evidence is required to prove
the existence of the relationship, and any competent and relevant evidence to prove the
relationship may be admitted, a finding that the relationship exists must nonetheless rest on

ATHENA M. SALAS | LABOR CASE DIGEST 2015 3

substantial evidence, which is that amount of relevant evidence that a reasonable mind might
accept as adequate to justify a conclusion.
Generally, the Court does not review factual questions, primarily because the Court is not a trier
of facts. However, where, like here, there is a conflict between the factual findings of the Labor
Arbiter and the NLRC, on the one hand, and those of the CA, on the other hand, it becomes
proper for the Court, in the exercise of its equity jurisdiction, to review and re-evaluate the
factual issues and to look into the records of the case and re-examine the questioned findings.
A review of the circumstances reveals that respondent was, indeed, petitioners employee. He
was undeniably employed as a pianist in petitioners Madison Coffee Shop/Tanglaw Restaurant
from September 1992 until his services were terminated on July 9, 1999.
A review of the records shows, however, that respondent performed his work as a pianist under
petitioners supervision and control. Specifically, petitioners control of both the end achieved
and the manner and means used to achieve that end was demonstrated by the following, to wit:
a. He could not choose the time of his performance, which petitioners had fixed from 7:00
pm to 10:00 pm, three to six times a week;
b. He could not choose the place of his performance;
c. The restaurants manager required him at certain times to perform only Tagalog songs or
music, or to wear barong Tagalog to conform to the Filipiniana motif; and
d. He was subjected to the rules on employees representation check and chits, a privilege
granted to other employees.
Substantive Issue No. 2: Validity of the Termination
Having established that respondent was an employee whom petitioner terminated to prevent
losses, the conclusion that his termination was by reason of retrenchment due to an authorized
cause under the Labor Code is inevitable.
The Court has laid down the following standards that an employer should meet to justify
retrenchment and to foil abuse, namely:
a) The expected losses should be substantial and not merely de minimis in extent;
b) The substantial losses apprehended must be reasonably imminent;
c) The retrenchment must be reasonably necessary and likely to effectively prevent the
expected losses; and
d) The alleged losses, if already incurred, and the expected imminent losses sought to be
forestalled must be proved by sufficient and convincing evidence.
Was the retrenchment of respondent valid?
In termination cases, the burden of proving that the dismissal was for a valid or authorized
cause rests upon the employer. Here, petitioner did not submit evidence of the losses to its
business operations and the economic havoc it would thereby imminently sustain. It only

ATHENA M. SALAS | LABOR CASE DIGEST 2015 4

claimed that respondents termination was due to its "present business/financial condition." This
bare statement fell short of the norm to show a valid retrenchment. Hence, we hold that there
was no valid cause for the retrenchment of respondent.
Indeed, not every loss incurred or expected to be incurred by an employer can justify
retrenchment.1wphi1 The employer must prove, among others, that the losses are substantial
and that the retrenchment is reasonably necessary to avert such losses. Thus, by its failure to
present sufficient and convincing evidence to prove that retrenchment was necessary,
respondents termination due to retrenchment is not allowed.
3. The New Philippine Skylanders, Inc., vs. Dakila, G.R. No. 199547, Sept. 24, 2012
PRINCIPLE: The issue of illegal dismissal is premised on the existence of an employeremployee relationship between the parties herein. It is essentially a question of fact, beyond the
ambit of a petition for review on certiorari under Rule 45 of the Rules of Court unless there is a
clear showing of palpable error or arbitrary disregard of evidence which does not obtain in this
case.
FACTS:
Respondent Dakila was employed by petitioner corporation as early as 1987 and terminated for
cause in April 1997 when the corporation was sold. In May 1997, he was rehired as consultant
by the petitioners under a Contract for Consultancy Services6 dated April 30, 1997.
Thereafter, in a letter7 dated April 19, 2007, respondent Dakila informed petitioners of his
compulsory retirement effective May 2, 2007 and sought for the payment of his retirement
benefits pursuant to the Collective Bargaining Agreement. His request, however, was not acted
upon. Instead, he was terminated from service effective May 1, 2007.
Consequently, respondent Dakila filed a complaint for constructive illegal dismissal, nonpayment of retirement benefits, under/non-payment of wages and other benefits of a regular
employee, and damages against petitioners.
On the other hand, petitioners, in their position paper,8 asserted that respondent Dakilawas a
consultant and not their regular employee. The latter was not included in petitioners' payroll and
paid a fixed amount under the consultancy contract. He was not required to observe regular
working hours and was free to adopt means and methods to accomplish his task except as to
the results of the work required of him. Hence, no employer-employee relationship existed
between them. Moreover, respondent Dakila terminated his contract in a letter dated April 19,
2007, thus, negating his dismissal.
ISSUE:
Whether petitioner was illegally dismissed.

ATHENA M. SALAS | LABOR CASE DIGEST 2015 5

RULING:
The issue of illegal dismissal is premised on the existence of an employer-employee
relationship between the parties herein. It is essentially a question of fact, beyond the ambit of a
petition for review on certiorari under Rule 45 of the Rules of Court unless there is a clear
showing of palpable error or arbitrary disregard of evidence which does not obtain in this case.
Records reveal that both the LA and the NLRC, as affirmed by the CA, have found substantial
evidence to show that respondent Dakila was a regular employee who was dismissed without
cause.
Following Article 279 of the Labor Code, an employee who is unjustly dismissed from work is
entitled to reinstatement without loss of seniority rights and other privileges and to his full
backwages computed from the time he was illegally dismissed. However, considering that
respondent Dakila was terminated on May 1, 2007, or one (1) day prior to his compulsory
retirement on May 2, 2007, his reinstatement is no longer feasible. Accordingly, the NLRC
correctly held him entitled to the payment of his retirement benefits pursuant to the CBA. On the
other hand, his backwages should be computed only for days prior to his compulsory retirement
which in this case is only a day. Consequently, the award of reinstatement wages pending
appeal must be deleted for lack of basis.
4. Tesoro et al., vs. Metro Manila Retreaders Inc., et al., GR No. 171482, March 12, 2014
This case concerns the effect on the status of employment of employees who entered into a
Service Franchise Agreement with their employer.
PRINCIPLE: Franchising involves the use of an established business expertise, trademark,
knowledge, and training. As such, the franchisee is required to follow a certain established
system. Accordingly, the franchisors may impose guidelines that somehow restrict the
petitioners conduct which do not necessarily indicate control. The important factor to consider
is still the element of control over how the work itself is done, not just its end result.
FACTS:
On various dates between 1991 and 1998, petitioners Ashmor M. Tesoro, Pedro Ang, and
Gregorio Sharp used to work as salesmen for respondents Metro Manila Retreaders, Inc.,
Northern Luzon Retreaders, Inc., or Power Tire and Rubber Corporation, apparently sister
companies, collectively called Bandag. Bandag offered repair and retread services for used
tires. In 1998, however, Bandag developed a franchising scheme that would enable others to
operate tire and retreading businesses using its trade name and service system.
Petitioners quit their jobs as salesmen and entered into separate Service Franchise Agreements
(SFAs) with Bandag for the operation of their respective franchises. Under the SFAs, Bandag
would provide funding support to the petitioners subject to a regular or periodic liquidation of
their revolving funds. The expenses out of these funds would be deducted from petitioners
sales to determine their incomes.

ATHENA M. SALAS | LABOR CASE DIGEST 2015 6

ISSUE:
The only issue presented in this case is whether or not petitioners remained to be Bandags
salesmen under the franchise scheme it entered into with them.
RULING:
Franchising is a business method of expansion that allows an individual or group of individuals
to market a product or a service and to use of the patent, trademark, trade name and the
systems prescribed by the owner.
The question is: did petitioners remain to be Bandags employees after they began operating
those branches? The tests for determining employeremployee relationship are: (a) the
selection and engagement of the employee; (b) the payment of wages; (c) the power of
dismissal; and (d) the employers power to control the employee with respect to the means and
methods by which the work is to be accomplished. The last is called the control test, the most
important element.
When petitioners agreed to operate Bandags franchise branches in different parts of the
country, they knew that this substantially changed their former relationships. They were to cease
working as Bandags salesmen, the positions they occupied before they ventured into running
separate Bandag branches. They were to cease receiving salaries or commissions. Their
incomes were to depend on the profits they made. Yet, petitioners did not then complain of
constructive dismissal. They took their chances, ran their branches, Gregorio Sharp in La Union
for several months and Ashmor Tesoro in Baguio and Pedro Ang in Pangasinan for over a year.
Clearly, their belated claim of constructive dismissal is quite hollow.
It is pointed out that Bandag continued, like an employer, to exercise control over petitioners
work. It points out that Bandag: (a) retained the right to adjust the price rates of products and
services; (b) imposed minimum processed tire requirement (MPR); (c) reviewed and regulated
credit applications; and (d) retained the power to suspend petitioners services for failure to meet
service standards.
But uniformity in prices, quality of services, and good business practices are the essence of all
franchises. A franchisee will damage the franchisors business if he sells at different prices,
renders different or inferior services, or engages in bad business practices. These business
constraints are needed to maintain collective responsibility for faultless and reliable service to
the same class of customers for the same prices.
This is not the control contemplated in employeremployee relationships. Control in such
relationships addresses the details of day to day work like assigning the particular task that has
to be done, monitoring the way tasks are done and their results, and determining the time during
which the employee must report for work or accomplish his assigned task.

ATHENA M. SALAS | LABOR CASE DIGEST 2015 7

Franchising involves the use of an established business expertise, trademark, knowledge, and
training. As such, the franchisee is required to follow a certain established system. Accordingly,
the franchisors may impose guidelines that somehow restrict the petitioners conduct which do
not necessarily indicate control. The important factor to consider is still the element of control
over how the work itself is done, not just its end result.
The Labor Arbiter, the NLRC, and the CA, are unanimous that petitioners were no longer route
salesmen, bringing previously ordered supplies and goods to dealers, taking back returned
items, collecting payments, remitting them, etc. They were themselves then the dealers, getting
their own supply and bringing these to their own customers and subdealers, if any.
5. Royale Homes Marketing Corp., vs. Alcantara, GR No. 195190, July 28, 2014
PRINCIPLE: Not every form of control that a hiring party imposes on the hired party is indicative
of employee-employer relationship. Rules and regulations that merely serve as guidelines
towards the achievement of a mutually desired result without dictating the means and methods
of accomplishing it do not establish employer-employee relationship.
FACTS:
Royale Homes, a corporation engaged in marketing real estates, appointed Alcantara as its
Marketing Director for a fixed period of one year. His work consisted mainly of marketing
Royale Homes real estate inventories on an exclusive basis. Royale Homes reappointed him
for several consecutive years, the last of which covered the period January 1 to December 31,
2003 where he held the position of Division 5 Vice-President-Sales.
Alcantara filed a Complaint for Illegal Dismissal9 against Royale Homes and its President
Matilde Robles, Executive Vice-President for Administration and Finance Ma. Melinda
Bernardino, and Executive Vice- President for Sales Carmina Sotto.
Royale Homes, on the other hand, vehemently denied that Alcantara is its employee. It argued
that the appointment paper of Alcantara is clear that it engaged his services as an independent
sales contractor for a fixed term of one year only. He never received any salary, 13th month
pay, overtime pay or holiday pay from Royale Homes as he was paid purely on commission
basis. In addition, Royale Homes had no control on how Alcantara would accomplish his tasks
and responsibilities as he was free to solicit sales at any time and by any manner which he may
deem appropriate and necessary. He is even free to recruit his own sales personnel to assist
him in pursuance of his sales target.
ISSUE:
The pivotal issue to be resolved in this case is whether Alcantara was an independent
contractor or an employee of Royale Homes.
RULING:

ATHENA M. SALAS | LABOR CASE DIGEST 2015 8

The juridical relationship of the parties based on their written contract


The primary evidence of the nature of the parties relationship in this case is the written contract
that they signed and executed in pursuance of their mutual agreement. While the existence of
employer-employee relationship is a matter of law, the characterization made by the parties in
their contract as to the nature of their juridical relationship cannot be simply ignored, particularly
in this case where the parties written contract unequivocally states their intention at the time
they entered into it.
In this case, the contract,27 duly signed and not disputed by the parties, conspicuously provides
that no employer-employee relationship exists between Royale Homes and Alcantara, as well
as his sales agents. It is clear that they did not want to be bound by employer-employee
relationship at the time of the signing of the contract. Since the terms of the contract are clear
and leave no doubt upon the intention of the contracting parties, the literal meaning of its
stipulations should control.
The juridical relationship of the parties based on Control Test
In determining the existence of an employer-employee relationship, this Court has generally
relied on the four-fold test, to wit: (1) the selection and engagement of the employee; (2) the
payment of wages; (3) the power of dismissal; and (4) the employers power to control the
employee with respect to the means and methods by which the work is to be accomplished.29
Among the four, the most determinative factor in ascertaining the existence of employeremployee relationship is the right of control test.
Not every form of control is indicative of employer-employee relationship. A person who
performs work for another and is subjected to its rules, regulations, and code of ethics does not
necessarily become an employee.34 As long as the level of control does not interfere with the
means and methods of accomplishing the assigned tasks, the rules imposed by the hiring party
on the hired party do not amount to the labor law concept of control that is indicative of
employer-employee relationship.
Logically, the line should be drawn between rules that merely serve as guidelines towards the
achievement of the mutually desired result without dictating the means or methods to be
employed in attaining it, and those that control or fix the methodology and bind or restrict the
party hired to the use of such means. The first, which aim only to promote the result, create no
employer-employee relationship unlike the second, which address both the result and the
means used to achieve it.
In this case, the Court agrees with Royale Homes that the rules, regulations, code of ethics, and
periodic evaluation alluded to by Alcantara do not involve control over the means and methods
by which he was to perform his job. Understandably, Royale Homes has to fix the price, impose
requirements on prospective buyers, and lay down the terms and conditions of the sale,

ATHENA M. SALAS | LABOR CASE DIGEST 2015 9

including the mode of payment, which the independent contractors must follow. It is also
necessary for Royale Homes to allocate its inventories among its independent contractors,
determine who has priority in selling the same, grant commission or allowance based on
predetermined criteria, and regularly monitor the result of their marketing and sales efforts. But
to the mind of this Court, these do not pertain to the means and methods of how Alcantara was
to perform and accomplish his task of soliciting sales. They do not dictate upon him the details
of how he would solicit sales or the manner as to how he would transact business with
prospective clients. In Tongko, this Court held that guidelines or rules and regulations that do
not pertain to the means or methods to be employed in attaining the result are not indicative of
control as understood in labor law.
Guidelines indicative of labor law control, as the first Insular Life case tells us, should not
merely relate to the mutually desirable result intended by the contractual relationship; they must
have the nature of dictating the means or methods to be employed in attaining the result, or of
fixing the methodology and of binding or restricting the party hired to the use of these means. In
fact, results-wise, the principal can impose production quotas and can determine how many
agents, with specific territories, ought to be employed to achieve the companys objectives.
These are management policy decisions that the labor law element of control cannot reach.

6. Sameer Overseas Placement Agency vs. Cabiles, GR No. 170139, August 3, 2014, En
Banc
Facts:
Petitioner, Sameer Overseas Placement Agency, Inc., is a recruitment and placement agency.
Respondent submitted her application for a quality control job in Taiwan. Joys application was
accepted.7 Joy was later asked to sign a one year employment contract for a monthly salary.
She alleged that Sameer Overseas Agency required her to pay a placement fee of 70,000.00
when she signed the employment contract. Joy was deployed to work for Taiwan Wacoal, Co.
Ltd. (Wacoal) on June 26, 1997.10 She alleged that in her employment contract, she agreed to
work as quality control for one year.11 In Taiwan, she was asked to work as a cutter.
Sameer Overseas Placement Agency claims that on July 14, 1997, a certain Mr. Huwang from
Wacoal informed Joy, without prior notice, that she was terminated and that she should
immediately report to their office to get her salary and passport. Thereafter, Joy filed a complaint
with the National Labor Relations Commission against petitioner and Wacoal. She claimed that
she was illegally dismissed.

ATHENA M. SALAS | LABOR CASE DIGEST 2015 10

Petitioner reiterates that there was just cause for termination because there was a finding of
Wacoal that respondent was inefficient in her work. Therefore, it claims that respondents
dismissal was valid.
Issue:
Whether respondent was illegally dismissed by her employer.
Ruling:
Protection to labor both local and overseas
Sameer Overseas Placement Agency failed to show that there was just cause for causing Joys
dismissal. The employer, Wacoal, also failed to accord her due process of law. Indeed,
employers have the prerogative to impose productivity and quality standards at work. They may
also impose reasonable rules to ensure that the employees comply with these standards.
Failure to comply may be a just cause for their dismissal. This prerogative, however, should not
be abused. It is tempered with the employees right to security of tenure. Employees are not
stripped of their security of tenure when they move to work in a different jurisdiction. With
respect to the rights of overseas Filipino workers, we follow the principle of lex loci contractus.
Thus, in Triple Eight Integrated Services, Inc. v. NLRC,65 this court noted:
Here in the Philippines, employment agreements are more than contractual in nature. The
Constitution itself, in Article XIII, Section 3, guarantees the special protection of workers, to wit:
The State shall afford full protection to labor, local and overseas, organized and unorganized,
and promote full employment and equality of employment opportunities for all. It shall guarantee
the rights of all workers to self-organization, collective bargaining and negotiations, and peaceful
concerted activities, including the right to strike in accordance with law. They shall be entitled to
security of tenure, humane conditions of work, and a living wage. They shall also participate in
policy and decision-making processes affecting their rights and benefits as may be provided by
law.
However, petitioners contend that the twin requirements of notice and hearing applies strictly
only when the employment is within the Philippines and that these need not be strictly observed
in cases of international maritime or overseas employment. The Court does not agree. The
provisions of the Constitution as well as the Labor Code which afford protection to labor apply to
Filipino employees whether working within the Philippines or abroad. Moreover, the principle of
lex loci contractus (the law of the place where the contract is made) governs in this jurisdiction.
By our laws, overseas Filipino workers (OFWs) may only be terminated for a just or authorized
cause and after compliance with procedural due process requirements.

ATHENA M. SALAS | LABOR CASE DIGEST 2015 11

Illegal dismissal (inefficiency in work)


Petitioners allegation that respondent was inefficient in her work and negligent in her duties69
may, therefore, constitute a just cause for termination under Article 282(b)[gross and habitual
negligence], but only if petitioner was able to prove it.
The burden of proving that there is just cause for termination is on the employer. The employer
must affirmatively show rationally adequate evidence that the dismissal was for a justifiable
cause.
To show that dismissal resulting from inefficiency in work is valid, it must be shown that: 1) the
employer has set standards of conduct and workmanship against which the employee will be
judged; 2) the standards of conduct and workmanship must have been communicated to the
employee; and 3) the communication was made at a reasonable time prior to the employees
performance assessment.
The employee cannot be expected to meet any standard of character or workmanship if such
standards were not communicated to him or her. Courts should remain vigilant on allegations of
the employers failure to communicate work standards that would govern ones employment if
[these are] to discharge in good faith [their] duty to adjudicate. In this case, petitioner merely
alleged that respondent failed to comply with her foreign employers work requirements and was
inefficient in her work.74 No evidence was shown to support such allegations.
Petitioner did not even bother to specify what requirements were not met, what efficiency
standards were violated, or what particular acts of respondent constituted. There was also no
showing that respondent was sufficiently informed of the standards against which her work
efficiency and performance were judged. Thus, respondent was illegally dismissed.

ATHENA M. SALAS | LABOR CASE DIGEST 2015 12

3.RIGHT TO HIRE
4.WAGES & WAGE RATIONALIZATION ACT
7. Royal Plant Workers Union vs. Coca-Cola Bottlers Phils Inc. -Cebu Plant, G.R. No.
198783, April 15, 2013
Facts:
Petitioner Coca-Cola Bottlers Philippines, Inc. (CCBPI) is a domestic corporation engaged in
the manufacture, sale and distribution of softdrink products. Under the employ of each bottling
plant are bottling operators. In the case of the plant in Cebu City, there are 20 bottling operators
who work for its Bottling Line 1 while there are 12-14 bottling operators who man its Bottling
Line 2. All of them are male and they are members of herein respondent Royal Plant Workers
Union (ROPWU). The bottling operators work in two shifts. The first shift is from 8 a.m. to 5 p.m.
and the second shift is from 5 p.m. up to the time production operations is finished. Thus, the
second shift varies and may end beyond eight (8) hours. However, the bottling operators are
compensated with overtime pay if the shift extends beyond eight (8) hours.
Each shift has rotations of work time and break time. Prior to September 2008, the rotation is
this: after two and a half (2 ) hours of work, the bottling operators are given a 30-minute break
and this goes on until the shift ends. In September 2008 and up to the present, the rotation has
changed and bottling operators are now given a 30-minute break after one and one half (1 )
hours of work.
In 1974, the bottling operators of then Bottling Line 2 were provided with chairs upon their
request. Their request was likewise granted. Sometime in September 2008, the chairs provided
for the operators were removed pursuant to a national directive of petitioner. This directive is in
line with the "I Operate, I Maintain, I Clean" program of petitioner for bottling operators.The
program reinforces the task of bottling operators to constantly move about in the performance of
their duties and responsibilities.
Furthermore, CCBPI rationalized that the removal of the chairs is implemented so that the
bottling operators will avoid sleeping, thus, prevent injuries to their persons. In addition, sleeping
will hamper the efficient flow of operations as the bottling operators would be unable to perform
their duties competently. . Petitioner argued that the removal of the chairs is valid as it is a
legitimate exercise of management prerogative.
The Union further claims that management prerogatives are not absolute but subject to certain
limitations found in law, a collective bargaining agreement, or general principles of fair play and
justice. The operators have been performing their assigned duties and responsibilities
satisfactorily for thirty (30) years using chairs. Thus, a violation of the principle of non-diminution
of benefits provided in Article 100 of the Labor Code.
Issue:

ATHENA M. SALAS | LABOR CASE DIGEST 2015 13

Whether theres a valid exercise of management prerogative. Whether the principle of nondiminution of benefits is applicable in this case.
Ruling:
Management Prerogative
The Court has held that management is free to regulate, according to its own discretion and
judgment, all aspects of employment, including hiring, work assignments, working methods,
time, place, and manner of work, processes to be followed, supervision of workers, working
regulations, transfer of employees, work supervision, lay-off of workers, and discipline, dismissal
and recall of workers. The exercise of management prerogative, however, is not absolute as it
must be exercised in good faith and with due regard to the rights of labor.
In the present controversy, it cannot be denied that CCBPI removed the operators chairs
pursuant to a national directive and in line with its "I Operate, I Maintain, I Clean" program,
launched to enable the Union to perform their duties and responsibilities more efficiently. The
chairs were not removed indiscriminately. They were carefully studied with due regard to the
welfare of the members of the Union. The removal of the chairs was compensated by: a) a
reduction of the operating hours of the bottling operators from a two-and-one-half (2 )-hour
rotation period to a one-and-a-half (1 ) hour rotation period; and b) an increase of the break
period from 15 to 30 minutes between rotations.
Apparently, the decision to remove the chairs was done with good intentions as CCBPI wanted
to avoid instances of operators sleeping on the job while in the performance of their duties and
responsibilities and because of the fact that the chairs were not necessary considering that the
operators constantly move about while working. In short, the removal of the chairs was
designed to increase work efficiency. Hence, CCBPIs exercise of its management prerogative
was made in good faith without doing any harm to the workers rights.
Principle of non-diminution of benefits
The operators chairs cannot be considered as one of the employee benefits covered in Article
10016 of the Labor Code. In the Courts view, the term "benefits" mentioned in the nondiminution rule refers to monetary benefits or privileges given to the employee with monetary
equivalents.
Such benefits or privileges form part of the employees wage, salary or compensation making
them enforceable obligations.Examples(13th month pay, legal/special holiday pay, night
premium pay and vacation and sick leaves; service awards with cash incentives, premium pay,
Christmas party with incidental benefits and promotional increase)

8. Vergara, Jr. vs. Coca-Cola Bottlers Phils Inc. G.R. No. 176985, April 1, 2013

ATHENA M. SALAS | LABOR CASE DIGEST 2015 14

Facts:
Vergara, Jr. was an employee of respondent Coca-Cola Bottlers Philippines, Inc. from May 1968
until he retired on January 31, 2002 as a District Sales Supervisor (DSS) for Las Pias City,
Metro Manila. As stipulated in respondents existing Retirement Plan Rules and Regulations at
the time, the Annual Performance Incentive Pay of RSMs, DSSs, and SSSs shall be considered
in the computation of retirement benefits.
Petitioner filed a complaint before the NLRC on June 11, 2002 for the payment of his "Full
Retirement Benefits, claiming that his entitlement to Service Management Incentive (SMI) was
not considered in the computation of his retirement package. Petitioner alleged that he is
entitled to the SMI because the provision of such benefit has ripened into a company practice.
Thus, the unilateral elimination of the SMI violates the principle of non-diminution of benefits.
Issue:
Whether the application of the principle of non-diminution of benefits is warranted by the facts
of the case.
Ruling:
No. Generally, employees have a vested right over existing benefits voluntarily granted to them
by their employer.Thus, any benefit and supplement being enjoyed by the employees cannot be
reduced, diminished, discontinued or eliminated by the employer. The principle of nondiminution of benefits is actually founded on the Constitutional mandate to protect the rights of
workers, to promote their welfare, and to afford them full protection. In turn, said mandate is the
basis of Article 4 of the Labor Code which states that "all doubts in the implementation and
interpretation of this Code, including its implementing rules and regulations, shall be rendered in
favor of labor."
There is diminution of benefits when the following requisites are present: (1) the grant or benefit
is founded on a policy or has ripened into a practice over a long period of time; (2) the practice
is consistent and deliberate; (3) the practice is not due to error in the construction or application
of a doubtful or difficult question of law; and (4) the diminution or discontinuance is done
unilaterally by the employer.
To be considered as a regular company practice, the employee must prove by substantial
evidence that the giving of the benefit is done over a long period of time, and that it has been
made consistently and deliberately. The common denominator in previously decided cases
appears to be the regularity and deliberateness of the grant of benefits over a significant period
of time. It requires an indubitable showing that the employer agreed to continue giving the
benefit knowing fully well that the employees are not covered by any provision of the law or
agreement requiring payment thereof. In sum, the benefit must be characterized by regularity,

ATHENA M. SALAS | LABOR CASE DIGEST 2015 15

voluntary and deliberate intent of the employer to grant the benefit over a considerable period of
time.
Upon review of the entire case records, We find no substantial evidence to prove that the grant
of SMI to all retired DSSs regardless of whether or not they qualify to the same had ripened into
company practice. Despite more than sufficient opportunity given him while his case was
pending before the NLRC, the CA, and even to this Court, petitioner utterly failed to adduce
proof to establish his allegation that SMI has been consistently, deliberately and voluntarily
granted to all retired DSSs without any qualification or conditions whatsoever. The only two
pieces of evidence that he stubbornly presented throughout the entirety of this case are the
sworn statements of Renato C. Hidalgo (Hidalgo) and Ramon V. Velazquez (Velasquez), former
DSSs of respondent who retired in 2000 and 1998, respectively. They claimed that the SMI was
included in their retirement package even if they did not meet the sales and collection qualifiers.
The declarations of Hidalgo and Velazquez were sufficiently countered by respondent through
the affidavits executed by Norman R. Biola (Biola), Moises D. Escasura (Escasura), and Ma.
Vanessa R. Balles (Balles).They attested that contrary to petitioners claim, Hidalgo was in fact
qualified for the SMI. As for Velazquez, Escasura asserted that even if he (Velazquez) did not
qualify for the SMI, respondents General Manager in its Calamba plant still granted his
(Velazquez) request, along with other numerous concessions, to achieve industrial peace in the
plant which was then experiencing labor relations problems. Lastly, Balles confirmed that
petitioner failed to meet the trade receivable qualifiers of the SMI. She also cited the cases of
Ed Valencia (Valencia) and Emmanuel Gutierrez (Gutierrez), both DSSs of respondent who
retired on January 31, 2002 and December 30, 2002, respectively. She noted that, unlike
Valencia, Gutierrez also did not receive the SMI as part of his retirement pay, since he failed to
qualify under the policy guidelines.
Therefore, respondent's isolated act of including the SMI in the retirement package of
Velazquez could hardly be classified as a company practice that may be considered an
enforceable obligation.It presupposes that a company practice, policy and tradition favorable to
the employees has been clearly established; and that the payments made by the company
pursuant to it have ripened into benefits enjoyed by them.

9. Royal Plant Workers Union vs. Coca-Cola Bottlers Phils Inc. -Cebu Plant, G.R. No.
198783, April 15, 2013
Facts:
Petitioner Coca-Cola Bottlers Philippines, Inc. (CCBPI) is a domestic corporation engaged in
the manufacture, sale and distribution of softdrink products. Under the employ of each bottling
plant are bottling operators. In the case of the plant in Cebu City, there are 20 bottling operators
who work for its Bottling Line 1 while there are 12-14 bottling operators who man its Bottling
Line 2. All of them are male and they are members of herein respondent Royal Plant Workers
Union (ROPWU). The bottling operators work in two shifts. The first shift is from 8 a.m. to 5 p.m.

ATHENA M. SALAS | LABOR CASE DIGEST 2015 16

and the second shift is from 5 p.m. up to the time production operations is finished. Thus, the
second shift varies and may end beyond eight (8) hours. However, the bottling operators are
compensated with overtime pay if the shift extends beyond eight (8) hours.
Each shift has rotations of work time and break time. Prior to September 2008, the rotation is
this: after two and a half (2 ) hours of work, the bottling operators are given a 30-minute break
and this goes on until the shift ends. In September 2008 and up to the present, the rotation has
changed and bottling operators are now given a 30-minute break after one and one half (1 )
hours of work.
In 1974, the bottling operators of then Bottling Line 2 were provided with chairs upon their
request. Their request was likewise granted. Sometime in September 2008, the chairs provided
for the operators were removed pursuant to a national directive of petitioner. This directive is in
line with the "I Operate, I Maintain, I Clean" program of petitioner for bottling operators.The
program reinforces the task of bottling operators to constantly move about in the performance of
their duties and responsibilities.
Furthermore, CCBPI rationalized that the removal of the chairs is implemented so that the
bottling operators will avoid sleeping, thus, prevent injuries to their persons. In addition, sleeping
will hamper the efficient flow of operations as the bottling operators would be unable to perform
their duties competently. . Petitioner argued that the removal of the chairs is valid as it is a
legitimate exercise of management prerogative.
The Union further claims that management prerogatives are not absolute but subject to certain
limitations found in law, a collective bargaining agreement, or general principles of fair play and
justice. The operators have been performing their assigned duties and responsibilities
satisfactorily for thirty (30) years using chairs. Thus, a violation of the principle of non-diminution
of benefits provided in Article 100 of the Labor Code.
Issues:
Whether theres a valid exercise of management prerogative.
Whether the principle of non-diminution of benefits is applicable in this case.
Ruling:
Management Prerogative
The Court has held that management is free to regulate, according to its own discretion and
judgment, all aspects of employment, including hiring, work assignments, working methods,
time, place, and manner of work, processes to be followed, supervision of workers, working
regulations, transfer of employees, work supervision, lay-off of workers, and discipline, dismissal
and recall of workers. The exercise of management prerogative, however, is not absolute as it
must be exercised in good faith and with due regard to the rights of labor.

ATHENA M. SALAS | LABOR CASE DIGEST 2015 17

In the present controversy, it cannot be denied that CCBPI removed the operators chairs
pursuant to a national directive and in line with its "I Operate, I Maintain, I Clean" program,
launched to enable the Union to perform their duties and responsibilities more efficiently. The
chairs were not removed indiscriminately. They were carefully studied with due regard to the
welfare of the members of the Union. The removal of the chairs was compensated by: a) a
reduction of the operating hours of the bottling operators from a two-and-one-half (2 )-hour
rotation period to a one-and-a-half (1 ) hour rotation period; and b) an increase of the break
period from 15 to 30 minutes between rotations.
Apparently, the decision to remove the chairs was done with good intentions as CCBPI wanted
to avoid instances of operators sleeping on the job while in the performance of their duties and
responsibilities and because of the fact that the chairs were not necessary considering that the
operators constantly move about while working. In short, the removal of the chairs was
designed to increase work efficiency. Hence, CCBPIs exercise of its management prerogative
was made in good faith without doing any harm to the workers rights.
Principle of non-diminution of benefits
The operators chairs cannot be considered as one of the employee benefits covered in Article
10016 of the Labor Code. In the Courts view, the term "benefits" mentioned in the nondiminution rule refers to monetary benefits or privileges given to the employee with monetary
equivalents.
Such benefits or privileges form part of the employees wage, salary or compensation making
them enforceable obligations.Examples(13th month pay, legal/special holiday pay, night
premium pay and vacation and sick leaves; service awards with cash incentives, premium pay,
Christmas party with incidental benefits and promotional increase)

10. The National Wages & Productivity Commission et al., vs. The Alliance of Progressive
Labor et al., GR No. 150326, March 12, 2014
Facts: The RTWPB-NCR issued Wage Order No. NCR-07 on October 14, 1999 imposing an
increase of P25.50/day on the wages of all private sector workers and employees in the NCR
and pegging the minimum wage rate in the NCR at P223.50/day.6 However, Section 2 and
Section 9 of Wage Order No. NCR-07 exempted certain sectors and industries from its
coverage, to wit:
Section 9. Upon application with and as determined by the Board, based on documentation and
other requirements in accordance with applicable rules and regulations issued by the
Commission, the following may be exempt from the applicability of this Order:
1. Distressed establishments as defined in the NPWC Guidelines No. 01, series of 1996;
2. Exporters including indirect exporters with at least 50% export sales and with forward
contracts with their foreign buyers/principals entered into on or twelve (12) months

ATHENA M. SALAS | LABOR CASE DIGEST 2015 18

before the
date of publication of this Order may be exempt during the lifetime of said
contract but not to
exceed twelve (12) months from the effectivity of this Order.
Feeling aggrieved by their non-coverage by the wage adjustment, the Alliance of Progressive
Labor (APL) and the Tunay na Nagkakaisang Manggagawa sa Royal (TNMR) filed an appeal
with the NWPC assailing Section 2(A) and Section 9(2) of Wage Order No. NCR-07. They
contended
that neither the NWPC nor the RTWPB-NCR had the authority to expand the non-coverage and
exemptible categories under the wage order. Exporters are not among those listed in the law.
Hence, the assailed sections of the wage order should be voided. NWPC upheld the validity of
the Wage Order. However, the C.A revered the decision of the NWPC.
Issue: Whether or not the RTWPB-NCR had the authority to provide additional exemptions from
the minimum wage adjustments embodied in Wage Order No. NCR-07.
Ruling: Yes. Indisputably, the NWPC had the authority to prescribe the rules and guidelines for
the determination of the minimum wage and productivity measures, and the RTWPB-NCR had
the power to issue wage orders.
Pursuant to its statutorily defined functions, the NWPC promulgated NWPC Guidelines No. 00195 (Revised Rules of Procedure on Minimum Wage Fixing) to govern the proceedings in the
NWPC and the RTWPBs in the fixing of minimum wage rates by region, province and industry.
Section 1 of Rule VIII of NWPC Guidelines No. 001-95 recognized the power of the RTWPBs to
issue exemptions from the application of the wage orders subject to the guidelines issued by the
NWPC, viz: (refer to regulation)
The NWPC also issued NWPC Guidelines No. 01, Series of 1996, to fix the rules on the
exemption from compliance with the wage increases prescribed by the RTWPBs. Section 2 of
the Guidelines No. 01 reads:
SECTION 2. CATEGORIES OF EXEMPTIBLE ESTABLISHMENTS:
1.
2.
3.
4.
5.

Distressed establishments
New business enterprises (NBEs)
Retail/Service establishments employing not more than ten (10)
workers
Establishments adversely affected by natural calamities

Exemptible categories outside of the abovementioned list may be allowed only if they are in
accord with the rationale for exemption\reflected in the first paragraph of this section. The
concerned Regional Board shall submit strong and justifiable reason/s for the inclusion of such
categories which shall be subject to review/approval by the Commission.
Under the guidelines, the RTWPBs could issue exemptions from the application of the wage
orders as long as the exemptions complied with the rules of the NWPC. In its rules, the NWPC
enumerated four exemptible establishments, but the list was not exclusive. The RTWPBs had
the authority to include in the wage orders establishments that belonged to, or to exclude from
the four enumerated exemptible categories. If the exemption was outside of the four exemptible

ATHENA M. SALAS | LABOR CASE DIGEST 2015 19

categories, like here, the exemptible category should be: (1) in accord with the rationale for
exemption; (2) reviewed/approved by the NWPC; and (3) upon review, the RTWPB issuing the
wage order must submit a strong and justifiable reason or reasons for the inclusion of such
category. It is the compliance with the second requisite that is at issue here. The very fact that
the validity of the assailed sections of Wage Order No. NCR-07 had been already passed upon
and upheld by the NWPC meant that the NWPC had already given the wage order its necessary
legal imprimatur. Accordingly, the requisite approval or review was complied with.
In creating the RTWPBs, Congress intended to rationalize wages, firstly, by establishing full time
boards to police wages round-the-clock, and secondly, by giving the boards enough powers to
achieve this objective. In Employers Confederation of the Phils. v. National Wages and
Productivity Commission, 16 this Court all too clearly pronounced that Congress meant the
RTWPBs to be creative in resolving the annual question of wages without Labor and
Management knocking on the doors of Congress at every turn. The R TWPBs are the thinking
group of men and women guided by statutory standards and bound by the rules and guidelines
prescribed by the NWPC.
In the nature of their functions, the R TWPBs investigate and study all the pertinent facts to
ascertain the conditions in their respective regions. Hence, they are logically vested with the
competence to determine the applicable minimum wages to be imposed as well as the
industries and sectors to exempt from the coverage of their wage orders.

11. Our Haus Realty Development Corp., vs. Parian et al., GR No. 204651, August 6, 2014
Principles:
i.

ii.

If the provision of articles/services to workers is mandated by law and required by law to


be integrated into the overall business cost, the employer cannot pass the burden of the
costs to its employees by deducting it as facilities, because this is the employers
obligation under the law.
The lodging and meals furnished by the employer to its workers in the construction
business are supplements, not facilities, because they redound to the benefit and
convenience of the employer. The success of its projects is largely a function of the
physical strength, vitality and efficiency of its laborers. Its business will be jeopardized if
its workers are weak, sickly, and lack the required energy to perform strenuous physical
activities. Moreover, it will be more convenient to the employer if its workers are housed
near the construction site to ensure their ready availability during urgent or emergency
circumstances.

Facts:
Respondents Alexander Parian, Jay Erinco, Alexander Canlas, Jerry Sabulao and Bernardo
Tenedero were all laborers working for petitioner Our Haus Realty Development Corporation
(Our Haus), a company engaged in the construction business.

ATHENA M. SALAS | LABOR CASE DIGEST 2015 20

They filed with the LA a complaint for underpayment of their daily wages. They claimed that
except for respondent Bernardo N. Tenedero, their wages were below the minimum rates
prescribed in the following wage orders from 2007 to 2010:
Wage Order No. NCR-13, which provides for a daily minimum wage rate ofP362.00 for the nonagriculture sector (effective from August 28, 2007 until June 13, 2008);
Wage Order No. NCR-14, which provides for a daily minimum wage rate ofP382.00 for the nonagriculture sector (effective from June 14, 2008 until June 30, 2010).
The respondents also alleged that Our Haus failed to pay them their holiday, service incentive
leave (SIL), 13th month and overtime pays.
Before the LA, Our Haus primarily argued that the respondents wages complied with the laws
minimum requirement. Aside from paying the monetary amount of the respondents wages, Our
Haus also subsidized their meals (3 times a day), and gave them free lodging near the
construction project they were assigned to.10 In determining the total amount of the
respondents daily wages, the value of these benefits should be considered, in line with Article
97(f)11 of
the
Labor
Code.
The respondents pointed out that Our Haus never presented any proof that they agreed in
writing to the inclusion of their meals value in their wages.16 Also, Our Haus failed to prove that
the value of the facilities it furnished was fair and reasonable.17 Finally, instead of deducting the
maximum amount of 70% of the value of the meals, Our Haus actually withheld its full value
(which was Php290.00 per week for each employee).
In its petition, Our Haus propounded a new theory. It made a distinction between deduction and
charging. A written authorization is only necessary if the facilitys value will be deducted and will
not be needed if it will merely be charged or included in the computation of wages.25 Our
Haus claimed that it did not actually deduct the values of the meals and housing benefits. It only
considered these in computing the total amount of wages paid to the respondents for purposes
of compliance with the minimum wage law. Hence, the written authorization requirement should
not apply.
Issues:
1. WON there is substantial distinction between deducting and charging a facilitys value to
the wages, justifying non-compliance with the written authorization requirement in the
case of merely charging the value of the facility
2. WON the lodging and subsidized meals furnished by Our Haus to respondents are
considered facilities and thus may be deducted from the latters wages.
Ruling:
1. No. In reality, deduction and charging both operate to lessen the actual take-home pay of an
employee; they are two sides of the same coin. In both, the employee receives a lessened
amount because supposedly, the facilitys value, which is part of his wage, had already been

ATHENA M. SALAS | LABOR CASE DIGEST 2015 21

paid to him in kind. As there is no substantial distinction between the two, the requirements set
by law must apply to both.
2. No. The requirements in order to deduct facilities from the workers wages are the
following:
a. Proof must be shown that such facilities are customarily furnished by the trade.
In a string of cases, we have concluded that one of the badges to show that a facility is
customarily furnished by the trade is the existence of a company policy or guideline showing
that provisions for a facility were designated as part of the employees salaries. Apart from
company policy, the employer may also prove compliance with the first requirement by showing
the existence of an industry-wide practice of furnishing the benefits in question among
enterprises engaged in the same line of business. If it were customary among construction
companies to provide board and lodging to their workers and treat their values as part of their
wages, we would have more reason to conclude that these benefits were really facilities.
However, Our Haus could not really be expected to prove compliance with the first requirement
since the living accommodation of workers in the construction industry is not simply a matter of
business practice. Peculiar to the construction business are the occupational safety and health
(OSH) services which the law itself mandates employers to provide to their workers. This is to
ensure the humane working conditions of construction employees despite their constant
exposure to hazardous working environments. Under Section 16 of DOLE Department Order
(DO) No. 13, series of 1998,43 employers engaged in the construction business are required to
provide suitable living accommodation for workers, and as may be applicable, for their families.
Moreover, DOLE DO No. 56, series of 2005, which sets out the guidelines for the
implementation of DOLE DO No. 13, mandates that the cost of the implementation of the
requirements for the construction safety and health of workers, shall be integrated to the overall
project cost.44 The rationale behind this is to ensure that the living accommodation of the
workers is not substandard and is strictly compliant with the DOLEs OSH criteria.
As part of the project cost that construction companies already charge to their clients, the value
of the housing of their workers cannot be charged again to their employees salaries. Our Haus
cannot pass the burden of the OSH costs of its construction projects to its employees by
deducting it as facilities. This is Our Haus obligation under the law.
Lastly, even if a benefit is customarily provided by the trade, it must still pass the purpose
test set by jurisprudence. Under this test, if a benefit or privilege granted to the employee is
clearly for the employers convenience, it will not be considered as a facility but a
supplement. Here, careful consideration is given to the nature of the employers business in
relation to the work performed by the employee. This test is used to address inequitable
situations wherein employers consider a benefit deductible from the wages even if the factual
circumstances show that it clearly redounds to the employers greater advantage.
Our Haus is engaged in the construction business, a labor-intensive enterprise. The success of
its projects is largely a function of the physical strength, vitality and efficiency of its laborers. Its
business will be jeopardized if its workers are weak, sickly, and lack the required energy to

ATHENA M. SALAS | LABOR CASE DIGEST 2015 22

perform strenuous physical activities. Thus, by ensuring that the workers are adequately and
well fed, the employer is actually investing on its business. Moreover, in the construction
business, contractors are usually faced with the problem of meeting target deadlines. More
often than not, work is performed continuously, day and night, in order to finish the project on
the designated turn-over date. Thus, it will be more convenient to the employer if its workers are
housed near the construction site to ensure their ready availability during urgent or emergency
circumstances. Based on these considerations, we conclude that even under the purpose test,
the subsidized meals and free lodging provided by Our Haus are actually supplements.
b. The provision of deductible facilities must be voluntarily accepted in writing by the
employee;
Again, in the motion for reconsideration with the NLRC, Our Haus belatedly submitted five
kasunduans, supposedly executed by the respondents, containing their conformity to the
inclusion of the values of the meals and housing to their total wages. Oddly, Our Haus only
offered these documents when the NLRC had already ruled that respondents did not
accomplish any written authorization, to allow deduction from their wages. These
five kasunduans were also undated, making us wonder if they had really been executed when
respondents first assumed their jobs.
c. The facilities must be charged at fair and reasonable value.
Our Haus valuation cannot be plucked out of thin air. The valuation of a facility must be
supported by relevant documents such as receipts and company records for it to be considered
as fair and reasonable.
In the present case, Our Haus never explained how it came up with the values it assigned for
the benefits it provided; it merely listed its supposed expenses without any supporting
document.

ATHENA M. SALAS | LABOR CASE DIGEST 2015 23

5.VIOLATION OF WAGE ORDERS


6.WAGE ENFORCEMENT AND RECOVERY
12. Peoples Broadcasting (Bombo Radyo Phils) vs. Sec. of DOLE et al., GR No. 179652,
March 6, 2012 Resolution on the main Decision of May 8, 2009
Principle: It is the DOLE who determines WON an ER-EE relationship exists pursuant to the
clause in cases where the relationship of employer-employee still exists in Art. 128(b). If a
complaint is brought before the DOLE to give effect to the labor standards provisions of the
Labor Code or other labor legislation, and there is a finding by the DOLE that there is an
existing employer-employee relationship, the DOLE exercises jurisdiction to the exclusion of the
NLRC. If the DOLE finds that there is no employer-employee relationship, the jurisdiction is
properly with the NLRC.
Facts:
Private respondent Jandeleon Juezan filed a complaint against petitioner with the Department
of Labor and Employment (DOLE) Regional Office No. VII, Cebu City, for illegal deduction,
nonpayment of service incentive leave, 13th month pay, premium pay for holiday and rest day
and illegal diminution of benefits, delayed payment of wages and noncoverage of SSS, PAGIBIG and Philhealth.[1] After the conduct of summary investigations, and after the parties
submitted their position papers, the DOLE Regional Director found that private respondent was
an employee of petitioner, and was entitled to his money claims.
The petitioner brought the matter before the CA. In the Decision of this Court, the CA Decision
was reversed and set aside, and the complaint against petitioner was dismissed. The Court
found that there was no employer-employee relationship between petitioner and private
respondent. It was held that while the DOLE may make a determination of the existence of an
employer-employee relationship, this function could not be co-extensive with the visitorial and
enforcement power provided in Art. 128(b) of the Labor Code, as amended by RA 7730. The
National Labor Relations Commission (NLRC) was held to be the primary agency in determining
the existence of an employer-employee relationship. This was the interpretation of the Court of
the clause in cases where the relationship of employer-employee still exists in Art. 128(b).
Issue:
WON the DOLE make a determination of whether or not an employer-employee relationship
exists, and if so, to what extent
Ruling:
Yes, and when the DOLE determines the existence of an employer-employee relationship, it
must be respected.
No limitation in the law was placed upon the power of the DOLE to determine the existence of
an employer-employee relationship. No procedure was laid down where the DOLE would only
make a preliminary finding, that the power was primarily held by the NLRC. The law did not say
that the DOLE would first seek the NLRCs determination of the existence of an employeremployee relationship, or that should the existence of the employer-employee relationship be

ATHENA M. SALAS | LABOR CASE DIGEST 2015 24

disputed, the DOLE would refer the matter to the NLRC. The DOLE must have the power to
determine whether or not an employer-employee relationship exists, and from there to decide
whether or not to issue compliance orders in accordance with Art. 128(b) of the Labor Code, as
amended by RA 7730.
The DOLE, in determining the existence of an employer-employee relationship, has a ready set
of guidelines to follow, the same guide the courts themselves use. The elements to determine
the existence of an employment relationship are: (1) the selection and engagement of the
employee; (2) the payment of wages; (3) the power of dismissal; (4) the employers power to
control the employees conduct.[9] The use of this test is not solely limited to the NLRC. The
DOLE Secretary, or his or her representatives, can utilize the same test, even in the course of
inspection, making use of the same evidence that would have been presented before the NLRC.
The determination of the existence of an employer-employee relationship by the DOLE must be
respected. The expanded visitorial and enforcement power of the DOLE granted by RA 7730
would be rendered nugatory if the alleged employer could, by the simple expedient of disputing
the employer-employee relationship, force the referral of the matter to the NLRC. The Court
issued the declaration that at least a prima facie showing of the absence of an employeremployee relationship be made to oust the DOLE of jurisdiction. But it is precisely the DOLE
that will be faced with that evidence, and it is the DOLE that will weigh it, to see if the same
does successfully refute the existence of an employer-employee relationship.
If the DOLE makes a finding that there is an existing employer-employee relationship, it takes
cognizance of the matter, to the exclusion of the NLRC. The DOLE would have no jurisdiction
only if the employer-employee relationship has already been terminated, or it appears, upon
review, that no employer-employee relationship existed in the first place.
This is not to say that the determination by the DOLE is beyond question or review. Suffice it to
say, there are judicial remedies such as a petition for certiorari under Rule 65 that may be
availed of, should a party wish to dispute the findings of the DOLE.

13. Superior Packaging Corp., vs. Balagsay et al., G.R. No. 178909, October 10, 2012
Principle: It can be assumed that the DOLE in the exercise of its visitorial and enforcement
power somehow has to make a determination of the existence of an employer-employee
relationship. Such determination, however, is merely preliminary, incidental and collateral to the
DOLEs primary function of enforcing labor standards provisions.
Facts:
The petitioner Superior Packaging engaged the services of Lancer to provide reliever services
to its business, which involves the manufacture and sale of commercial and industrial
corrugated boxes.
Pursuant to a complaint filed by the respondents against the petitioner and its President, Cesar
Luz (Luz), for underpayment of wages, nonpayment of premium pay for worked rest, overtime
pay and non-payment of salary, the Department of Labor and Employment (DOLE) conducted

ATHENA M. SALAS | LABOR CASE DIGEST 2015 25

an inspection of the petitioners premises and found several violations, to wit: (1) nonpresentation of payrolls and daily time records; (2) non-submission of annual report of safety
organization; (3) medical and accident/illness reports; (4) non-registration of establishment
under Rule 1020 of Occupational and Health Standards; and (5) no trained first aide.1 Due to
the petitioners failure to appear in the summary investigations conducted by the DOLE, an
Order2 was issued on June 18, 2003 finding in favor of the respondents and adopting the
computation of the claims submitted.
Petitioner and Luz were ordered, among others, to pay respondents their total claims in the
amount of Eight Hundred Forty Thousand Four Hundred Sixty-Three Pesos and 38/100
(P840,463.38).
They filed a motion for reconsideration on the ground that respondents are not its employees
but of Lancer and that they pay Lancer in lump sum for the services rendered.
The LA, NLRC, and CA ruled in favor of the respondents.
The petitioner objects to the finding that it is engaged in labor-only contracting and is
consequently an indirect employer, considering that it is beyond the visitorial and enforcement
power of the DOLE to make such conclusion. According to the petitioner, such conclusion may
be made only upon consideration of evidentiary matters and cannot be determined solely
through a labor inspection.
Issue:
WON the DOLE has the authority to make a finding of an employer-employee relationship
concomitant to its visitorial and enforcement power.
Ruling: Yes.
The DOLE clearly acted within its authority when it determined the existence of an employeremployee relationship between the petitioner and respondents as it falls within the purview of its
visitorial and enforcement power under Article 128(b) of the Labor Code.
In Peoples Broadcasting (Bombo Radyo Phils., Inc.) v. Secretary of the Department of Labor
and Employment, the Court stated that it can be assumed that the DOLE in the exercise of its
visitorial and enforcement power somehow has to make a determination of the existence of an
employer-employee relationship. Such determination, however, is merely preliminary, incidental
and collateral to the DOLEs primary function of enforcing labor standards provisions.
Also, the existence of an employer-employee relationship is ultimately a question of fact. The
determination made in this case by the DOLE, albeit provisional, and as affirmed by the
Secretary of DOLE and the CA is beyond the ambit of a petition for review on certiorari.

ATHENA M. SALAS | LABOR CASE DIGEST 2015 26

7.WAGE PROTECTION PROVISIONS & PROHIBITIONS REGARDING WAGES


14. Netlink Computer Inc. vs. Delmo, GR No. 160827, June 18, 2014
Principle: In the absence of a written agreement between the employer and the employee that
sales commissions shall be paid in a foreign currency, the latter has the right to be paid in such
foreign currency once the same has become an established practice of the former. The rate of
exchange at the time of payment, not the rate of exchange at the time of the sales, controls.
Facts:
On November 3, 1991, Netlink Computer, Inc. Products and Services (Netlink) hired Eric S.
Delmo (Delmo) as account manager tasked to canvass and source clients and convince them to
purchase the products and services of Netlink. Delmo worked in the field most of the time. He
and his fellow account managers were not required to accomplish time cards to record their
personal presence in the office of Netlink.1 He was able to generate sales worth
P35,000,000.00, more or less, from which he earned commissions amounting to P993,558.89
and US$7,588.30. He then requested payment of his commissions, but Netlink refused and only
gave him partial cash advances chargeable to his commissions. Later on, Netlink began to
nitpick and fault find, like stressing his supposed absences and tardiness. In order to force him
to resign, Netlink issued several memoranda detailing his supposed infractions of the
companys attendance policy. Despite the memoranda, Delmo continued to generate huge sales
for Netlink.
On November 28, 1996, Delmo was shocked when he was refused entry into the company
premises by the security guard pursuant to a memorandum to that effect. His personal
belongings were still inside the company premises and he sought their return to him. This
incident prompted Delmo to file a complaint for illegal dismissal.
The NLRC and CA ruled that the respondent was terminated for just causes, but ordered
petitioner to pay the latter his unpaid commissions, including his dollar commissions for US
dollar-denominated sales.
Issues:
1. WON the payment of the commissions should be in US dollars
2. WON the award of attorneys fees was warranted.
Ruling:
1. Yes. As a general rule, all obligations shall be paid in Philippine currency. However, the
contracting parties may stipulate that foreign currencies may be used for settling obligations.
This
is
pursuant
to
Republic Act
No.
8183,which
provides
as
follows:
Section 1. All monetary obligations shall be settled in the Philippine currency which is legal
tender in the Philippines. However, the parties may agree that the obligation or transaction shall
be settled in any other currency at the time of payment.

ATHENA M. SALAS | LABOR CASE DIGEST 2015 27

The rate of exchange at the time of payment, not the rate of exchange at the time of the sales,
controls.
There was no written contract between Netlink and Delmo stipulating that the latters
commissions would be paid in US dollars. The absence of the contractual stipulation
notwithstanding, Netlink was still liable to pay Delmo in US dollars because the practice of
paying its sales agents in US dollars for their US dollar-denominated sales had become a
company policy. This was impliedly admitted by Netlink when it did not refute the allegation that
the commissions earned by Delmo and its other sales agents had been paid in US dollars.
Instead of denying the allegation, Netlink only sought a declaration that the US dollar
commissions be paid using the exchange rate at the time of sale. The principle of nondiminution of benefits, which has been incorporated in Article 10013 of the Labor Code, forbade
Netlink from unilaterally reducing, diminishing, discontinuing or eliminating the practice. Verily,
the phrase supplements, or other employee benefits in Article 100 is construed to mean the
compensation and privileges received by an employee aside from regular salaries or wages.
2. Yes. The award of attorney's fees must, likewise, be upheld in line with the decision of the
Supreme Court in the case of Consolidated Rural Bank (Cagayan Valley), Inc. vs. National
Labor Relations Commission, 301 SCRA 223, 235, where it was held that "in actions for
recovery of wages or where an employee was forced to litigate and thus incur expenses to
protect her rights and interests, even if not so claimed, an award of attorney's fees equivalent to
ten percent (10%) of the total award is legally and morally justifiable. There is no doubt that in
the present case, the private respondent has incurred expenses for the protection and
enforcement of his right to his commissions.

15. Locsin II vs. Mekeni Food Corp., GR No. 192105, December 9, 2013
Principle: In the absence of specific terms and conditions governing a car plan agreement
between the employer and employee, the former may not retain the installment payments made
by the latter on the car plan and treat them as rents for the use of the service vehicle in the
event that the employee ceases his employment and is unable to complete the installment
payments on the vehicle. The underlying reason is that the service vehicle was precisely used
in the former's business; any personal benefit obtained by the employee from its use is merely
incidental.
Facts:
In February 2004, respondent Mekeni Food Corporation (Mekeni)a Philippine company
engaged in food manufacturing and meat processing offered petitioner Antonio Locsin II the
position of Regional Sales Manager to oversee Mekenis National Capital Region
Supermarket/Food Service and South Luzon operations.
In addition to a compensation and benefit package, Mekeni offered petitioner a car plan, under
which one-half of the cost of the vehicle is to be paid by the company and the other half to be
deducted from petitioners salary.

ATHENA M. SALAS | LABOR CASE DIGEST 2015 28

Mekenis offer was contained in an Offer Sheet which was presented to petitioner. Petitioner
began his stint as Mekeni Regional Sales Manager on March 17, 2004. To be able to effectively
cover his appointed sales territory, Mekeni furnished petitioner with a used Honda Civic car
valued at P280,000.00, which used to be the service vehicle of petitioners immediate
supervisor. Petitioner paid for his 50% share through salary deductions of P5,000.00 each
month. Subsequently, Locsin resigned effective February 25, 2006. By then, a total of
P112,500.00 had been deducted from his monthly salary and applied as part of the employees
share in the car plan.
Mekeni supposedly put in an equivalent amount as its share under the car plan. In his
resignation letter, petitioner made an offer to purchase his service vehicle by paying the
outstanding balance thereon. The parties negotiated, but could not agree on the terms of the
proposed purchase. Petitioner thus returned the vehicle to Mekeni on May 2, 2006.
In May 3, 2007, petitioner filed against Mekeni and/or its President, Prudencio S. Garcia, a
Complaint for the recovery of monetary claims consisting of unpaid salaries, commissions,
sick/vacation leave benefits, and recovery of monthly salary deductions which were earmarked
for his cost-sharing in the car plan.
The NLRC held that petitioners amortization payments on his service vehicle amounting to
P112,500.00 should be reimbursed; if not, unjust enrichment would result, as the vehicle
remained in the possession and ownership of Mekeni. The CA reversed the decision of the
NLRC and held that this award should be deleted.
Issue:
WON petitioner is entitled to a refund of all the amounts applied to the cost of the service
vehicle under the car plan.
Ruling:
Partly, yes. The petitioner is only entitled to a refund of his contributions to the car plan, but is
not entitled to the amount contributed by employer Mekeni.
From the evidence on record, it is seen that the Mekeni car plan offered to petitioner was
subject to no other term or condition than that Mekeni shall cover one-half of its value, and
petitioner shall in turn pay the other half through deductions from his monthly salary. Mekeni has
not shown, by documentary evidence or otherwise, that there are other terms and conditions
governing its car plan agreement with petitioner. There is no evidence to suggest that if
petitioner failed to completely cover one-half of the cost of the vehicle, then all the deductions
from his salary going to the cost of the vehicle will be treated as rentals for his use thereof while
working with Mekeni, and shall not be refunded. Installments made on the car plan may be
treated as rentals only when there is an express stipulation in the car plan agreement to such
effect.
Indeed, the Court cannot allow that payments made on the car plan should be forfeited by
Mekeni and treated simply as rentals for petitioners use of the company service vehicle. Nor
may they be retained by it as purported loan payments, as it would have this Court believe. In

ATHENA M. SALAS | LABOR CASE DIGEST 2015 29

the first place, there is precisely no stipulation to such effect in their agreement. Secondly, it may
not be said that the car plan arrangement between the parties was a benefit that the petitioner
enjoyed; on the contrary, it was an absolute necessity in Mekenis business operations, which
benefited it to the fullest extent: without the service vehicle, petitioner would have been unable
to rapidly cover the vast sales territory assigned to him, and sales or marketing of Mekenis
products could not have been booked or made fast enough to move Mekenis inventory. Poor
sales, inability to market Mekenis products, a high rate of product spoilage resulting from
stagnant inventory, and poor monitoring of the sales territory are the necessary consequences
of lack of mobility.
Any benefit or privilege enjoyed by petitioner from using the service vehicle was merely
incidental and insignificant, because for the most part the vehicle was under Mekenis control
and supervision. Thus, it is clear that while petitioner was paying for half of the vehicles value,
Mekeni was reaping the full benefits from the use thereof.
In light of the foregoing, it is unfair to deny petitioner a refund of all his contributions to the car
plan. In the absence of specific terms and conditions governing the car plan arrangement
between the petitioner and Mekeni, a quasi-contractual relation was created between them.
Consequently, Mekeni may not enrich itself by charging petitioner for the use of its vehicle which
is otherwise absolutely necessary to the full and effective promotion of its business. Conversely,
petitioner cannot recover the monetary value of Mekenis counterpart contribution to the cost of
the vehicle; that is not property or money that belongs to him, nor was it intended to be given to
him in lieu of the car plan. In other words, Mekenis share of the vehicles cost was not part of
petitioners compensation package. To start with, the vehicle is an asset that belonged to
Mekeni. Just as Mekeni is unjustly enriched by failing to refund petitioners payments, so should
petitioner notbe awarded the value of Mekenis counterpart contribution to the car plan, as this
would unjustly enrich him at Mekenis expense.

16. TH Shopfitters Corp., et al., vs. T&H Shopfitters Corp., Union, GR No. 191714, Feb 26,
2014
Principle: On the award of attorneys fees, the applicable law concerning the grant thereof in
labor cases is Article 111 of the Labor Code. Pursuant thereto, the award of 10% attorneys fees
is limited to cases of unlawful withholding of wages. In this case, however, the Court cannot find
any claim or proof that petitioners unlawfully withheld the wages of respondents. Consequently,
the grant of 10% attorneys fees in favor of respondents is not justified under the circumstances.
Accordingly, the Court deems it proper to delete the same.
Facts:
T&H Shopfitters Corporation/Gin Queen Corporation workers union (THSGQ Union) filed their
Complaint for Unfair Labor Practice (ULP) by way of union busting, and Illegal Lockout, with
moral and exemplary damages and attorneys fees, against T&H Shopfitters and Gin Queen
Corporation before the Labor Arbiter.
The LA dismissed respondents complaint and all their money claims for lack of merit.

ATHENA M. SALAS | LABOR CASE DIGEST 2015 30

NLRC reversed the LA decision and ruled in favor of respondents. CA sustained the ruling of the
NLRC.
Issues:
Whether ULP acts were committed by petitioners against respondents in the case at bench.

Ruling:
ULP was committed.
In the case at bench, petitioners are being accused of violations of paragraphs (a), (c), and (e)
of Article 257 (formerly Article 248) of the Labor Code, to wit:
Article 257. Unfair labor practices of employers.It shall be unlawful for an employer to commit
any of the following unfair labor practices:
(a) To interfere with, restrain or coerce employees in the exercise of their right to self
organization;
xxxx
(c) To contract out services or functions being performed by union members when such will
interfere with, restrain, or coerce employees in the exercise of their right to selforganization;
xxxx
(e) To discriminate in regard to wages, hours of work, and other terms and conditions of
employment in order to encourage or discourage membership in any labor organization. x x x
The concept of ULP is embodied in Article 256 (formerly Article 247) of the Labor Code, which
provides:
Article 256. Concept of unfair labor practice and procedure for prosecution thereof.Unfair
labor practices violate the constitutional right of workers and employees to selforganization,
are inimical to the legitimate interests of both labor and management, including their right to
bargain collectively and otherwise deal with each other in an atmosphere of freedom and mutual
respect, disrupt industrial peace and hinder the promotion of healthy and stable labor
management relations.
In essence, ULP relates to the commission of acts that transgress the workers right to organize.
As specified in Articles 248 [now Article 257] and 249 [now Article 258] of the Labor Code, the
prohibited acts must necessarily relate to the workers right to selforganization x x x.
In the case of Insular Life Assurance Co., Ltd. Employees Association NATU v. Insular Life
Assurance Co. Ltd.,16 this Court had occasion to lay down the test of whether an employer has
interfered with and coerced employees in the exercise of their right to selforganization, that is,
whether the employer has engaged in conduct which, it may reasonably be said, tends to
interfere with the free exercise of employees rights; and that it is not necessary that there be

ATHENA M. SALAS | LABOR CASE DIGEST 2015 31

direct evidence that any employee was in fact intimidated or coerced by statements of threats of
the employer if there is a reasonable inference that antiunion conduct of the employer does
have an adverse effect on selforganization and collective bargaining.
The questioned acts of petitioners, namely: 1) sponsoring a field trip to Zambales for its
employees, to the exclusion of union members, before the scheduled certification election; 2)
the active campaign by the sales officer of petitioners against the union prevailing as a
bargaining agent during the field trip; 3) escorting its employees after the field trip to the polling
center; 4) the continuous hiring of subcontractors performing respondents functions; 5)
assigning union members to the Cabangan site to work as grass cutters; and 6) the
enforcement of work on a rotational basis for union members, all reek of interference on the part
of petitioners.
Indubitably, the various acts of petitioners, taken together, reasonably support an inference that,
indeed, such were all orchestrated to restrict respondents free exercise of their right to self
organization. The Court is of the considered view that petitioners undisputed actions prior and
immediately before the scheduled certification election, while seemingly innocuous, unduly
meddled in the affairs of its employees in selecting their exclusive bargaining representative.
In fine, mindful of the nature of the charge of ULP, including its civil and/or criminal
consequences, the Court finds that the NLRC, as correctly sustained by the CA, had sufficient
factual and legal bases to support its finding of ULP.
Anent the issue on the award of attorneys fess, the applicable law concerning the grant thereof
in labor cases is Article 11120 of the Labor Code. Pursuant thereto, the award of 10% attorneys
fees is limited to cases of unlawful withholding of wages. In this case, however, the Court cannot
find any claim or proof that petitioners unlawfully withheld the wages of respondents.
Consequently, the grant of 10% attorneys fees in favor of respondents is not justified under the
circumstances. Accordingly, the Court deems it proper to delete the same.

17. Wesleyan University-Phils., vs. Wesleyan University-Phils., Faculty & Staff Asso., GR
No. 181806, March 12, 2014
Principle: The Non-Diminution Rule found in Article 100 of the Labor Code explicitly prohibits
employers from eliminating or reducing the benefits received by their employees. This rule,
however, applies only if the benefit is based on an express policy, a written contract, or has
ripened into a practice. To be considered a practice, it must be consistently and deliberately
made by the employer over a long period of time.
An exception to the rule is when the practice is due to error in the construction or application of
a doubtful or difficult question of law. The error, however, must be corrected immediately after
its discovery; otherwise, the rule on Non-Diminution of Benefits would still apply.
Facts:
Petitioner and respondent signed a 5-year CBA. Petitioner, through its President, Atty. Maglaya
issued a Memorandum providing guidelines on the implementation of vacation and sick leave

ATHENA M. SALAS | LABOR CASE DIGEST 2015 32

credits as well as vacation leave commutation. The pertinent portions of the Memorandum to
wit:
VACATION AND SICK LEAVE CREDITS
Vacation and sick leave credits are not automatic. They have to be earned. Monthly, a qualified
employee earns an equivalent of 1.25 days credit each for VL and SL. Vacation Leave and Sick
Leave credits of 15 days become complete at the cut off date of May 31 of each year.
(Example, only a total of 5 days credit will be given to an employee for each of sick leave [or]
vacation leave, as of month end September, that is, 4 months from June to September
multiplied by 1.25 days). An employee, therefore, who takes VL or SL beyond his leave credits
as of date will have to file leave without pay for leaves beyond his credit.
VACATION LEAVE COMMUTATION
Only vacation leave is commuted or monetized to cash. Vacation leave commutation is effected
after the second year of continuous service of an employee. Hence, an employee who started
working June 1, 2005 will get his commutation on May 31, 2007 or thereabout.
Respondents President, Cynthia L. De Lara (De Lara) wrote a letter to Atty. Maglaya informing
him that respondent is not amenable to the unilateral changes made by petitioner. De Lara
questioned the guidelines for being violative of existing practices and the CBA, specifically
Sections
1
and
2,
Article
XII
of
the
CBA,
to
wit:
ARTICLE XII
VACATION LEAVE AND SICK LEAVE
SECTION 1. VACATION LEAVE - All regular and non-tenured rank-and-file faculty and staff who
are entitled to receive shall enjoy fifteen (15) days vacation leave with pay annually.
1.1 All unused vacation leave after the second year of service shall be converted into cash and
be paid to the entitled employee at the end of each school year to be given not later than August
30 of each year.
SECTION 2. SICK LEAVE - All regular and non-tenured rank-and-file faculty and staff shall
enjoy fifteen (15) days sick leave with pay annually.
Subsequently, a Labor Management Committee (LMC) Meeting was held during which
petitioner advised respondent to file a grievance complaint on the implementation of the
vacation and sick leave policy. In the same meeting, petitioner announced its plan of
implementing a one-retirement policy, which was unacceptable to respondent.
Unable to settle their differences at the grievance level, the parties referred the matter to a
Voluntary Arbitrator. During the hearing, respondent submitted affidavits to prove that there is
an established practice of giving two retirement benefits, one from the Private Education
Retirement Annuity Association (PERAA) Plan and another from the CBA Retirement Plan. The
Voluntary Abitrator rendered a Decision declaring the one-retirement policy and the
Memorandum contrary to law.

ATHENA M. SALAS | LABOR CASE DIGEST 2015 33

The CA affirmed the findings and the ruling of the VA.


Issues:
Whether the CA committed grave and palpable error in sustaining the Voluntary Arbitrators
ruling that a university practice of granting its employees two (2) sets of Retirement Benefits had
already been established as defined by the law and jurisprudence especially in light of the
illegality and lack of authority of such alleged grant.
Ruling:
The practice of giving two retirement benefits is supported by substantial evidence.
The Non-Diminution Rule found in Article 100 of the Labor Code explicitly prohibits employers
from eliminating or reducing the benefits received by their employees. This rule, however,
applies only if the benefit is based on an express policy, a written contract, or has ripened into a
practice. To be considered a practice, it must be consistently and deliberately made by the
employer
over
a
long
period
of
time.
An exception to the rule is when the practice is due to error in the construction or application of
a doubtful or difficult question of law. The error, however, must be corrected immediately after
its discovery; otherwise, the rule on Non-Diminution of Benefits would still apply.
The practice of giving two retirement benefits to petitioners employees is supported by
substantial evidence.
In this case, respondent was able to present substantial evidence in the form of affidavits to
support its claim that there are two retirement plans. Based on the affidavits, petitioner has
been giving two retirement benefits as early as 1997. Petitioner, on the other hand, failed to
present any evidence to refute the veracity of these affidavits. Petitioners contention that these
affidavits are self-serving holds no water. The retired employees of petitioner have nothing to
lose or gain in this case as they have already received their retirement benefits. Thus, they
have no reason to perjure themselves. Obviously, the only reason they executed those
affidavits is to bring out the truth. As we see it then, their affidavits, corroborated by the
affidavits of incumbent employees, are more than sufficient to show that the granting of two
retirement benefits to retiring employees had already ripened into a consistent and deliberate
practice.
Moreover, petitioners assertion that there is only one retirement plan as the CBA Retirement
Plan and the PERAA Plan are one and the same is not supported by any evidence. There is
nothing in Article XVI of the CBA to indicate or even suggest that the Plan referred to in the
CBA is the PERAA Plan. Besides, any doubt in the interpretation of the provisions of the CBA
should be resolved in favor of respondent.

18. Bluer Than Blue Joint Ventures Co., vs. Esteban, GR No. 192582, April 7, 2014, citing
2011 Nina Jewelry Manufacturing of Metal Arts Inc. vs. Montecillo

ATHENA M. SALAS | LABOR CASE DIGEST 2015 34

Principle: Article 113 of the Labor Code provides that no employer, in his own behalf or in
behalf of any person, shall make any deduction from the wages of his employees, except in
cases where the employer is authorized by law or regulations issued by the Secretary of Labor
and Employment, among others.
Facts:
Respondent Glyza Esteban (Esteban) was employed as Sales Clerk, and assigned at Bluer
Than Blue Joint Ventures Company's (petitioner) EGG boutique in SM City Marilao, Bulacan,
beginning the year 2006. Part of her primary tasks were attending to all customer needs,
ensuring efficient inventory, coordinating orders from clients, cashiering and reporting to the
accounting department.
In November 2006, the petitioner received a report that several employees have access to its
point-of-sale (POS) system through a universal password given by Elmer Flores (Flores). Upon
investigation, it was discovered that it was Esteban who gave Flores the password. The
petitioner sent a letter memorandum to Esteban asking her to explain in writing why she should
not be disciplinary dealt with for tampering with the companys POS system through the use of
an unauthorized password. Esteban was also placed under preventive suspension for ten days.
In her explanation, Esteban admitted that she used the universal password three times on the
same day in December 2005, after she learned of it from two other employees who she saw
browsing through the petitioners sales inquiry. She inquired how the employees were able to
open the system and she was told that they used the "123456" password.
On November 13, 2006, Estebans preventive suspension was lifted, but at the same time, a
notice of termination was sent to her, finding her explanation unsatisfactory and terminating her
employment immediately on the ground of loss of trust and confidence.
Esteban was given her final pay, including benefits and bonuses, less inventory variances
incurred by the store amounting to P8,304.93..
Esteban filed a complaint for illegal dismissal, illegal suspension, holiday pay, rest day and
separation pay.
The Labor Arbiter (LA) ruled in favor of Esteban and found that she was illegally dismissed.
NLRC reversed the decision of the LA. However, the CA reinstated the decision of the LA.
Issues:
WON the deduction of P8,304.93, representing the stores negative variance, from Estebans
salary violates Article 113 of the Labor Code, which prohibits wage deduction.
Ruling:
Yes, petitioner violated the Labor Code provision on wage deduction.
The petitioner deducted the amount of P8,304.93 from Estebans last salary. According to the
petitioner, this represents the stores negative variance for the year 2005 to 2006. The petitioner

ATHENA M. SALAS | LABOR CASE DIGEST 2015 35

justifies the deduction on the basis of alleged trade practice and that it is allowed by the Labor
Code.
Article 113 of the Labor Code provides that no employer, in his own behalf or in behalf of any
person, shall make any deduction from the wages of his employees, except in cases where the
employer is authorized by law or regulations issued by the Secretary of Labor and Employment,
among others. The Omnibus Rules Implementing the Labor Code, meanwhile, provides:
SECTION 14. Deduction for loss or damage. Where the employer is engaged in a trade,
occupation or business where the practice of making deductions or requiring deposits is
recognized to answer for the reimbursement of loss or damage to tools, materials, or equipment
supplied by the employer to the employee, the employer may make wage deductions or require
the employees to make deposits from which deductions shall be made, subject to the following
conditions:
a) That the employee concerned is clearly shown to be responsible for the loss or damage;
b) That the employee is given reasonable opportunity to show cause why deduction should
not be made;
c) That the amount of such deduction is fair and reasonable and shall not exceed the
actual loss or damage; and
d) That the deduction from the wages of the employee does not exceed 20 percent of the
employees wages in a week.
In this case, the petitioner failed to sufficiently establish that Esteban was responsible for the
negative variance it had in its sales for the year 2005 to 2006 and that Esteban was given the
opportunity to show cause the deduction from her last salary should not be made. The Court
cannot accept the petitioners statement that it is the practice in the retail industry to deduct
variances from an employees salary, without more. In Nia Jewelry Manufacturing of Metal Arts,
Inc. v. Montecillo,the Court ruled that:
The petitioners should first establish that the making of deductions from the salaries is
authorized by law, or regulations issued by the Secretary of Labor. Further, the posting of cash
bonds should be proven as a recognized practice in the jewelry manufacturing business, or
alternatively, the petitioners should seek for the determination by the Secretary of Labor through
the issuance of appropriate rules and regulations that the policy the former seeks to implement
is necessary or desirable in the conduct of business. The petitioners failed in this respect. It
bears stressing that without proofs that requiring deposits and effecting deductions are
recognized practices, or without securing the Secretary of Labor's determination of the
necessity or desirability of the same, the imposition of new policies relative to deductions and
deposits can be made subject to abuse by the employers. This is not what the law intends.

ATHENA M. SALAS | LABOR CASE DIGEST 2015 36

8.PAYMENT OF WAGES
9.CONDITIONS OF EMPLOYMENT
10.MINIMUM LABOR STANDARD BENEFITS
19. Radio Mindanao Network Inc. et al., vs. Ybarola, Jr. G.R. No. 198662, Sept. 12, 2012
Facts:
Respondents Domingo Z. Ybarola, Jr. and Alfonso E. Rivera, Jr. were hired on June 15, 1977
and June 1, 1983, respectively, by RMN. They eventually became account managers, soliciting
advertisements and servicing various clients of RMN.
On September 15, 2002, the respondents' services were terminated as a result of RMN's
reorganization/restructuring; they were given their separation pay P631,250.00 for Ybarola,
and P481,250.00 for Rivera. Sometime in December 2002, they executed release/quitclaim
affidavits.
Dissatisfied with their separation pay, the respondents filed separate complaints (which were
later consolidated) against RMN and its President, Eric S. Canoy, for illegal dismissal with
several money claims, including attorney's fees. They indicated that their monthly salary rates
were P60,000.00 for Ybarola and P40,000.00 for Rivera.

Issues:
Whether or not the commissions received by respondents be included in the computation of
their separation pay.

Ruling:
Yes. If these commissions had been really profit-sharing bonuses to the respondents, they
should have received the same amounts, yet, as the NLRC itself noted, Ybarola and Rivera
received P372,173.11 and P586,998.50 commissions, respectively, in 2002. The variance in
amounts the respondents received as commissions supports the CA's finding that the salary
structure of the respondents was such that they only received a minimal amount as guaranteed
wage; a greater part of their income was derived from the commissions they get from soliciting
advertisements; these advertisements are the "products" they sell. As the CA aptly noted, this
kind of salary structure does not detract from the character of the commissions being part of the
salary or wage paid to the employees for services rendered to the company, as the Court held
in Philippine Duplicators, Inc. v. NLRC.

ATHENA M. SALAS | LABOR CASE DIGEST 2015 37

20. David/Yiels Hog Dealer vs. Macasio, GR No. 195466, July 2, 2014
Facts:
Macasio filed before the LA a complaint against petitioner Ariel L. David, doing business under
the name and style "Yiels Hog Dealer," for non-payment of overtime pay, holiday pay and 13th
month pay. He also claimed payment for moral and exemplary damages and attorney's
fees. Macasio also claimed payment for service incentive leave (SIL).
Macasio alleged before the LA that he had been working as a butcher for David since January
6, 1995. Macasio claimed that David exercised effective control and supervision over his work,
pointing out that David: (1) set the work day, reporting time and hogs to be chopped, as well as
the manner by which he was to perform his work; (2) daily paid his salary of P700.00, which was
increased from P600.00 in 2007, P500.00 in 2006 and P400.00 in 2005; and (3) approved and
disapproved his leaves. Macasio added that David owned the hogs delivered for chopping, as
well as the work tools and implements; the latter also rented the workplace. Macasio further
claimed that David employs about twenty-five (25) butchers and delivery drivers.
In his defense, David claimed that he started his hog dealer business in 2005 and that he only
has ten employees. He alleged that he hired Macasio as a butcher or chopper on "pakyaw" or
task basis who is, therefore, not entitled to overtime pay, holiday pay and 13th month pay
pursuant to the provisions of the Implementing Rules and Regulations (IRR) of the Labor Code.
David pointed out that Macasio: (1) usually starts his work at 10:00 p.m. and ends at 2:00 a.m.
of the following day or earlier, depending on the volume of the delivered hogs; (2) received the
fixed amount of P700.00 per engagement, regardless of the actual number of hours that he
spent chopping the delivered hogs; and (3) was not engaged to report for work and, accordingly,
did not receive any fee when no hogs were delivered.
David maintains that Macasio's engagement was on a "pakyaw" or task basis. Hence, the latter
is excluded from the coverage of holiday, SIL and 13th month pay.|||

Issue:
Whether or not a worker engaged on pakyaw or task basis is entitled to Holiday, SIL, and 13 th
month pay.
Ruling:
Petition was partially granted.

ATHENA M. SALAS | LABOR CASE DIGEST 2015 38

SIL
The general rule is that holiday and SIL pay provisions cover all employees. To be excluded
from their coverage, an employee must be one of those that these provisions expressly exempt,
strictly in accordance with the exemption.
Under the IRR, exemption from the coverage of holiday and SIL pay refer to "field
personnel and other employees whose time and performance is unsupervised by the employer
including those who are engaged on task or contract basis[.]" Note that unlike Article 82 of the
Labor Code, the IRR on holiday and SIL pay do not exclude employees "engaged on task
basis" as a separate and distinct category from employees classified as "field personnel."
Rather, these employees are altogether merged into one classification of exempted employees.
The payment of an employee on task or pakyaw basis alone is insufficient to exclude one from
the coverage of SIL and holiday pay. They are exempted from the coverage of Title I (including
the holiday and SIL pay) only if they qualify as "field personnel." The IRR therefore validly
qualifies and limits the general exclusion of "workers paid by results" found in Article 82 from the
coverage of holiday and SIL pay. This is the only reasonable interpretation since the
determination of excluded workers who are paid by results from the coverage of Title I is
"determined by the Secretary of Labor in appropriate regulations."|||

In short, in determining whether workers engaged on "pakyaw" or task basis" is entitled to


holiday and SIL pay, the presence (or absence) of employer supervision as regards the worker's
time and performance is the key: if the worker is simply engaged on pakyaw or task basis, then
the general rule is that he is entitled to a holiday pay and SIL pay unless exempted from the
exceptions specifically provided under Article 94 (holiday pay) and Article 95 (SIL pay) of the
Labor Code. However, if the worker engaged on pakyaw or task basis also falls within the
meaning of "field personnel" under the law, then he is not entitled to these monetary benefits.|||

Based on the definition of field personnel under Article 82, that Macasio does not fall under the
definition of "field personnel." The finding is supported by the established facts of this case: first,
Macasio regularly performed his duties at David's principal place of business; second, his actual
hours of work could be determined with reasonable certainty; and, third, David supervised his
time and performance of duties. Since Macasio cannot be considered a "field personnel," then
he is not exempted from the grant of holiday, SIL pay even as he was engaged on "pakyaw" or
task basis.|||

13th month pay


The governing law on 13th month pay is PD No. 851. As with holiday and SIL pay, 13th month
pay benefits generally cover all employees; an employee must be one of those expressly
enumerated to be exempted. Section 3 of the Rules and Regulations Implementing P.D. No. 851
enumerates the exemptions from the coverage of 13th month pay benefits. Under Section 3 (e),

ATHENA M. SALAS | LABOR CASE DIGEST 2015 39

"employers of those who are paid on . . . task basis, and those who are paid a fixed amount
for performing a specific work, irrespective of the time consumed in the
performancethereof" are exempted. DSAICa
Note that unlike the IRR of the Labor Code on holiday and SIL pay, Section 3 (e) of the Rules
and Regulations Implementing PD No. 851 exempts employees "paid on task basis" without any
reference to "field personnel." This could only mean that insofar as payment of the 13th month
pay is concerned, the law did not intend to qualify the exemption from its coverage with the
requirement that the task worker be a "field personnel" at the same time.

11.OTHER SPECIAL BENEFITS


21. Radio Mindanao Network Inc, et al., vs. Ybarola, Jr. et al., G.R. No. 198662, September
12, 2012
Facts:
Respondents Domingo Z. Ybarola, Jr. and Alfonso E. Rivera, Jr. were hired on June 15, 1977
and June 1, 1983, respectively, by RMN. They eventually became account managers, soliciting
advertisements and servicing various clients of RMN.

On September 15, 2002, the respondents' services were terminated as a result of RMN's
reorganization/restructuring; they were given their separation pay P631,250.00 for Ybarola,
and P481,250.00 for Rivera. Sometime in December 2002, they executed release/quitclaim
affidavits.

Dissatisfied with their separation pay, the respondents filed separate complaints (which were
later consolidated) against RMN and its President, Eric S. Canoy, for illegal dismissal with
several money claims, including attorney's fees. They indicated that their monthly salary rates
were P60,000.00 for Ybarola and P40,000.00 for Rivera.

Issues:

ATHENA M. SALAS | LABOR CASE DIGEST 2015 40

Whether or not the commissions received by respondents be included in the computation of


their separation pay.
Ruling:
Yes. If these commissions had been really profit-sharing bonuses to the respondents, they
should have received the same amounts, yet, as the NLRC itself noted, Ybarola and Rivera
received P372,173.11 and P586,998.50 commissions, respectively, in 2002. The variance in
amounts the respondents received as commissions supports the CA's finding that the salary
structure of the respondents was such that they only received a minimal amount as guaranteed
wage; a greater part of their income was derived from the commissions they get from soliciting
advertisements; these advertisements are the "products" they sell. As the CA aptly noted, this
kind of salary structure does not detract from the character of the commissions being part of the
salary or wage paid to the employees for services rendered to the company, as the Court held
in Philippine Duplicators, Inc. v. NLRC.
22. Padillo vs. Rural Bank of Nabunturan Inc. G.r. No. 199338, Jan. 21, 2013
Facts:
Padillo, was employed by respondent Bank as its SA Bookkeeper. Due to liquidity problems
which arose sometime in 2003, the Bank took out retirement/insurance plans with Philippine
American Life and General Insurance Company (Philam Life) for all its employees in anticipation
of its possible closure and the concomitant severance of its personnel. Respondent Mark S.
Oropeza (Oropeza), the President of the Bank, bought majority shares of stock in the Bank and
took over its management which brought about its gradual rehabilitation. The Bank's finances
improved and eventually, its liquidity was regained.
Padillo suffered a mild stroke due to hypertension which consequently impaired his ability to
effectively pursue his work. In particular, he was diagnosed with Hypertension S/P CVA
(Cerebrovascular Accident) with short term memory loss, the nature of which had been
classified as a total disability. On September 10, 2007, he wrote a letter addressed to
respondent Oropeza expressing his intention to avail of an early retirement package. Despite
several follow-ups, his request remained unheeded. Padillo was separated from employment
due to his poor and failing health
Issue:
Whether or not petitioner is entitled to retirement pay.
Ruling:
In the absence of any applicable agreement, an employee must (1) retire when he is at least
sixty (60) years of age and (2) serve at least (5) years in the company to entitle him/her to a
retirement benefit of at least one-half (1/2) month salary for every year of service, with a fraction
of at least six (6) months being considered as one whole year. Notably, these age and tenure

ATHENA M. SALAS | LABOR CASE DIGEST 2015 41

requirements are cumulative and non-compliance with one negates the employee's entitlement
to the retirement benefits under Article 300 of the Labor Code altogether.
In this case, it is undisputed that there exists no retirement plan, collective bargaining
agreement or any other equivalent contract between the parties which set out the terms and
condition for the retirement of employees, with the sole exception of the Philam Life Plan which
premiums had already been paid by the Bank. Neither was it proven that there exists an
established company policy of giving early retirement packages to the Bank's aging employees.
All told, in the absence of any applicable contract or any evolved company policy, Padillo should
have met the age and tenure requirements set forth under Article 300 of the Labor Code to be
entitled to the retirement benefits provided therein. Unfortunately, while Padillo was able to
comply with the five (5) year tenure requirement as he served for twenty-nine (29) years
he, however, fell short with respect to the sixty (60) year age requirement given that he was only
fifty-five (55) years old when he retired. Therefore, without prejudice to the proceeds due under
the Philam Life Plan, petitioners' claim for retirement benefits must be denied.
Nevertheless, financial assistance should be awarded but at an increased amount. With a
veritable understanding that the award of financial assistance is usually the final refuge of the
laborer, considering as well the supervening length of time which had sadly overtaken the point
of Padillo's death an employee who had devoted twenty-nine (29) years of dedicated service
to the Bank the Court, in light of the dictates of social justice, holds that the CA's financial
assistance award should be increased from P50,000.00 to P75,000.00, still exclusive of the
P100,000.00 benefit receivable by the petitioners under the Philam Life Plan which remains
undisputed.

12.RIGHT TO SECURITY OF TENURE


23. Gapayao vs. Fulo et al., G.R. No. 193493, June 13, 2013
Facts:
Fulo (deceased) died of "acute renal failure secondary to 1st degree burn 70% secondary
electrocution" while doing repairs at the residence and business establishment of petitioner
located at San Julian, Irosin, Sorsogon. The ER and the wife executed a compromise

ATHENA M. SALAS | LABOR CASE DIGEST 2015 42

agreement whereby the ER agreed to pay 40K to the suriving spouse of Fulo. Thereafter,
private respondent filed a claim for social security benefits with the Social Security System
(SSS) Sorsogon Branch. However, upon verification and evaluation, it was discovered that
the deceased was not a registered member of the SSS.
Private respondent claimed that the deceased was not the former's employee, but was rather an
independent contractor whose tasks were not subject to petitioner's control and supervision.
Assuming arguendo that the deceased was petitioner's employee, he was still not entitled to be
paid his SSS premiums for the intervening period when he was not at work, as he was an
"intermittent worker who [was] only summoned every now and then as the need [arose]."
Hence, petitioner insisted that he was under no obligation to report the former's demise to the
SSS for social security coverage.
Issue:
Whether or not there exists between the deceased Jaime Fulo and petitioner an employeremployee relationship that would merit an award of benefits in favor of private respondent under
social security laws.
Ruling:
Jurisprudence has identified the three types of employees mentioned in the provision: (1)
regular employees or those who have been engaged to perform activities that are usually
necessary or desirable in the usual business or trade of the employer; (2) project employees or
those whose employment has been fixed for a specific project or undertaking, the completion or
termination of which has been determined at the time of their engagement, or those whose work
or service is seasonal in nature and is performed for the duration of the season; and (3) casual
employees or those who are neither regular nor project employees.
Farm workers generally fall under the definition of seasonal employees. We have consistently
held that seasonal employees may be considered as regular employees. Regular seasonal
employees are those called to work from time to time. The nature of their relationship with the
employer is such that during the off season, they are temporarily laid off; but reemployed during
the summer season or when their services may be needed. They are in regular employment
because of the nature of their job, and not because of the length of time they have worked.
The rule, however, is not absolute. Seasonal workers who have worked for one season only
may not be considered regular employees. When such seasonal employees are free to contract
their services with other farm owners, then the former are not regular employees.
For regular employees to be considered as such, the primary standard used is the reasonable
connection between the particular activity they perform and the usual trade or business of the
employer. The deceased was indeed a farm worker who was in the regular employ of petitioner.
From year to year, starting January 1983 up until his death, the deceased had been working on

ATHENA M. SALAS | LABOR CASE DIGEST 2015 43

petitioner's land by harvesting abaca and coconut, processing copra, and clearing weeds. His
employment was continuous in the sense that it was done for more than one harvesting season.
Moreover, no amount of reasoning could detract from the fact that these tasks were necessary
or desirable in the usual business of petitioner.
The other tasks allegedly done by the deceased outside his usual farm work only bolster the
existence of an employer-employee relationship. As found by the SSC, the deceased was a
construction worker in the building and a helper in the bakery, grocery, hardware, and piggery
all owned by petitioner. 63 This fact only proves that even during the off season, the deceased
was still in the employ of petitioner.
Pakyaw workers are considered employees for as long as their employers exercise control over
them. Being the owner of the farm on which the latter worked, petitioner on his own or
through his overseer necessarily had the right to review the quality of work produced by his
laborers
The right of an employee to be covered by the Social Security Act is premised on the existence
of an employer-employee relationship. The Court ruled in favor of private respondent.
24. Concrete Solutions Inc. et al., vs. Cabusas, G.R. No. 177812, June 19, 2013
Facts:
Respondent Arthur Cabusas (respondent) was hired by petitioner Primary Structures
Corporation (PSC) as transit mixer driver for petitioner Concrete Solutions, Inc. (CSI)
Batching Plant Project. The appointment letter dated June 27, 2000, which was signed by
petitioner PSC's Human Resource Division Assistant with respondent's conformity, provided,
among others: that respondent was hired for the period from June 28, 2000 until June 23, 2001;
the status of his employment was that of a project employee and, as such, his employment was
co-terminus with the completion of the project or any phase thereof; that upon completion of the
particular project or phase, he was free to seek other employment of his choice; and, that within
the duration of the work, petitioners shall have the right to terminate his employment without any
liability on their part if his performance did not meet the company standards, or if he violated
petitioners' rules and regulations.
On February 16, 2001, a report reached petitioners that at around 5 o'clock in the afternoon of
that day, respondent, as the driver of Transit Mixer 13, unloaded less than a cubic meter of
concrete mix at Cabancalan, Mandaue City, more than two kilometers away from its project site
located at Wireless, Mandaue City, instead of returning the excess concrete mix to the plant;
and that respondent sold the excess concrete mix to the residents of the place where he
unloaded the same.
On March 7, 2001, petitioners' Administrative Assistant, Carlo E. Gimena, submitted an Incident
Report where he stated that it is a company policy that washing/cleaning of drums must be done
inside petitioners' plant to maximize the utilization of concrete residues for precast use; and

ATHENA M. SALAS | LABOR CASE DIGEST 2015 44

nearly a cubic meter of concrete mix as excess would have been a substantial quantity for such
purpose.
On March 8, 2001, petitioners' Manager, Anastacio G. Ardiente, Jr., required respondent to
explain in writing why he should not be meted with a disciplinary action for the alleged act of
theft or dishonesty under the company's Code of Conduct and Discipline. In his
explanation, 7 respondent stated that he threw away the concrete mix at Cabancalan, Mandaue
City, instead of turning them over to the plant as he will wash the transit mixer at A.S. Fortuna,
Mandaue City. Respondent was meted a three (3)-day suspension effective March 20, 2001 to
March 22, 2001.
On April 19, 2001, petitioners received an information that respondent allegedly took the
company's plastic drum for personal gain. In his Incident Report dated April 20, 2001,
petitioners' Administrative Assistant Gimena reported that at 10:00 a.m. of April 19, 2001,
respondent took an empty plastic drum and hid it in the Transit Mixer 13 he was driving on his
way to deliver concrete mix to Ayala Heights; and that respondent even admitted the
commission of such act which another transit mixer driver could attest to. Gimena
recommended further investigation to include the security guards on duty at the time of the
incident. Respondent was asked to explain why no disciplinary action should be meted on him
for such violation, and to attend the formal investigation on April 26, 2001
In his written explanation, respondent denied the accusation against him and claimed that he
could not had driven the transit mixer out of the company's premises without passing through
the guard house; hence, it was impossible to steal the plastic drum without the knowledge of the
guard. He personally delivered his letter of explanation to the company, but was refused entry
by the security guards. Respondent was placed under preventive suspension from April 20,
2001 to April 27, 2001 pending investigation of his case.
The administrative investigation which was scheduled on April 26, 2001 was postponed to May
4, 2001 and respondent's preventive suspension was extended up to May 5, 2001. Respondent
alleged that after the investigation on May 4, 2001, he and his counsel had asked for the result
of the investigation and were waiting for such result.
While petitioners were deliberating on the violation committed by respondent, they went over the
latter's 201 file and discovered that he appeared not to be registered with the Social Security
System as the SSS number he submitted was that of another person in the name of Alex
Cabusas. Thus, petitioners needed clarifications from respondent, but the latter had been
absent since May 6, 2001. On May 25, 2001, petitioners sent respondent a telegram, to wit:
"You have been absent without official leave since May [6], 2001. Please notify CSI as soon as
possible."
On June 12, 2001, petitioners, thru Manager Ardiente, sent respondent a termination letter.

ATHENA M. SALAS | LABOR CASE DIGEST 2015 45

Petitioners submitted to the Department of Labor and Employment an Establishment


Termination Report indicating that the project where respondent was assigned was already
completed and also that respondent was terminated for being absent without leave (AWOL).
Earlier, however, on May 30, 2001, respondent had filed with Regional Arbitration Branch No.
VII of Cebu City a Complaint for unfair labor practice, illegal dismissal, non-payment of holiday
pay, premium pay for holiday, rest day, night shift premium, separation pay and moral damages
against petitioners. In his position paper, respondent alleged among others: that it was not true
that he went on AWOL. He alleged that when the administrative investigation on his alleged theft
of company property was conducted and terminated on May 4, 2001, his counsel asked to be
furnished a copy of the result of the investigation; that since then, they eagerly waited for such
result, thus they were surprised to receive a telegram on May 26, 2001 where he was said to
have been AWOL since May 5, 2001; that immediately upon receipt of the telegram, respondent
went to petitioners' office, but he was refused entry for the reason that he was AWOL; that there
was no valid cause for his dismissal and petitioners found the lame excuse of declaring him
AWOL if only to create a semblance of justification for his unlawful termination; that he had
previously tendered a follow-up letter for a copy of the resolution of the administrative
investigation that was terminated on May 4, 2001, however, petitioners unceremoniously
refused to receive a copy of the letter he personally delivered, thus his counsel was compelled
to send the letter by way of registered mail on May 29, 2001; that petitioners did not reply to his
letter and did not even furnish his counsel with a copy of the suspension letter; that petitioners'
imputation that he committed dishonest acts was founded on falsehood and fabrications as no
evidence was presented during the so-called administrative hearing, except the self-serving and
perjured statements of petitioners' employees who were merely cajoled into making unfounded
stories. Respondent had prayed for his reinstatement, among others.
Issue
1. Whether or not respondent deliberately abandoned his work which is a just cause for his
dismissal.
2. Whether or not there was illegal dismissal.
Ruling
It is well settled that in termination cases, the burden of proof rests upon the employer to show
that the dismissal was for a just and valid cause, and failure to discharge the same would mean
that the dismissal is not justified and, therefore, illegal. In this case, petitioners claim that
respondent was validly dismissed as he abandoned his work as shown by the following
circumstances, to wit: He did not go back to work on May 6, 2001, i.e., after his preventive
suspension expired on May 5, 2001; he did not report to work despite receipt of the telegram on
May 25, 2001 stating that "he was absent without official leave since May 5, 2001, and to notify
CSI as soon as possible," but instead, through his lawyer, sent a letter asking for a copy of the
result of the investigation; despite not being given the result of the investigation, respondent still
did not bother to report back to work; and the complaint he filed with the LA did not pray for
reinstatement.

ATHENA M. SALAS | LABOR CASE DIGEST 2015 46

To constitute abandonment, two elements must concur, to wit: (1) the failure to report for work or
absence without valid or justifiable reason; and (2) a clear intention to sever the employeremployee relationship, with the second element as the more determinative factor and being
manifested by some overt acts. Abandonment is a matter of intention and cannot lightly be
presumed from certain equivocal acts. To be a valid cause for dismissal for abandonment, there
must be clear proof of deliberate and unjustified intent to sever the employer-employee
relationship. Clearly, the operative act is still the employee's ultimate act of putting an end to his
employment.
We find that the elements of abandonment are lacking. The CA did not commit any reversible
error in affirming the NLRC's decision that respondent was illegally dismissed for petitioners'
failure to substantiate their claim that the former abandoned his work. The circumstances
obtaining in this case do not indicate abandonment.
Respondent explained that his absence from work was due to the fact that he and his counsel
had asked and were waiting for a copy of result of the investigation on his alleged act of theft or
dishonesty conducted on May 4, 2001 but were not given at all. We find his absence from work
not sufficient to establish that he already had intention of abandoning his job. Besides, settled is
the rule that mere absence or failure to report for work is not tantamount to abandonment of
work. Even the failure to report for work after a notice to return to work has been served does
not necessarily constitute abandonment. In fact, when respondent received petitioners'
telegram on May 25, 2001 stating that "he was absent without official leave since May 5, 2001,
and to notify CSI as soon as possible", he went to petitioners premises but was refused entry for
reason that he was AWOL. He also tried to give them a letter dated May 26, 2001 from his
counsel requesting for a copy of the resolution of the investigation conducted on May 4, 2001
but petitioners refused to receive the same which prompted respondent's counsel to send the
letter dated May 26, 2001 to petitioners by registered mail on May 29, 2001. The fact of
petitioners' refusal to receive the letter was stated in that letter but they never refuted the same
which in effect, negates petitioners' claim that respondent did not comply with the telegram sent
to him.
There is no showing of respondent's intent to sever the employer-employee relationship. It is
also notable that when respondent was refused entry to petitioners' premises and the letter of
former's counsel was refused acceptance by the latter, there is already constructive dismissal
which led respondent to seek recourse by filing an illegal dismissal case against petitioners on
May 30, 2001. The proximity of respondent's filing of the complaint from the time he received
the telegram and was refused entry to petitioners' premises showed that he had the least
intention of abandoning his job. Well-settled that the filing by an employee of a complaint for
illegal dismissal with a prayer for reinstatement is proof enough of his desire to return to work,
thus, negating the employer's charge of abandonment.
There is also no merit to petitioners' claim that respondent did not ask for reinstatement. While
in his complaint filed with the LA, respondent failed to ask for reinstatement however, in his

ATHENA M. SALAS | LABOR CASE DIGEST 2015 47

position paper, he specifically prayed for reinstatement. 39 which showed that he had no
intention of abandoning his work.
Petitioners' claim that respondent's violations of company rules also warranted his termination
on account of loss of trust and confidence deserves scant consideration since the latter's
dismissal was not due to those alleged dishonest acts but due to abandonment.
Petitioners contend that respondent was a project employee and the project to which he was
hired was already completed, thus he could not be reinstated anymore.
Project employee is one whose employment has been fixed for a specific project or undertaking
the completion or termination of which has been determined at the time of the engagement of
the employee or where the work or services to be performed is seasonal in nature and the
employment is for the duration of the season. We held that the length of service of a project
employee is not the controlling test of employment tenure but whether or not the employment
has been fixed for a specific project or undertaking the completion or termination of which has
been determined at the time of the engagement of the employee.
We rule that respondent is a project employee. His appointment letter showed that he was hired
as transit mixer driver for the Concrete Solutions, Inc. (CSI) Batching Plant Project for the
period from June 28, 2000 until June 23, 2001. The same letter provided that he was a project
employee whose employment was co-terminus with the completion of the project or any phase
thereof and upon completion of the particular project or phase, he was free to seek other
employment of his choice. There is no evidence showing that respondent did not sign the
conforme part of the appointment letter voluntarily. Hence, respondent was bound by the
provisions in the appointment letter. Moreover, there is also no showing that the period fixed in
the appointment letter was imposed to preclude acquisition of tenurial security by the employee
and should be disregarded for being contrary to public policy as ruled by the NLRC since no
evidence exists on the record to support such conclusion.
Considering that respondent was dismissed prior to the expiration of the duration of his
employment and without a valid or just cause, his termination was therefore illegal. However,
respondent could no longer be reinstated since the project he was assigned to was already
completely finished. However, we find that he is entitled to the salary corresponding to the
unexpired portion of his employment. Respondent is entitled to the payment of his salary from
the time he was not admitted back to work on May 26, 2001 up to June 23, 2001, the expiration
of his employment contract.

ATHENA M. SALAS | LABOR CASE DIGEST 2015 48

25. D.M. Consunji vs. Bello, G.R. No. 159371, July 29, 2013
Facts:
Bello brought a complaint for illegal dismissal and damages against DMCI and/or Rachel
Consunji. In his position paper, he claimed that DMCI had employed him as a mason without
any interruption from February 1, 1990 until October 10, 1997 at an hourly rate of P25.081;
that he had been a very diligent and devoted worker and had served DMCI as best as he
could and without any complaints; that he had never violated any company rules; that his job
as a mason had been necessary and desirable in the usual business or trade of DMCI; that he
had been diagnosed to be suffering from pulmonary tuberculosis, thereby necessitating his
leave of absence; that upon his recovery, he had reported back to work, but DMCI had refused
to accept him and had instead handed to him a termination paper; that he had been
terminated due to "RSD" effective November 5, 1997; that he did not know the meaning of
"RSD" as the cause of his termination; that the cause had not been explained to him; that he
had not been given prior notice of his termination; that he had not been paid separation pay as
mandated by law; that at that time of his dismissal, DMCI's projects had not yet been
completed; and that even if he had been terminated due to an authorized cause, he should
have been given at least one month pay or at least one-half month pay for every year of
service he had rendered, whichever was higher.
In its position paper submitted on March 6, 2000, DMCI contended that Bello had only been a
project employee, as borne out by his contract of employment and appointment papers; that
after his termination from employment, it had complied with the reportorial requirements of the
Department of Labor and Employment (DOLE) pursuant to the mandates of Policy Instruction
No. 20, as revised by Department Order No. 19, series of 1993; and that although his last
project employment contract had been set to expire on October 7, 1997, he had tendered his
voluntary resignation on October 4, 1997 for health reasons that had rendered him incapable of
performing his job, per his resignation letter.
Issues
I. WHETHER OR NOT PRIVATE RESPONDENT WAS A REGULAR EMPLOYEE; AND
II.WHETHER OR NOT PRIVATE RESPONDENT WAS DISMISSED OR VOLUNTARILY
RESIGNED.
Ruling
The provision that governs the first issue is Article 280 of the Labor Code, which is quoted
hereunder as to its relevant part, viz.:
Article 280.Regular and Casual Employment. The provisions of written agreement to the
contrary notwithstanding and regardless of the oral agreement of the parties, an employment

ATHENA M. SALAS | LABOR CASE DIGEST 2015 49

shall be deemed to be regular where the employee has been engaged to perform activities
which are usually necessary and desirable to the usual business or trade of the
employer, except where the employment has been fixed for a specific project or
undertaking the completion or termination of which has been determined at the time of
the engagement of the employee or where the work or service to be performed is seasonal in
nature and the employment is for the duration of the season. (Emphasis supplied)
xxx xxx xxx
A project employee is, therefore, one who is hired for a specific project or undertaking, and the
completion or termination of such project or undertaking has been determined at the time of
engagement of the employee. In the context of the law, Bello was a project employee of DMCI
at the beginning of their employer-employee relationship. The project employment contract they
then entered into clearly gave notice to him at the time of his engagement about his
employment being for a specific project or phase of work. He was also thereby notified of the
duration of the project, and the determinable completion date of the project.
However, the history of Bello's appointment and employment showed that he performed his
tasks as a mason in DMCI's various constructions projects.
We affirm the CA's conclusion that Bello acquired in time the status of a regular employee by
virtue of his continuous work as a mason of DMCI. The work of a mason like him a skilled
workman working with stone or similar material was really related to building or constructing,
and was undoubtedly a function necessary and desirable to the business or trade of one
engaged in the construction industry like DMCI. His being hired as a mason by DMCI in not one,
but several of its projects revealed his necessity and desirability to its construction business.
It is settled that the extension of the employment of a project employee long after the supposed
project has been completed removes the employee from the scope of a project employee and
makes him a regular employee. In this regard, the length of time of the employee's service,
while not a controlling determinant of project employment, is a strong factor in determining
whether he was hired for a specific undertaking or in fact tasked to perform functions vital,
necessary and indispensable to the usual business or trade of the employer. On the other
hand, how DMCI chose to categorize the employment status of Bello was not decisive of his
employment status. What were of consequence in that respect were his actual functions and the
length of his stay with DMCI. Verily, the principal test for determining whether an employee is a
project employee, as distinguished from a regular employee, is whether or not he is assigned to
carry out a specific project or undertaking, the duration and scope of which are specified at the
time he is engaged for the project.
Still, DMCI contends that Bello's services as a mason were deemed necessary and desirable in
its usual business only for the period of time it had taken it to complete the project.
The contention may be correct if each engagement of Bello as a mason over the span of eight
years was to be treated separately. The contention cannot be upheld, however, simply because

ATHENA M. SALAS | LABOR CASE DIGEST 2015 50

his successive re-engagement in order to perform the same kind of work as a mason firmly
manifested the necessity and desirability of his work in DMCI's usual business of construction.
Lastly, DMCI claims that Bello voluntarily resigned from work. It presented his supposed
handwritten resignation letter to support the claim. However, Bello denied having resigned,
explaining that he had signed the letter because DMCI had made him believe that the letter was
for the purpose of extending his sick leave.
In resolving the matter against DMCI, the CA relied on the conclusion by ELA PanganibanOrtiguerra that she could not give credence to the voluntary resignation for health reasons in the
face of Bello's declaration that he had been led to sign the letter to obtain the extension of his
leave of absence due to illness, and on her observation that "the handwriting in the supposed
resignation letter is undeniably different from that of complainant," something that she said
DMCI had not rebutted.
The CA's reliance on the conclusion and finding by ELA Panganiban-Ortiguerra was warranted.
Her observation that the handwriting in the resignation letter was "undeniably different" from that
of Bello could not be ignored or shunted aside simply because she had no expertise to make
such a determination, as the NLRC tersely stated in its decision. To begin with, her supposed
lack of expertise did not appear in the records, rendering the NLRC's statement speculative and
whimsical. If we were now to outrightly discount her competence to make that observation, we
would disturb the time-honored practice of according respect to the findings of the first-line trier
of facts in order to prefer the speculative and whimsical statement of an appellate forum like the
NLRC. Yet, even had the letter been actually signed by him, the voluntariness of the resignation
could not be assumed from such fact alone. His claim that he had been led to believe that the
letter would serve only as the means of extending his sick leave from work should have alerted
DMCI to the task of proving the voluntariness of the resignation. It was obvious that, if his claim
was true, then he did not fully comprehend the import of the letter, rendering the resignation
farcical. The doubt would then be justifiably raised against the letter being at all intended to end
his employment. Under the circumstances, DMCI became burdened with the obligation to prove
the due execution and genuineness of the document as a letter of resignation.
We reiterate that it is axiomatic in labor law that the employer who interposes the defense of
voluntary resignation of the employee in an illegal dismissal case must prove by clear, positive
and convincing evidence that the resignation was voluntary; and that the employer cannot rely
on the weakness of the defense of the employee. The requirement rests on the need to resolve
any doubt in favor of the working man.

26. Colegio Del Santisimo Rosario et al., vs. Rojo, G.R. No. 170388, Sept. 4, 2013
Facts:
Petitioner Colegio del Santisimo Rosario (CSR) hired respondent as a high school teacher on
probationary basis for the school years 1992-1993, 1993-1994 and 1994-1995.

ATHENA M. SALAS | LABOR CASE DIGEST 2015 51

On April 5, 1995, CSR, through petitioner Sr. Zenaida S. Mofada, OP (Mofada), decided not to
renew respondent's services.
Thus, on July 13, 1995, respondent filed a Complaint for illegal dismissal. He alleged that since
he had served three consecutive school years which is the maximum number of terms allowed
for probationary employment, he should be extended permanent employment. Citing paragraph
75 of the 1970 Manual of Regulations for Private Schools (1970 Manual), respondent asserted
that "full-time teachers who have rendered three (3) consecutive years of satisfactory services
shall be considered permanent."
On the other hand, petitioners argued that respondent knew that his Teacher's Contract for
school year 1994-1995 with CSR would expire on March 31, 1995. Accordingly, respondent was
not dismissed but his probationary contract merely expired and was not renewed. Petitioners
also claimed that the "three years" mentioned in paragraph 75 of the 1970 Manual refer to "36
months," not three school years. And since respondent served for only three school years of 10
months each or 30 months, then he had not yet served the "three years" or 36 months
mentioned in paragraph 75 of the 1970 Manual.
Issue
WILL A BASIC EDUCATION (ELEMENTARY) TEACHER HIRED FOR THREE (3)
CONSECUTIVE SCHOOL YEARS AS A PROBATIONARY EMPLOYEE AUTOMATICALLY
AND/OR BY LAW BECOMES A PERMANENT EMPLOYEE UPON COMPLETION OF HIS
THIRD YEAR OF PROBATION NOTWITHSTANDING [A] THE PRONOUNCEMENT OF THIS
HONORABLE COURT IN COLEGIO SAN AGUSTIN V. NLRC, 201 SCRA 398 [1991] THAT A
PROBATIONARY TEACHER ACQUIRES PERMANENT STATUS "ONLY WHEN HE IS
ALLOWED TO WORK AFTER THE PROBATIONARY PERIOD" AND [B] DOLE-DECS-CHEDTESDA ORDER NO. 01, S. 1996 WHICH PROVIDE THAT TEACHERS WHO HAVE SERVED
THE PROBATIONARY PERIOD "SHALL BE MADE REGULAR OR PERMANENT IF ALLOWED
TO WORK AFTER SUCH PROBATIONARY PERIOD?"

Ruling
Cases dealing with employment on probationary status of teaching personnel are not governed
solely by the Labor Code as the law is supplemented, with respect to the period of probation, by
special rules found in the Manual of Regulations for Private Schools (the Manual). With regard
to the probationary period, Section 92 of the 1992 Manual provides:
Section 92.Probationary Period. Subject in all instances to compliance with the
Department and school requirements, the probationary period for academic personnel
shall not be more than three (3) consecutive years of satisfactory service for those in the
elementary and secondary levels, six (6) consecutive regular semesters of satisfactory
service for those in the tertiary level, and nine (9) consecutive trimesters of satisfactory service

ATHENA M. SALAS | LABOR CASE DIGEST 2015 52

for those in the tertiary level where collegiate courses are offered on a trimester basis.
(Emphasis supplied)
In this case, petitioners' teachers who were on probationary employment were made to enter
into a contract effective for one school year. Thereafter, it may be renewed for another school
year, and the probationary employment continues. At the end of the second fixed period of
probationary employment, the contract may again be renewed for the last time.
Such employment for fixed terms during the teachers' probationary period is an accepted
practice in the teaching profession
However, this scheme "of fixed-term contract is a system that operates during the probationary
period and for this reason is subject to Article 281 of the Labor Code," which provides:
. . . The services of an employee who has been engaged on a probationary basis may be
terminated for a just cause or when he fails to qualify as a regular employee in
accordance with reasonable standards made known by the employer to the employee at
the time of his engagement. An employee who is allowed to work after a probationary period
shall be considered a regular employee. [Emphasis supplied]
That teachers on probationary employment also enjoy the protection afforded by Article 281 of
the Labor Code is supported by Section 93 of the 1992 Manual which provides:
Sec. 93.Regular or Permanent Status. Those who have served the probationary period shall
be made regular or permanent. Full-time teachers who have satisfactorily completed their
probationary period shall be considered regular or permanent. (Emphasis supplied)
The above provision clearly provides that full-time teachers become regular or permanent
employees once they have satisfactorily completed the probationary period of three school
years. 37 The use of the term satisfactorily necessarily connotes the requirement for schools to
set reasonable standards to be followed by teachers on probationary employment. For how else
can one determine if probationary teachers have satisfactorily completed the probationary
period if standards therefore are not provided?
As such, "no vested right to a permanent appointment shall accrue until the employee has
completed the prerequisite three-year period necessary for the acquisition of a permanent
status. [However, it must be emphasized that] mere rendition of service for three consecutive
years does not automatically ripen into a permanent appointment. It is also necessary that the
employee be a full-time teacher, and that the services he rendered are satisfactory."
In this case, glaringly absent from petitioners' evidence are the reasonable standards that
respondent was expected to meet that could have served as proper guidelines for purposes of
evaluating his performance. Nowhere in the Teacher's Contract could such standards be
found. Neither was it mentioned that the same were ever conveyed to respondent. Even
assuming that respondent failed to meet the standards set forth by CSR and made known to the

ATHENA M. SALAS | LABOR CASE DIGEST 2015 53

former at the time he was engaged as a teacher on probationary status, still, the termination
was flawed for failure to give the required notice to respondent. This is because Book VI, Rule I,
Section 2 of the IRR of the Labor Code provides:
Section 2.Security of Tenure. (a) In cases of regular employment, the employer shall not
terminate the services of an employee except for just or authorized causes as provided by law,
and subject to the requirements of due process.
(b)The foregoing shall also apply in cases of probationary employment; provided, however, that
in such cases, termination of employment due to failure of the employee to qualify in
accordance with the standards of the employer made known to the former at the time of
engagement may also be a ground for termination of employment.
xxx xxx xxx
(d)In all cases of termination of employment, the following standards of due process shall be
substantially observed:
xxx xxx xxx
If the termination is brought about by the completion of a contract or phase thereof, or by
failure of an employee to meet the standards of the employer in the case of probationary
employment, it shall be sufficient that a written notice is served the employee, within a
reasonable time from the effective date of termination. (Emphasis supplied) ATHCac
Curiously, despite the absence of standards, Mofada mentioned the existence of alleged
performance evaluations 47 in respondent's case. We are, however, in a quandary as to what
could have been the basis of such evaluation, as no evidence were adduced to show the
reasonable standards with which respondent's performance was to be assessed or that he was
informed thereof. Notably too, none of the supposed performance evaluations were presented.
These flaws violated respondent's right to due process. As such, his dismissal is, for all intents
and purposes, illegal.
As a matter of due process, teachers on probationary employment, just like all probationary
employees, have the right to know whether they have met the standards against which their
performance was evaluated. Should they fail, they also have the right to know the reasons
therefore.
It should be pointed out that absent any showing of unsatisfactory performance on the part of
respondent, it can be presumed that his performance was satisfactory, especially taking into
consideration the fact that even while he was still more than a year into his probationary
employment, he was already designated Prefect of Discipline. In such capacity, he was able to
uncover the existence of a drug syndicate within the school and lessen the incidence of drug
use therein. Yet despite respondent's substantial contribution to the school, petitioners chose to
disregard the same and instead terminated his services; while most of those who were involved

ATHENA M. SALAS | LABOR CASE DIGEST 2015 54

in drug activities within the school were punished with a slap on the wrist as they were merely
made to write letters promising that the incident will not happen again.
Mofada would also have us believe that respondent chose to resign as he feared for his life,
thus, the school's decision not to renew his contract. However, no resignation letter was
presented. Besides, this is contrary to respondent's act of immediately filing the instant case
against petitioners.

27. Philippine Spring Water Resources Inc vs. Court of Appeals, GR No. 205278, June 11,
2014
Facts:
Mahilum was hired by petitioner as Vice President for Sales and Marketing. Sometime in Nov.
2004, the inauguration of the company's Bulacan plant was to be celebrated, and he was
designated to oversee the affair. However on the inaugural day, he was not able to supervise
the program due to tending to some visitors of the company, and according to him, he delegated
the task to Evangelista, the VP of Admin and Finance.
Mahilum's attention was called however when the CEO became furious because his presence
was not acknowledged nor was he called to deliver his speech during the event. According to
the MCs Lua's speech was labelled optional in the program because Lua declined participation
in the program due to being busy. Mahilum was placed on preventive suspension the following
day. An investigation was conducted. Upon expiration of the suspension, he tried to go back to
work but was prevented from entering. He received a Memorandum terminating his services,
and was made to sign a Quitclaim after receiving an amount. He then filed for illegal dismissal.
Ruling:
Mahilum was a regular employee.
Based on the agreement of the parties, he was to be on probationary status for 6 months and
may be extended permanent appointment if he could satisfactorily perform his duties. It was the
theory of the company that the probationary status of Mahilum did not end for failing to perform
his duties. Mahilum insists that he was a regular employee since at the time of termination he
had been serving for 8 months. Pursuant to Article 281 of the Labor Code, "an employee who is
allowed to work after the probationary period shall be considered a regular employee." Mahilum
was therefore a regular employee.
A probationary employee, like a regular employee, enjoys security of tenure. In cases of
probationary employment, however, aside from just or authorized causes of termination, an
additional ground is provided under Article 281 of the Labor Code, that is, the probationary
employee may also be terminated for failure to qualify as a regular employee in accordance with
reasonable standards made known by the employer to the employee at the time of the
engagement. Thus, the services of an employee who has been engaged on probationary basis
may be terminated for any of the following: (1) a just or (2) an authorized cause and (3) when he

ATHENA M. SALAS | LABOR CASE DIGEST 2015 55

fails to qualify as a regular employee in accordance with reasonable standards prescribed by


the employer.
Mahilum was illegally dismissed
According to the petitioners, Mahilums behavior during the inauguration/party was allegedly
tantamount to: 1] serious misconduct, as displayed by a drinking binge with his own visitors
causing the shame and humiliation of Lua; and 2] willful disobedience, as shown by his refusal
to carry out legitimate orders. His designation as the chairman of the whole affair did not form
part of his duty as a supervisor. Mahilum was engaged to supervise the sales and marketing
aspects of PSWRIs Bulacan Plant. Verily, the charge of loss of trust and confidence had no leg
to stand on, as the act complained of was not work-related.
Accordingly, Philippine Spring Water Resources Inc. is hereby ordered to pay Juvenstein B.
Mahilum, his separation pay, full backwages inclusive of his basic salary, proportionate 13th
month pay, and unused leave credits, to be computed based on his salary at the time of his
illegal termination and attorneys fees.

28. Concrete Solutions Inc. et al., vs. Cabusas, G.R. No. 177812, June 19, 2013
Facts:
Respondent Arthur Cabusas (respondent) was hired by petitioner Primary Structures
Corporation (PSC) as transit mixer driver for petitioner Concrete Solutions Inc. (CSI) Batching
Plant Project. The appointment letter3 dated June 27, 2000, which was signed by petitioner
PSC's Human Resource Division Assistant with respondent's conformity, provided, among
others: that respondent was hired for the period from June 28, 2000 until June 23, 2001; the
status of his employment was that of a project employee and, as such, his employment was coterminus with the completion of the project or any phase thereof; that upon completion of the
particular project or phase, he was free to seek other employment of his choice; and, that within
the duration of the work, petitioners shall have the right to terminate his employment without any
liability on their part if his performance did not meet the company standards, or if he violated
petitioners' rules and regulations.
On February 16, 2001, a report reached petitioners that at around 5 oclock in the afternoon of
that day, respondent, as the driver of Transit Mixer 13, unloaded less than a cubic meter of
concrete mix at Cabancalan, Mandaue City, more than two kilometers away from its project site
located at Wireless, Mandaue City, instead of returning the excess concrete mix to the plant;
and that respondent sold the excess concrete mix to the residents of the place where he
unloaded the same.
Petitioners' Manager required respondent to explain in writing why he should not be meted with
a disciplinary action for the alleged act of theft or dishonesty under the companys Code of
Conduct and Discipline. In his explanation, respondent stated that he threw away the concrete
mix at Cabancalan, Mandaue City, instead of turning them over to the plant as he will wash the

ATHENA M. SALAS | LABOR CASE DIGEST 2015 56

transit mixer at A.S. Fortuna, Mandaue City. Respondent was meted a three (3)-day suspension
effective March 20, 2001 to March 22, 2001.
While petitioners were deliberating on the violation committed by respondent, they went over the
latter's 201 file and discovered that he appeared not to be registered with the Social Security
System as the SSS number he submitted was that of another person in the name of Alex
Cabusas. Thus, petitioners needed clarifications from respondent, but the latter had been
absent since May 6, 2001. On May 25, 2001, petitioners sent respondent a telegram,14 to wit:
"You have been absent without official leave since May [6], 2001. Please notify CSI as soon as
possible." On June 12, 2001, petitioners, thru Manager Ardiente, sent respondent a termination
letter. Earlier, however, on May 30, 2001, respondent had filed with Regional Arbitration Branch
No. VII of Cebu City a Complaint for unfair labor practice, illegal dismissal, non-payment of
holiday pay, premium pay for holiday, rest day, night shift premium, separation pay and moral
damages against petitioners.
Ruling:
1st issue: Whether respondent deliberately abandoned his work which is a just cause for
his dismissal or whether he was illegally dismissed by petitioners.
In this case, petitioners claim that respondent was validly dismissed as he abandoned his work.
To constitute abandonment, two elements must concur, to wit: (1) the failure to report for work or
absence without valid or justifiable reason; and (2) a clear intention to sever the employeremployee relationship, with the second element as the more determinative factor and being
manifested by some overt acts. Abandonment is a matter of intention and cannot lightly be
presumed from certain equivocal acts. To be a valid cause for dismissal for abandonment, there
must be clear proof of deliberate and unjustified intent to sever the employer-employee
relationship. Clearly, the operative act is still the employee's ultimate act of putting an end to his
employment.
Respondent explained that his absence from work was due to the fact that he and his counsel
had asked and were waiting for a copy of result of the investigation on his alleged act of theft or
dishonesty conducted on May 4, 2001 but were not given at all. We find his absence from work
not sufficient to establish that he already had intention of abandoning his job. Besides, settled is
the rule that mere absence or failure to report for work is not tantamount to abandonment of
work.
Petitioners' claim that respondent's violations of company rules also warranted his termination
on account of loss of trust and confidence deserves scant consideration since the latter's
dismissal was not due to those alleged dishonest acts but due to abandonment.
2nd Issue: whether or not he was a project employee.
Petitioners contend that respondent was a project employee and the project to which he was
hired was already completed, thus he could not be reinstated anymore.
Project employee is one whose employment has been fixed for a specific project or undertaking
the completion or termination of which has been determined at the time of the engagement of
the employee or where the work or services to be performed is seasonal in nature and the
employment is for the duration of the season. We held that the length of service of a project

ATHENA M. SALAS | LABOR CASE DIGEST 2015 57

employee is not the controlling test of employment tenure but whether or not the employment
has been fixed for a specific project or undertaking the completion or termination of which has
been determined at the time of the engagement of the employee.
We rule that respondent is a project employee. His appointment letter showed that he was hired
as transit mixer driver for the Concrete Solutions Inc. (CSI) Batching Plant Project for the
period from June 28, 2000 until June 23, 2001. The same letter provided that he was a project
employee whose employment was co-terminus with the completion of the project or any phase
thereof and upon completion of the particular project or phase, he was free to seek other
employment of his choice. There is no evidence showing that respondent did not sign the
conforme part of the appointment letter voluntarily. Hence, respondent was bound by the
provisions in the appointment letter. Moreover, there is also no showing that the period fixed in
the appointment letter was imposed to preclude acquisition of tenurial security by the employee
and should be disregarded for being contrary to public policy as ruled by the NLRC since no
evidence exists on the record to support such conclusion.
Considering that respondent was dismissed prior to the expiration of the duration of his
employment and without a valid or just cause, his termination was therefore illegal. However,
respondent could no longer be reinstated since the project he was assigned to was already
completely finished. However, we find that he is entitled to the salary corresponding to the
unexpired portion of his employment. Respondent is entitled to the payment of his salary from
the time he was not admitted back to work on May 26, 2001 up to June 23, 2001, the expiration
of his employment contract.

29. D.M. Consunji vs. Bello, G.R. No. 159371, July 29, 2013
FACTS:
Bello brought a complaint for illegal dismissal and damages against DMCI. Bello claimed that
DMCI had employed him as a mason without any interruption from February 1, 1990 until
October 10, 1997; that his job as a mason had been necessary and desirable in the usual
business or trade of DMCI; that he had been diagnosed to be suffering from pulmonary
tuberculosis, thereby necessitating his leave of absence; that upon his recovery, he had
reported back to work, but DMCI had refused to accept him and had instead handed to him a
termination paper effective November 5, 1997.
DMCI contended that Bello had only been a project employee, as borne out by his contract of
employment and appointment papers; and that although his last project employment contract
had been set to expire on October 7, 1997, he had tendered his voluntary resignation on

ATHENA M. SALAS | LABOR CASE DIGEST 2015 58

October 4, 1997 for health reasons that had rendered him incapable of performing his job, per
his resignation letter.
ISSUE:
WHETHER OR NOT BELLO WAS A REGULAR EMPLOYEE OF D.M. CONSUNJI
RULING:
A project employee is one who is hired for a specific project or undertaking, and the completion
or termination of such project or undertaking has been determined at the time of engagement of
the employee. In the context of the law, Bello was a project employee of DMCI at the beginning
of their employer-employee relationship. The project employment contract they then entered
into clearly gave notice to him at the time of his engagement about his employment being for a
specific project or phase of work. He was also thereby notified of the duration of the project, and
the determinable completion date of the project.
However, the history of Bellos appointment and employment showed that he performed his
tasks as a mason in DMCIs various constructions projects. Based on the foregoing, we affirm
the CAs conclusion that Bello acquired in time the status of a regular employee by virtue of his
continuous work as a mason of DMCI. The work of a mason like him a skilled workman
working with stone or similar material was really related to building or constructing, and was
undoubtedly a function necessary and desirable to the business or trade of one engaged in the
construction industry like DMCI. His being hired as a mason by DMCI in not one, but several of
its projects revealed his necessity and desirability to its construction business.
It is settled that the extension of the employment of a project employee long after the supposed
project has been completed removes the employee from the scope of a project employee and
makes him a regular employee. In this regard, the length of time of the employees service,
while not a controlling determinant of project employment, is a strong factor in determining
whether he was hired for a specific undertaking or in fact tasked to perform functions vital,
necessary and indispensable to the usual business or trade of the employer. On the other hand,
how DMCI chose to categorize the employment status of Bello was not decisive of his
employment status. What were of consequence in that respect were his actual functions and the
length of his stay with DMCI. Verily, the principal test for determining whether an employee
is a project employee, as distinguished from a regular employee, is whether or not he is
assigned to carry out a specific project or undertaking, the duration and scope of which are
specified at the time he is engaged for the project.
Still, DMCI contends that Bellos services as a mason were deemed necessary and desirable in
its usual business only for the period of time it had taken it to complete the project. The
contention may be correct if each engagement of Bello as a mason over the span of eight years
was to be treated separately. The contention cannot be upheld, however, simply because his
successive reengagement in order to perform the same kind of work as a mason firmly
manifested the necessity and desirability of his work in DMCIs usual business of construction.

ATHENA M. SALAS | LABOR CASE DIGEST 2015 59

30. Colegio Del Santisimo Rosario et al., vs. Rojo, G.R. No. 170388, Sept. 4, 2013
FACTS:
Colegio del Santisimo Rosario (CSR) hired Rojo as a high school teacher on probationary basis
for the school years 1992-1993, 1993-1994 and 1994-1995. On April 5, 1995, CSR, through Sr.
Zenaida S. Mofada, OP (Mofada), decided not to renew respondents services.
Rojo filed a Complaint for illegal dismissal. He alleged that since he had served three
consecutive school years which is the maximum number of terms allowed for probationary
employment, he should be extended permanent employment. Citing paragraph 75 of the 1970
Manual of Regulations for Private Schools (1970 Manual), Rojo asserted that "full- time teachers
who have rendered three (3) consecutive years of satisfactory services shall be considered
permanent."
On the other hand, CSR argued that respondent knew that his Teachers Contract for school
year 1994-1995 with CSR would expire on March 31, 1995. Accordingly, Rojo was not
dismissed but his probationary contract merely expired and was not renewed. Petitioners
maintain that upon the expiration of the probationary period, both the school and the respondent
were free to renew the contract or let it lapse. Petitioners insist that a teacher hired for three
consecutive years as a probationary employee does not automatically become a regular
employee upon completion of his third year of probation. It is the positive act of the school the
hiring of the teacher who has just completed three consecutive years of employment on
probation for the next school year that makes the teacher a regular employee of the school.
ISSUE:
WHETHER OR NOT ROJO BECAME A PERMANENT EMPLOYEE UPON COMPLETION OF
HIS THIRD YEAR PROBATION.
RULING:
That teachers on probationary employment also enjoy the protection afforded by Article 281 of
the Labor Code is supported by Section 93 of the 1992 Manual which provides:
Sec. 93. Regular or Permanent Status. - Those who have served the probationary period shall
be made regular or permanent. Full-time teachers who have satisfactorily completed their
probationary period shall be considered regular or permanent. (Emphasis supplied)
The above provision clearly provides that full-time teachers become regular or
permanent employees once they have satisfactorily completed the probationary period of
three school years. The use of the term satisfactorily necessarily connotes the requirement for
schools to set reasonable standards to be followed by teachers on probationary employment.

ATHENA M. SALAS | LABOR CASE DIGEST 2015 60

For how else can one determine if probationary teachers have satisfactorily completed the
probationary period if standards therefor are not provided?
As such, "no vested right to a permanent appointment shall accrue until the employee has
completed the prerequisite three-year period necessary for the acquisition of a permanent
status. [However, it must be emphasized that] mere rendition of service for three
consecutive years does not automatically ripen into a permanent appointment. It is also
necessary that the employee be a full-time teacher, and that the services he rendered are
satisfactory."
In Mercado, this Court, speaking through J. Brion, held that:
xxx
When fixed-term employment is brought into play under the above probationary period rules, the
situation as in the present case may at first blush look muddled as fixed-term employment is
in itself a valid employment mode under Philippine law and jurisprudence. The conflict, however,
is more apparent than real when the respective nature of fixed-term employment and of
employment on probationary status are closely examined.
The fixed-term character of employment essentially refers to the period agreed upon between
the employer and the employee; employment exists only for the duration of the term and ends
on its own when the term expires. In a sense, employment on probationary status also refers to
a period because of the technical meaning "probation" carries in Philippine labor law a
maximum period of six months, or in the academe, a period of three years for those engaged in
teaching jobs. Their similarity ends there, however, because of the overriding meaning
that being "on probation" connotes, i.e., a process of testing and observing the character
or abilities of a person who is new to a role or job.
Understood in the above sense, the essentially protective character of probationary status for
management can readily be appreciated. But this same protective character gives rise to the
countervailing but equally protective rule that the probationary period can only last for a specific
maximum period and under reasonable, well-laid and properly communicated standards.
Otherwise stated, within the period of the probation, any employer move based on the
probationary standards and affecting the continuity of the employment must strictly conform
to the probationary rules.
x x x If we pierce the veil, so to speak, of the parties so-called fixed-term employment contracts,
what undeniably comes out at the core is a fixed-term contract conveniently used by the school
to define and regulate its relations with its teachers during their probationary period.
In the same case, this Court has definitively pronounced that "in a situation where the
probationary status overlaps with a fixed-term contract not specifically used for the fixed term it
offers, Article 281 should assume primacy and the fixed-period character of the contract must
give way."

ATHENA M. SALAS | LABOR CASE DIGEST 2015 61

An example given of a fixed-term contract specifically used for the fixed term it offers is a
replacement teacher or a reliever contracted for a period of one year to temporarily take the
place of a permanent teacher who is on leave. The expiration of the relievers fixed-term
contract does not have probationary status implications as he or she was never employed on
probationary basis. This is because his or her employment is for a specific purpose with
particular focus on the term. There exists an intent to end his or her employment with the school
upon expiration of this term.
However, for teachers on probationary employment, in which case a fixed term contract is not
specifically used for the fixed term it offers, it is incumbent upon the school to have not only set
reasonable standards to be followed by said teachers in determining qualification for regular
employment, the same must have also been communicated to the teachers at the start of the
probationary period, or at the very least, at the start of the period when they were to be applied.
These terms, in addition to those expressly provided by the Labor Code, would serve as the just
cause for the termination of the probationary contract. The specific details of this finding of just
cause must be communicated to the affected teachers as a matter of due process. Corollarily,
should the teachers not have been apprised of such reasonable standards at the time
specified above, they shall be deemed regular employees.
In Tamsons Enterprises, Inc. v. Court of Appeals,we held that "[t]he law is clear that in all cases
of probationary employment, the employer shall [convey] to the employee the standards under
which he will qualify as a regular employee at the time of his engagement. Where no standards
are made known to the employee at that time, he shall be deemed a regular employee.
In this case, glaringly absent from petitioners evidence are the reasonable standards that
respondent was expected to meet that could have served as proper guidelines for purposes of
evaluating his performance. Nowhere in the Teachers Contract could such standards be found.
Neither was it mentioned that the same were ever conveyed to respondent.

31. Herrera-Manaois vs. St. Scholasticas College, GR No. 188914, December 11, 2013
Facts:
SSC, situated in the City of Manila, is a private educational institution offering elementary,
secondary, and tertiary education. Manaois graduated from SSC in October 1992 with a degree
in Bachelor of Arts in English. In 1994, she returned to her alma mater as a part-time English
teacher. After taking a leave of absence for one year, she was again rehired by SSC for the
same position. Four years into the service, she was later on recommended by her Department
Chairperson to become a full-time faculty member of the English Department.
Manaois thus applied for a position as full-time instructor for school year 2000-2001.

ATHENA M. SALAS | LABOR CASE DIGEST 2015 62

She mentioned in her application letter that she had been taking the course Master of Arts in
English Studies, Major in Creative Writing, at the University of the Philippines, Diliman (UP); that
she was completing her master's thesis; and that her oral defense was scheduled for June
2000. In a reply letter 4 dated 17 April 2000, the Dean of Arts and Sciences informed her of the
SSC Administrative Council's approval of her application. SSC hired her as a probationary fulltime faculty member with the assigned task of instructor for the school year 2000-2001. Her
probationary employment continued for a total of three consecutive years.
Because of the forthcoming completion of her third year of probationary employment, Manaois
wrote the Dean of Arts and Sciences requesting an extension of her teaching load for the school
year 2003-2004. Manaois eventually received a letter from the Dean of College and
Chairperson of the Promotions and Permanency Board officially informing her of the board's
decision not to renew her contract.
Issue:
Whether the completion of a master's degree is required in order for a tertiary level educator to
earn the status of permanency in a private educational institution.
Ruling:
Probationary employment refers to the trial stage or period during which the employer examines
the competency and qualifications of job applicants, and determines whether they are qualified
to be extended permanent employment status. Such an arrangement affords an employer the
opportunity before the full force of the guarantee of security of tenure comes into play to
fully scrutinize and observe the fitness and worth of probationers while on the job and to
determine whether they would become proper and efficient employees. It also gives the
probationers the chance to prove to the employer that they possess the necessary qualities and
qualifications to meet reasonable standards for permanent employment.
Viewed next to the statements and actions of Manaois i.e., the references to obtaining a
master's degree in her application letter, in the subsequent correspondences between her and
SSC, and in the letter seeking the extension of a teaching load for the school year 2003-2004;
and her submission of certifications from UP and from her thesis adviser we find that there is
indeed substantial evidence proving that she knew about the necessary academic qualifications
to obtain the status of permanency.
At this juncture, we reiterate the rule that mere completion of the three-year probation, even with
an above-average performance, does not guarantee that the employee will automatically
acquire a permanent employment status. It is settled jurisprudence that the probationer can only
qualify upon fulfillment of the reasonable standards set for permanent employment as a member
of the teaching personnel.

ATHENA M. SALAS | LABOR CASE DIGEST 2015 63

Thus, pursuant to the 1992 Manual, private educational institutions in the tertiary level may
extend "full-time faculty" status only to those who possess, inter alia, a master's degree in the
field of study that will be taught. This minimum requirement is neither subject to the prerogative
of the school nor to the agreement between the parties. For all intents and purposes, this
qualification must be deemed impliedly written in the employment contracts between private
educational institutions and prospective faculty members. The issue of whether probationers
were informed of this academic requirement before they were engaged as probationary
employees is thus no longer material, as those who are seeking to be educators are presumed
to know these mandated qualifications. In the light of the failure of Manaois to satisfy the
academic requirements for the position, she may only be considered as a part-time instructor
pursuant to Section 45 of the 1992 Manual.
32. Universal Robina Sugar Milling Corp., vs. Acibo et al., GR No. 186439, January 15,
2014
Facts:
URSUMCO is a domestic corporation engaged in the sugar cane milling business; Cabati is
URSUMCO's Business Unit General Manager. The complainants were employees of
URSUMCO. They were hired on various dates (between February 1988 and April 1996) and on
different capacities, 8 i.e., drivers, crane operators, bucket hookers, welders, mechanics,
laboratory attendants and aides, steel workers, laborers, carpenters and masons, among
others. At the start of their respective engagements, the complainants signed contracts of
employment for a period of one (1) month or for a given season. URSUMCO repeatedly hired
the complainants to perform the same duties and, for every engagement, required the latter to
sign new employment contracts for the same duration of one month or a given season.
The complainants filed before the LA complaints for regularization, entitlement to the benefits
under the existing Collective Bargaining Agreement (CBA), and attorney's fees.
Issue:
Whether or not respondents are regular employees of URSUMCO
Ruling:
We find the respondents to be regular seasonal employees of URSUMCO.
Seasonal employment operates much in the same way as project employment, albeit it involves
work or service that is seasonal in nature or lasting for the duration of the season. As with
project employment, although the seasonal employment arrangement involves work that is
seasonal or periodic in nature, the employment itself is not automatically considered seasonal
so as to prevent the employee from attaining regular status. To exclude the asserted "seasonal"
employee from those classified as regular employees, the employer must show that: (1) the

ATHENA M. SALAS | LABOR CASE DIGEST 2015 64

employee must be performing work or services that are seasonal in nature; and (2) he had been
employed for the duration of the season. Hence, when the "seasonal" workers are continuously
and repeatedly hired to perform the same tasks or activities for several seasons or even after
the cessation of the season, this length of time may likewise serve as badge of regular
employment. In fact, even though denominated as "seasonal workers," if these workers are
called to work from time to time and are only temporarily laid off during the off-season, the law
does not consider them separated from the service during the off-season period. The law simply
considers these seasonal workers on leave until re-employed.
The nature of the employment depends on the nature of the activities to be performed by the
employee, considering the nature of the employer's business, the duration and scope to be
done, and, in some cases, even the length of time of the performance and its continued
existence. In light of the above legal parameters laid down by the law and applicable
jurisprudence, the respondents are neither project, seasonal nor fixed-term employees, but
regular seasonal workers of URSUMCO.
1

The respondents were made to perform various tasks that did not at all pertain to any
specific phase of URSUMCO's strict milling operations that would ultimately cease upon
completion of a particular phase in the milling of sugar; rather, they were tasked to
perform duties regularly and habitually needed in URSUMCO's operations during the
milling season.

The respondents were regularly and repeatedly hired to perform the same tasks year
after year.

33. DIONARTO Q. NOBLEJAS, v. ITALIAN MARITIME ACADEMY PHILS., INC., CAPT.


NICOLO S. TERREI, RACELI B. FERREZ AND MA. TERESA R. MENDOZA . G.R. No.
207888, June 09, 2014.

FACTS:
Petitioner Noblejas filed a complaint for illegal dismissal, tax refund, moral and exemplary
damages, non-payment of 13th month pay, food, gasoline and schooling allowances, health
insurance, monetized leave, and attorneys fees, against Italian Maritime Academy Phils., Inc.
(IMAPI) and its officers.
Record shows that Procerfina SA. Terrei, IMAPI President, wrote a letter to Noblejas informing
him that he had been appointed as training instructor/assessor of the company on a contractual
basis for a period of three (3) months. After the expiration of the 3-month period, IMAPI hired
Noblejas anew as training instructor/assessor with the same salary rate, but no written contract
was
drawn
for
his
rehiring.

ATHENA M. SALAS | LABOR CASE DIGEST 2015 65

The absence of a written contract to cover the renewal of his employment became Noblejas
major concern. To address all his apprehensions, he wrote Capt. Terrei a letter requesting that a
new contract be executed.
Noblejas averred that the company did not act on his letter-request, so he sought an audience
with Capt. Terrei. During the meeting, an altercation between them ensued. He claimed that
after that incident, Capt. Terrei instructed Ferrez, his secretary, to dismiss him from
employment. He claimed that when he asked from Ferrez for a copy of his old contract, she
allegedly replied, No, you better pack up all your things now and go, you are now dismissed
and you are no longer part in this office clearly, you are terminated from this day on.
Respondents submitted that they could not be adjudged guilty of illegal dismissal because there
was no positive and overt act of dismissing Noblejas from employment.
Respondents presented a different version of what took place. According to respondents,
Noblejas got angry, hurled invectives against Ferrez and even threatened to file a case against
them after she had relayed to him the response of Capt. Terrei to his letter to the effect that
there was no previous agreement to grant him tax refund, health insurance and food, schooling
and gasoline allowances and that he had to render at least one year of service before the
company could decide whether to accord him the status of a regular employee. The following
day, March 17, 2010, he did not report for work anymore and filed the complaint against them.
The Labor Arbiter found that Noblejas was illegally dismissed from his employment, and
awarded him limited backwages.
NLRC reversed the LA decision.
ISSUES:
1. WON petitioner is a contractual employee.
2. WON petitioner was illegally dismissed.
3. WON petitioner is entitled to his money claims.

RULING:
As to the

first

issue,

petitioner

Noblejas

is

regular

employee

of

IMAPI.

Pursuant to Article 280 of the Labor Code, there are two kinds of regular employees, namely: (1)
those who are engaged to perform activities which are usually necessary or desirable in the
usual business or trade of the employer; and (2) those who have rendered at least one year of
service, whether continuous or broken, with respect to the activities in which they are employed.
Regular employees are further classified into (1) regular employees - by nature of work and (2)
regular employees - by years of service. The former refers to those employees who perform a
particular function which is necessary or desirable in the usual business or trade of the

ATHENA M. SALAS | LABOR CASE DIGEST 2015 66

employer, regardless of their length of service; while the latter refers to those employees who
have been performing the job, regardless of its nature thereof, for at least a year.
In the case at bench, Noblejas was employed by IMAPI as a training instructor/assessor for a
period of three (3) months effective May 20, 2009. After the end of the 3-month period, he was
rehired by IMAPI for the same position and continued to work as such until March 16, 2010.
There is no dispute that the work of Noblejas was necessary or desirable in the business or
trade of IMAPI, a training and assessment center for seamen and officers of vessels. Moreover,
such continuing need for his services is sufficient evidence of the necessity and indispensability
of his services to IMAPIs business. Taken in this light, Noblejas had indeed attained the status
of a regular employee at the time he ceased to report for work on March 17, 2010.
As to the second issue, petitioner was not illegally dismissed.
The rule in labor cases is that the employer has the burden of proving that the termination was
for a valid or authorized cause. It is likewise incumbent upon the employees, however, that they
should first establish by competent evidence the fact of their dismissal from employment. It is
an age-old rule that the one who alleges a fact has the burden of proving it and the proof should
be
clear,
positive
and
convincing.
Mere
allegation
is
not
evidence.
Aside from his mere assertion, no corroborative and competent evidence was adduced by
Noblejas to substantiate his claim that he was dismissed from employment. The record is bereft
of any indication that he was prevented from returning to work or otherwise deprived of any
work assignment. It is also noted that no evidence was submitted to show that respondent
Ferrez, the secretary of Capt. Terrei, was actually authorized by IMAPI to terminate the
employment of the companys employees or that Ferrez was indeed instructed by Capt. Terrei to
dismiss him from employment.
Respondents refusal to grant complainants demands does not constitute an overt act of
dismissal. On the contrary, it is rather the apparent disinterest of complainant to continue his
employment with respondent company that may be considered a covert act that severed his
employment when the latter did not grant the litany of his demands.
As to the third issue, the SC sustains the LA in granting Noblejas proportionate 13th month pay
covering the period of January 1, 2010 to March 15, 2010 in the aggregate amount of
P15,625.00.
Furthermore, the respondents should accept him back and reinstate him to his former position.
However, there can be no payment of backwages under the principle of no work, no pay.

34. OMNI HAULING SERVICES, INC., v. BERNARDO BON, ROBERTO TORTOLES, et al.
G.R. No. 199388, September 03, 2014
FACTS:
Petitioner Omni was awarded a one (1) year service contract by the local government of Quezon
City to provide garbage hauling services for the period July 1, 2002 to June 30, 2003. For this
purpose, Omni hired respondents as garbage truck drivers and paleros who were then paid on a

ATHENA M. SALAS | LABOR CASE DIGEST 2015 67

per

trip

basis.

When the service contract was renewed for another year, or for the period July 1, 2003 to June
30, 2004, petitioners required each of the respondents to sign employment contracts which
provided that they will be re-hired only for the duration of the same period. However,
respondents refused to sign the employment contracts, claiming that they were regular
employees since they were engaged to perform activities which were necessary and desirable
to Omnis usual business or trade.
For this reason, Omni terminated the employment of respondents which, in turn, resulted in the
filing of cases for illegal dismissal, nonpayment of Emergency Cost of Living Allowance
(ECOLA) and 13th month pay, and actual, moral, and exemplary damages.
LA ruled in favor of petitioners, finding that respondents were not illegally dismissed.
The LA found that respondents were informed that their employment will be limited for a specific
period of one year and was co-terminus with the service contract with the Quezon City
government. Thus, respondents were not regular but merely project employees whose hiring
was solely dependent on the aforesaid service contract.
The NLRC affirmed in toto the decision of the Labor Arbiter.
The Court of Appeals reversed and set aside the decision of the NLRC and held that the
respondents are regular employees and that NLRC failed to consider the glaring fact that no
contract of employment exists to support petitioners allegation that respondents are fixed-term
or project employees.
ISSUES:
WON the respondents are project employees.
RULING:
The respondents are not project employees. They are regular employees.
Article 280 of the Labor Code distinguishes a project employee from a regular employee in
this wise:
Art. 280. Regular and casual employment. The provisions of written agreement to the contrary
notwithstanding and regardless of the oral agreement of the parties, an employment shall be
deemed to be regular where the employee has been engaged to perform activities which are
usually necessary or desirable in the usual business or trade of the employer, except where
the employment has been fixed for a specificproject or undertaking the completion or
termination of which has been determined at the time of the engagement of the
employee or where the work or service to be performed is seasonal in nature and the
employment
is
for
the
duration
of
the
season.
A project employee is assigned to a project which begins and ends at determined or

ATHENA M. SALAS | LABOR CASE DIGEST 2015 68

determinable times. Unlike regular employees who may only be dismissed for just and/or
authorized causes under the Labor Code, the services of employees who are hired as project
employees may be lawfully terminated at the completion of the project.
According to jurisprudence, the principal test for determining whether particular employees are
properly characterized as project employees as distinguished from regular employees, is
whether or not the employees were assigned to carry out a specific project or undertaking, the
duration (and scope) of which were specified at the time they were engaged for that project. The
project could either be (1) a particular job or undertaking that is within the regular or usual
business of the employer company, but which is distinct and separate, and identifiable as such,
from the other undertakings of the company; or (2) a particular job or undertaking that is not
within the regular business of the corporation. In order to safeguard the rights of workers
against the arbitrary use of the word project to prevent employees from attaining a regular
status, employers claiming that their workers are project employees should not only
prove that the duration and scope of the employment was specified at the time they were
engaged,
but
also
that
there
was
indeed
a
project.cr
Hence, even though the absence of a written contract does not by itself grant regular status to
respondents, such a contract is evidence that respondents were informed of the duration and
scope of their work and their status as project employees. In this case, where no other
evidence was offered, the absence of an employment contract puts into serious question
whether the employees were properly informed at the onset of their employment status
as project employees. It is doctrinally entrenched that in illegal dismissal cases, the employer
has the burden of proving with clear, accurate, consistent and convincing evidence that a
dismissal was valid.
In this case, records are bereft of any evidence to show that respondents were made to sign
employment contracts explicitly stating that they were going to be hired as project employees,
with the period of their employment to be co-terminus with the original period of Omnis service
contract with the Quezon City government. Neither is petitioners allegation that respondents
were duly apprised of the project-based nature of their employment supported by any other
evidentiary proof.Thus, the logical conclusion is that respondents were not clearly and
knowingly informed of their employment status as mere project employees, with the duration
and scope of the project specified at the time they were engaged. As such, the presumption of
regular employment should be accorded in their favor pursuant to Article 280 of the Labor Code
which provides that [employees] who have rendered at least one year of service, whether such
service is continuous or broken [ as respondents in this case ] shall be considered as
[regular employees] with respect to the activity in which [they] are employed and [their]
employment shall continue while such activity actually exists. Add to this the obvious fact that
respondents have been engaged to perform activities which are usually necessary or desirable
in the usual business or trade of Omni, i.e., garbage hauling, thereby confirming the strength of
the
aforesaid
conclusion.
The determination that respondents are regular and not merely project employees resultantly

ATHENA M. SALAS | LABOR CASE DIGEST 2015 69

means that their services could not have been validly terminated at the expiration of the project,
or, in this case, the service contract of Omni with the Quezon City government. As regular
employees, it is incumbent upon petitioners to establish that respondents had been dismissed
for a just and/or authorized cause. However, petitioners failed in this respect; hence,
respondents
were
illegally
dismissed.

35. Hacienda Ledd vs. Villegas, GR No. 179654, Sept. 22, 2014
Facts:
Villegas is an employee at the Hacienda Leddy as early as 1960, when it was still named
Hacienda Teresa. Later on named Hacienda Leddy owned by Ricardo Gamboa Sr., the same
was succeeded by his son Ricardo Gamboa, Jr. During his employment up to the time of his
dismissal, Villegas performed sugar farming job 8 hours a day, 6 days a week work,
continuously for not less than 302 days a year, and for which services he was paid P45.00 per
day. He likewise worked in petitioner's coconut lumber business where he was paid P34.00 a
day for 8 hours work.
On June 9, 1993, Gamboa went to Villegas' house and told him that his services were no longer
needed without prior notice or valid reason. Hence, Villegas filed the instant complaint for illegal
dismissal.
Gamboa, on the other hand, denied having dismissed Villegas but admitted in his earlier
position paper that Villegas indeed worked with the said farm owned by his father, doing casual
and odd jobs until the latter's death in 1993. He was even given the benefit of occupying a
small portion of the land where his house was erected. He, however, maintained that Villegas
ceased working at the farm as early as 1992, contrary to his allegation that he was dismissed.
However, later, Gamboa apparently retracted and instead insisted that the farm records reveal
that the only time Villegas rendered service for the hacienda was only in the year 1993,
specifically February 9, 1993 and February 11, 1993 when he was contracted by the farm to cut
coconut lumber which were given to regular workers for the repairs of their houses. Gamboa
added that they informed Villegas that they need the property, hence, they requested that he
vacate it, but he refused. Thus, Gamboa surmised that Villegas filed the instant complaint to
gain leverage so he would not be evicted from the land he is occupying. He further argued that
during his employment, Villegas was paid in accordance with the rate mandated by law and that
his claim for illegal dismissal was merely a fabrication as he was the one who opted not to work.
Issue

ATHENA M. SALAS | LABOR CASE DIGEST 2015 70

1. Whether or nor there was an ER-EE relationship.


2. Whether or not there was illegal dismissal.
Ruling
A perusal of the records would show that respondent, having been employed in the subject
Hacienda while the same was still being managed by petitioner's father until the latter's death in
1993, is undisputed as the same was even admitted by Gamboa in his earlier pleadings. While
refuting that Villegas was a regular employee, petitioner however failed to categorically deny
that Villegas was indeed employed in their hacienda albeit he insisted that Villegas was merely
a casual employee doing odd jobs.
The rule is long and well settled that, in illegal dismissal cases like the one at bench, the burden
of proof is upon the employer to show that the employee's termination from service is for a just
and valid cause. The employer's case succeeds or fails on the strength of its evidence and not
the weakness of that adduced by the employee, in keeping with the principle that the scales of
justice should be tilted in favor of the latter in case of doubt in the evidence presented by them.
Often described as more than a mere scintilla, the quantum of proof is substantial evidence
which is understood as such relevant evidence as a reasonable mind might accept as adequate
to support a conclusion, even if other equally reasonable minds might conceivably opine
otherwise.
In the instant case, if we are to follow the length of time that Villegas had worked with the
Gamboas, it should be more than 20 years of service. Even Gamboa admitted that by act of
generosity and compassion, Villegas was given a privilege of erecting his house inside the
hacienda during his employment. While it may indeed be an act of good will on the part of the
Gamboas, still, such act is usually done by the employer either out of gratitude for the
employee's service or for the employer's convenience as the nature of the work calls for it.
Indeed, petitioner's length of service is an indication of the regularity of his employment. Even
assuming that he was doing odd jobs around the farm, such long period of doing said odd jobs
is indicative that the same was either necessary or desirable to petitioner's trade or business.
Owing to the length of service alone, he became a regular employee, by operation of law, one
year after he was employed.
Article 280 of the Labor Code, describes a regular employee as one who is either (1) engaged
to perform activities which are necessary or desirable in the usual business or trade of the
employer; and (2) those casual employees who have rendered at least one year of service,
whether continuous or broken, with respect to the activity in which he is employed.
While length of time may not be the controlling test to determine if Villegas is indeed a regular
employee, it is vital in establishing if he was hired to perform tasks which are necessary and
indispensable to the usual business or trade of the employer. If it was true that Villegas worked
in the hacienda only in the year 1993, specifically February 9, 1993 and February 11, 1993, why
would then he be given the benefit to construct his house in the hacienda? More significantly,

ATHENA M. SALAS | LABOR CASE DIGEST 2015 71

petitioner admitted that Villegas had worked in the hacienda until his father's demise. Clearly,
even assuming that Villegas' employment was only for a specific duration, the fact that he was
repeatedly re-hired over a long period of time shows that his job is necessary and indispensable
to the usual business or trade of the employer.
Gamboa likewise argued that Villegas was paid on a piece-rate basis. However, payment on a
piece-rate basis does not negate regular employment. "The term 'wage' is broadly defined in
Article 97 of the Labor Code as remuneration or earnings, capable of being expressed in terms
of money whether fixed or ascertained on a time, task, piece or commission basis. Payment by
the piece is just a method of compensation and does not define the essence of the relations."
We are likewise unconvinced that it was Villegas who suddenly stopped working. Considering
that he was employed with the Gamboas for more than 20 years and was even given a place to
call his home, it does not make sense why Villegas would suddenly stop working therein for no
apparent reason. To justify a finding of abandonment of work, there must be proof of a
deliberate and unjustified refusal on the part of an employee to resume his employment. The
burden of proof is on the employer to show an unequivocal intent on the part of the employee to
discontinue employment. Mere absence is not sufficient. It must be accompanied by manifest
acts unerringly pointing to the fact that the employee simply does not want to work anymore.
Petitioner failed to discharge this burden. Other than the self-serving declarations in the affidavit
of his employee, petitioner did not adduce proof of overt acts of Villegas showing his intention to
abandon his work. Abandonment is a matter of intention; it cannot be inferred or presumed from
equivocal acts. On the contrary, the filing of the instant illegal dismissal complaint negates any
intention on his part to sever their employment relationship. The delay of more than 1 year in
filing the instant illegal dismissal case likewise is non-issue considering that the complaint was
filed within a reasonable period during the three-year period provided under Article 291 of the
Labor Code. As aptly observed by the appellate court, Villegas appeared to be without
educational attainment. He could not have known that he has rights as a regular employee that
is protected by law.
Thus, notwithstanding any agreements to the contrary, what determines whether a certain
employment is regular or casual is not the will and word of the employer, to which the desperate
worker often accedes, much less the procedure of hiring the employee or the manner of paying
his salary. It is the nature of the activities performed in relation to the particular business or
trades considering all circumstances, and in some cases the length of time of its performance
and its continued existence.
All these having discussed, as a regular worker, Villegas is entitled to security of tenure under
Article 279 of the Labor Code and can only be removed for cause. We found no valid cause
attending to his dismissal and found also that his dismissal was without due process.
Article 277 (b) of the Labor Code provides that:

ATHENA M. SALAS | LABOR CASE DIGEST 2015 72

. . . Subject to the constitutional right of workers to security of tenure and their right to be
protected against dismissal except for a just and authorized cause and without prejudice to the
requirement of notice under Article 283 of this Code, the employer shall furnish the worker
whose employment is sought to be terminated a written notice containing a statement of the
causes for termination and shall afford the latter ample opportunity to be heard and to defend
himself with the assistance of his representative if he so desires in accordance with company
rules and regulations promulgated pursuant to guidelines set by the Department of Labor and
Employment. . . .
The failure of the petitioner to comply with these procedural guidelines renders its dismissal of
Villegas illegal. An illegally dismissed employee should be entitled to either reinstatement if
viable, or separation pay if reinstatement is no longer viable, plus backwages in either instance.
Considering that reinstatement is no longer feasible because of strained relations between the
employee and the employer, separation pay should be granted. The basis for computing
separation pay is usually the length of the employee's past service, while that for backwages is
the actual period when the employee was unlawfully prevented from working. 25 It should be
emphasized, however, that the finality of the illegal dismissal decision becomes the reckoning
point. In allowing separation pay, the final decision effectively declares that the employment
relationship ended so that separation pay and backwages are to be computed up to that point.
The decision also becomes a judgment for money from which another consequence flows
the payment of interest in case of delay.

36. FVR Skills & Services Exponents Inc. et al., vs. Seva, et al., GR No. 200857, Oct. 22,
2014

Facts:
The twenty-eight (28) respondents in this case were employees of petitioner FVR Skills and
Services Exponents, Inc. (petitioner), an independent contractor engaged in the business of
providing janitorial and other manpower services to its clients. As early as 1998, some of the
respondents had already been under the petitioner's employ.
On April 21, 2008, the petitioner entered into a Contract of Janitorial Service (service
contract) with Robinsons Land Corporation (Robinsons). Both agreed that the petitioner shall
supply janitorial, manpower and sanitation services to Robinsons Place Ermita Mall for a period
of one year from January 1, 2008 to December 31, 2008. Pursuant to this, the respondents
were deployed to Robinsons.
Halfway through the service contract, the petitioner asked the respondents to execute individual
contracts which stipulated that their respective employments shall end on December 31, 2008,
unless earlier terminated.

ATHENA M. SALAS | LABOR CASE DIGEST 2015 73

The petitioner and Robinsons no longer extended their contract of janitorial services.
Consequently, the petitioner dismissed the respondents as they were project employees whose
duration of employment was dependent on the petitioner's service contract with Robinsons.
The respondents responded to the termination of their employment by filing a complaint for
illegal dismissal with the NLRC. They argued that they were not project employees; they were
regular employees who may only be dismissed for just or authorized causes. The respondents
also asked for payment of their unpaid wage differential, 13th month pay differential, service
incentive leave pay, holiday pay and separation pay.
Issues:
1. Whether or not the respondents regular or project employees.
2. Whether or not the employment contract they belatedly sign valid.
3. Whether or not there was valid dismissal.

Ruling:
The
respondents
employees, not project employees.

are

regular

Article 280 (now Article 294) of the Labor Code governs the determination of whether an
employee is a regular or a project employee.
Under this provision, there are two kinds of regular employees, namely: (1) those who were
engaged to perform activities which are usually necessary or desirable in the usual business or
trade of the employer; and (2) those casual employees who became regular after one year of
service, whether continuous or broken, but only with respect to the activity for which they have
been hired.
We distinguish these two types of regular employees from a project employee, or one whose
employment was fixed for a specific project or undertaking, whose completion or termination
had been determined at the time of engagement.
A careful look at the factual circumstances of this case leads us to the legal conclusion that the
respondents are regular and not project employees.
The primary standard in determining regular employment is the reasonable
connection between the particular activity performed by the employee and the employer's
business or trade. This connection can be ascertained by considering the nature of the work
performed and its relation to the scheme of the particular business, or the trade in its entirety. 25

ATHENA M. SALAS | LABOR CASE DIGEST 2015 74

Guided by this test, we conclude that the respondents' work as janitors, service crews and
sanitation aides, are necessary or desirable to the petitioner's business of providing
janitorial and manpower services to its clients as an independent contractor.
Also, the respondents had already been working for the petitioner as early as 1998. Even
before the service contract with Robinsons, the respondents were already under the
petitioner's employ. 26 They had been doing the same type of work and occupying the
same positions from the time they were hired and until they were dismissed in January
2009. The petitioner did not present any evidence to refute the respondents' claim that from the
time of their hiring until the time of their dismissal, there was no gap in between the projects
where they were assigned to. The petitioner continuously availed of their services by constantly
deploying them to its clients.
Lastly, under Department Order (DO) 18-02, the applicable labor issuance to the petitioner's
case, the contractor or subcontractor is considered as the employer of the contractual employee
for purposes of enforcing the provisions of the Labor Code and other social legislation.
DO 18-02 grants contractual employees all the rights and privileges due a regular employee,
including the following: (a) safe and healthful working conditions; (b) labor standards such as
service incentive leave, rest days, overtime pay, holiday pay, 13th month pay and
separation pay; (c) social security and welfare benefits; (d) self-organization, collective
bargaining and peaceful concerted action; and (e) security of tenure.
In this light, we thus conclude that although the respondents were assigned as contractual
employees to the petitioner's various clients, under the law, they remain to be the petitioner's
regular employees, who are entitled to all the rights and benefits of regular employment.
The
contracts,
signed, are voidable.

respondents'
which

were

employment
belatedly

The records show that at the time of the respondents' dismissal, they had already been
continuously working for the petitioner for more than a year. Despite this, they never signed any
employment contracts with the petitioner, except the contracts they belatedly signed when the
petitioner's own contract of janitorial services with Robinsons neared expiration.
As already discussed, for an employee to be validly categorized as a project employee, it is
necessary that the specific project or undertaking had been identified and its period and
completion date determined and made known to the employee at the time of his
engagement. This provision ensures that the employee is completely apprised of the terms of
his hiring and the corresponding rights and obligations arising from his undertaking. Notably, the
petitioner's service contract with Robinsons was from January 1 to December 31, 2008. The
respondents were only asked to sign their employment contracts for their deployment with
Robinsons halfway through 2008, when the petitioner's service contract was about to expire.

ATHENA M. SALAS | LABOR CASE DIGEST 2015 75

We find the timing of the execution of the respondents' respective employment contracts to be
indicative of the petitioner's calculated plan to evade the respondents' right to security of tenure,
to ensure their easy dismissal as soon as the Robinsons' contract expired. The attendant
circumstances cannot but raise doubts as to the petitioner's good faith.
If the petitioner really intended the respondents to be project employees, then the
contracts should have been executed right from the time of hiring, or when the
respondents were first assigned to Robinsons, not when the petitioner's service contract
was winding up. The terms and conditions of the respondents' engagement should have been
disclosed and explained to them from the commencement of their employment. The petitioner's
failure to do so supports the conclusion that it had been in bad faith in evading the respondents'
right to security of tenure.
Moreover, under Article 1390 of the Civil Code, contracts where the consent of a party was
vitiated by mistake, violence, intimidation, undue influence or fraud, are voidable or annullable.
The petitioner's threat of non-payment of the respondents' salaries clearly amounted to
intimidation. Under this situation, and the suspect timing when these contracts were executed,
we rule that these employment contracts were voidable and were effectively questioned when
the respondents filed their illegal dismissal complaint.
The
dismissed.

respondents

were

illegally

To be valid, an employee's dismissal must comply with the substantive and procedural
requirements of due process. Substantively, a dismissal should be supported by a just or
authorized cause. Procedurally, the employer must observe the twin notice and hearing
requirements in carrying out an employee's dismissal.
The petitioner argues that these substantive and procedural requisites do not apply to the
respondents' case since they were employed under fixed term contracts. According to the
petitioner, the respondents' employment contracts lapsed by operation of law as the necessary
consequence of the termination and non-renewal of its service contract with Robinsons.
Because of this, there was no illegal dismissal to speak of, only contract expiration.
We do not agree with the petitioner.
Having already determined that the respondents are regular employees and not project
employees, and that the respondents' belated employment contracts could not be given any
binding effect for being signed under duress, we hold that illegal dismissal took place when the
petitioner failed to comply with the substantive and procedural due process requirements of the
law.

ATHENA M. SALAS | LABOR CASE DIGEST 2015 76

The petitioner also asserts that the respondents' subsequent absorption by Robinsons' new
contractors Fieldmen Janitorial Service Corporation and Altaserv negates their illegal
dismissal. This reasoning is patently erroneous. The charge of illegal dismissal was made only
against the petitioner which is a separate juridical entity from Robinsons' new contractors; it
cannot escape liability by riding on the goodwill of others.
By law, the petitioner must bear the legal consequences of its violation of the respondents' right
to security of tenure. The facts of this case show that since the respondents' hiring, they had
been under the petitioner's employ as janitors, service crews and sanitation aides. Their
services had been continuously provided to the petitioner without any gap. Notably, the
petitioner never refuted this allegation of the respondents. Further, there was no
allegation that the petitioner went out of business after the non-renewal of the
Robinsons' service contract. Thus, had it not been for the respondents' dismissal, they would
have been deployed to the petitioner's other existing clients.
With regard to the award of separation pay, we agree with the CA's finding that this litigation
resulted to strained relations between the petitioner and the respondents. Thus, we also affirm
the CA's ruling that instead of reinstatement, the respondents should be paid their respective
separation pays equivalent to one (1) month pay for every year of service.
We cannot give credence to the petitioner's assertion that under Section 10 of DO 18-02, the
respondents are not entitled to separation pay because their employment was terminated due to
the completion of the project where they had been engaged. This provision must be construed
with the rest of DO 18-02's other provisions.
As earlier pointed out, Section 7 of DO 18-02 treats contractual employees as the independent
contractor's regular employees for purposes of enforcing the Labor Code and other social
legislation laws. Consequently, a finding of regular employment entitles them to the rights
granted to regular employees, particularly the right to security of tenure and to separation pay.
Thus, a holistic reading of DO 18-02, guides us to the conclusion that Section 10 only pertains
to contractual employees who are really project employees. They are not entitled to separation
pay since the end of the project for which they had been hired necessarily results to the
termination of their employment. On the other hand, we already found that the respondents are
the petitioner's regular employees. Thus, their illegal dismissal entitles them to backwages and
reinstatement or separation pay, in case reinstatement is no longer feasible.

ATHENA M. SALAS | LABOR CASE DIGEST 2015 77

13. MANAGEMENT PREROGATIVE


37. Barba vs Liceo De Cagayan University, GR No. 193857, November 28, 2012
Facts

Petitioner Dr. Ma. Mercedes L. Barba was the Dean of the College of Physical Therapy of
respondent Liceo de Cagayan University, Inc., a private educational institution with school
campus located at Carmen, Cagayan de Oro City.
Petitioner started working for respondent on July 8, 1993 as medical officer/school physician for
a period of one school year or until March 31, 1994. In July 1994, she was chosen by
respondent to be the recipient of a scholarship grant to pursue a three-year residency training in
Rehabilitation Medicine at the Veterans Memorial Medical Center (VMMC).
After completing her residency training with VMMC in June 1997, petitioner returned to continue
working for respondent. She was appointed as Acting Dean of the College of Physical Therapy
and at the same time designated as Doctor-In-Charge of the Rehabilitation Clinic of the Rodolfo
N. Pelaez Hall, City Memorial Hospital.

ATHENA M. SALAS | LABOR CASE DIGEST 2015 78

On June 19, 2002, petitioner's appointment as Doctor-In-Charge of the Rehabilitation Clinic was
renewed and she was appointed as Dean of the College of Physical Therapy by respondent's
President, Dr. Jose Ma. R. Golez.
Petitioner accepted her appointment and assumed the position of Dean of the College of
Physical Therapy. In the school year 2003 to 2004, the College of Physical Therapy suffered a
dramatic decline in the number of enrollees from a total of 1,121 students in the school year
1995 to 1996 to only 29 students in the first semester of school year 2003 to 2004. This
worsened in the next year or in school year 2004 to 2005 where a total of only 20 students
enrolled.
Due to the low number of enrollees, respondent decided to freeze the operation of the College
of Physical Therapy indefinitely. Respondent's President Dr. Rafaelita Pelaez-Golez wrote
petitioner a letter dated March 16, 2005 informing her that her services as dean of the said
college will end at the close of the school year. Thereafter, the College of Physical Therapy
ceased operations on March 31, 2005, and petitioner went on leave without pay starting on April
9, 2005. Subsequently, respondent's Executive Vice President, Dr. Marian M. Lerin, through Dr.
Glory S. Magdale, respondent's Vice President for Academic Affairs, sent petitioner a
letter dated April 27, 2005 instructing petitioner to return to work on June 1, 2005 and report to
Ma. Chona Palomares, the Acting Dean of the College of Nursing, to receive her teaching load
and assignment as a full-time faculty member in that department for the school year 20052006. TIEHSA
In reply, petitioner informed Dr. Lerin that she had not committed to teach in the College of
Nursing and that as far as she can recall, her employment is not dependent on any teaching
load. She then requested for the processing of her separation benefits in view of the closure of
the College of Physical Therapy. She did not report to Palomares on June 1, 2005.
On June 8, 2005, petitioner followed up her request for separation pay and other benefits but Dr.
Lerin insisted that she report to Palomares; otherwise, sanctions will be imposed on her. Thus,
petitioner through counsel wrote Dr. Golez directly, asking for her separation pay and other
benefits.
On June 21, 2005, Dr. Magdale wrote petitioner a letter directing her to report for work and to
teach her assigned subjects on or before June 23, 2005. Otherwise, she will be dismissed from
employment on the ground of abandonment. Petitioner, through counsel, replied that teaching in
the College of Nursing is in no way related to her scholarship and training in the field of
rehabilitation medicine. Petitioner added that coercing her to become a faculty member from her
position as College Dean is a great demotion which amounts to constructive dismissal.
Dr. Magdale sent another letter to petitioner on June 24, 2005 ordering her to report for work as
she was still bound by the Scholarship Contract to serve respondent for two more years. But
petitioner did not do so. Hence, on June 28, 2005, Dr. Magdale sent petitioner a notice
terminating her services on the ground of abandonment.
Meanwhile, on June 22, 2005, prior to the termination of her services, petitioner filed a
complaint before the Labor Arbiter for illegal dismissal, payment of separation pay and

ATHENA M. SALAS | LABOR CASE DIGEST 2015 79

retirement benefits against respondent, Dr. Magdale and Dr. Golez. She alleged that her
transfer to the College of Nursing as a faculty member is a demotion amounting to constructive
dismissal.
Respondent claimed that petitioner was not terminated and that it was only petitioner's
appointment as College Dean in the College of Physical Therapy that expired as a necessary
consequence of the eventual closure of the said college. Respondent further averred that
petitioner's transfer as full-time professor in the College of Nursing does not amount to
constructive dismissal since the transfer was without loss of seniority rights and without
diminution of pay. Also, respondent added that pursuant to the Scholarship Contract, petitioner
was still duty bound to serve respondent until 2007 in whatever position related to her studies
the school desires.

Issus
The decisive issue in the present petition is whether petitioner was an employee or a corporate
officer of respondent university. Resolution of this issue resolves the question of whether the
appellate court was correct in ruling that the Labor Arbiter and the NLRC had no jurisdiction over
petitioner's complaint for constructive dismissal against respondent.

Ruling
After a careful review and examination of the records, we find that the CA's previous ruling that
petitioner was respondent's employee and not a corporate officer is supported by the totality of
the evidence and more in accord with law and prevailing jurisprudence.
Corporate officers are elected or appointed by the directors or stockholders, and are those who
are given that character either by the Corporation Code or by the corporation's by-laws. Section
25 of the Corporation Code enumerates corporate officers as the president, the secretary, the
treasurer and such other officers as may be provided for in the by-laws.

In declaring petitioner a corporate officer, the CA considered respondent's by-laws and gave
weight to the certifications of respondent's secretary attesting to the resolutions of the board of
directors appointing the various academic deans for the School Years 1991-2002 and 20022005, including petitioner. However, an assiduous perusal of these documents does not
convince us that petitioner occupies a corporate office position in respondent university.

On the other hand, there were two board resolutions that were presented appointed the various
academic deans in the university which included petitioner.
In respondent's by-laws, there are four officers specifically mentioned, namely, a president, a
vice president, a secretary and a treasurer. In addition, it is provided that there shall be other

ATHENA M. SALAS | LABOR CASE DIGEST 2015 80

appointive officials, a College Director and heads of departments whose appointments,


compensations, powers and duties shall be determined by the board of directors. It is worthy to
note that a College Dean is not among the corporate officers mentioned in respondent's bylaws. Petitioner, being an academic dean, also held an administrative post in the university but
not a corporate office as contemplated by law. Petitioner was not directly elected nor appointed
by the board of directors to any corporate office but her appointment was merely approved by
the board together with the other academic deans of respondent university in accordance with
the procedure prescribed in respondent's Administrative Manual. The act of the board of
directors in approving the appointment of petitioner as Dean of the College of Therapy did not
make her a corporate officer of the corporation. CIAHDT
Moreover, the CA, in its amended decision erroneously equated the position of a College
Director to that of a College Dean thereby concluding that petitioner is an officer of respondent.

It bears stressing that the appointive officials mentioned in Article V of respondent's by-laws are
not corporate officers under the contemplation of the law. Though the board of directors may
create appointive positions other than the positions of corporate officers, the persons occupying
such positions cannot be deemed as corporate officers as contemplated by Section 25 of
the Corporation Code. On this point, the SEC Opinion dated November 25, 1993 is instructive:
Thus, pursuant to the above provision (Section 25 of the Corporation Code), whoever are the
corporate officers enumerated in the by-laws are the exclusive Officers of the corporation and
the Board has no power to create other Offices without amending first the corporate Bylaws. However, the Board may create appointive positions other than the positions of
corporate Officers, but the persons occupying such positions are not considered as
corporate officers within the meaning of Section 25 of the Corporation Code and are not
empowered to exercise the functions of the corporate Officers, except those functions
lawfully delegated to them. Their functions and duties are to be determined by the Board of
Directors/Trustees.
But even assuming that a College Director may be considered a corporate officer of respondent,
a review of the records as well as the other documents submitted by the parties fails to
persuade that petitioner was the "College Director" mentioned in the by-laws of respondent.
Nowhere in petitioner's appointment letter was it stated that petitioner was designated as the
College Director or that petitioner was to assume the functions and duties of a College Director.
Neither can it be inferred in respondent's by-laws that a dean of a college is the same as a
College Director of respondent. Respondent's lone surviving incorporating director Yolanda
Rollo even admitted that no College Director has ever been appointed by respondent.
Undoubtedly, petitioner is not a College Director and she is not a corporate officer but an
employee of respondent. Applying the four-fold test concerning (1) the selection and
engagement of the employee; (2) the payment of wages; (3) the power of dismissal; (4) the
employer's power to control the employee with respect to the means and methods by which the
work is to be accomplished, it is clear that there exists an employer-employee relationship
between petitioner and respondent. Records show that petitioner was appointed to her position

ATHENA M. SALAS | LABOR CASE DIGEST 2015 81

as Dean by Dr. Golez, the university president and was paid a salary of P32,500 plus
transportation allowance. It was evident that respondent had the power of control over petitioner
as one of its deans. It was also the university president who informed petitioner that her services
as Dean of the College of Physical Therapy was terminated effective March 31, 2005 and she
was subsequently directed to report to the Acting Dean of the College of Nursing for assignment
of teaching load. ETHaDC

Thus, petitioner, being an employee of respondent, her complaint for illegal/constructive


dismissal against respondent was properly within the jurisdiction of the Labor Arbiter and the
NLRC. Article 217 of the Labor Code provides:

ART. 217.Jurisdiction of Labor Arbiters and the Commission. (a) Except as otherwise
provided under this Code,
the Arbiters shall have original and exclusive jurisdiction to hear and decide . . . the following
cases involving all workers, whether agricultural or non-agricultural:

1.Unfair labor practice cases;


2.Termination disputes;
3.If accompanied with a claim for reinstatement, those cases that workers may file involving
wage, rates of pay, hours of work and other terms and conditions of employment;
4.Claims for actual, moral, exemplary and other forms of damages arising from the employeremployee relations;
5.Cases arising from any violation of Article 264 of this Code, including questions involving the
legality of strikes and lockouts; and
6.Except claims for Employees Compensation, Social Security, Medicare and maternity
benefits, all other claims arising from employer-employee relations, including those of persons
in domestic or household service, involving an amount exceeding five thousand pesos
(P5,000.00) regardless of whether accompanied with a claim for reinstatement.
(b)The Commission shall have exclusive appellate jurisdiction over all cases decided by Labor
Arbiters.
xxx xxx xxx
Moreover, we agree with the CA's earlier pronouncement that since respondent actively
participated in the proceedings before the Labor Arbiter and the NLRC, it is already estopped
from belatedly raising the issue of lack of jurisdiction. In this case, respondent filed position
papers and other supporting documents to bolster its defense before the labor tribunals but in all
these pleadings, the issue of lack of jurisdiction was never raised. It was only in its

ATHENA M. SALAS | LABOR CASE DIGEST 2015 82

Supplemental Petition filed before the CA that respondent first brought the issue of lack of
jurisdiction. We have consistently held that while jurisdiction may be assailed at any stage, a
party's active participation in the proceedings will estop such party from assailing its jurisdiction.
It is an undesirable practice of a party participating in the proceedings and submitting his case
for decision and then accepting the judgment, only if favorable, and attacking it for lack of
jurisdiction, when adverse.

Under Section 6, Rule 10 of the 1997 Rules of Civil Procedure, as amended, governing
supplemental pleadings, the court "may" admit supplemental pleadings, such as the
supplemental petition filed by respondent before the appellate court, but the admission of these
pleadings remains in the sound discretion of the court. Nevertheless, we have already found no
credence in respondent's claim that petitioner is a corporate officer, consequently, the alleged
lack of jurisdiction asserted by respondent in the supplemental petition is bereft of merit.
On the issue of constructive dismissal, we agree with the Labor Arbiter and the appellate court's
earlier ruling that petitioner was not constructively dismissed. Petitioner's letter of appointment
specifically appointed her as Dean of the College of Physical Therapy and Doctor-in-Charge of
the Rehabilitation Clinic "for a period of three years effective July 1, 2002 unless sooner
revoked for valid cause or causes." Evidently, petitioner's appointment as College Dean was for
a fixed term, subject to reappointment and revocation or termination for a valid cause. When
respondent decided to close its College of Physical Therapy due to drastic decrease in
enrollees, petitioner's appointment as its College Dean was validly revoked and her subsequent
assignment to teach in the College of Nursing was justified as it is still related to her scholarship
studies in Physical Therapy.
In constructive dismissal cases, the employer has the burden of proving that its conduct and
action or the transfer of an employee are for valid and legitimate grounds such as genuine
business necessity. Particularly, for a transfer not to be considered a constructive dismissal, the
employer must be able to show that such transfer is not unreasonable, inconvenient, or
prejudicial to the employee. In this case, petitioner's transfer was not unreasonable,
inconvenient or prejudicial to her. On the contrary, the assignment of a teaching load in the
College of Nursing was undertaken by respondent to accommodate petitioner following the
closure of the College of Physical Therapy. Respondent further considered the fact that
petitioner still has two years to serve the university under the Scholarship Contract.
Petitioner's subsequent transfer to another department or college is not tantamount to demotion
as it was a valid transfer. There is therefore no constructive dismissal to speak of. That
petitioner ceased to enjoy the compensation, privileges and benefits as College Dean was but a
logical consequence of the valid revocation or termination of such fixed-term position. Indeed, it
would be absurd and unjust for respondent to maintain a deanship position in a college or
department that has ceased to exist. Under the circumstances, giving petitioner a teaching load
in another College/Department that is related to Physical Therapy thus enabling her to serve
and complete her remaining two years under the Scholarship Contract is a valid exercise of
management prerogative on the part of respondent.

ATHENA M. SALAS | LABOR CASE DIGEST 2015 83

38. Best Wear Garments vs De Lemos, GR No. 191281, November 28, 2012
Facts
Petitioner Best Wear Garments is a sole proprietorship represented by its General Manager
Alex Sitosta. Respondents Cecile M. Ocubillo and Adelaida B. De Lemos were hired as sewers
on piece-rate basis by petitioners on October 27, 1993 and July 12, 1994, respectively.
On May 20, 2004, De Lemos filed a complaint for illegal dismissal with prayer for backwages
and other accrued benefits, separation pay, service incentive leave pay and attorney's fees. A
similar complaint was filed by Ocubillo on June 10, 2004. Both alleged in their position paper
that in August 2003, Sitosta arbitrarily transferred them to other areas of operation of petitioner's
garments company, which they said amounted to constructive dismissal as it resulted in less
earnings for them.
De Lemos claimed that after two months in her new assignment, she was able to adjust but
Sitosta again transferred her to a "different operation where she could not earn [as] much as
before because by-products require long period of time to finish." She averred that the reason
for her transfer was her refusal "to render [overtime work] up to 7:00 p.m." Her request to be
returned to her previous assignment was rejected and she was "constrained not to report for
work as Sitosta had become indifferent to her since said transfer of operation." She further
alleged that her last salary was withheld by petitioner company.

On her part, Ocubillo alleged that her transfer was precipitated by her having "incurred
excessive absences since 2001." Her absences were due to the fact that her father became
very sick since 2001 until his untimely demise on November 9, 2003; aside from this, she
herself became very sickly. She claimed that from September to October 2003, Sitosta assigned
her to different machines "whichever is available" and that "there were times, she could not earn
for a day because there was no available machine to work for [sic]." Sitosta also allegedly
required her to render overtime work up to 7:00 p.m. which she refused "because she was only
paid up to 6:25 p.m."
Petitioners denied having terminated the employment of respondents who supposedly
committed numerous absences without leave (AWOL). They claimed that sometime in February
2004, De Lemos informed Sitosta that due to personal problem, she intends to resign from the
company. She then demanded the payment of separation pay. In March 2004, Ocubillo likewise
intimated her intention to resign and demanded separation pay. Sitosta explained to both De
Lemos and Ocubillo that the company had no existing policy on granting separation pay, and
hence he could not act on their request. De Lemos never reported back to work since March
2004, while Ocubillo failed to report for work from October 2004 to the present.
As to the allegation of respondents that the reason for their transfer was their refusal to render
overtime work until 7:00 p.m., petitioners asserted that respondents are piece-rate workers and
hence they are not paid according to the number of hours worked.

ATHENA M. SALAS | LABOR CASE DIGEST 2015 84

Issue
Were the respondents constructively dismissed or were their transfers of assignment an
exercise of management prerogative?

Ruling

The right of employees to security of tenure does not give them vested rights to their positions
to the extent of depriving management of its prerogative to change their assignments or to
transfer them. Thus, an employer may transfer or assign employees from one office or area of
operation to another, provided there is no demotion in rank or diminution of salary, benefits, and
other privileges, and the action is not motivated by discrimination, made in bad faith, or effected
as a form of punishment or demotion without sufficient cause.
We rule that respondents were not constructively dismissed.
Being piece-rate workers assigned to individual sewing machines, respondents' earnings
depended on the quality and quantity of finished products. That their work output might have
been affected by the change in their specific work assignments does not necessarily imply that
any resulting reduction in pay is tantamount to constructive dismissal. Workers under piece-rate
employment have no fixed salaries and their compensation is computed on the basis of
accomplished tasks. As admitted by respondent De Lemos, some garments or by-products took
a longer time to finish so they could not earn as much as before. Also, the type of sewing jobs
available would depend on the specifications made by the clients of petitioner company. Under
these circumstances, it cannot be said that the transfer was unreasonable, inconvenient or
prejudicial to the respondents. Such deployment of sewers to work on different types of
garments as dictated by present business necessity is within the ambit of management
prerogative which, in the absence of bad faith, ill motive or discrimination, should not be
interfered with by the courts.
The records are bereft of any showing of clear discrimination, insensibility or disdain on the part
of petitioners in transferring respondents to perform a different type of sewing job. It is unfair to
charge petitioners with constructive dismissal simply because the respondents insist that their
transfer to a new work assignment was against their will. We have long stated that "the
objection to the transfer being grounded on solely upon the personal inconvenience or hardship
that will be caused to the employee by reason of the transfer is not a valid reason to disobey an
order of transfer." That respondents eventually discontinued reporting for work after their plea to
be returned to their former work assignment was their personal decision, for which the
petitioners should not be held liable particularly as the latter did not, in fact, dismiss them.
Indeed, there was no evidence that respondents were dismissed from employment. In fact,
petitioners expressed willingness to accept them back to work. There being no termination of
employment by the employer, the award of backwages cannot be sustained. It is well settled

ATHENA M. SALAS | LABOR CASE DIGEST 2015 85

that backwages may be granted only when there is a finding of illegal dismissal. In cases where
there is no evidence of dismissal, the remedy is reinstatement but without backwages.
The constitutional policy of providing full protection to labor is not intended to oppress or destroy
management. While the Constitution is committed to the policy of social justice and the
protection of the working class, it should not be supposed that every labor dispute will be
automatically decided in favor of labor. Management also has its rights which are entitled to
respect and enforcement in the interest of simple fair play. Thus, where management
prerogative to transfer employees is validly exercised, as in this case, courts will decline to
interfere.
39. Royal Plant Workers Union vs. Coca-Cola Bottlers Phils Inc. -Cebu Plant, G.R. No.
198783, April 15, 2013
Facts:
Under the employ of each bottling plant are bottling operators. In the case of the plant in Cebu
City, there are 20 bottling operators who work for its Bottling Line 1 while there are 12-14
bottling operators who man its Bottling Line 2. All of them are male and they are members of
herein respondent Royal Plant Workers Union (ROPWU).
The bottling operators work in two shifts. The first shift is from 8 a.m. to 5 p.m. and the second
shift is from 5 p.m. up to the time production operations is finished. Thus, the second shift varies
and may end beyond eight (8) hours. However, the bottling operators are compensated with
overtime pay if the shift extends beyond eight (8) hours. For Bottling Line 1, 10 bottling
operators work for each shift while 6 to 7 bottling operators work for each shift for Bottling Line
2.
Each shift has rotations of work time and break time. Prior to September 2008, the rotation is
this: after two and a half (2 1/2) hours of work, the bottling operators are given a 30-minute
break and this goes on until the shift ends. In September 2008 and up to the present, the
rotation has changed and bottling operators are now given a 30-minute break after one and one
half (1 1/2) hours of work.
In 1974, the bottling operators of then Bottling Line 2 were provided with chairs upon their
request. In 1988, the bottling operators of then Bottling Line 1 followed suit and asked to be
provided also with chairs. Their request was likewise granted. Sometime in September 2008,
the chairs provided for the operators were removed pursuant to a national directive of petitioner.
With this task of moving constantly to check on the machinery and equipment assigned to him,
a bottling operator does not need a chair anymore, hence, petitioner's directive to remove them.
Furthermore, CCBPI rationalized that the removal of the chairs is implemented so that the
bottling operators will avoid sleeping, thus, prevent injuries to their persons.

ATHENA M. SALAS | LABOR CASE DIGEST 2015 86

The bottling operators took issue with the removal of the chairs.
Issue:
Whether or not the removal of the bottling operators' chairs
production/manufacturing lines a valid exercise of a management prerogative

from

CCBPI's

Ruling:
Yes.
The Court has held that management is free to regulate, according to its own discretion and
judgment, all aspects of employment, including hiring, work assignments, working methods,
time, place, and manner of work, processes to be followed, supervision of workers, working
regulations, transfer of employees, work supervision, lay-off of workers, and discipline, dismissal
and recall of workers. The exercise of management prerogative, however, is not absolute as it
must be exercised in good faith and with due regard to the rights of labor.
In the present controversy, it cannot be denied that CCBPI removed the operators' chairs
pursuant to a national directive and in line with its "I Operate, I Maintain, I Clean" program,
launched to enable the Union to perform their duties and responsibilities more efficiently. The
chairs were not removed indiscriminately. They were carefully studied with due regard to the
welfare of the members of the Union. The removal of the chairs was compensated
by: a) a reduction of the operating hours of the bottling operators from a two-and-one-half (2
1/2)-hour rotation period to a one-and-a-half (1 1/2) hour rotation period; and b) anincrease of
the break period from 15 to 30 minutes between rotations.
Apparently, the decision to remove the chairs was done with good intentions as CCBPI wanted
to avoid instances of operators sleeping on the job while in the performance of their duties and
responsibilities and because of the fact that the chairs were not necessary considering that the
operators constantly move about while working. In short, the removal of the chairs was
designed to increase work efficiency. Hence, CCBPI's exercise of its management prerogative
was made in good faith without doing any harm to the workers' rights. The rights of the Union
under any labor law were not violated. There is no law that requires employers to provide chairs
for bottling operators. Further, The operators' chairs cannot be considered as one of the
employee benefits covered in Article 100 of the Labor Code. In the Court's view, the term
"benefits" mentioned in the non-diminution rule refers to monetary benefits or privileges given to
the employee with monetary equivalents. Such benefits or privileges form part of the employees'
wage, salary or compensation making them enforceable obligations
40. Peckson vs. Robinsons Supermarket Corp. G.R. No. 198534, July 3, 2013
Facts:

ATHENA M. SALAS | LABOR CASE DIGEST 2015 87

The petitioner first joined the Robinsons Supermarket Corporation (RSC) as a Sales Clerk on
November 3, 1987. On October 26, 2006, she was holding the position of Category Buyer when
respondent Roena Sarte (Sarte), RSC's Assistant Vice-President for Merchandising, reassigned
her to the position of Provincial Coordinator, effective November 1, 2006. Claiming that her new
assignment was a demotion because it was non-supervisory and clerical in nature, the petitioner
refused to turn over her responsibilities to the new Category Buyer, or to accept her new
responsibilities as Provincial Coordinator.
Issue: Whether or not petitioners lateral transfer from Category Buyer to Provincial Coordinator
is considered a demotion amounting to constructive dismissal
Ruling:
Under the doctrine of management prerogative, every employer has the inherent right to
regulate, according to his own discretion and judgment, all aspects of employment, including
hiring, work assignments, working methods, the time, place and manner of work, work
supervision, transfer of employees, lay-off of workers, and discipline, dismissal, and recall of
employees. The only limitations to the exercise of this prerogative are those imposed by labor
laws and the principles of equity and substantial justice.
Concerning the transfer of employees, these are the following jurisprudential guidelines: (a) a
transfer is a movement from one position to another of equivalent rank, level or salary without
break in the service or a lateral movement from one position to another of equivalent rank or
salary; (b) the employer has the inherent right to transfer or reassign an employee for legitimate
business purposes; (c) a transfer becomes unlawful where it is motivated by discrimination or
bad faith or is effected as a form of punishment or is a demotion without sufficient cause; (d) the
employer must be able to show that the transfer is not unreasonable, inconvenient, or prejudicial
to the employee.
If the transfer of an employee is not unreasonable, or inconvenient, or prejudicial to him, and it
does not involve a demotion in rank or a diminution of his salaries, benefits and other
privileges, the employee may not complain that it amounts to a constructive dismissal. The
respondents had the burden of proof that the transfer of the petitioner was not tantamount to
constructive dismissal. The respondents have discharged the burden of proof that the transfer of
the petitioner was not tantamount to constructive dismissal.
In the case at bar, we agree with the appellate court that there is substantial showing that the
transfer of the petitioner from Category Buyer to Provincial Coordinator was not unreasonable,
inconvenient, or prejudicial to her. The petitioner failed to dispute that the job classifications of
Category Buyer and Provincial Coordinator are similar, or that they command a similar salary
structure and responsibilities. We agree with the NLRC that the Provincial Coordinator's position
does not involve mere clerical functions but requires the exercise of discretion from time to time,
as well as independent judgment, since the Provincial Coordinator gives appropriate

ATHENA M. SALAS | LABOR CASE DIGEST 2015 88

recommendations to management and ensures the faithful implementation of policies and


programs of the company. It even has influence over a Category Buyer because of its
recommendatory function that enables the Category Buyer to make right decisions on
assortment, price and quantity of the items to be sold by the store.

14. TERMINATION OF EMPLOYMENT


41. Ramirez et al., vs. Mar Fishing Co Inc. et al., G.R. No. 168208, June 13, 2012
Facts:
Mar Fishing sold its principal assets to Miramar Fishing Co through public bidding. In view of
that transfer, Mar Fishing issued a Memorandum informing all its workers that the company
would cease to operate by the end of the month. two days prior to the months end, it notified
(DOLE) of the closure of its business operations.
Thereafter, Mar Fishings labor union, Mar Fishing Workers Union entered into a Memorandum
of Agreement. The Agreement provided that the acquiring company, Miramar, shall absorb Mar
Fishings regular rank and file employees whose performance was satisfactory, without loss of
seniority rights and privileges previously enjoyed.
Unfortunately, petitioners, who worked as rank and file employees, were not hired or given
separation pay by Miramar. Thus, petitioners filed Complaints for illegal dismissal with money
claims before the Arbitration Branch of (NLRC).
Issue:
Whether or not dismissal based on the closure of business is valid.
Ruling:
YES.
3 requirements for closure of business to be valid , to wit:
1) the cessation of or withdrawal from business operations must be bona fide in
character;
2) there must be payment to the employees of termination pay amounting to at least
one-half (1/2) month pay for each year of service, or one (1) month pay, whichever is higher;
3) the company must serve a written notice on the employees and on the DOLE at least
one (1) month before the intended termination.
Petitioners did not dispute the conclusion of the LA and the NLRC that Mar Fishing had an
authorized cause to dismiss its workers. Neither did petitioners challenge the computation of
their separation pay. Petitioners only questioned the holding that only Mar Fishing was liable for
their monetary claims.

ATHENA M. SALAS | LABOR CASE DIGEST 2015 89

42. Prudential Guarantee & Assurance Employee Labor Union vs. NLRC et al., G.R. No.
185335, June 13, 2012
Facts:
Vallota commenced his employment with Prudential Guarantee and Assurance, Inc. (PGAI) on
1995 as a Junior Programmer assigned to the Electronic Data Processing (EDP) . He reported
directly to Gerald Dy Victory, then head of the EDP, until his replacement by respondent Jocelyn
Retizos in 1997.
PGAIs Human Resource Manager, invited Union President, Mike Apostol to his office. Atty. Rillo
informed Apostol that PGAI was going to conduct an on-the-spot security check in the
Information and Technology (IT) Department. Atty. Rillo also requested that Union
representatives witness the inspection to which Apostol agreed.
The inspection team proceeded to the IT Department, PGAI network administrator Angelo
Gutierrez (Gutierrez), initiated the spot check beginning with the one assigned to Vallota.
Gutierrez apparently did not find anything unusual with Vallotas computer and said "Wala
naman, saan dito?" Retizos insisted, "Nandyan yan," and took over the inspection until she
found a folder named "MAA." She then exclaimed, "Heto oh! Ano to? Bakit may MAA dito?"
Retizos asked Vallota, "Are you working for MAA?" Vallota replied, "Hindi po, MAA mutual life
po yan na makikita po sa internet." Gutierrez saved a copy of the contents of the MAA folder in
a floppy disk.
Vallota received a memorandum directing him to explain within 72 hours why highly confidential
files were stored in his computer. The same memorandum also informed him that he was being
placed under preventive suspension for 30 days effective upon receipt of the said notice. A
second memorandum , notified Vallota of the extension of his preventive suspension for another
30 days, in view of the fact that the management needed more time to evaluate the
administrative case against him.
PGAI sent him another memorandum requesting further details on some of the matters he
raised in his response. Vallota requested a conference, to be attended by a Union
representative and counsel. In reply, PGAI sent Vallota another memorandum which, among
others, set a new deadline for Vallota to submit his reply and evidence in his defense.
Meanwhile, the Union sent a letter to PGAI President Philip K. Rico (Rico) requesting that a
grievance committee be convened and that the contents of the computers of other IT personnel
be similarly produced. The request for the convening of a grievance committee was ignored.
Vallota was given a notice of termination of his employment effective January 10, 2006 on the
ground of loss of trust and confidence. Thus, the petitioners filed a complaint for illegal dismissal
with claims for full backwages, moral and exemplary damages, and attorneys fees.
Issues:
1. Whether or not the petitioner was validly dismissed on the ground of loss of trust and
confidence.

ATHENA M. SALAS | LABOR CASE DIGEST 2015 90

2. Whether or not the procedural due process requirements for termination were observed.
Ruling:
1. No.
Loss of confidence as a just cause for dismissal was never intended to provide employers with a
blank check for terminating their employees. Such a vague, all-encompassing pretext as loss of
confidence, if unqualifiedly given the seal of approval by this Court, could readily reduce to
barren form the words of the constitutional guarantee of security of tenure.
Having this in mind, loss of confidence should ideally apply only to cases involving employees
occupying positions of trust and confidence or to those situations where the employee is
routinely charged with the care and custody of the employer's money or property. To the first
class belong managerial employees, i.e., those vested with the powers or prerogatives to lay
down management policies and/or to hire, transfer, suspend, lay-off, recall, discharge, assign or
discipline employees or effectively recommend such managerial actions; and to the second
class belong cashiers, auditors, property custodians, etc., or those who, in the normal and
routine exercise of their functions, regularly handle significant amounts of money or property.
The first question to be addressed is whether Vallota held a position of trust and confidence.
Vallota was employed by PGAI as a Junior Programmer assigned to the EDP Department.
Based on the standards set by previous jurisprudence, Vallotas position as Junior Programmer
is analogous to the second class of positions of trust and confidence. Though he did not
physically handle money or property, he became privy to confidential data or information by the
nature of his functions. At a time when the most sensitive of information is found not printed on
paper but stored on hard drives and servers, an employee who handles or has access to data in
electronic form naturally becomes the unwilling recipient of confidential information.
Having addressed the nature of his position, the next question is whether the act complained of
justified the loss of trust and confidence of Vallotas employer so as to constitute a valid cause
for dismissal. It must, thus, be determined whether the alleged basis for dismissal was based on
clearly established facts.
The act alleged to have caused the loss of trust and confidence of PGAI in Vallota was the
presence in his computers hard drive of a folder named "MAA" allegedly containing files with
information on MAA Mutual Life Philippines.
In this case, there was no other evidence presented to prove fraud in the manner of securing or
obtaining the files found in Vallotas computer. In fact, aside from the presence of these files in
Vallotas hard drive, there was no other evidence to prove any gross misconduct on his part.
There was no proof either that the presence of such files was part of an attempt to defraud his
employer or to use the files for a purpose other than that for which they were intended. If
anything, the presence of the files reveals some degree of carelessness or neglect in his failure
to delete them, but it is an extremely farfetched conclusion bordering on paranoia to state that it
is part of a larger conspiracy involving corporate espionage.

ATHENA M. SALAS | LABOR CASE DIGEST 2015 91

Moreover, contrary to the respondents allegations, the MAA files found in Vallotas computer,
the prospectus and corporate profile, are not sensitive corporate documents. These are
documents routinely made available to the public.
2. Yes.
The following are the guiding principles in connection with the hearing requirement in dismissal
cases:
1. Ample opportunity to be heard means any meaningful opportunity (verbal or written)
given to the employee to answer the charges against him and submit evidence in
support of his defense, whether in a hearing, conference or some other fair, just and
reasonable way.
2. A formal hearing or conference becomes mandatory only when requested by the
employee in writing or substantial evidentiary disputes exist or a company rule or
practice requires it, or when similar circumstances justify it.
3. The ample opportunity to be heard standard in the Labor Code prevails over the
hearing or conference requirement in the implementing rules and regulations.
In this case, the two-notice requirement was complied with. By the petitioners own admission,
PGAI issued to Valota a written Notice of Charges & Preventive Suspension (Ref. No. AC-0502) dated November 14, 2005. After an exchange of memoranda, PGAI then informed Vallota of
his dismissal in its decision dated December 21, 2005.
But the petitioners expressly requested a conference or a convening of a grievance committee,
such formal hearing became mandatory. After PGAI failed to affirmatively respond to such
request, it follows that the hearing requirement was not complied with and, therefore, Vallota
was denied his right to procedural due process.
43. Romeo Paulino v. NLRC, PLDT. 13 JUNE 2012
Loss of trust and confidence, termination for just cause
Facts:
Involved here is a Paulino, a Cable Splicer III who was terminated from employment because of
serious misconduct (theft of company policy). Undisputed are the following series of facts:
1. Paulino surrendered his service vehicle to PLDTs motor pool for body repairs. Because
of this, he unloaded several company-issued plant materials contained in the vehicle and
stored them in his residence. He kept these properties in his residence for 1 month and
11 days, thus on February, armed with a Search Warrant, PNP searched his house and
found many plant materials. He was thereafter charged with Theft.

ATHENA M. SALAS | LABOR CASE DIGEST 2015 92

2. During this interim, PLDT likewise received security report that Paulino was engaged in
illicit disposal of company properties.
3. Following this, PLDT issued a Memo requiring Paulino to explain why he should not be
terminated. The Memo likewise gave him the option to ask for a formal hearing. Paulino,
in his reply, merely requested that the proceedings be held in abeyance until the criminal
case for theft had been concluded.
4. Thereafter, PLDT informed Paulino that since his reply did not provide any clarification,
his employment is terminated.
5. Three years later, after the criminal case for Theft had been dismissed for failure to
prove guilt beyond reasonable doubt, Paulino filed a complaint for Illegal Dismissal.
Issue:
Whether or not Paulinos employment was illegally terminated
Ruling:
Dismissal was valid, petition is denied.
The Labor Code recognizes that an employer, for just cause, may validly terminate the services
of an employee for serious misconduct or willful disobedience of the lawful orders of the
employer or representative in connection with the employees work. Fraud or willful breach by
the employee of the trust reposed by the employer in the former, or simply loss of confidence,
also justifies an employees dismissal from employment.
1. The LA, the NLRC and the CA all acknowledged that, notwithstanding petitioners
acquittal in the criminal case for qualified theft, respondent PLDT had adequately
established the basis for the companys loss of confidence as a just cause to terminate
petitioner. This Court finds that approach to be correct, since proof beyond reasonable
doubt of an employees misconduct is not required in dismissing an employee. Rather,
as opposed to the proof beyond reasonable doubt standard of evidence required in
criminal cases, labor suits require only substantial evidence to prove the validity of the
dismissal.
2. Willful breach of trust or loss of confidence requires that the employee (1) occupied a
position of trust or (2) was routinely charged with the care of the employers property. As
correctly appreciated by the CA, petitioner was charged with the care and custody of
PLDTs property.
3. To warrant dismissal based on loss of confidence, there must be some basis for the
loss of trust or the employer must have reasonable grounds to believe that the employee
is responsible for misconduct that renders the latter unworthy of the trust and confidence
demanded by his or her position. Here, petitioner disputes the sufficiency of PLDTs
basis for loss of trust and confidence. He alleges that he did not steal the plant materials,
considering that he had lawful possession.

ATHENA M. SALAS | LABOR CASE DIGEST 2015 93

4. However, assuming that he lawfully possessed the materials, PLDT still had ample
reason or basis to already distrust petitioner. For more than a month, he did not even
inform PLDT of the whereabouts of the plant materials. Instead, he stocked these
materials at his residence even if they were needed in the daily operations of the
company. In keeping with the honesty and integrity demanded by his position, he should
have turned over these materials to the plants warehouse. The fact that petitioner did
not present any documents or requisition slips at the time that the PNP took the plant
materials logically excites suspicion. In addition, PLDT received a security report stating
that petitioner had engaged in the illicit disposal of its plant materials, which were
recovered during the search conducted at his residence.
5. In a final effort to impugn his dismissal, petitioner claims that he could only be faulted for
breaching PLDTs rules and regulations which prohibited the employees from bringing
home company materials.
a. In this regard, petitioner exacerbates his position. By admitting that he breached
company rules, he buttressed his employers claim that he committed serious
misconduct.
b. Employees cannot take company rules for granted, especially in this case where
petitioners breach involved various plant materials that may cause major
disruption in the companys operations. Indeed, an employer may discharge an
employee for refusal to obey a reasonable company rule. As a rule, although this
Court leans over backwards to help workers and employees continue with their
employment, acts of dishonesty in the handling of company property are a
different matter.
c. Given these circumstances, it would have been unfair for PLDT to keep petitioner
in its employ. Petitioner displayed actions that made him untrustworthy. Thus, as
a measure of self-protection, PLDT validly terminated his services for serious
misconduct and loss of confidence.

44. Ever Electrical Mfg. (EEMI) and Vicente Go v. Samahang Mangagawas ng Ever
Electrical, et al. 13 June 2012
Serious business losses, termination of employment, prevented from entering work by virtue of
writ of execution against employer, solidary liability of corporate officers
Facts: EEMI is a corporation engaged in manufacturing electrical parts/supplies. Respondents
are employees of said corporation, whose employment was terminated. EEMI reasoned that
said termination was due to serious business losses resulting in closure of its business
operations which progressed as follows:
EEMI invested in Orient Commercial Banking Corporation a sum of P500 000 000. During the
Asian Currency Crisis, EEMI was one of those who suffered huge losses. It further suffered
losses due to the continued entry of cheaper goods from China. EEMI had to obtain a loan from
UCPB in the amount of P121 000 000, secured by a mortgage on some of its properties. To cap
it all off, Orient Bank, where most of EEMIs resources were invested, closed.

ATHENA M. SALAS | LABOR CASE DIGEST 2015 94

As a result, EEMI was not able to meet its loan obligations with UCPB. EEMI and UCPB then
entered into a dacion en pago, where EEMI transferred ownership of the companys property to
UCPB. EEMI then, through an affiliate company, leased the transferred property so that it can
continue its business operations. However, this lease agreement came to a halt after the affiliate
company (and ultimately EEMI) was evicted by virtue of an unlawful detainer case. A writ of
execution ordered the affiliate company to vacate the premises. Hence, when said writ was
implemented, herein employee-respondents were prevented from entering the factory, causing
said employees to file a Complaint for Illegal Dismissal, payment of 13 th month pay and
separation pay.
Vicente Go, an officer-stockholder of the corporation was also joined as defendant in the
Complaint.
Issues:
1. Whether or not the closure of EEMIs operation was due to serious business losses.
2. Whether or not Vicente Go is solidarily liable.

Ruling:
Partially granted. EEMI is liable for 13th month and separation pay. Vicente go is absolved.
1. Article 283 of the Labor Code provides:
Art. 283. Closure of establishment and reduction of personnel. The employer may
also terminate the employment of any employee due to the installation of labor saving
devices, redundancy, retrenchment to prevent losses or the closing or cessation of
operation of the establishment or undertaking unless the closing is for the purpose of
circumventing the provisions of this Title, by serving a written notice on the workers and
the Ministry of Labor and Employment at least one (1) month before the intended date
thereof. In case of termination due to the installation of labor saving devices or
redundancy, the worker affected thereby shall be entitled to a separation pay equivalent
to at least his one (1) month pay or to at least one (1) month pay for every year of
service, whichever is higher. In case of retrenchment to prevent losses and in cases of
closures or cessation of operations of establishment or under taking not due to serious
business losses or financial reverses, the separation pay shall be equivalent to one (1)
month pay or at least one-half ( 1/2) month pay for every year of service, whichever is
higher. A fraction of at least six (6) months shall be considered one (1) whole year.
In this case, EEMI failed to establish that the main reason for its closure was business
reverses. As aptly observed by the CA, the cessation of EEMIs business was not directly
brought about by serious business losses or financial reverses, but by reason of the
enforcement of a judgment against it. Thus, EEMI should be required to pay separation pay to
its affected employees.

ATHENA M. SALAS | LABOR CASE DIGEST 2015 95

2. As to whether or not Go should be held solidarily liable with EEMI, the Court agrees with the
petitioner.
The SC rejected the view of the Labor Arbiter, to wit: the LA was of the view that Go, as
President of the corporation, actively participated in the management of EEMIs corporate
obligations, and, accordingly, rendered judgment ordering EEMI and Go in solidum to pay the
complainants[18] their due. He explained that [r]espondent Gos negligence in not paying the
lease rental of the plant in behalf of the lessee EGO Electrical Supply, Inc., where EEMI was
operating and reimburse expenses of UCPB for real estate taxes and the like, prompted the
bank to file an unlawful detainer case against the lessee, EGO Electrical Supply Co. This
evasion of an existing obligation, made respondent Go as liable as respondent EEMI, for
complainants money awards.[19] Added the LA, being the President and the one actively
representing respondent EEMI, in major contracts i.e. Real Estate Mortgage, loans,dacion en
pago, respondent Go has to be liable in the case. [20] As earlier stated, the CA affirmed the LA
decision citing the case of Restaurante Las Conchas v. Llego,[21] where it was held that when
the employer corporation is no longer existing and unable to satisfy the judgment in favor of the
employees, the officers should be held liable for acting on behalf of the corporation.
As a general rule, corporate officers should not be held solidarily liable with the corporation for
separation pay for it is settled that a corporation is invested by law with a personality separate
and distinct from those of the persons composing it as well as from that of any other legal entity
to which it may be related. Mere ownership by a single stockholder or by another corporation of
all or nearly all of the capital stock of a corporation is not of itself sufficient ground for
disregarding the separate corporate personality.[17]
In the case at bench, the records do not warrant an application of the exception. The rule, which
requires the presence of malice or bad faith, must still prevail. In the recent case of Wensha
Spa Center and/or Xu Zhi Jie v. Yung, the Court absolved the corporations president from
liability in the absence of bad faith or malice. In the said case, the Court stated:
In labor cases, corporate directors and officers may be held solidarily liable with the
corporation for the termination of employment only if done with malice or in bad faith Bad
faith does not connote bad judgment or negligence; it imports a dishonest purpose or
some moral obliquity and conscious doing of wrong; it means breach of a known duty
through some motive or interest or ill will; it partakes of the nature of fraud
In the present case, Go may have acted in behalf of EEMI but the companys failure to operate
cannot be equated to bad faith. Cessation of business operation is brought about by various
causes like mismanagement, lack of demand, negligence, or lack of business foresight. Unless
it can be shown that the closure was deliberate, malicious and in bad faith, the Court must apply
the general rule that a corporation has, by law, a personality separate and distinct from that of
its owners. As there is no evidence that Go, as EEMIs President, acted maliciously or in bad

ATHENA M. SALAS | LABOR CASE DIGEST 2015 96

faith in handling their business affairs and in eventually implementing the closure of its business,
he cannot be held jointly and solidarily liable with EEMI.

45. WATERFRONT CEBU CITY HOTEL VS. MA. MELANIE JIMENEZ ET AL.
174214, JUNE 13, 2012

GR NO.

FACTS:
The closure of a department or division of a company constitutes retrenchment by, and not
closure of, the company itself.
It is upon this principle that both parties to this case, the employer Hotel and the complaining
employees repose their argument.
Respondents Ma. Melanie P. Jimenez et were hired for Club Waterfront (the Club), a division
under petitioner Waterfront Cebu City Hotel (the Hotel) which catered to foreign high stakes
gamblers.
On 12 May 2003, respondents received identical letters of termination from petitioners Director
of Human Resources informing them of the temporary suspension of business of the Club. A
total of 45 employees were notified of the imminent closure. Petitioner averred that since April
2002, the Club has been incurring losses that it had to temporarily cease its operations effective
15 June 2003. To support the allegations of losses, petitioner presented financial statements of
Waterfront Promotion, Ltd. Petitioner argued that pursuant to Article 286 of the Labor Code, the
temporary suspension of business operations does not terminate employment. Thus,
respondents have no cause of action against them.
The dismissed employees were offered separation pay equivalent to half-month pay for every
year of service.
On 26 June 2003, respondents filed a complaint before the Labor Arbiter for illegal dismissal,
illegal suspension, and non-payment of salaries and other monetary benefits. They refused to
believe that the Club was suffering from losses because they knew exactly the number of
arrivals as well as junket clients of the Club. They presented documents to show the arrival of
foreign guests at the Club.
The Labor Arbiter ruled in favor of petitioner and upheld the closure of the Clubs business
operations as a management prerogative.
The NLRC also ruled in favor of petitioner.
After the denial of respondents motion for reconsideration, they elevated the case to the Court
of Appeals which reversed the decision of the NLRC.
ISSUE:
WHETHER OR NOT THERE WAS A VALID RETRENCHMENT
RULING:

ATHENA M. SALAS | LABOR CASE DIGEST 2015 97

Retrenchment is the termination of employment initiated by the employer through no fault of and
without prejudice to the employees. It is resorted to during periods of business recession,
industrial depression, or seasonal fluctuations or during lulls occasioned by lack of orders,
shortage of materials, conversion of the plant for a new production program or the introduction
of new methods or more efficient machinery or of automation. It is an act of the employer of
dismissing employees because of losses in the operation of a business, lack of work, and
considerable reduction on the volume of his business.
In case of retrenchment, proof of financial losses becomes the determining factor in proving its
legitimacy. In establishing a unilateral claim of actual or potential losses, financial statements
audited by independent external auditors constitute the normal method of proof of profit and loss
performance of a company. The condition of business losses justifying retrenchment is normally
shown by audited financial documents like yearly balance sheets and profit and loss statements
as well as annual income tax returns.
Retrenchment is subject to faithful compliance with the substantive and procedural requirements
laid down by law and jurisprudence. For a valid retrenchment, the following elements must be
present:
(1) That retrenchment is reasonably necessary and likely to prevent business losses
which, if already incurred, are not merely de minimis, but substantial, serious, actual
and real, or if only expected, are reasonably imminent as perceived objectively and in
good faith by the employer
(2) That the employer served written notice both to the employees and to the
Department of Labor and Employment at least one month prior to the intended date of
retrenchment;
(3) That the employer pays the retrenched employees separation pay equivalent to
one (1) month pay or at least month pay for every year of service, whichever is
higher;
(4) That the employer exercises its prerogative to retrench employees in good faith
for the advancement of its interest and not to defeat or circumvent the employees
right to security of tenure; and
(5) That the employer used fair and reasonable criteria in ascertaining who would be
dismissed and who would be retained among the employees, such as status,
efficiency, seniority, physical fitness, age, and financial hardship for certain workers.
All these elements were successfully proven by petitioner. First, the huge losses suffered by the
Club for the past two years had forced petitioner to close it down to avert further losses which
would eventually affect the operations of petitioner. Second, all 45 employees working under
the Club were served with notice of termination. The corresponding notice was likewise served
to the DOLE one month prior to retrenchment. Third, the employees were offered separation
pay, most of whom have accepted and opted not to join in this complaint. Fourth, cessation of
or withdrawal from business operations was bona fide in character and not impelled by a motive
to defeat or circumvent the tenurial rights of employees. As a matter of fact, as of this writing,

ATHENA M. SALAS | LABOR CASE DIGEST 2015 98

the Club has not resumed operations. Neither is there a showing that petitioner carried out the
closure of the business in bad faith. No labor dispute existed between management and the
employees when the latter were terminated.
46. Manila Electric Co., vs Dejan, GR No. 194106, June 18, 2012
FACTS:
Respondent Herminigildo Dejan commenced employment with the Manila Electric
Company (Meralco) on July 7, 1992. He was then Meralco's branch representative in its San
Pedro, Laguna branch, with a monthly salary of P30,500.00. His work consisted of accepting
payments of the required fees from applicants for electric service installation and issuing the
corresponding meter sockets/bases after payment of a deposit, preceded by an inspection of
the premises to be energized by a Meralco field personnel.
In the mid-afternoon of March 18, 2005, the security guard on duty at the branch, Warlito
Silverio, noticed a certain Estanislao Gozarin a.k.a. Mang Islao, a private electrician, take out
from the branch premises 20 pieces of meter sockets which were then loaded into a parked
Meralco contracted jeep belonging to one Cesar Reyes. Reyes brought the meter sockets to his
house. The meter sockets were thereafter allegedly picked up by Gil Duenas, a Meralco field
representative. Dejan was asked to explain the incident. aCITEH
In his letter-explanation, dated March 23, 2005, to a certain Emilia SJ Reaso, Dejan admitted
that he released the meter sockets in question because the deposit fees had already been paid.
The payor, a certain Antonio A. Depante a.k.a. Bruce, also an electrician, asked for the release
of the items. Allegedly, he had several contracts for service installation with the branch. Dejan
indicated the list of contracts covering the released meter sockets. Sometime in September,
October and November 2005, Meralco asked Gozarin, Dejan, and Reyes to give their sworn
statements on the incident.
On February 10, 2006, Dejan received a letter from Marcelino Rosario, head of Meralco's
Investigation-Paralegal Services, charging him with the unauthorized taking of 20 meter
sockets, in violation of Section 7, paragraphs 4 and 11 of the Company Code of Employee
Discipline, in relation to Article 282 of the Labor Code. On February 17, 2006, Meralco
conducted a formal investigation where Dejan admitted issuing the meter sockets without the
authorization of the applicants for electric connection. He alleged that he released the items
even without authorization as it had been the accepted practice in the office, provided the
deposit fee had been paid. He claimed that he talked with Depante, through the cell phone of
Duenas, about it, after Duenas himself requested him (Dejan) to release the meter sockets to
Gozarin. When Dejan released the meter sockets, Duenas instructed Gozarin to take them out
of the Meralco premises and load them in Reyes' jeep.
Also testifying at the investigation, Depante corroborated Dejan's account of the incident. He
alleged that he made the request for the release of the meter sockets due to his inability to pick
up the items himself as he was busy with another project at the time. He and Duenas retrieved

ATHENA M. SALAS | LABOR CASE DIGEST 2015 99

the meter sockets from Reyes' house the next day.


Unconvinced with Dejan's explanation, Meralco served Dejan a letter on April 6,
2006, 9 terminating his employment effective the following day, with forfeiture of all rights and
privileges. On April 20, 2006, Dejan filed his complaint with the National Labor Relations
Commission (NLRC).
ISSUE:
The validity of dejans termination.
RULING:
We found for petitioner.
Dejan is liable as charged. More specifically, he is liable for violation of Section 7, paragraphs 4
and 11 of the Company Code of Employee Discipline, constituting serious misconduct, fraud
and willful breach of trust of the employer, just causes for termination of employment under the
law. The facts and the evidence on record clearly bear this out and we wonder how the CA
could have missed the seriousness or gravity of Dejans transgressions.
There is no dispute about the release of the meter sockets. Also, the persons involved were
clearly identified Dejan; Gozarin or Mang Islao, a private electrician who received the meter
sockets; Reyes, the owner of the jeep where the meter sockets were loaded by Gozarin;
Duenas, a Meralco field representative; and Depante, another private electrician who
purportedly owned the meter sockets.
There is also no question that Dejan released the meter sockets to Gozarin without the written
authority or SPA from the customer or customers who applied for electric connection (as a
matter of company policy). Dejan released the meter sockets to Gozarin on the mere say-so of
Depante, as he claimed, through a call to Duenas cell phoneand justified his act to be in accord
with accepted company practice.
We cannot blame Meralco for losing its trust and confidence in Dejan. He is no ordinary
employee. As branch representative, "he was principally charged with the function and
responsibility to accept payment of fees required for the installation of electric service and
facilitate issuance of meter sockets." The duties of his position require him to always act with
the highest degree of honesty, integrity and sincerity, as the company puts it. In light of his
fraudulent act, Meralco, an enterprise imbued with public interest, cannot be compelled to
continue Dejan's employment, as it would be inimical to its interest. Needless to say, "[t]he law,
in protecting the rights of the laborer, authorizes neither oppression nor self-destruction of the
employer." For sure, Dejan was validly dismissed for serious misconduct, and loss of
trust and confidence.

ATHENA M. SALAS | LABOR CASE DIGEST 2015 100

47. Apo Cement vs Baptisma, GR 176671


Principle: The act of a power plant manager in receiving commissions or kickbacks from the companys
suppliers constitutes breach of the trust and confidence reposed in him, justifying his dismissal. For
managerial employees, the mere existence of a basis for believing that such employee has breached
the trust of his employer would suffice for his dismissal.
Facts:
On June 16, 1998, respondent Zaldy E. Baptisma was employed by petitioner Apo Cement Corporation.
Sometime in September 2003, petitioner received information from one of its employees, Armando
Moralda (Moralda), that some of its personnel, including respondent who was then the manager of
petitioners Power Plant Department, were receiving commissions or kickbacks from suppliers. [5] To
ascertain the veracity of the information given by Moralda, the top management of petitioner conducted
an investigation during which Jerome Lobitaa (Lobitaa), one of petitioners accredited suppliers, doing
business under the name and style Precision Process, came forward to corroborate the statement of
Moralda.
Having been implicated in the irregularities, respondent, on November 3, 2003, received a Show Cause
Letter with Notice of Preventive Suspension from Plant Director Ariel Mendoza. On November 5, 2003,
respondent submitted his written explanation denying the accusations hurled against him. To further
afford respondent ample opportunity to defend himself, petitioner conducted a series of administrative
investigation hearings during which respondent was able to face his accusers.
On March 22, 2004, respondent received the Notice of Termination dated March 19, 2004 informing him
of his dismissal from employment effective immediately on the ground of loss of trust and confidence. [ At
the time of his termination, respondent was a Power Plant Manager earning a monthly salary
of P71,100.00. On March 31, 2004, respondent filed with the Regional Arbitration Branch VII of the
National Labor Relations Commission (NLRC) in Cebu City a complaint for illegal dismissal
Issue:
WON respondent was validly dismissed
Ruling:
Yes.
To validly dismiss an employee on the ground of loss of trust and confidence under Article 282 (c) of the
Labor Code of the Philippines, the following guidelines must be observed: 1) loss of confidence should
not be simulated; 2) it should not be used as subterfuge for causes which are improper, illegal or
unjustified; 3) it may not be arbitrarily asserted in the face of overwhelming evidence to the contrary; and
4) it must be genuine, not a mere afterthought to justify earlier action taken in bad faith. More important,
it must be based on a willful breach of trust and founded on clearly established facts.

ATHENA M. SALAS | LABOR CASE DIGEST 2015 101

In this case, we agree with the NLRC that the termination of respondent on the ground of loss of trust
and confidence was justified. Unlike the Labor Arbiter and the CA, we find the testimony of Lobitaa
credible and truthful. There are no inconsistensies between the two affidavits executed by Lobitaa, and
there appears to be no ill-motive on the part of Lobitaa to falsely accuse respondent of accepting
commissions and/or kickbacks.

All told, we find that the testimony of Lobitaa constitutes substantial evidence to prove that respondent,
as the then Power Plant Manager, accepted commissions and/or kickbacks from suppliers, which is a
clear violation of Section 2.04 of petitioners Company Rules and Regulations. Jurisprudence
consistently holds that for managerial employees the mere existence of a basis for believing that such
employee has breached the trust of his employer would suffice for his dismissal. [60] As we then see it,
respondents termination was for a just and valid cause.

48. Cosmos Bottling vs Fermin, GR 193676


Principle: Theft committed against a co-employee is considered as a case analogous to
serious misconduct, and an employee terminated for this cause is considered dismissed
for a just cause. Such employee is not entitled to payment of his full retirement benefits
or backwages. The award of financial compensation or assistance to an employee
validly dismissed from service due to serious misconduct has no basis in law.
Facts:
Wilson B. Fermin (Fermin) was a forklift operator at Cosmos Bottling Corporation (COSMOS),
where he started his employment on 27 August 1976. [4] On 16 December 2002, he was accused
of stealing the cellphone of his fellow employee, Luis Braga (Braga). [5] Fermin was then given a
Show Cause Memorandum, requiring him to explain why the cellphone was found inside his
locker.[6] In compliance therewith, he submitted an affidavit the following day, explaining that he
only hid the phone as a practical joke and had every intention of returning it to Braga.
After conducting an investigation, COSMOS found Fermin guilty of stealing Bragas phone in
violation of company rules and regulations.[9] Consequently, on 2 October 2003,[10] the company
terminated Fermin from employment after 27 years of service,[11] effective on 6 October 2003.[12] [
Fermin filed a Complaint for Illegal Dismissal.
Issue:
WON Fermin was validly dismissed
Ruling:
Yes.

ATHENA M. SALAS | LABOR CASE DIGEST 2015 102

It must be noted that in the case at bar, all the lower tribunals were in agreement that
Fermins act of taking Bragas cellphone amounted to theft. Factual findings made by
administrative agencies, if established by substantial evidence as borne out by the records, are
final and binding on this Court.
Theft committed against a co-employee is considered as a case analogous to serious
misconduct, for which the penalty of dismissal from service may be meted out to the erring
employee.
Finally, it must be emphasized that the award of financial compensation or assistance to an
employee validly dismissed from service has no basis in law. Therefore, considering that
Fermins act of taking the cellphone of his co-employee is a case analogous to serious
misconduct, this Court is constrained to reverse the CAs ruling as regards the payment of his
full retirement benefits. In the same breath, neither can this Court grant his prayer for
backwages.

49. Reyes-Ravel vs. Philippine Luen Thai Holdings Corp., G.R. No. 174893, July 11, 2012
50. Reyes-Rayel vs. Philippine Luen Thai Holdings Corp., G.R. No. 174893, July 11, 2012
Facts:
In February 2000, PLTHC hired petitioner as Corporate Human Resources (CHR) Director for
Manufacturing for its subsidiary/affiliate company, L&T. In the employment contract, 5 petitioner
was tasked to perform functions in relation to administration, recruitment, benefits,
audit/compliance, policy development/structure, project plan, and such other works as may be
assigned by her immediate superior, Frank Sauceda (Sauceda), PLTHC's Corporate Director for
Human Resources.
On September 6, 2001, petitioner received a Prerequisite Notice from Sauceda and the
Corporate Legal Counsel of PLTHC, Ma. Lorelie T. Edles with reference to her failure to perform
in accordance with management directives in various instances, which collectively have resulted
in loss of confidence because on numerous occasions. Also, in the presence of colleagues and
subordinates, she made statements that serve to undermine the Company's efforts at pursuing
the HR2 Program of which the other colleagues complained about her.
She explained that her alleged failure to perform management directives could be attributed to
the lack of effective communication with her superiors due to malfunctioning email system. This
caused her to miss certain directives coming from her superiors and likewise, for her superiors
to overlook the reports she was submitting. She denied uttering negative comments about the
HR2 Program and instead claimed to have intimated her support for it.

ATHENA M. SALAS | LABOR CASE DIGEST 2015 103

Petitioner was dismissed from the service for loss of confidence on her ability to promote the
interests of the company. This led petitioner to file a Complaint for illegal dismissal, payment of
separation pay, 13th month pay, moral and exemplary damages, attorney's fees, and other
unpaid company benefits against respondents and its officers.
Issue:
Whether or not there was a valid dismissal.
Ruling:
There exists a valid ground for petitioner's termination for employment.
Jurisprudence provides that an employer has a distinct prerogative and wider latitude of
discretion in dismissing a managerial personnel who performs functions which by their nature
require the employer's full trust and confidence. As distinguished from a rank and file personnel,
mere existence of a basis for believing that a managerial employee has breached the trust of
the employer justifies dismissal. Loss of confidence as a ground for dismissal does not require
proof beyond reasonable doubt as the law requires only that there be at least some basis to
justify it.
Petitioner was L&T's CHR Director for Manufacturing. As such, she was directly responsible for
managing her own departmental staff. It is therefore without question that the CHR Director for
Manufacturing is a managerial position saddled with great responsibility. Because of this,
petitioner must enjoy the full trust and confidence of her superiors. Not only that, she ought to
know that she is "bound by more exacting work ethics" and should live up to this high standard
of responsibility.
Records show that petitioner indeed unreasonably failed to effectively communicate with her
immediate superior. There was an apparent neglect in her obligation to maintain constant
communication with Sauceda in order to ensure that her work is up to par. Second, the affidavits
of petitioner's co-workers revealed her negative attitude and unprofessional behavior towards
them and the company. The third and most important is petitioner's display of inefficiency and
ineptitude in her job.
An employer has the right to regulate, according to its discretion and best judgment, all aspects
of employment, including work assignment, working methods, processes to be followed, working
regulations, transfer of employees, work supervision, lay-off of workers and the discipline,
dismissal and recall of workers soo long as they are exercised in good faith.

ATHENA M. SALAS | LABOR CASE DIGEST 2015 104

51. JOMAR S. VERDADERO, Petitioner, vs. BARNEY AUTOLINES GROUP OF COMPANIES


TRANSPORT, INC., and/or BARNEY D. CHITO, ROSELA F. CHITO and GEIURDO
GIMENEZ, G.R. No. 195428 . 29 August 2012

Facts:
Respondent Barney Autolines Group of Companies Transport, Inc. (BALGCO) hired Verdadero
as bus conductor and paid him a salary on commission basis. an altercation took place between
Verdadero and respondent Atty. Gerardo Gimenez (Gimenez), BALGCOs Disciplinary Officer.
Gimenez was on board BALGCO Bus together with his wife and four other companions.
Verdadero was then the assigned bus conductor. BALGCO has a company policy of granting
free rides to company employees and their wives. The story started when Verdadero began
issuing fare tickets to passengers, including the wife of Gimenez.
Gimenez filed an unverified complaint for serious misconduct against Verdadero before the
BALGCO Management. He requested Barney D. Chito (Barney) and Rosela F. Chito (Rosela),
owners of BALGCO, to preside over the conciliation proceedings. Verdadero, accompanied by
his father, appeared at the BALGCO Office. Verdadero was said to have shown willingness to
be penalized for his misconduct provided no record of the proceedings would be made.
Gimenez, on the other hand, was willing to waive the imposition of any penalty if Verdadero
would give a simple letter of apology, which the latter supposedly agreed with his father
guaranteeing the same. Verdadero, instead, submitted his counter-affidavit refuting all
allegations in the written complaint against him. Verdadero furtively reported for work for fear of
having another confrontation with Gimenez. Rosela sent Verdadero a letter requiring him to
immediately report for work and finish the pending disciplinary proceedings against him.
Verdadero submitted his Letter-Reply, explaining that he had been receiving threats. He likewise
believed he was already illegally dismissed as he was not given any work assignment.
Verdadero filed a complaint for illegal dismissal before the Labor Arbiter (LA), claiming, as well,
non-payment of holiday pay, premium on holiday, 13th month pay, separation pay, retirement
benefits, moral and exemplary damages, and reinstatement plus backwages. LA rendered a
Decision dismissing Verdadero's complaint and declaring that no dismissal took place but
merely an administrative investigation. The LA reasoned that Verdadero made it impossible for
BALGCO to give him any trip assignment as he reported for work only when the respondents
were not around. Verdadero filed an appeal before the NLRC. The sworn statement of BALGCO
Electrician, who witnessed the incident, was given weight by the NLRC. It apparently found the
sworn affidavit to be corroborative of Verdadero's testimonies. For said reason, the NLRC
partially granted the appeal. It ruled that Verdadero was illegally dismissed and awarded him
backwages and separation pay. The Court of Appeals found no constructive dismissal. Hence
this petition.
Issue:

ATHENA M. SALAS | LABOR CASE DIGEST 2015 105

Whether or not petitioner Verdadero was constructively dismissed.


Ruling:
None.
Verdadero alleges that he was employed as bus conductor of BALGCO from September 10,
2004 until January 28, 2008 when he was no longer allowed to report for work. He claims that
he was not given any trip assignment since the January 27, 2008 incident. He argues that when
Gimenez committed the verbal abuse against him in the presence of the bus passengers and
threatened him with physical harm, there was termination by the employee of his employment
under the doctrine of constructive dismissal. BALGCO contends that Verdadero was not given
any trip assignment because he was surreptitiously reporting for work and would come to the
office only when Gimenez was not around. This was confirmed in the letterreply by Rosela to
Verdadero. Verdadero admitted not reporting for work after the incident because of his mortal
fear of being harmed by the Disciplinary Officer and his friends.
Constructive dismissal exists where there is cessation of work, because "continued employment
is rendered impossible, unreasonable or unlikely, as an offer involving a demotion in rank or a
diminution in pay" and other benefits. Aptly called a dismissal in disguise or an act amounting to
dismissal but made to appear as if it were not, constructive dismissal may, likewise, exist if an
act of clear discrimination, insensibility, or disdain by an employer becomes so unbearable on
the part of the employee that it could foreclose any choice by him except to forego his continued
employment. In this case, Verdadero cannot be deemed constructively dismissed. Records do
not show any demotion in rank or a diminution in pay made against him. Neither was there any
act of clear discrimination, insensibility or disdain committed by BALGCO against Verdadero
which would justify or force him to terminate his employment from the company. To support his
contention of constructive dismissal, Verdadero considers the verbal abuse by Gimenez against
him as an act which rendered his continued employment impossible, unreasonable or unlikely.
The claimed abuse was corroborated by the sworn written statement executed by Mascaria,
which was given credence by the NLRC and the CA. With the alleged threats of Gimenez,
Verdadero believed that he could no longer stay and work for BALGCO. It is to be emphasized
that the abovementioned acts should have been committed by the employer against the
employee. Unlawful acts committed by a co-employee will not bring the matter within the ambit
of constructive dismissal.
Assuming arguendo that, Gimenez did commit the alleged unlawful acts, still, this fact will not
suffice to conclude that constructive dismissal was proper. Contrary to the arguments of
Verdadero, Gimenez is not the employer. He may be the disciplinary officer, but his functions
as such, as can be gleaned from the BALGCO Rules and Regulations, do not involve the power
or authority to dismiss or even suspend an employee. Such power is exclusively lodged in the
BALGCO management. Gimenez remains to be a mere employee of BALGCO and, thus,
cannot cause the dismissal or even the constructive dismissal of Verdadero. The employers are
BALGCO and its owners, Barney and Rosela. Moreover, it was not established that BALGCO

ATHENA M. SALAS | LABOR CASE DIGEST 2015 106

itself or its owners had been, in any way, forcing Verdadero to resign from his employment. In
fact, records show that the management had been urging him to report back to work, not only to
face the administrative charge against him, but also because of the scarcity and necessity of
bus conductors in the company. Verdadero, however, failed to present himself before the
management, more specifically, to Rosela. This situation provided no opportunity for BALGCO
to give him any trip assignment. The abovementioned act of BALGCO was even misinterpreted
by Verdadero as yet another means of harassment. Furthermore, records are bereft of any
showing that Verdadero was no longer allowed to report for work starting January 28, 2008
when Gimenez lodged a complaint for serious misconduct against him before the BALGCO
management. It was Verdadero himself who terminated his employment.
In the case at bench, considering that there has been no dismissal at all, there can be no
reinstatement. In the same vein, no separation pay can be awarded as it is given only in lieu of
reinstatement. Consequently, there is likewise no justification for the award of backwages.

52. ALEX Q. NARANJO, DONNALYN DE GUZMAN, RONALD V. CRUZ, ROSEMARIE P.


PIMENTEL, and ROWENA B. BARDAJE, Petitioners, vs BIOMEDICA HEALTH CARE, INC.
and CARINA "KAREN" J. MOTOL, Respondents. G.R. No. 193789. 19 September 2012
Facts:
Respondent Biomedica Health Care, Inc. (Biomedica) was, during the material period, engaged
in the distribution of medical equipment. Respondent Carina Karen J. Motol (Motol) was then
its President. Petitioners were former employees of Biomedica. On November 7, 2006, which
happened to be Motols birthday, petitioners with two (2) other employees, were all absent for
various personal reasons. Notably, these are the same employees who filed a letter-complaint
addressed to Director Lourdes M. Transmonte, National Director, National Capital RegionDepartment of Labor and Employment (DOLE) against Biomedica for lack of salary increases,
failure to remit Social Security System and Pag-IBIG contributions, and violation of the minimum
wage law, among other grievances. Per available records, the complaint has not been acted
upon.
Later that day, petitioners reported for work after receiving text messages for them to proceed to
Biomedica. They were, however, refused entry and told to start looking for another workplace.
The next day, petitioners allegedly came in for work but were not allowed to enter the premises.
Motol purportedly informed petitioners, using foul language, to just find other employment.
Biomedica issued a notice of preventive suspension and notices to explain within 24 hours
(Notices) to petitioners. In the Notices, Biomedica accused the petitioners of having conducted
an illegal strike and were accordingly directed to explain why they should not be held guilty of
and dismissed for violating the company policy against illegal strikes. Petitioners filed a
Complaint with the NLRC for constructive dismissal and nonpayment of salaries, overtime pay,

ATHENA M. SALAS | LABOR CASE DIGEST 2015 107

13th month pay as well as non-remittance of SSS, Pag-IBIG and Philhealth contributions as well
as loan payments. Thereafter, Biomedica served Notices of Termination on petitioners.
The Labor Arbiter dismissed the complaint for lack of merit but ordered respondents to pay. The
Labor Arbiter found that, indeed, petitioners engaged in a mass leave akin to a strike. He added
that, assuming that petitioners were not aware of the company policies on illegal strikes, such
mass leave can sufficiently be deemed as serious misconduct under Art. 282 of the Labor Code.
Thus, the Labor Arbiter concluded that petitioners were validly dismissed. Unlike the Labor
Arbiter, the NLRC found and so declared petitioners to have been illegally dismissed. The Court
of Appeals, however, reversed the decision of the NLRC and reinstated the decision of the
Labor Arbiter.
Issue:
Whether or not petitioners were illegally dismissed.
Ruling:
Petitioners were illegally dismissed.
The fundamental law of the land guarantees security of tenure, thus:
Sec. 3. The State shall afford full protection to labor x x x.
x x x They shall be entitled to security of tenure, humane conditions of work and a living wage. x
xx
On the other hand, the Labor Code promotes the right of the worker to security of tenure
protecting them against illegal dismissal under ARTICLE 279.
It bears pointing out that in the dismissal of an employee, the law requires that due process be
observed. Such due process requirement is twofold, procedural and substantive, that is, the
termination of employment must be based on a just or authorized cause of dismissal and the
dismissal must be effected after due notice and hearing. In the instant case, petitioners were
not afforded both procedural and substantive due process.
Petitioners were not afforded procedural due process
Art. 277(b) of the Labor Code contains the procedural due process requirements in the
dismissal of an employee. On the other hand, Rule XIII, Book V, Sec. 2 I (a) of the Implementing
Rules and Regulations of the Labor Code states:
SEC. 2. Standards of due process; requirements of notice.In all cases of termination of
employment, the following standards of due process shall be substantially observed:
I. For termination of employment based on just causes as defined in Article 282 of the Code:

ATHENA M. SALAS | LABOR CASE DIGEST 2015 108

(a) A written notice served on the employee specifying the ground or grounds for
termination, and giving said employee reasonable opportunity within which to explain his
side.
xxx
The Court elaborated in King of Kings Transport, Inc. v. Mamac that a mere general description
of the charges against an employee by the employer is insufficient to comply with the provisions
of the law:
x x x Moreover, in order to enable the employees to intelligently prepare their explanation and
defenses, the notice should contain a detailed narration of the facts and circumstances
that will serve as basis for the charge against the employees. A general description of the
charge will not suffice. Lastly, the notice should specifically mention which company rules, if
any, are violated and/or which among the grounds under Art. 282 is being charged against the
employees.
In the instant case, the notice specifying the grounds for termination dated November 9, 2006
states:
Effective upon receipt hereof, you are placed under preventive suspension for willfully
organizing and/or engaging in illegal strike on November 7, 2006. Your said illegal act-in
conspiracy with your other co-employees, paralyzed the company operation on that day and
resulted to undue damage and prejudice to the company and is direct violation of Article
XI, Category Four Section 6, 8, 12, 18 & 25 of our Company Policy, which if found guilty,
you will be meted a penalty of dismissal. Please explain in writing within 24 hours from
receipt hereof why you should not be held guilty of violating the company policy considering
further that you committed and timed such act during the birthday of our Company president.
Clearly, petitioners were charged with conducting an illegal strike, not a mass leave, without
specifying the exact acts that the company considers as constituting an illegal strike or violative
of company policies. Such allegation falls short of the requirement in King of Kings Transport,
Inc. of a detailed narration of the facts and circumstances that will serve as basis for the charge
against the employees. A bare mention of an illegal strike will not suffice.
Further, while Biomedica cites the provisions of the company policy which petitioners
purportedly violated, it failed to quote said provisions in the notice so petitioners can be
adequately informed of the nature of the charges against them and intelligently file their
explanation and defenses to said accusations. The notice is bare of such description of the
company policies. Moreover, it is incumbent upon respondent company to show that petitioners
were duly informed of said company policies at the time of their employment and were given
copies of these policies. No such proof was presented by respondents. There was even no
mention at all that such requirement was met. Worse, respondent Biomedica did not even quote
or reproduce the company policies referred to in the notice.

ATHENA M. SALAS | LABOR CASE DIGEST 2015 109

Moreover, the period of 24 hours allotted to petitioners to answer the notice was severely
insufficient and in violation of the implementing rules of the Labor Code. Under the
implementing rule of Art. 277, an employee should be given reasonable opportunity to file a
response to the notice. Reasonable opportunity under the Omnibus Rules means every kind of
assistance that management must accord to the employees to enable them to prepare
adequately for their defense. This should be construed as a period of at least five (5)
calendar days from receipt of the notice to give the employees an opportunity to study
the accusation against them, consult a union official or lawyer, gather data and evidence,
and decide on the defenses they will raise against the complaint.
In addition, Biomedica did not set the charges against petitioners for hearing or conference in
accordance with Sec. 2, Book V, Rule XIII of the Implementing Rules and Regulations of the
Labor Code. While petitioners did not submit any written explanation to the charges, it is
incumbent for Biomedica to set the matter for hearing or conference to hear the defenses and
receive evidence of the employees. More importantly, Biomedica is duty-bound to exert efforts,
during said hearing or conference, to hammer out a settlement of its differences with petitioners.
These prescriptions Biomedica failed to satisfy.
Lastly, Biomedica again deviated from the dictated contents of a written notice of termination as
laid down in Sec. 2, Book V, Rule XIII of the Implementing Rules that it should embody the facts
and circumstances to support the grounds justifying the termination. The Notice of Termination
issued by Biomedica miserably failed to satisfy the requisite contents of a valid notice of
termination, as it simply mentioned the failure of petitioners to submit their respective written
explanations without discussing the facts and circumstances to support the alleged violations of
Secs. 6, 8, 12, 18 and 25 of Category Four, Art. XI of the alleged company rules.
Petitioners were denied substantive due process
In any event, petitioners were also not afforded substantive due process, that is, they were
illegally dismissed. The just causes for the dismissal of an employee are exclusively found in
Art. 282(a) of the Labor Code. It was on the ground of serious misconduct that the CA upheld
the dismissal of petitioners from their employment.
Misconduct has been defined as improper or wrong conduct; the transgression of some
established and definite rule of action, a forbidden act, a dereliction of duty, unlawful in
character implying wrongful intent and not mere error of judgment. The misconduct to be
serious must be of such grave and aggravated character and not merely trivial and unimportant.
In the instant case, Biomedica failed to even establish that petitioners indeed violated company
rules, failing to even present a copy of the rules and to prove that petitioners were made aware
of such regulations. In fact, from the records of the case, Biomedica has failed to prove that
petitioners are guilty of a wrongdoing that is punishable with termination from employment. Art.
277(b) of the Labor Code states, The burden of proving that the termination was for a valid or
authorized cause shall rest on the employer. In the instant case, Biomedica failed to overcome
such burden.

ATHENA M. SALAS | LABOR CASE DIGEST 2015 110

Petitioners did not stage a mass leave


The accusation is for engaging in a mass leave tantamount to an illegal strike. The term Mass
Leave has been left undefined by the Labor Code. Plainly, the legislature intended that the
terms ordinary sense be used. Mass is defined as participated in, attended by, or affecting a
large number of individuals; having a large-scale character. While the term Leave is defined
as an authorized absence or vacation from duty or employment usually with pay.
Thus, the phrase mass leave may refer to a simultaneous availment of authorized leave
benefits by a large number of employees in a company. It is undeniable that going on leave or
absenting ones self from work for personal reasons when they have leave benefits available is
an employees right. In addition to sick leave, the company, as a policy or practice or as agreed
to in a CBA, grants vacation leave to employees. Lastly, even the Labor Code grants a service
incentive leave of 5 days to employees.
In the factual milieu at bar, Biomedica did not submit a copy of the CBA or a company
memorandum or circular showing the authorized sick or vacation leaves which petitioners can
avail of. Neither is there any document to show the procedure by which such leaves can be
enjoyed. Absent such pertinent documentary evidence, the Court can only conclude that the
availment of petitioners of their respective leaves on November 7, 2006 was authorized, valid
and in accordance with the company or CBA rules on entitlement to and availment of such
leaves.
Moreover, a mass leave involves a large number of people or in this case, workers. Here, the
five (5) petitioners were absent on November 7, 2006. The records are bereft of any evidence to
establish how many workers are employed in Biomedica. There is no evidence on record that 5
employees constitute a substantial number of employees of Biomedica. Having failed to show
that there was a mass leave, the Court concludes that there were only individual availment of
their leaves by petitioners and they cannot be held guilty of any wrongdoing, much less anything
to justify their dismissal from employment. On this ground alone, the petition must be granted.
Petitioners did not go on strike
Granting for the sake of argument that the absence of the 5 petitioners on November 7, 2006 is
considered a mass leave, still, their actions cannot be considered a strike. Art. 212(o) of the
Labor Code defines a strike as any temporary stoppage of work by the concerted action of
employees as a result of any industrial or labor dispute. Concerted is defined as mutually
contrived or planned or performed in unison.38 In the case at bar, the 5 petitioners went on
leave for various reasons. Biomedica did not prove that the individual absences can be
considered as temporary stoppage of work. Biomedicas allegation that the mass leave
paralyzed the company operation on that day has remained unproved.
Dismissal is not the proper penalty

ATHENA M. SALAS | LABOR CASE DIGEST 2015 111

But setting aside from the nonce the facts established above, the most pivotal argument against
the dismissal of petitioners is that the penalty of dismissal from employment cannot be imposed
even if we assume that petitioners went on an illegal strike. It has not been shown that
petitioners are officers of the Union. In the instant case, Biomedica has not alleged, let alone,
proved the commission by petitioners of any illegal act during the alleged mass leave. There
being none, the mere fact that petitioners conducted an illegal strike cannot be a legal basis for
their dismissal.
Petitioners are entitled to separation pay in lieu of reinstatement, backwages and
nominal damages.

53. THE NEW PHILIPPINE SKYLANDERS, INC. and/or JENNIFER M. ENANO-BOTE , vs.
FRANCISCO N. DAKILA, G.R. No. 199547. September 24, 2012
FACTS:
Respondent Dakila was employed by petitioner corporation as early as 1987 and terminated for
cause in April 1997 when the corporation was sold. In May 1997, he was rehired as consultant
by the petitioners under a Contract for Consultancy Services dated April 30, 1997.
Thereafter, in a letter dated April 19, 2007, respondent Dakila informed petitioners of his
compulsory retirement effective May 2, 2007 and sought for the payment of his retirement
benefits pursuant to the Collective Bargaining Agreement. His request, however, was not acted
upon. Instead, he was terminated from service effective May 1, 2007.
Consequently, respondent Dakila filed a complaint for constructive illegal dismissal, nonpayment of retirement benefits, under/non-payment of wages and other benefits of a regular
employee, and damages against petitioners before the NLRC. He averred, among others, that
the consultancy contract was a scheme to deprive him of the benefits of regularization, claiming
to have assumed tasks necessary and desirable in the trade or business of petitioners and
under their direct control and supervision.
On the other hand, petitioners, in their position paper, asserted that respondent Dakila was a
consultant and not their regular employee. The latter was not included in petitioners' payroll and
paid a fixed amount under the consultancy contract. He was not required to observe regular
working hours and was free to adopt means and methods to accomplish his task except as to
the results of the work required of him. Hence, no employer-employee relationship existed
between them. Moreover, respondent Dakila terminated his contract in a letter dated April 19,
2007, thus, negating his dismissal.
ISSUES:

ATHENA M. SALAS | LABOR CASE DIGEST 2015 112

Whether or not respondent Dakila is a regular employee; and whether or not he has been
illegally dismissed.
RULING:
The issue of illegal dismissal is premised on the existence of an employer-employee
relationship between the parties herein. Records reveal that both the LA and the NLRC, as
affirmed by the CA, have found substantial evidence to show that respondent Dakila was a
regular employee who was dismissed without cause. Respondent Dakila is found to be a regular
employee on the basis of the unrebutted documentary evidence showing that he was under the
petitioners' direct control and supervision and performed tasks that were either incidental or
usually desirable and necessary in the trade or business of petitioner corporation for a period of
ten years.
Following Article 279 of the Labor Code, an employee who is unjustly dismissed from work is
entitled to reinstatement without loss of seniority rights and other privileges and to his full
backwages computed from the time he was illegally dismissed. However, considering that
respondent Dakila was terminated on May 1, 2007, or one (1) day prior to his compulsory
retirement on May 2, 2007, his reinstatement is no longer feasible. Accordingly, the NLRC
correctly held him entitled to the payment of his retirement benefits pursuant to the CBA. On the
other hand, his backwages should be computed only for days prior to his compulsory retirement
which in this case is only a day. Consequently, the award of reinstatement wages pending
appeal must be deleted for lack of basis.

54. LENN MORALES vs. METROPOLITAN BANK AND TRUST COMPANY, G.R. No. 182475
November 21, 2012
FACTS:
Petitioner Lenn Morales was hired by Solidbank as Teller for its Rizal Avenue Branch in
Tacloban City. With said banks merger with respondent Metrobank, the latter, as surviving
entity, absorbed Morales and assigned him to its Customer Service Relations-Reserve Pool
(CSR-RP) which was composed of employees who, with no permanent places of assignment,
acted as relievers whenever temporary vacancies arise in other branches. Designated as
reliever for Metrobanks Main Branch in Tacloban City, Morales was likewise assigned to work in
the same capacity for the banks other Visayas Region III branches. From a job with a grade
four rank, Morales was subsequently promoted to the position of Customer Service
Representative (CSR), with a job grade 6 rank. It was while occupying the latter position that
Morales was informed by Federico Mariano, the Senior Manager of Metrobanks Tacloban City
Main Branch, that he was covered by the banks Special Separation Program (SSP) and that, in
accordance therewith, his employment was going to be terminated on the ground of
redundancy.

ATHENA M. SALAS | LABOR CASE DIGEST 2015 113

Having signed a form on the same day signifying his unqualified and unconditional acceptance
of Metrobanks decision to terminate his employment, Morales execute a Release, Waiver and
Quitclaim acknowledging receipt of the sum of P158,496.95 as full payment of his monetary
entitlements.
On 20 February 2004, Morales filed against Metrobank a complaint for illegal dismissal,
separation pay, backwages, moral and exemplary damages as well as attorneys fees.
ISSUE:
Whether or not the dismissal of petitioner was based on authorized cause of redundancy.
RULING:
One of the authorized causes for the dismissal of an employee, redundancy exists when the
service capability of the workforce is in excess of what is reasonably needed to meet the
demands of the business enterprise. A position is redundant when it is superfluous, and
superfluity of a position or positions could be the result of a number of factors, such as the
overhiring of workers, a decrease in the volume of business or the dropping of a particular line
or service previously manufactured or undertaken by the enterprise. Time and again, it has been
ruled that an employer has no legal obligation to keep more employees than are necessary for
the operation of its business. For the implementation of a redundancy program to be valid,
however, the employer must comply with the following requisites: (1) written notice served on
both the employees and the DOLE at least one month prior to the intended date of termination
of employment; (2) payment of separation pay equivalent to at least one month pay for every
year of service; (3) good faith in abolishing the redundant positions; and (4) fair and reasonable
criteria in ascertaining what positions are to be declared redundant and accordingly abolished.
In implementing a redundancy program, it has been ruled that the employer is required to adopt
a fair and reasonable criteria, taking into consideration such factors as (a) preferred status; (b)
efficiency; and (c) seniority, among others. Consistent with this principle, Metrobank established
that, as a direct result of the adoption of the HRP, it was determined that the volume of
transactions in Visayas Region III required the further reduction of its eight-man reserve pool by
two employees. As these employees had no permanent place of assignment and merely acted
as relievers whenever temporary vacancies arise in other branches, they were the most logical
candidates for inclusion in the SSP. Already lacking preferred status in Metrobanks hierarchy of
positions, Morales was included in the SSP because of his poor work performance which
reportedly caused complaints from the branches where he was temporarily assigned as
reliever. To our mind, the foregoing circumstances contradict Morales claim that he was
arbitrarily singled out for termination by Metrobank which, having validly determined the surplus
in its manpower complement, appears to have appropriately identified him as a candidate for
the SSP on account of his work attitude.

55. Mirant vs. Sario G.R. No. 197598, 21 Nov 2012

ATHENA M. SALAS | LABOR CASE DIGEST 2015 114

FACTS:
Danilo A. Sario (Sario) filed a complaint for illegal dismissal, backwages, damages and
attorneys fees against the petitioner,Mirant (Philippines) Corporation (company), and its
officers, namely: Harris (President); Sliman(Executive Vice-President forOperations);
and Aprieto (Officer-in-Charge, MMD). The company owns shares in Mirant Sual Corporation
and Mirant PagbilaoCorporation which operate power stations in the provinces of Pangasinan
and Quezon.Sario worked for the company as procurement officer from March 1998 to October
2005. Allegedly, the companydiscovered that some of its employees had been involved in the
rampant practice of favoring certain suppliers, therebyseriously impairing transparency in its
procurement process and compromising the quality of purchased materials. To curbthe practice,
the company issued the 2002 MMD Policies and Procedures Manual (2002 Procurement
Manual) for theguidance of its employees and officers in soliciting bid quotations and proposals
from vendors, suppliers and contractors. The2002 Procurement Manual was duly disseminated
through seminars and it became effective in January 2002.The 2002 Procurement Manual was
replaced by the 2004 Procurement Policies and Procedures Manual (2004Procurement Manual)
which was disseminated and which became effective on August 31, 2004. Again, seminars
wereconducted and a proficiency examination was administered to familiarize the company
buyers/procurement officers and theteam leaders with the 2004 Procurement Manual.
Sario took the proficiency examination on September 28, 2004.On September 8, 2005, Sario
received a Show Cause Notice from the company, advising him that based on an internalaudit,
he was found to have committed the following violations:1. Non-compliance with the Minimum
Bid/Quotation Requirements;2. Non-compliance with the Single Tender Justification
Requirement;3. No Evidence of Independent Approval of the PRF;4. No Evidence of Authorized
Recommendation or Approval of the PO;5. PO not Awarded to the lowest Bidder; and6. No TAS
Attached.Sario was given ten (10) days, or until September 18, 2005, to explain why no
disciplinary action should be taken against himfor the violations. He was also notified that an
investigation would be conducted on the matter. He was placed on preventivesuspension
pending the investigation.As a result of the administrative hearing, Sliman sent Sario a letter
informing him of the termination of hisemployment for his failure to comply with the standard operating
procedures/instructions; for his serious misconduct orwillful disobedience of the lawful orders of
the company in connection with his work; and for his gross and habitual neglect of his duties.
The company found Sario liable for his failure to comply with the 2002 and 2004 Procurement
Manuals, especiallyhis unabated practice of sending Requests for Quotation (RFQs) to
suppliers who have a history of not responding to requestsor of not sending quotes. The
practice, the company lamented, resulted in the issuance of purchase orders to the lone
bidders.Labor Arbiter (LA) Anni declared Sario to have been illegally dismissed. Consequently,
he ordered: (1) Sarios immediate reinstatement without loss of seniority rights and other
privileges; and (2) the company, Sliman and Aprieto,jointly and severally, to pay Sario back
wages, moral damages, exemplary damages and attorneys fees. Labor Arbiter Anniabsolved
Harris from liability.The respondents appealed to the NLRC, which reversed the labor arbiters
ruling and dismissed the complaint forlack of merit. Sario moved for reconsideration, but the
NLRC denied the motion. He then sought relief from the CA, through apetition for certiorari

ATHENA M. SALAS | LABOR CASE DIGEST 2015 115

under Rule 65 of the Rules of Court.In its decision, the CA granted the petition. It set aside the
NLRC rulings and reinstated the Labor Arbiters decision, with modifications. It deleted the
award of moral and exemplary damages, and absolved Harris, Sliman and Aprieto fromliability
in the case. Like the Labor Arbiter, it found the penalty of dismissal meted on Sario too harsh.
The company moved forpartial reconsideration, but the CA denied the motion; hence, the
present recourse.
ISSUE:
Whether Sario was legally dismissed.
RULING:
We find the petition meritorious. The Supreme Court held in favor of Mirant, finding that Sario
was legally dismissed. Under the law, the burden of proving that the termination of a workers
employment was for a valid or authorized cause rests on the employer. In this case, the
company was able to prove that Sarios dismissal was for a valid cause. Throughhis repeated
violations of the companys 2002 and 2004 Procurement Manuals, Sario committed a serious
misconduct or willful disobedience of the lawful directives or orders of his employer, constituting
a just cause for termination of employment. Sario was not an ordinary rank-and-file employee.
He was a procurement officer. While he did not occupy a highposition in the company hierarchy,
the nature of his work made him, as the company avers, a vital cog in its procurement program.
The effectiveness of the program depended in no small measure on the people running it, i.e.,
from the lowliest employee to the highest official. Sario was one of these people and he was
occupying, not a lowly but, a middle position.
This position carries with it responsibilities which only he can, and should, answer for.As the
records show, Sario failed to faithfully discharge his duties as procurement officer. Over a span
of almost one-and-a-half years, from January 2004 to May 2005 (not two years as the company
claims), Sario committed 27violations of the2002 and 2004 Procurement Manuals in critical
areas of the procurement process, in particular, non-compliance with theminimum bid/quotation
requirements, non-compliance with the single tender justification requirement, failure to
provideproof of approval of the purchase requisition form, failure to provide proof of authorized recommendation
of the purchaseorder, failure to award purchase order to the lowest bidder, and no tender analysis
summary. Sarios transgressions cannot be mitigated by the supposed approval of his actions
by his superiors. Sario has toaccount for his own actions.
The circumstance that his recommendations were approved by his superiors does not erase
thefact that he repeatedly violated the 2002 and 2004 Procurement Manuals. He was well
aware of his duties and theirparameters, based on the 2002 and 2004 Procurement Manuals.
He committed the violations for one-and-a-half years. Theserepeated violations can only
indicate a willful disobedience to reasonable company rules and regulations.Based on the facts,
the law and jurisprudence, Sario deserves to be dismissed for willful disobedience.

ATHENA M. SALAS | LABOR CASE DIGEST 2015 116

In Gold City Integrated Port Services, Inc. v. NLRC the Court stressed that willful disobedience
of an employee contemplates the concurrence of at least two requisites: the employees
assailed conduct must have been willful or intentional, the willfulnes sbeing characterized by a
"wrongful and perverse attitude"; and the order violated must have been reasonable, lawful
andmade known to the employee, and must pertain to the duties which he had been engaged to
discharge. We find the tworequisites present in this case. Sarios repeated violations of the
companys 2002 and 2004 Procurement Manuals lawful orders in themselves as they provide
the dos and, necessarily, the donts of a procurement officer constitute willful disobedience. He
committed therepeated violations because he knew or was confident that he would not get
caught since his actions were being approved, ashe claims, by his superiors, evidencing wrongful or
perverse intent. While the Constitution urges the moderation of thesanction that may be applied to an
employee where a penalty less punitive would suffice, as the Court pronounced in
MarivalTrading, Inc. v. NLRC, cited by the CA, we do not believe that such a moderation is
proper in this case. Sario has become unfit to remain in employment. A contrary view would be
oppressive to the employer. "The law, in protecting the rights of thelaborer, authorizes neither oppression nor selfdestruction of the employer."
56. Barba vs. Liceo De Cagayan University
Facts:
Petitioner Dr. Ma. Mercedes L. Barba was the Dean of the College of Physical Therapy of
respondent Liceo de CagayanUniversity, Inc., a private educational institution with school
campus located at Carmen, Cagayan de Oro City.Petitioner started working for respondent on
July 8, 1993 as medical officer/school physician for a period of oneschool year or until March
31, 1994. In July 1994, she was chosen by respondent to be the recipient of a scholarship grant
topursue a three-year residency training in Rehabilitation Medicine at the Veterans Memorial
Medical Center (VMMC). TheScholarship Contract provides:
That the SCHOLAR after the duration of her study and training shall serve the SCHOOL in whatever positionthe
SCHOOL desires related to the SCHOLARs studies for a period of not less than ten (10) years;
After completing her residency training with VMMC in June 1997, petitioner returned to continue working for
respondent. Shewas appointed as Acting Dean of the College of Physical Therapy and at the
same time designated as Doctor-In-Charge of theRehabilitation Clinic of the Rodolfo N. Pelaez
Hall, City Memorial Hospital.
On June 19, 2002, petitioners appointment as Doctor-In-Charge of the Rehabilitation Clinic was
renewed and she wasappointed as Dean of the College of Physical Therapy by respondents
President, Dr. Jose Ma. R. Golez. Petitioner accepted herappointment and assumed the
position of Dean of the College of Physical Therapy.In the school year 2003 to 2004, the
College of Physical Therapy suffered a dramatic decline in the number of enrollees from a total
of 1,121 students in the school year 1995 to 1996 to only 29 students in the first semester of

ATHENA M. SALAS | LABOR CASE DIGEST 2015 117

school year2003 to 2004. This worsened in the next year or in school year 2004 to 2005 where
a total of only 20 students enrolled.
Due to the low number of enrollees, respondent decided to freeze the operation of the College
of Physical Therapy indefinitely. Respondents President wrote petitioner a letter informing her
that her services as dean of the said college willend at the close of the school year. Thereafter,
the College of Physical Therapy ceased operations on March 31, 2005, and petitioner went on
leave without pay starting on April 9, 2005. Subsequently, respondents Executive Vice
President, Dr.Lerin,sent petitioner a letter instructing petitioner to return to work on June 1, 2005
and report to Palomares, the Acting Dean of the College of Nursing, to receive her teaching load
and assignment as a full-time faculty member in that department for theschool year 20052006.In reply, petitioner informed Dr. Lerin that she had not committed to teach in the College of
Nursing and that as far asshe can recall, her employment is not dependent on any teaching
load. She then requested for the processing of her separationbenefits in view of the closure of
the College of Physical Therapy. Dr. Magdale, Vice-President for Academic Affairs, sent another
letter to petitioner ordering her to report for work asshe was still bound by the Scholarship Contract to
serve respondent for two more years. But petitioner did not do so.
Hence,on June 28, 2005, Dr. Magdale sent petitioner a notice terminating her services on the
ground of abandonment.Meanwhile, on June 22, 2005, prior to the termination of her services,
petitioner filed a complaint before the LaborArbiter for illegal dismissal, payment of separation
pay and retirement benefits against respondent, Dr. Magdale and Dr. Golez.She alleged that her
transfer to the College of Nursing as a faculty member is a demotion amounting to constructive
dismissal.Labor Arbiter (LA) found that respondent did not constructively dismiss petitioner;
therefore, she was not entitled to separation pay. The Labor Arbiter held that petitioners
assignment as full-time professor in the College of Nursing was not ademotion tantamount to
constructive dismissal.Petitioner appealed the above decision to the NLRC. NLRC reversed the
LAs decision and held that petitioner wasconstructively dismissed. The NLRC held that
petitioner was demoted when she was assigned as a professor in the College of Nursing
because there are functions and obligations and certain allowances and benefits given to a
College Dean but not to anordinary professor.Respondent went to the CA on a petition for certiorari
alleging that the NLRC committed grave abuse of discretion when it declared that petitioners
transfer to the College of Nursing as full-time professor but without diminution of salariesand
without loss of seniority rights amounted to constructive dismissal because there was
a demotion involved in the transferand because petitioner was compelled to accept her new
assignment.The CA reversed and set aside the NLRC resolutions and reinstated the decision of
the Labor Arbiter. The CA did not find merit in respondents assertion in its Supplemental
Petition that the position of petitioner as College Dean was a corporate office. Instead, the
appellate court held that petitioner was respondents employee
ISSUE:
Whether petitioner was constructively dismissed.

ATHENA M. SALAS | LABOR CASE DIGEST 2015 118

RULING:
We grant the petition. The Supreme Court ruled in favor of the University, finding that Barba was
not constructivelydismissed, but was instead validly transferred.On the issue of constructive
dismissal, we agree with the Labor Arbiter and the appellate courts earlier ruling that petitioner
was not constructively dismissed. Petitioners letter of appointment specifically appointed her as
Dean of the
College of Physical Therapy and Doctor-in-Charge of the Rehabilitation Clinic for a period of
three years effective July 1, 2002 unless sooner revoked for valid cause or causes. Evidently,
petitioners appointment as College Dean was for a fixed term, subject to reappointment and
revocation or termination for a valid cause. When respondent decided to close its College
of Physical Therapy due to drastic decrease in enrollees, petitioners appointment as its College
Dean was validly revoked andher subsequent assignment to teach in the College of Nursing
was justified as it is still related to her scholarship studies inPhysical Therapy.As we observed in
Brent School, Inc. v. Zamora also cited by the CA, it is common practice in educational institutionsto
have fixed-term contracts in administrative positions, thus:
Some familiar examples may be cited of employment contracts which may be neither for seasonal work nor
forspecific projects, but to which a fixed term is an essential and natural appurtenance: overseas
employment contracts, for one, to which, whatever the nature of the engagement, the concept of regular
employment with allthat it implies does not appear ever to have been applied, Article 280 of the Labor Code
notwithstanding; alsoappointments to the positions of dean, assistant dean, college secretary,
principal, and otheradministrative offices in educational institutions, which are by practice
or tradition rotated among thefaculty members, and where fixed terms are a necessity without
which no reasonable rotation would bepossible.x x x
In constructive dismissal cases, the employer has the burden of proving that its conduct and
action or the transfer of anemployee are for valid and legitimate grounds such as genuine
business necessity.
Particularly, for a transfer not to beconsidered a constructive dismissal, the employer must be
able to show that such transfer is not unreasonable, inconvenient, or prejudicial to the
employee. In this case, petitioners transfer was not unreasonable, inconvenient or prejudicial to
her. Onthe contrary, the assignment of a teaching load in the College of Nursing was
undertaken by respondent to accommodatepetitioner following the closure of the College of
Physical Therapy. Respondent further considered the fact that petitioner stillhas two years to
serve the university under the Scholarship Contract. Petitioners subsequent transfer to another
department or college is not tantamount to demotion as it was a valid transfer. There is therefore
no constructive dismissal to speak of. That petitioner ceased to enjoy the compensation,
privilegesand benefits as College Dean was but a logical consequence of the valid revocation or
termination of such fixed-term position.Indeed, it would be absurd and unjust for respondent to
maintain a deanship position in a college or department that hasceased to exist. Under the
circumstances, giving petitioner a teaching load in another College/Department that is related
toPhysical Therapy thus enabling her to serve and complete her remaining two years under the
Scholarship Contract is avalid exercise of management prerogative on the part of respondent.

ATHENA M. SALAS | LABOR CASE DIGEST 2015 119

CA held that contrary to theallegation of petitioner, the position of Dean does not appear to be
the same as that of a College Director.Aside from the obvious disparity in name, the By-Laws of
LDCU provides for onlyone College Director. But as shownby LDCU itself,
Numerous persons have been appointed as Deans. If it is indeed the intention of LDCU to give its manyDeans
the rank of College Director, then it exceeded the authority given to it by its By-Laws because only one
CollegeDirector is authorized to be appointed. It must amend its By-Laws.Petitioner filed a motion for
reconsideration from the above decision, but her motion was denied by the CA. Hence,
petitioner filed the present petition.

57. SAMPAGUITA AUTO TRANSPORT CORPORATION vs. NLRC, G.R. No.


197384. January 30, 2013
Facts:
Sagad alleged that on May 14, 2006, the company hired him as a regular bus driver, not as a
probationary employee as the company claimed. He disowned his purported signature on the
contract of probationary employment 5 submitted in evidence by the company. He maintained
that his signature was forged. He further alleged that on November 5, 2006, he was dismissed
by the company for allegedly conniving with conductor Vitola in issuing tickets outside their
assigned route. EScIAa
The company countered that it employed Sagad as a probationary bus driver 6 from May 14,
2006 to October 14, 2006; he was duly informed of his corresponding duties and
responsibilities. 7 He was further informed that during the probationary period, his attendance,
performance and work attitude shall be evaluated to determine whether he would qualify for
regular employment. For this purpose and as a matter of company policy, an evaluator was
deployed on a company bus (in the guise of a passenger) to observe the driver's work
performance and attitude.
Allegedly, on September 21, 2006, an evaluator boarded Sagad's bus. The evaluator described
Sagad's manner of driving as "reckless driver, nakikipaggitgitan, nakikipaghabulan, nagsasakay
sa gitna ng kalsada, sumusubsob ang pasahero[.]"8 Sagad disputed the evaluator's
observations. In an explanation (rendered in Filipino), 9 he claimed that he could not have been
driving as reported because his wife (who was pregnant) and one of his children were with him
on the bus. He admitted though that at one time, he chased an "Everlasting" bus to serve
warning on its driver not to block his bus when he was overtaking. He also admitted that once in
a while, he sped up to make up for lost time in making trips.
The company further alleged that on October 13, 2006, it conducted a thorough evaluation of
Sagad's performance. It requested conductors who had worked with Sagad to comment on his
work. Conductors A. Hemoroz and Israel Lucero revealed that Sagad proposed that they cheat
on the company by way of an unreported early bus trip. 10 The company also cited Sagad's

ATHENA M. SALAS | LABOR CASE DIGEST 2015 120

involvement in a hit-and-run accident on September 9, 2006 along Commonwealth Avenue in


Quezon City while on a trip. Allegedly, Sagad did not report the accident to the company.
On October 15, 2006, upon conclusion of the evaluation, the company terminated Sagad's
employment for his failure to qualify as a regular employee.
Issue:
Whether or not Saga was dismissed illegally.
Ruling:
Sagad was dismissed, not as a probationary employee, but as one who had attained regular
status. The company's evidence on Sagad's purported hiring as a probationary employee is
inconclusive.
The NLRC resolved the doubt in Sagad's favor, ruling that Sagad's signature on the contract
was not his, because it was a forgery. It declared that his signature on the contract "is extremely
different from those in his pleadings and from the other documents on record[,]"33 without
explaining how and why the two sets of signatures were vastly different. Lending further support
to the NLRC conclusion, which the CA upheld, is its finding that the company failed to refute
Sagad's denial of his signature in the contract, which the labor tribunal considered as an
admission of the veracity of Sagad's statement, pursuant to the Rules of Court.
Independently of the above discussion and even if we were to consider that Sagad went through
a probationary period, the records indicate that he was retained even beyond the expiration of
his supposed probationary employment on October 14, 2006. As the NLRC noted, Sagad
claimed that he was dismissed by the company on November 5, 2006, after he was accused of
conniving with conductor Vitola in issuing tickets outside their assigned route.
The company never refuted this particular assertion of Sagad and its silence can only mean that
Sagad remained in employment until November 4, 2006, thereby attaining regular status as of
that date. Under the law, "an employee who is allowed to work after a probationary period shall
be considered a regular employee."
Is Sagad's dismissal illegal?
We disagree with the finding that Sagad's dismissal had no basis.
First. It is not disputed that the company called Sagad's attention to his negative actuations as
a bus driver, which were reported by a company evaluator 38 who boarded his bus on
September 21, 2006. The evaluator reported that he was driving recklessly, racing and jostling
for position on the road, thereby jarring the passengers on their seats, and picking up
passengers on the middle of the road. He disputed the evaluator's observations, 39 claiming
that he could not have been driving as reported because his pregnant wife and one of his

ATHENA M. SALAS | LABOR CASE DIGEST 2015 121

children were with him on the bus at the time. He admitted, however, that on one occasion, he
chased an "Everlasting" bus to warn its driver not to block him. He also admitted that once in a
while, he sped up to compensate for lost time in his trips.
Sagad's explanation reveals more than what it stated. During his brief employment with the
company, he exhibited the tendency to speed up when he finds the need for it, very obviously in
violation of traffic rules, regulations and company policy. Instead of negating the evaluator's
observations, his admissions make them credible.
Second. He was also asked to react to the comments of conductors who had worked with him
(Hemoroz and Lucero) to the effect that he proposed to them that they cheat on the company by
making early (but not to be reported) bus trips. Further, there was Castillo's evaluation dated
October 13, 2006, rating Sagad's work performance as poor on account of: (1) the low revenue
of Sagad's bus; (2) his inability to make all his scheduled trips; and (3) his habit of bringing his
wife with him on his trips. Castillo also heard of talks of Sagad's orders to the conductors to earn
money in a questionable way.
During the arbitration, Sagad disputed the conductors' comments, maintaining that they were
not under oath and that the fraudulent proposal they mentioned could only be committed by
conductors. With respect to Castillo's evaluation, Sagad invoked the CA's pronouncement that
the infractions mentioned in the report are not just causes for the termination of his employment.
Sagad's position fails to convince us. We find no evidence that Hemoroz and Lucero had an
ax to grind against Sagad so that they would lie about their impression of him as a bus driver.
Significantly, their statements validate Castillo's own observation that he heard talks of Sagad's
orders to the conductors for them to cheat on the company. The scheme, contrary to Sagad's
explanation, can only be committed with the cooperation, or even at the behest, of the driver, as
the proposed scheme is for the bus to make unscheduled, but unreported, early trips.
Lastly, the company cites Sagad's involvement in a hit-and-run incident on September 9, 2006
while driving his assigned bus (with Plate No. NYK-216 and Body No. 3094). Once more, he
denies the charge, claiming that it was not his bus, but two other vehicles, a Honda City and an
Elf truck, which figured in the incident. To prove his point, he submitted the "SALAYSAY" of his
replacement driver, Carlito Laude, for September 10, 2006, saying that there was no dents or
scratches on the bus.
Again, Sagad's stance fails to persuade us. Sagad's statements vis--vis the incident, as well
as those of Laude, are belied by the Traffic Accident Investigation Report which mentioned the
"Unidentified driver of Public Utility Bus with plate No. NYK-216 and Body No. 3094." The report
was corroborated by the sworn statements of Ronald Apura, driver of the Elf truck, UFF-597, the
second party in the incident, and Bibiana Fuentes, driver of the White Honda City, WDV-422
(owned by Purefoods Hormel Co.), the first party in the vehicular accident. There was also the
letter to the company of Standard Insurance Co., Inc. dated February 14, 2007 47 demanding
the reimbursement of P24,667.54 it paid to Purefoods Hormel Co. by way of damages sustained

ATHENA M. SALAS | LABOR CASE DIGEST 2015 122

by the Honda City.


Third. The CA misappreciated the law when it declared that the grounds relied upon by the
company in terminating Sagad's employment are not among those enumerated under Article
282 of the Labor Code as just causes for employee dismissals. Article 282 of the Code provides:
Art. 282. Termination by employer. An employer may terminate an employment
for any of the following causes:
(a) Serious misconduct or willful disobedience by the employee of the lawful orders of
his employer or representative in connection with his work;
(b) Gross and habitual neglect by the employee of his duties;
(c) Fraud or willful breach by the employee of the trust reposed in him by his employer
or duly authorized representative;
(d) Commission of a crime or offense by the employee against the person of his
employer or any immediate member of his family or his duly authorized
representative; and
(e) Other causes analogous to the foregoing.
The irregularities or infractions committed by Sagad in connection with his work as a bus
driver constitute a serious misconduct or, at the very least, conduct analogous to serious
misconduct, under the above-cited Article 282 of the Labor Code. To be sure, his tendency to
speed up during his trips, his reckless driving, his picking up passengers in the middle of the
road, his racing with other buses and his jostling for vantage positions do not speak well of him
as a bus driver. While he denies being informed, when he was hired, of the duties and
responsibilities of a driver contained in a document submitted in evidence by the
company the requirement ". to obey traffic rules and regulations as well as the
company policies. . to ensure the safety of the riding public as well as the other vehicles
and motorist (sic)" is so fundamental and so universal that any bus driver is expected to
satisfy the requirement whether or not he has been so informed.
Sagad tries to minimize the adverse effect of the evaluator's report of September 21, 2006
about his conduct as a driver with the argument that he had already been penalized with a fiveday suspension for chasing an "Everlasting" bus at one time. The suspension is of no moment.
He was penalized for one reckless driving incident, but it does not erase all the other infractions
he committed. The conductors' comments and the dispatcher's evaluation, together with the
earlier on-board evaluation, all paint a picture of a reckless driver who endangers the safety of
his passengers, other motorists and the general public. With this record, it is not surprising that
he figured in a hit-and-run accident on September 9, 2006.
Under the circumstances, Sagad has become a liability rather than an asset to his
employer, more so when we consider that he attempted to cheat on the company or could
have, in fact, defrauded the company during his brief tenure as a bus driver. This calls to mind
Castillo's report on the low revenue of Sagad's bus, an observation which is validated by the
company's Daily Operation Reports from June to October 2006.

ATHENA M. SALAS | LABOR CASE DIGEST 2015 123

All told, we find substantial evidence supporting Sagad's removal as a bus driver.
Through his reckless driving and his schemes to defraud the company, Sagad committed
serious misconduct and breach of the trust and confidence of his employer, which,
without doubt, are just causes for his separation from the service. It is well to stress, at this
point, an earlier pronouncement of the Court "that justice is in every case for the deserving, to
be dispensed in the light of the established facts and applicable law and doctrine."
The twin-notice requirement
Even as we find a just cause for Sagad's dismissal, we agree with the CA that the company
failed to comply with the two-notice rule. It failed to serve notice of: (1) the particular acts for
which Sagad was being dismissed on November 5, 2006 and (2) his actual dismissal.
Consistent with our ruling in Agabon v. NLRC, 52 we hold that the violation of Sagad's right to
procedural due process entitles him to an indemnity in the form of nominal damages.
Considering the circumstances in the present case, we deem it appropriate to award Sagad
P30,000.00.

58. Sampaguita Auto Transport Corp. vs. NLRC, et al., G.R. No. 197384, Jan. 30,
2013
Facts:
On May 14, 2006, Sagad was hired by the company hired him as a regular bus driver until his
dismissal on November 5, 2006 for allegedly conniving with conductor Vitola in issuing tickets
outside their assigned route
However, the company claimed that it employed Sagad as a probationary bus driver (evidenced
by a probationary employment contract) from May 14, 2006 to October 14, 2006; he was duly
informed of his corresponding duties and responsibilities. He was further informed that during
the probationary period, his attendance, performance and work attitude shall be evaluated to
determine whether he would qualify for regular employment. For this purpose and as a matter of
company policy, an evaluator was deployed on a company bus (in the guise of a passenger) to
observe the driver's work performance and attitude.
On September 21, 2006, an evaluator boarded Sagad's bus. The evaluator described Sagad's
manner of driving as "reckless driver, nakikipaggitgitan, nakikipaghabulan, nagsasakay sa gitna
ng kalsada, sumusubsob ang pasahero[.]" Sagad disputed the evaluator's observations. In an
explanation, he claimed that he could not have been driving as reported because his wife (who
was pregnant) and one of his children were with him on the bus. He admitted though that at one
time, he chased an "Everlasting" bus to serve warning on its driver not to block his bus when he
was overtaking. He also admitted that once in a while, he sped up to make up for lost time in
making trips.

ATHENA M. SALAS | LABOR CASE DIGEST 2015 124

The company further alleged that on October 13, 2006, it conducted a thorough evaluation of
Sagad's performance. It requested conductors who had worked with Sagad to comment on his
work. Conductors A. Hemoroz and Israel Lucero revealed that Sagad proposed that they cheat
on the company by way of an unreported early bus trip. Dispatcher E. Castillo likewise
submitted a negative report and even recommended the termination of Sagad's employment.
The company also cited Sagad's involvement in a hit-and-run accident on September 9, 2006
along Commonwealth Avenue in Quezon City while on a trip (bus with Plate No. NYK-216 and
Body No. 3094). Allegedly, Sagad did not report the accident to the company.
On October 15, 2006, upon conclusion of the evaluation, the company terminated Sagad's
employment for his failure to qualify as a regular employee.
Issues:
I.

WON the employee was dismissed as a regular employee, and not as a probationary.

II.

WON the employee was dismissed for just cause.

III.
WON the employer failed to observe procedural due process. Hence, the employee is
entitled to nominal damages.
Ruling:
I. The employee was dismissed as a regular employee, and not as a probationary.
The employer was not able to clearly establish that the employee was a probationary. The
employee denied having signed any probationary contract. Moreover, the employer had retained
the employee beyond the supposed probationary period of employment. The payslips presented
by the employee evidences such extension.
In labor law, it is an elementary rule that "an employee who is allowed to work after a
probationary period shall be considered a regular employee."
II. The employee was dismissed for just cause.
The employee was evidently dismissed for just cause.
First, it was established that the employer called the employee's attention to his "negative
actuations as a bus driver". These were reported by a company evaluator who boarded the
employee's bus. The evaluator reported that the employee was "driving recklessly, racing and
jostling for position on the road, thereby jarring the passengers on their seats, and picking up
passengers on the middle of the road."

ATHENA M. SALAS | LABOR CASE DIGEST 2015 125

By way of reply, the employee claimed that he could not have been driving recklessly as his wife
and one of his children were with him on the bus at the time of evaluation. However, he admitted
to two things: (a) that, on one occasion, he chased on "Everlasting" bus in order "to warn the
driver not to block him", and (b) that, once in a while, "he sped up to compensate for lost time in
his trips".
"Sagad's explanation reveals more than what is stated. During his brief employment with the
company, he exhibited the tendency to speed up when he finds the need for it, very obviously in
violation of traffic rules, regulations and company policy. Instead of negating the evaluator's
observations, his admissions make them credible."
Second, the two conductors who had worked with the employee commented that the employee
"proposed to them that they cheat on the company by making early trips (but not to be reported)
bus trips". The two conductors had no ax to grind against the employee. Furthermore, the
evaluator's evaluation report rated the employee's work performance as poor on account of: "(1)
the low revenue of Sagad's bus; (2) his inability to make all his scheduled trips; and (3) his habit
of bringing his wife with him on trips". He likewise heard of talks of the employee's orders to the
conductors "to earn money in a questionable way".
As the two conductors have no ax to grind with the employee, their testimony is credible. In
addition, their testimony validates the evaluator's observation that he heard talks of the
employee's orders to the conductors "for them to cheat on the company".
"x x x The scheme, contrary to Sagad's explanation, can only be committed with the
corporation, or even at the behest of the driver, as the proposed scheme is for the bus to make
unscheduled, but unreported, early trips.
Third, the employee's involvement in a hit-and-run incident while driving his assigned bus was
clearly established. The following were the substantial evidence presented: (1) the Traffic
Accident Investigation Report; (2) Sworn Statement of the driver of the Elf truck, which was hit;
(3) Sworn Statement of the driver of the White Honda City (owned by Purefoods Hormel Co.),
the first party to the vehicular accident; and, (4) Demand Letter by the insurance company
demanding reimbursement it paid to Purefoods.
In view of the above-mentioned grounds, the "irregularities or infractions committed by Sagad in
connection with his work as a bus driver constitutes a serious misconduct under Article 282 of
the Labor Code.
"The irregularities or infractions committed by Sagad in connection with his work as a bus driver
constitutes serious misconduct or, at the very least, conduct analogous to serious misconduct,
under the above-cited Article 282 of the Labor Code. To be sure his tendency to speed up
during his trips, his reckless driving, his picking up passengers in the middle of the road, his
racing with other buses and his jostling for vantage positions do not speak well of him as a bus

ATHENA M. SALAS | LABOR CASE DIGEST 2015 126

driver. While he denies being informed when he was hired, of the duties and responsibilities of a
driver contained in a document submitted in evidence by the company - the requirement "3. to
obey traffic rules and regulations as well as the company policies. 4. to ensure the safety of the
riding public as well as the other vehicles and motorist (sic)" is so fundamental and so universal
that any bus driver is expected to satisfy the requirement whether or not he has been so
informed." (Citations omitted.)
While the employee tried to minimize the adverse effect of the evaluator's report by claiming that
he had already been penalized by a 5-day suspension, the same is of no moment. "He was
penalized for one reckless driving incident, but it does not erase all other infractions he
committed."
III. The employer failed to observe procedural due process. Hence, the employee is entitled to
nominal damages.
While the employee was dismissed based on a just cause, the employer failed to comply with
the two-notice rule.That is to say, the employer failed to serve: (1) the initial notice - stating the
particular acts on which the employee is being dismissed on 05 November 2006, and (2) the
termination notice.
Following Agabon v. NLRC, the violation of the employee's right to procedural due process
entitles him to nominal damages in the amount of Php30,000.00.

59. GENERAL MILLING CORPORATION, vs. VIOLETA L. VIAJAR. G.R. No. 181738. January 30,
2013.
Facts:
GMC is a domestic corporation with principal office in Makati City and a manufacturing plant in LapuLapu City.
In October 2003, GMC terminated the services of thirteen (13) employees for redundancy, including
herein respondent, Violeta Viajar (Viajar). GMC alleged that it has been gradually downsizing its Vismin
(Visayas-Mindanao) Operations in Cebu where a sizeable number of positions became redundant over
a period of time.
When Viajar reported for work on October 31, 2003, almost a month before the effectivity of her
severance from the company, the guard on duty barred her from entering GMC's premises. She was
also denied access to her office computer and was restricted from punching her daily time record in the
bundy clock.

ATHENA M. SALAS | LABOR CASE DIGEST 2015 127

On November 7, 2003, Viajar was invited to the HRD Cebu Office where she was asked to sign certain
documents, which turned out to be an "Application for Retirement and Benefits." The respondent
refused to sign and sought clarification because she did not apply for retirement and instead asserted
that her services were terminated for alleged redundancy. Almocera told her that her signature on the
Application for Retirement and Benefits was needed to process her separation pay. The respondent
also claimed that between the period of July 4, 2003 and October 13, 2003, GMC hired fifteen (15) new
employees which aroused her suspicion that her dismissal was not necessary. At the time of her
termination, the respondent was receiving the salary rate of P19,651.41 per month.
GMC presented the following as proof of validity of its redundancy plan, the notification letter to
respondent Viajar; the "Establishment Termination Report" it submitted to the DOLE Office; the two (2)
checks issued in the respondent's name amounting to P440,253.02 and P21,211.35; and the list of
terminated employees as of June 6, 2006.
Issue:
Whether or not the dismissal of Viajar was valid on the ground of redundancy.
Ruling:
No. Viajar was illegally dismissed.
Article 283 of the Labor Code provides that redundancy is one of the authorized causes for dismissal. It
reads:
Article 283.Closure of establishment and reduction of personnel. The employer may also terminate
the employment of any employee due to the installment of labor-saving devices, redundancy,
retrenchment to prevent losses or the closing or cessation of operation of the establishment or
undertaking unless the closing is for the purpose of circumventing the provisions of this Title, by
serving a written notice on the worker and the Ministry of Labor and Employment at least one
(1) month before the intended date thereof. In case of termination due to the installation of
labor-saving devices or redundancy, the worker affected thereby shall be entitled to a
separation pay equivalent to at least his one (1) month pay or to at least one (1) month pay for
every year of service, whichever is higher. In case of retrenchment to prevent losses and in cases of
closures or cessation of operations of establishment or undertaking not due to serious business losses
or reverses, the separation pay shall be equivalent to one (1) month pay or at least one-half (1/2) month
pay for every year of service, whichever is higher. A fraction of at least six (6) months shall be
considered one (1) whole year.
From the above provision, it is imperative that the employer must comply with the requirements for a
valid implementation of the company's redundancy program, to wit: (a) the employer must serve a
written notice to the affected employees and the DOLE at least one (1) month before the intended date
of retrenchment; (b) the employer must pay the employees a separation pay equivalent to at least one
month pay or at least one month pay for every year of service, whichever is higher; (c) the employer

ATHENA M. SALAS | LABOR CASE DIGEST 2015 128

must abolish the redundant positions in good faith; and (d) the employer must set fair and reasonable
criteria in ascertaining which positions are redundant and may be abolished.
While it is true that the "characterization of an employee's services as superfluous or no longer
necessary and, therefore, properly terminable, is an exercise of business judgment on the part of the
employer," the exercise of such judgment, however, must not be in violation of the law, and must not be
arbitrary or malicious. The Court has always stressed that a company cannot simply declare
redundancy without basis. To exhibit its good faith and that there was a fair and reasonable criteria in
ascertaining redundant positions, a company claiming to be over manned must produce adequate proof
of the same.
The letter-memorandum which contains general allegations is not enough to convince this Court that
Viajar's termination of employment due to redundancy was warranted under the circumstances. There
is no showing that GMC made an evaluation of the existing positions and their effect to the company.
Neither did GMC exert efforts to present tangible proof that it was experiencing business slow down or
over hiring. The "Establishment Termination Report" it submitted to the DOLE Office did not account for
anything to justify declaring the positions redundant. The Court notes that the list of terminated
employees presented by GMC was a list taken as of June 6, 2006 or almost three years after the
respondent was illegally dismissed and almost a year after the LA promulgated its decision. While the
petitioner had been harping that it was on a "reduction mode" of its employees, it has not presented any
evidence (such as new staffing pattern, feasibility studies or proposal, viability of newly created
positions, job description and the approval of the management of the restructuring, audited financial
documents like balance sheets, annual income tax returns and others) which could readily show that
the company's declaration of redundant positions was justified. Such proofs, if presented, would suffice
to show the good faith on the part of the employer or that this business prerogative was not whimsically
exercised in terminating respondent's employment on the ground of redundancy. Unfortunately, these
are wanting in the instant case. The petitioner only advanced a self-serving general claim that it was
experiencing business reverses and that there was a need to reduce its manpower complement.
On the other hand, the respondent presented proof that the petitioner had been hiring new employees
while it was firing the old ones, negating the claim of redundancy. It must, however, be pointed out that
in termination cases, like the one before us, the burden of proving that the dismissal of the employees
was for a valid and authorized cause rests on the employer. It was incumbent upon the petitioner to
show by substantial evidence that the termination of the employment of the respondent was validly
made and failure to discharge that duty would mean that the dismissal is not justified and therefore
illegal.

60. JONATHAN I. SANG-AN, vs. EQUATOR KNIGHTS DETECTIVE AND SECURITY


AGENCY, G.R. No. 173189. February 13, 2013.

Facts:

ATHENA M. SALAS | LABOR CASE DIGEST 2015 129

Jonathan was the Assistant Operation Manager of respondent Equator Knights Detective and Security
Agency, Inc. (Equator). He was tasked, among others, with the duty of assisting in the operations of the
security services; he was also in charge of safekeeping Equator's firearms.
On April 21, 2001, Equator discovered that two firearms were missing from its inventory. The
investigation revealed that it was Jonathan who might have been responsible for the loss. He was then
suspended.
While Jonathan was under suspension, a security guard from Equator was apprehended by policemen
for violating the Commission on Elections' gun ban rule. The security guard stated in his
affidavit that the unlicensed firearm had been issued to him by Jonathan.
He then filed a complaint before the NLRC for illegal suspension. He treated his case as one for illegal
dismissal and alleged that he was denied due process when he was dismissed.
Equator argued that Jonathan was dismissed for just cause and making his dismissal valid.
Issue:
Whether or not there was illegal dismissal.
Ruling:
Equator complied with the substantive requirements for the dismissal of Jonathan on the ground of
serious misconduct. Misconduct is improper or wrongful conduct; it is the transgression of some
established and definite rule of action, a forbidden act, a dereliction of duty, willful in character, and
implies wrongful intent and not mere error of judgment. The misconduct, to be serious within the
meaning of the Labor Code, must be of such grave and aggravated character and not merely trivial or
unimportant. It is also important that the misconduct be in connection with the employee's work to
constitute just cause for his separation.
By losing two firearms and issuing an unlicensed firearm, Jonathan committed serious misconduct. He
did not merely violate a company policy; he violated the law itself (Presidential Decree No.
1866 or Codifying the Laws on Illegal/Unlawful Possession, Manufacture, Dealing in, Acquisition or
Disposition, of Firearms, Ammunition or Explosives or Instruments Used in the Manufacture of
Firearms, Ammunition or Explosives, and Imposing Stiffer Penalties for Certain Violations Thereof and
for Relevant Purposes), and placed Equator and its employees at risk of being made legally liable.
Thus, Equator had a valid reason that warranted Jonathan's dismissal from employment as Assistant
Operation Manager.
However, Equator failed to observe the proper procedure in dismissing Jonathan.
Jurisprudence has expounded on the guarantee of due process, requiring the employer to furnish the
employee with two written notices before termination of employment can be effected: a first written
notice that informs the employee of the particular acts or omissions for which his or her dismissal is

ATHENA M. SALAS | LABOR CASE DIGEST 2015 130

sought, and a second written notice which informs the employee of the employer's decision to dismiss
him. In considering whether the charge in the first notice is sufficient to warrant dismissal under the
second notice, the employer must afford the employee ample opportunity to be heard.
A review of the records shows that Jonathan was not furnished with any written notice that informed
him of the acts he committed justifying his dismissal from employment. The notice of suspension given
to Jonathan only pertained to the first offense, i.e., the loss of Equator's firearms under Jonathan's
watch. With respect to his second offense (i.e., the issuance of an unlicensed firearm to Equator's
security guard that became the basis for his dismissal), Jonathan was never given any notice that
allowed him to air his side and to avail of the guaranteed opportunity to be heard. That Equator brought
the second offense before the LA does not serve as notice because by then, Jonathan had already
been dismissed.
In order to validly dismiss an employee, the observance of both substantive and procedural due
process by the employer is a condition sine qua non. Procedural due process requires that the
employee be given a notice of the charge against him, an ample opportunity to be heard, and a notice
of termination.

61. Leopard Security & Investigation Agency vs. Quitoy, G.R. No. 186344, Feb. 20, 2013
Facts:
Respondents were hired as security guards by Petitioner (LSIA). They were assigned by the
petitioner to the different branches of Union Bank in Cebu city. On April 1, 2005, Union Bank
served notice to LSIA terminating the security service contract effective at the end of business
hours of April 30, 2005. However, the respondents were only informed of the termination of the
contract with Union Bank on April 29, 2005. The respondents went to union bank on April 30,
2005 for the turnover of their service firearms to Cortes, Union Bank Chief security officer.
Respondents filed a complaint for illegal dismissal against LSIA. CA sustained the award of
separation pay of NLRC to respondents on the ground that the parties' relationship had already
been strained.
Issues:
WON there was illegal dismissal.
WON the award of separation pay was proper.
Ruling:
NO. Applying Article 286 of the Labor Code of the Philippines by analogy, this Court has
repeatedly recognized that security guards may be temporarily sidelined by their security
agency as their assignments primarily depend on the contracts entered into by the latter with

ATHENA M. SALAS | LABOR CASE DIGEST 2015 131

third parties. Temporary "off-detail" or "floating status" is the period of time when security
guards are in between assignments or when they are made to wait after being relieved from a
previous post until they are transferred to a new one. It takes place when, as here, the security
agency's clients decide not to renew their contracts with the agency, resulting in a situation
where the available posts under its existing contracts are less than the number of guards in its
roster. For as long as such temporary inactivity does not continue for a period exceeding six
months, it has been ruled that placing an employee on temporary "off-detail" or "floating status"
is not equivalent to dismissal.
Considering that a security guard is only considered illegally dismissed from service when he is
sidelined from duty for a period exceeding six months, respondents were not illegally dismissed
by LSIA. Under Article 279 of the Labor Code, an illegally dismissed employee is entitled to the
twin reliefs of full backwages and reinstatement without loss of seniority rights. Aside from the
instances provided under Articles 283 and 284 of the Labor Code, separation pay is, however,
granted when reinstatement is no longer feasible because of strained relations between the
employer and the employee. In cases of illegal dismissal, the accepted doctrine is that
separation pay is available in lieu of reinstatement when the latter recourse is no longer
practical or in the best interest of the parties.
As a relief granted in lieu of reinstatement, however, it consequently goes without saying that an
award of separation pay is inconsistent with a finding that there was no illegal dismissal. Even in
cases of illegal dismissal, the doctrine of strained relations is not applied indiscriminately as to
bar reinstatement, especially when the employee has not indicated an aversion to returning to
work or does not occupy a position of trust and confidence in or has no say in the operation of
the employer's business. Although litigation may also engender a certain degree of hostility, it
has likewise been ruled that the understandable strain in the parties' relations would not
necessarily rule out reinstatement which would, otherwise, become the rule rather than the
exception in illegal dismissal cases.
Our perusal of the position paper they filed a quo shows that, despite erroneously believing
themselves to have been illegally dismissed, respondents had alleged no circumstance
indicating the strained relations between them and LSIA and had even alternatively prayed for
reinstatement alongside the payment of separation pay. Since application of the doctrine of
strained relations presupposes a question of fact which must be demonstrated and adequately
supported by evidence, the CA clearly erred in ruling that the parties' relations had already
soured and that an award of separation pay in favor of respondents is proper.

62. Philippine Plaza Holdings Inc. vs. Episcope, G.R. No. 192826, Feb. 27, 2013
Facts:

ATHENA M. SALAS | LABOR CASE DIGEST 2015 132

Petitioner Philippine Plaza Holdings, Inc. (PPHI) is the owner and operator of the Westin
Philippine Plaza Hotel (Hotel). Respondent Ma. Flora M. Episcope (Episcope) was employed by
PPHI since July 24, 1984 until she was terminated on November 4, 2004 for dishonesty, willful
disobedience and serious misconduct amounting to loss of trust and confidence.
In order to check the performance of the employees and the services in the different outlets of
the Hotel, PPHI regularly employed the services of independent auditors and/or professional
shoppers. For this purpose, Sycip, Gorres and Velayo auditors dined at the Hotel's Caf Plaza
on August 28, 2004. After dining, the auditors were billed the total amount of P2,306.65,
representing the cost of the food and drinks they had ordered under Check No. 565938. 4
Based on the audit report 5 submitted to PPHI, Episcope was one of those who attended to the
auditors and was the one who handed the check and received the payment of P2,400.00. She
thereafter returned Check No. 565938, which was stamp marked "paid," together with the
change.
Upon verification of the foregoing check receipt with the sales report of Caf Plaza, it was
discovered that the Hotel's copy of the receipt bore a discount of P906.45 6 on account of the
use of a Starwood Privilege Discount Card registered in the name of Peter A. Pamintuan, while
the receipt issued by Episcope to the auditors reflected the undiscounted amount of P2,306.65
considering that none of the auditors had such discount card. In view of the foregoing, the
amount actually remitted to the Hotel was only P1,400.20 thus, leaving a shortage of P906.45.
On September 30, 2004, the Hotel issued a Show-Cause Memo 7 directing Episcope to explain
in writing why no disciplinary action should be taken against her for the questionable and invalid
discount application on the settlement check issued to the auditors on August 28, 2004.
Finding Episcope to have failed to sufficiently explain the questionable discount application on
the settlement bill of the auditors, her employment was terminated for committing acts of
dishonesty, which was classified as a Class D offense under the Hotel's Code of Discipline, as
well as for willful disobedience, serious misconduct and loss of trust and confidence.
Aggrieved, Episcope filed a complaint for illegal dismissal with prayer for payment of damages
and attorney's fees against PPHI before the NLRC docketed as NLRC-NCR Case No. 00-1213621-04.
Issue:
WON there was illegal dismissal.
Ruling:

ATHENA M. SALAS | LABOR CASE DIGEST 2015 133

Article 293 (formerly Article 279) of the Labor Code 25 provides that the employer shall not
terminate the services of an employee except only for a just or authorized cause. If an employer
terminates the employment without a just or authorized cause, then the employee is considered
to have been illegally dismissed and is thus, entitled to reinstatement or in certain instances,
separation pay in lieu thereof, as well as the payment of backwages.
Among the just causes for termination is the employer's loss of trust and confidence in its
employee. Article 296 (c) (formerly Article 282 [c]) of the Labor Code provides that an employer
may terminate the services of an employee for fraud or willful breach of the trust reposed in him.
But in order for the said cause to be properly invoked, certain requirements must be complied
with namely, (1) the employee concerned must be holding a position of trust and confidence and
(2) there must be an act that would justify the loss of trust and confidence.
It is noteworthy to mention that there are two classes of positions of trust: on the one hand,
there are managerial employees whose primary duty consists of the management of the
establishment in which they are employed or of a department or a subdivision thereof, and to
other officers or members of the managerial staff; on the other hand, there are fiduciary rankand-file employees, such as cashiers, auditors, property custodians, or those who, in the normal
exercise of their functions, regularly handle significant amounts of money or property. These
employees, though rank-and-file, are routinely charged with the care and custody of the
employer's money or property, and are thus classified as occupying positions of trust and
confidence. 27 Episcope belongs to this latter class and therefore, occupies a position of trust
and confidence.
As may be readily gleaned from the records, Episcope was employed by PPHI as a service
attendant in its Caf Plaza. In this regard, she was tasked to attend to dining guests, handle
their bills and receive their payments for transmittal to the cashier. It is also apparent that
whenever discount cards are presented, she maintained the responsibility to take them to the
cashier for the application of discounts. Being therefore involved in the handling of company
funds, Episcope is undeniably considered an employee occupying a position of trust and
confidence and as such, was expected to act with utmost honesty and fidelity.
Anent the second requisite, records likewise reveal that Episcope committed an act which
justified her employer's (PPHI's) loss of trust and confidence in her.
Primarily, it is apt to point out that proof beyond reasonable doubt is not required in dismissing
an employee on the ground of loss of trust and confidence; it is sufficient that there lies some
basis to believe that the employee concerned is responsible for the misconduct and that the
nature of the employee's participation therein rendered him absolutely unworthy of trust and
confidence demanded by his position.
Perforce, having substantially established the actual breach of duty committed by Episcope and
the due observance of due process, no grave abuse of discretion can be imputed against the
NLRC in sustaining the finding of the LA that her dismissal was proper under the circumstances.

ATHENA M. SALAS | LABOR CASE DIGEST 2015 134

Finally, with respect to Episcope's other monetary claims, namely, service incentive leave
credits and 13th month pay, the Court finds no error on the part of the LA when it denied the
foregoing claims considering that Episcope failed to proffer any legitimate basis to substantiate
her entitlement to the same

63. TORRES vs. RURAL BANK OF SAN JUAN, INC., et. al. G.R. No. 184520. March 13, 2013
FACTS:
The petitioner was hired by RBSJI. On September 24, 1996, the petitioner was temporarily
assigned as the manager of RBSJI's N. Domingo branch in view of the resignation of Jacinto
Figueroa (Jacinto). On the same month, Jacinto requested the petitioner to sign a standard
employment clearance pertaining to his accountabilities with RBSJI. When the petitioner
declined his request, Jacinto threw a fit and shouted foul invectives. To pacify him, the petitioner
bargained to issue a clearance but only for Jacinto's paid cash advances and salary loan. About
seven months later, respondent issued a memorandum to the petitioner requiring him to explain
why no administrative action should be imposed on him for his unauthorized issuance of a
clearance to Jacinto whose accountabilities were yet to be audited. The petitioner submitted his
explanation on the same day clarifying that the clearance was limited only to Jacinto's paid cash
advances and salary loan based on the receipts presented by the cashier. He emphasized that
he had no foreknowledge nor was he forewarned of Jacinto's unliquidated cash advances and
questionable transactions and that the clearance did not extend to those matters.
After conducting an investigation, RBSJI's Human Resources Department recommended the
petitioner's termination from employment for the following reasons, to wit: the issuance of
clearance to Mr. Jacinto Figueroa have been prejudicial to the Bank; the petitioner is not in any
authority to issue said clearance which is a violation of the Company Code of Conduct and
Discipline under Category B Grave Offense No. 1 (falsifying or misrepresenting persons or other
company records, documents or papers); and the nature of his participation in the issuance of
the said clearance could be a reasonable ground for the Management to believe that he is
unworthy of the trust and confidence demanded by his position which is also a ground for
termination under Article [282] of the Labor Code. Feeling aggrieved, the petitioner filed the
herein complaint for illegal dismissal. The petitioner averred that the supposed loss of trust and
confidence on him was a sham as it is in fact the calculated result of the respondents' dubious
plot to conveniently oust him from RBSJI.
The LA sustained the claims of the petitioner as against the factually unsubstantiated allegation
of loss of trust and confidence propounded by the respondents. The NLRC disagreed with the
LA's conclusion. But on motion for reconsideration, the NLRC reversed its earlier ruling and
reinstated the LA's Decision. The CA reversed and set aside the NLRC Decision and ruled that
the petitioner was dismissed for a just cause.

ATHENA M. SALAS | LABOR CASE DIGEST 2015 135

ISSUE:
Whether or not petitioner was illegally dismissed.

RULING:

YES. The respondents failed to prove that the petitioner was dismissed for a just cause. The law
mandates that before validity can be accorded to a dismissal premised on loss of trust and
confidence, two requisites must concur, viz.: (1) the employee concerned must be holding a
position of trust; and (2) the loss of trust must be based on willful breach of trust founded on
clearly established facts.
The presence of the first requisite is certain. Ever since the petitioner was employed, he has
occupied positions that entail the power or prerogative to dictate management policies. Anent
the second requisite, the Court finds that the respondents failed to meet their burden of proving
that the petitioner's dismissal was for a just cause.
The unsubstantiated claims of the respondents fall short of the standard proof required for valid
termination of employment. They failed to clearly and convincingly establish that the petitioner's
act of issuing a clearance to Jacinto rendered him unfit to continue working for RBSJI.
Furthermore, the Court has repeatedly emphasized that the act that breached the trust must be
willful such that it was done intentionally, knowingly, and purposely, without justifiable excuse, as
distinguished from an act done carelessly, thoughtlessly, heedlessly or inadvertently. The
conditions under which the clearance was issued exclude any finding of deliberate or conscious
effort on the part of the petitioner to prejudice his employer.
Also, the petitioner did not commit an irregular or prohibited act. He did not falsify or
misrepresent any company record as it was officially confirmed by the cashier that the items
covered by the clearance were truly settled by Jacinto. Hence, the respondents had no factual
basis in declaring that the petitioner violated Category B Grave Offense No. 1 of the Company
Code of Conduct and Discipline. All told, the petitioner was illegally dismissed from employment
and is entitled to back wages, to be computed from the date he was illegally dismissed until the
finality of this decision.
In addition to his back wages, the petitioner is also entitled to separation pay. It cannot be
gainsaid that animosity and antagonism have been brewing between the parties since the
petitioner was gradually eased out of key positions in RBSJI and to reinstate him will only
intensify their hostile working atmosphere. Thus, based on strained relations, separation pay
equivalent to one (1) month salary for every year of service, with a fraction of a year of at least
six (6) months to be considered as one (1) whole year, should be awarded in lieu of
reinstatement, to be computed from date of his engagement by RBSJI up to the finality of this
decision.
The award of separation pay in case of strained relations is more beneficial to both parties in
that it liberates the employee from what could be a highly oppressive work environment in as

ATHENA M. SALAS | LABOR CASE DIGEST 2015 136

much as it releases the employer from the grossly unpalatable obligation of maintaining in its
employ a worker it could no longer trust.
Moral damages are recoverable only where the dismissal of the employee was attended by bad
faith or fraud, or constituted an act oppressive to labor, or was done in a manner contrary to
morals, good customs or public policy. Such an award cannot be justified solely upon the
premise that the employer fired his employee without just cause or due process. Here, the
petitioner failed to prove that his dismissal was attended by explicit oppressive, humiliating or
demeaning acts. Since no moral damages can be granted under the facts of the case,
exemplary damages cannot also be awarded.
64. THE ORCHARD GOLF AND COUNTRY CLUB vs. FRANCISCO G.R. No. 178125. March
18, 2013
FACTS:
Respondent Francisco was employed by petitioner as Club Accountant. The respondent reports
directly to the Club's Financial Comptroller, Jose Ernilo P. Famy. On May 18, 2000, Famy
directed Francisco to draft a letter to SGV & Co. (SGV), the Club's external auditor, inquiring
about the accounting treatment that should be accorded property that will be sold or donated to
the Club. Francisco failed to prepare the letter, even after Famy's repeated verbal and written
reminders. For this reason, Famy issued a memorandum suspending Francisco without pay for
a period of 15 days.
Francisco filed an amended Complaint for illegal suspension. Meanwhile, she continued to
report for work. A day after serving her suspension, Francisco again received a memorandum
placing her on forced leave with pay. After the expiration of her forced leave, Francisco reported
back to work. This time she was handed a memorandum informing her that, due to strained
relations between her and Famy and the pending evaluation of her betrayal of company trust
charge, she has been permanently transferred, without diminution of benefits, to the Club's Cost
Accounting Section.
On account of her transfer, Francisco once more amended her Complaint to include
illegal/constructive dismissal. And in her prayer, she sought to be reinstated to her former
position as Club Accountant.
The Labor Arbiter dismissed Francisco's Complaint for lack of merit. The NLRC reversed the
ruling of LA and the CA sustained the NLRC ruling.

ISSUE:
Whether or not the respondent was constructively dismissed.

RULING:

ATHENA M. SALAS | LABOR CASE DIGEST 2015 137

YES. There was constructive dismissal when Francisco was transferred to the Cost Accounting
Section. We agree with the NLRC and the CA that Francisco's transfer to the position of Cost
Controller was without valid basis and that it amounted to a demotion in rank. Hence, there was
constructive dismissal.
For one, there was no valid reason to temporarily transfer Francisco to Cost Accounting Section.
She had already served her penalty for her failure to draft the SGV letter, through the 15-day
suspension period which she just completed. Secondly, the transfer was not even rooted in any
new infraction she is accused of committing. There was thus an absolute lack of basis for her
temporary transfer.
As for her permanent transfer, the same is null and void for lack of just cause. Also, the transfer
is a penalty imposed on a charge that has not yet been resolved. Definitely, to punish one for an
offense that has not been proved is truly unfair; this is deprivation without due process. Finally,
the Court sees no necessity for Francisco's transfer; on the contrary, such transfer is
outweighed by the need to secure her office and documents from Famy's possible intervention
on account of the complaint she filed against him.
We also agree with the findings of the NLRC, as affirmed by the CA, that Francisco's transfer
constituted a demotion, It is obvious, therefore, that Complainant's position of Club Accountant
is higher in level/rank than that of Cost Controller/Accountant. Patently, Complainant's transfer
from the position of Club Accountant to the position of Cost Accountant resulted to her demotion
in level/rank. Complainant's transfer resulting to her demotion is, therefore, tantamount to
constructive dismissal. The fact that Francisco continued to report for work does not necessarily
suggest that constructive dismissal has not occurred, nor does it operate as a waiver.
Constructive dismissal occurs not when the employee ceases to report for work, but when the
unwarranted acts of the employer are committed to the end that the employee's continued
employment shall become so intolerable. In these difficult times, an employee may be left with
no choice but to continue with his employment despite abuses committed against him by the
employer, and even during the pendency of a labor dispute between them. This should not be
taken against the employee.

65. Banares vs. Tabaco Womens Transport Service Cooperative, G.R. No. 197353, April 1, 2013
Facts:
PETITIONER Alexander B. Baares worked for some time as general manager of respondent
Tabaco Womens Transport Service Cooperative (TAWTRASCO). He filed a complaint for illegal
dismissal and payment of monetary claims against TAWTRASCO. Among others, the Labor
Arbiter ordered TAWTRASCO to immediately reinstate petitioner to his former position.

ATHENA M. SALAS | LABOR CASE DIGEST 2015 138

In compliance with the decision, TAWTRASCO directed the petitioner to report at the companys
Virac, Catanduanes terminal. Petitioner asked for a lodging allowance, which he used to enjoy
in his previous assignment but was told to just stay at the Virac office.
He, however, found the Virac office very dilapidated and empty of an office table, chairs, filing
cabinet and other office supplies. He asked for expenses for renovation, which respondents
turned deaf ears to. Hence, he stopped reporting to work and filed a complaint for non-payment
of salaries.
Issue:
WON the complaint will prosper.
Ruling:
Yes.
Under Article 223 of the Labor Code, an employee entitled to reinstatement shall either be
admitted back to work under the same terms and conditions prevailing prior to his dismissal or
separation x x x. An illegally dismissed employee is entitled to reinstatement without loss of
seniority rights and to other established employment privileges, and to his full back wages. The
boarding house privilege, being an established perk accorded to petitioner, ought to have been
granted him if a real and authentic reinstatement to his former position as general manager is to
be posited.
It cannot be stressed enough that TAWTRASCO withheld petitioners salaries for and after his
purported refusal to report for work at the Virac terminal. The reality, however, is that
TAWTRASCO directed petitioner to work under terms and conditions prejudicial to him, the most
hurtful cut being that he was required to work without a decent office, partly performing a
checkers job. This embarrassing work arrangement is what doubtless triggered the refusal to
work, which under the premises appears justified.
Generally, employees have a demandable right over benefits voluntarily granted to them by their
employers. And if the grant or benefit is founded on an express policy or has, for a considerable
period, been given regularly and deliberately, then the grant ripens into a vested right that the
employer cannot unilaterally diminish, discontinue or eliminate. So it must be here with respect,
at the minimum, to the lodging accommodation which TAWTRASCO, as found by the National
Labor Relations Commission, appears to have regularly extended for free for some time to
petitioner.

66. Reyes, et al., vs. RP Guardians security Agency Inc. G.R. No. 193756, April 10, 2013

ATHENA M. SALAS | LABOR CASE DIGEST 2015 139

Facts:
Petitioners Venancio S. Reyes, Edgardo C. Dabbay, Walter A. Vigilia, Nemesio M. Calanno,
Rogelio A. Supe, Jr., Roland R. Trinidad, and Aurelio A. Duldulao (petitioners) were hired by
respondent RP Guardians Security Agency, Inc. (respondent) as security guards. They were
deployed to various clients of respondent, the last of which were the different branches of Banco
Filipino Savings and Mortgage Bank (Banco Filipino).
In September 2006, respondents security contract with Banco Filipino was terminated. In
separate letters, petitioners were individually informed of the termination of the security contract
with Banco de Oro. In two (2) memoranda, dated September 21, 2006 and September 29,
2006, petitioners were directed to turnover their duties and responsibilities to the incoming
security agency and were advised that they would be placed on floating status while waiting for
available post. Petitioners waited for their next assignment, but several months lapsed and they
were not given new assignments.
Consequently, on April 10, 2007, petitioners filed a complaint7 for constructive dismissal.
In its position paper, respondent claimed that there was no dismissal, of petitioners, constructive
or otherwise, and asserted that their termination was due to the expiration of the service
contract which was coterminus with their contract of employment.
On August 20, 2007, the Labor Arbiter (LA) rendered a decision9 in favor of petitioners ordering
respondent to pay petitioners separation pay, backwages, refund of trust fund, moral and
exemplary damages, and attorneys fees. Aggrieved, respondent appealed to the NLRC.
On April 9, 2008, the NLRC promulgated its decision sustaining the finding of constructive
dismissal by the LA, and the awards she made in the decision. The award of moral and
exemplary damages, however, were deleted.
Upon denial of its motion for reconsideration, respondent filed a petition for certiorari before the
CA.
On February 26, 2010, the CA rendered a decision12 dismissing the petition and affirming the
assailed NLRC decision and resolution.
On motion for reconsideration, the CA issued the Amended Decision13 dated May 18, 2010,
modifying its earlier decision. Citing Section 6.5 (4) of Department Order No. 14 of the
Department of Labor and Employment (DOLE D.O. No. 14), otherwise known as Guidelines
Governing the Employment and Working Conditions of Security Guards and Similar Personnel
in the Private Security Industry, the CA reduced the computation of the separation pay from one
month pay per year of service to one-half month pay for every year of service; reduced the
refund of trust fund contribution from Sixty (P60.00) Pesos to Thirty (P30.00)Pesos; and deleted
the award of backwages and attorneys fees.
Issue:
WON the first decision of the CA is correct in affirming the decision of the NLRC awarding both
backwages and separation pay of one month pay for every year of service

ATHENA M. SALAS | LABOR CASE DIGEST 2015 140

WON petitioners are entitled to backwages for the period covered from the time the Labor
Arbiter rendered the decision in their favor on August 20, 2007 until said decision was reversed
by the Court of Appeals in its Amended Decision promulgated on May 18, 2010.
Ruling:
There is no doubt that petitioners were constructively dismissed. The LA, the NLRC and the CA
were one in their conclusion that respondent was guilty of illegal dismissal when it placed
petitioners on floating status beyond the reasonable six-month period after the termination of
their service contract with Banco de Oro. Temporary displacement or temporary off-detail of
security guard is, generally, allowed in a situation where a security agencys client decided not
to renew their service contract with the agency and no post is available for the relieved security
guard. Such situation does not normally result in a constructive dismissal. Nonetheless, when
the floating status lasts for more than six (6) months, the employee may be considered to have
been constructively dismissed. No less than the Constitution guarantees the right of workers to
security of tenure, thus, employees can only be dismissed for just or authorized causes and
after they have been afforded the due process of law.
Settled is the rule that that an employee who is unjustly dismissed from work shall be entitled to
reinstatement without loss of seniority rights and other privileges, and to his full backwages,
inclusive of allowances and to his other benefits or their monetary equivalent computed from the
time his compensation was withheld up to the time of actual reinstatement. If reinstatement is
not possible, however, the award of separation pay is proper.
Backwages and reinstatement are separate and distinct reliefs given to an illegally dismissed
employee in order to alleviate the economic damage brought about by the employees
dismissal.21 Reinstatement is a restoration to a state from which one has been removed or
separated while the payment of backwages is a form of relief that restores the income that was
lost by reason of the unlawful dismissal. Therefore, the award of one does not bar the other.
In the case of Aliling v. Feliciano, citing Golden Ace Builders v. Talde, the Court explained:
Thus, an illegally dismissed employee is entitled to two reliefs: backwages and reinstatement.
The two reliefs provided are separate and distinct. In instances where reinstatement is no longer
feasible because of strained relations between the employee and the employer, separation pay
is granted. In effect, an illegally dismissed employee is entitled to either reinstatement, if viable,
or separation pay if reinstatement is no longer viable, and backwages.
The normal consequences of respondents illegal dismissal, then, are reinstatement without loss
of seniority rights, and payment of backwages computed from the time compensation was
withheld up to the date of actual reinstatement. Where reinstatement is no longer viable as an
option, separation pay equivalent to one (1) month salary for every year of service should be
awarded as an alternative. The payment of separation pay is in addition to payment of
backwages.
Furthermore, the entitlement of the dismissed employee to separation pay of one month for
every year of service should not be confused with Section 6.5 (4) of DOLE D.O. No. 14 which
grants a separation pay of one-half month for every year service, to wit:

ATHENA M. SALAS | LABOR CASE DIGEST 2015 141

6.5 Other Mandatory Benefits. In appropriate cases, security guards/similar personnel are
entitled to the mandatory benefits as listed below, although the same may not be included in the
monthly cost distribution in the contracts, except the required premiums for their coverage:
a. Maternity benefit as provided under the SSS Law;
b. Separation pay if the termination of employment is for authorized cause as provided by law
and as enumerated below:
Half-Month Pay Per Year of Service, but in no case less than One Month Pay, if separation is
due to:
1. Retrenchment or reduction of personnel effected by management to prevent serious losses;
2. Closure or cessation of operation of an establishment not due to serious losses or financial
reverses;
3. Illness or disease not curable within a period of 6 months and continued employment is
prohibited by law or prejudicial to the employee's health or that of co-employees; or
4. Lack of service assignment for a continuous period of 6 months.
The said provision contemplates a situation where a security guard is removed for authorized
causes such as when the security agency experiences a surplus of security guards brought
about by lack of clients. In such a case, the security agency has the option to resort to
retrenchment upon compliance with the procedural requirements of two-notice rule set forth in
the Labor Code and to pay separation pay of one-half month for every year of service.
In this case, respondent would have been liable for reinstatement and payment of backwages.
Reinstatement, however, was no longer feasible because, as found by the LA, respondent had
already ceased operation of its business. Thus, backwages and separation pay, in the amount
of one month for every year of service, should be paid in lieu of reinstatement.
As to their claim of attorneys fees, petitioners were compelled to file an action for the recovery
of their lawful wages and other benefits and, in the process, incurred expenses. Hence,
petitioners are entitled to attorneys fees equivalent to ten percent (10%) of the monetary award.

67. Celdran vs. Forza Integrated Services et al., G.R. No. 189460, June 5, 2013, Res.
Facts:
Petitioner Leo Mario C. Celdran was working for [(Philamlife)] before he was hired by
respondent City Service Corporation [(City Service)] as Vice-President in Cebu City.
Respondents City Service Corporation and [(Peerless Integrated)] are affiliate companies of
respondent Forza Integrated)]. CELDRAN was paid monthly basis. Celdran alleges that his
compensation package included a car benefit which to him, was a car plan wherein respondent

ATHENA M. SALAS | LABOR CASE DIGEST 2015 142

City Service Corporation would assume 50% of the purchase price while Celdran pays the other
50% thereof.
On the contrary, respondents City Service, states that it was a car lease arrangement whereby
an agreed percentage of the [amount paid to Philamlife] was to be set as residual value and the
remaining amount was to be spread over a certain period as monthly car rental with an option to
buy the vehicle at its residual value after the end of the lease period.
Deductions were then made from Celdran's salary with the monthly deductions in place and the
vehicle being already in the possession and use of Celdran, he was required to sign a motor
vehicle lease contract in order to formalize the arrangement. Celdran consistently refused to
sign the said lease contract as it was contrary to what he allegedly agreed with respondent
Valentin B. Prieto, Jr. [(V. Prieto)] during his employment interview, since Celdran avers it was a
car plan not a car lease arrangement.
Thereafter, Santiago, the Chief Operating Officer of City Service, accused Celdran of dishonesty
for allegedly charging a personal lunch to the company.
Respondent Santiago then demanded Celdran's resignation. Eight days after, Celdran received
a termination notice signed by private respondent Santiago which led the former to file a
complaint for illegal dismissal before the LA. However, the said case was settled as he was
reinstated to his position.
Upon his return, Celdran was told to occupy the last open cubicle at the ground floor and not to
his former office. He was given a new copy of the motor vehicle lease contract for his signature
which he refused to sign. Subsequently, he was relieved as Mancom Chairman for no reason at
all. He was subjected to check and inspection by the security guard and his transportation and
cellular phone allowances were subjected to new guidelines. City Service Corporation gave
Celdran the option to buy the Honda CRV at its residual value, otherwise, the former would
recover the vehicle from the latter.
Then, Celdran filed a complaint again with the LA charging private respondents with violation of
the terms of the car plan. Celdran filed an injunction case with a prayer for the issuance of a
TRO with the NLRC, to prevent City Service from taking the car. NLRC granted the prayed-for
TRO as well as the writ of preliminary injunction.
Soon, Celdran was placed under preventive suspension for 30 days due to his belligerent
attitude and required to explain why he should not be terminated. He was not asked to return to
work after his suspension for which reason he amended his complaint, to include charges of
illegal suspension, constructive dismissal and unpaid money claims.
Afterwards, private respondent City Service Corporation informed all its employees, including
Celdran, that by virtue of a board resolution, the company decided to replace the Visayas
Regional Office with a small Liaison Office. Consequently, Celdran made a second amendment
of his complaint to include charges of illegal lay-off/downsizing.
The Executive Labor Arbiter Ruling, found: 1) the long-standing policy is that of granting Car
Lease Agreement with option to buy at residual value after end of the lease term and NOT car
plan. 2) That because Celdran amended his complaint to subsequently include illegal dismissal
due to retrenchment (when his original complaint alleges constructive dismissal, earlier), this
has the effect of Celdran waiving his claim for constructive dismissal, because employment can

ATHENA M. SALAS | LABOR CASE DIGEST 2015 143

only be terminated once. 3) That the suspension was not made illegal by the fact that it
continued after 30 days since City Service reinstated Celdran in the payroll. 4) That the
downsizing or retrenchment was legal justified by management prerogative.
On appeal NLRC affirmed LA, but found that it was a carplan not car lease agreement. CA
affirm NLRC.
Hence the petition
Issue:
Whether or not there is constructive dismissal
Ruling:
Petition of CELDRAN is denied.
There is constructive dismissal when employees resort to involuntary resignation, because
continued employment is rendered impossible, unreasonable, or unlikely; when there is a
demotion in rank and/or a diminution in pay; or when a clear discrimination, insensibility, or
disdain by the employer becomes unbearable to them. That is, they find themselves in a
situation in which there is a belief that personal reasons cannot be sacrificed in favor of the
exigency of the service, and that they have no other choice but to disassociate themselves from
their employment.
We rule that there was no constructive dismissal in the case at bar. According to petitioner, he
had been experiencing a kind of treatment that rendered "employment impossible and
unreasonable" as early as in the last quarter of 2005. However, he never resigned. In fact, when
he filed a complaint in March 2006 regarding his car plan benefit, he did not make any allegation
concerning his inability to continue working for respondents due to an alleged ill working
environment. We thus find that he was still willing and able to continue his employment despite
any alleged ill treatment. To repeat, for there to be constructive dismissal, the employer must be
shown to have committed an act of clear discrimination, insensibility, or disdain, which had
become so unbearable on the part of the employee that it foreclosed any choice other than for
the latter to forego continued employment.
In this case downsizing or retrenchment was valid (since city service had been experiencing a
downtrend in their Visayas operations since three years before they decided to downsize).
Pursuant to Article 283 of the Labor Code, an employer may reduce the number of its
employees based on economic grounds in order to protect and preserve the employer's viability
and ensure its survival. 5 Consequently, employers are given the management prerogative to
implement a retrenchment program for the purpose of preventing losses or cessation of
business operations due to business recession, industrial depression, seasonal fluctuations,
lack of work, or considerable reduction in the volume of their business.

68. Surigao Del Norte Electric Cooperative Inc. vs. Gonzaga, G.R. No. 187722, June 10,
2013
Facts:

ATHENA M. SALAS | LABOR CASE DIGEST 2015 144

Petitioner (SURNECO) hired Gonzaga as its lineman.


Temporary Teller at SURNECO's sub-office.

Thereafter he was assigned as

On June 26, 2001, petitioner (Escalante), General Manager of SURNECO, issued


Memorandum Order with attached report of SURNECO's Internal Auditor, (Collection Report)
and two (2) sets of summaries of collections and remittances (Summaries), seeking an
explanation from Gonzaga regarding his remittance shortages in the total amount of
P314,252.23,
Later, Gonzaga asked for an extension of three (3) weeks within which to submit his explanation
since he needed to go over the voluminous receipts of collections and remittances with the
assistance of an accountant. On the same day, he sent another letter, denying any unremitted
amount on his part and thereby, requesting that the charges against him be lifted. Attached to
the same letter is an Audit Opinion prepared by one (Laluna), a certified public accountant
(CPA), stating that the Internal Auditor's Report cannot accurately establish any remittance
shortage on Gonzaga's part since the amount of collections stated in the Summaries was not
supported by any bills or official receipts.
Meanwhile, SURNECO formed an Investigation Committee to investigate Gonzaga's alleged
remittance shortages. Soon, the Committee sent Gonzaga an invitation to attend the
investigation proceedings, in which he participated. Pending investigation, Gonzaga was placed
under preventive suspension.
Afterwards, the Committee tendered its report, finding Gonzaga guilty of (a) gross and habitual
neglect of duty); (b) misappropriation of REC funds and (c) failure to remit collections/monies
under the Code of Ethics and Discipline for Rural Electric Cooperative (REC) Employees (Code
of Ethics. Thereafter, a notice of termination was served on Gonzaga. Gonzaga sought
reconsideration before SURNECO's Board of Directors but the latter denied the same after he
presented his case. Another notice of termination (Final Notice of Termination) was then served
on Gonzaga. Consequently, he was dismissed from the service.
In view thereof, Gonzaga filed a complaint with the LA for illegal dismissal claiming that he was
denied due process and dismissed without just cause. He alleged that while he was asked in
the Memorandum to explain the remittance shortage, he was nonetheless denied due process
since the actual grounds for his dismissal, i.e., gross and habitual neglect of duties and
responsibilities, misappropriation of REC funds and failure to remit collections/monies, were not
indicated in the said memorandum. 16 He also claimed that petitioners' evidence failed to show
any missing collection since (a) the attached Summary of Collections and Remittances did not
bear any receipt numbers, both with respect to collections and remittances and (b) the other
Summary of Collections and Remittances only contained receipt numbers for the remittances
and none for the collections.
The LA's Ruling: 1) Gonzagas dismissal was illegal since it was not founded on just cause.
Hence he should be reinstated. 2) Shortages in remittances not proved since actual receipts not
presented in evidence. 3) Gonzaga not afforded due process under the said code of ethics.
Thereafter, petitioner elevated matter to NLRC. Petitioner also submitted additional audit report
by a CPA to corroborate the findings of shortage.

ATHENA M. SALAS | LABOR CASE DIGEST 2015 145

NLRC reversed LA decision. NLRC found that the dismissal was valid. CA reversed NLRC
decision, finding that dismissal was illegal. Hence the petition.
Issue:
WON the dismissal was based on just and valid cause
Ruling:
Yes, the dismissal was valid.
In termination cases, the burden of proof rests on the employer to show that the dismissal is for
a valid cause. Failing in which, the law considers the matter a case of illegal dismissal.
The quantum of proof which the employer must discharge is substantial evidence which, as
defined in case law, means that amount of relevant evidence as a reasonable mind might
accept as adequate to support a conclusion, even if other minds, equally reasonable, might
conceivably opine otherwise.
Petitioners were able to prove, by substantial evidence, that there lies a valid cause to terminate
Gonzaga's employment.
The Court concurs with the NLRC's finding that petitioners' evidence which consists of the
Collection Report, the Summaries, and the September 15, 2003 Audit Report with attached
Cash Flow Summary adequately supports the conclusion that Gonzaga misappropriated the
funds of the cooperative.
Labor tribunals, such as the NLRC, are not precluded from receiving evidence submitted on
appeal as technical rules are not binding in cases submitted before them. In fact, labor officials
should use every and reasonable means to ascertain the facts in each case speedily and
objectively, without regard to technicalities of law or procedure, all in the interest of due process.
The evidence presented is sufficient to constitute substantial evidence to prove that he
committed serious misconduct and gross and habitual neglect of duty to warrant his dismissal
from employment. Such are just causes for termination which are explicitly enumerated under
Article 296 of the Labor Code:
Article 296.Termination by Employer. An employer may terminate an employment for any of
the following causes:
(a)Serious Misconduct or wilful disobedience by the employee of the lawful orders of his
employer or representative in connection with his work;
(b)Gross and habitual neglect by the employee of his duties;
xxx xxx xxx
B. Termination procedure; statutory compliance.
The statutory procedure for terminating an employee is found in Section 2 (III), Rule XXIII, Book
V of the Omnibus Rules Implementing the Labor Code (Omnibus Rules) which states:
SEC. 2.Standards of due process: requirements of notice. In all cases of termination of
employment, the following standards of due process shall be substantially observed:

ATHENA M. SALAS | LABOR CASE DIGEST 2015 146

For termination of employment based on just causes as defined in Article 282 of the Labor
Code:
(i)A written notice served on the employee specifying the ground or grounds for termination, and
giving said employee reasonable opportunity within which to explain his side.
(ii)A hearing or conference during which the employee concerned, with the assistance of
counsel if he so desires is given opportunity to respond to the charge, present his evidence, or
rebut the evidence presented against him.
(iii)A written notice of termination served on the employee, indicating that upon due
consideration of all the circumstances, grounds have been established to justify his termination.
Succinctly put, the foregoing procedure consists of (a) a first written notice stating the intended
grounds for termination; (b) a hearing or conference where the employee is given the
opportunity to explain his side; and (c) a second written notice informing the employee of his
termination and the grounds therefor. Records disclose that petitioners were able to prove that
they sufficiently complied with these procedural requirements:
At this juncture, it must be pointed out that while petitioners have complied with the procedure
laid down in the Omnibus Rules, they, however, failed to show that the established company
policy in investigating employees was adhered to. In this regard, SURNECO's breach of its
company procedure necessitates the payment of nominal damages as will be discussed below.
C. Company procedure;
Consequences of breach.
Jurisprudence dictates that it is not enough that the employee is given an "ample opportunity to
be heard" if company rules or practices require a formal hearing or conference. In such
instance, the requirement of a formal hearing and conference becomes mandatory. In Perez v.
Philippine Telegraph and Telephone Company, 56 the Court laid down the following principles in
dismissing employees:
(a)"ample opportunity to be heard" means any meaningful opportunity (verbal or written) given
to the employee to answer the charges against him and submit evidence in support of his
defense, whether in a hearing, conference or some other fair, just and reasonable way.
(b) A formal hearing or conference becomes mandatory only when requested by the employee
in writing or substantial evidentiary disputes exists or a company rule or practice requires it, or
when similar circumstances justify it.
(c) The "ample opportunity to be heard" standard in the Labor Code prevails over the "hearing
and conference" requirement in the implementing rules and regulations. The rationale behind
this mandatory characterization is premised on the fact that company rules and regulations
which regulate the procedure and requirements for termination, are generally binding on the
employer. Thus, as pronounced in Suico v. NLRC, et al.:
Company policies or practices are binding on the parties. Some can ripen into an obligation on
the part of the employer, such as those which confer benefits on employees or regulate the
procedures and requirements for their termination.

ATHENA M. SALAS | LABOR CASE DIGEST 2015 147

Records reveal that while Gonzaga was given an ample opportunity to be heard within the
purview of the foregoing principles, SURNECO, however, failed to show that it followed its own
rules which mandate that the employee who is sought to be terminated be afforded a formal
hearing or conference.
Accordingly, since only an informal inquiry was conducted in investigating Gonzaga's alleged
cash shortages, SURNECO failed to comply with its own company policy, violating the proper
termination procedure altogether.
In this relation, case law states that an employer who terminates an employee for a valid cause
but does so through invalid procedure is liable to pay the latter nominal damages.
In Agabon v. NLRC (Agabon), the Court pronounced that where the dismissal is for a just cause,
the lack of statutory due process should not nullify the dismissal, or render it illegal, or
ineffectual. However, the employer should indemnify the employee for the violation of his
statutory rights. Thus, in Agabon, the employer was ordered to pay the employee nominal
damages in the amount of P30, 000.00.
By analogy, the Court finds that the same principle should apply to the case at bar for the
reason that an employer's breach of its own company procedure is equally violative of the
laborer's rights, albeit not statutory in source. Hence, although the dismissal stands, the Court
deems it appropriate to award Gonzaga nominal damages in the amount of P30, 000.00.

69. Univac Developments Inc. vs. Soriano, G.R. No. 182072, June 19, 2013
Facts:
Soriano was hired by Univac Development, Inc., as legal assistant on probationary basis with a
monthly salary of P15, 000.00. Eight days prior to the completion of his probationary period, he
was informed that he was being terminated due to cost-cutting measures. He asked for 30 day
notice but the termination was effective immediately. He then filed for illegal dismissal against
Univac Development, Inc. It was found out in this case that Soriano was not apprised of the
standards prescribed y the company for him to qualify as regular employee at the time of his
engagement. It was not even shown that performance evaluation was conducted to determine
whether his performance was unsatisfactory.
Issue:
Whether or not there was illegal dismissal.
Ruling:
YES. It is undisputed that respondent was hired as a probationary employee. As such, he did
not enjoy a permanent status. Nevertheless, he is accorded the constitutional protection of
security of tenure which means that he can only be dismissed from employment for a just cause
or when he fails to qualify as a regular employee in accordance with reasonable standards
made known to him by the employer at the time of his engagement.
It is primordial that at the start of the probationary period, the standards for regularization be
made known to the probationary employee. Equally important is the requirement that in order to

ATHENA M. SALAS | LABOR CASE DIGEST 2015 148

invoke "failure to meet the probationary standards" as a justification for dismissal, the employer
must show how these standards have been applied to the subject employee.
Indeed, the power of the employer to terminate a probationary employee is subject to three
limitations, namely: (1) it must be exercised in accordance with the specific requirements of the
contract; (2) the dissatisfaction on the part of the employer must be real and in good faith, not
feigned so as to circumvent the contract or the law; and (3) there must be no unlawful
discrimination in the dismissal. In this case, not only did petitioner fail to show that respondent
was apprised of the standards for regularization but it was likewise not shown how these
standards had been applied in his case.
Pursuant to well-settled doctrine, petitioner's failure to specify the reasonable standards by
which respondent's alleged poor performance was evaluated as well as to prove that such
standards were made known to him at the start of his employment, makes respondent a regular
employee. In other words, because of this omission on the part of petitioner, respondent is
deemed to have been hired from day one as a regular employee.
To justify the dismissal of an employee, the employer must, as a rule, prove that the dismissal
was for a just cause and that the employee was afforded due process prior to dismissal.
Respondent's termination from employment is without just and valid ground. Neither was due
process observed, making his termination illegal.

70. Unilever Phils vs. Rivera, G.R. No. 201701, June 3, 2013
Facts:
Rivera was hired by Unilever as its Area Activation Executive in the cities of Cotabato and
Davao. She was primarily tasked with managing the sales, distribution and promotional activities
in her area and supervising Ventureslink International, Inc., a third party service provider for the
company's activation projects. It was the policy of the company that the allocated budget for
specific activity must be used for such activity only. However, sometime in 2007, a random audit
was conducted and found out that there were fictitious billings and fabricated receipts
supposedly coming from Ventureslink amounting to P11, 200,000.00. It was also discovered that
some funds were diverted from the original intended projects, and the deviation was upon the
instruction of Rivera. She was made to explain the incident by Unilever where she admitted the
diversion of funds. She explained that diversion was made because of the difficulty in procuring
budget from the head office, and that the funds were utilized in the companys promotional
venture in her area of coverage. Unilever found Rivera guilty of serious breach of the company's
Code of Business Principles compelling it to sever their professional relations. When the case
reached the Court of appeals, it upheld the validity of the dismissal but awarded Rivera a
separation pay as a measure of social justice.
Issue:
Whether or not a validly dismissed employee, like Rivera, is entitled to an award of separation
pay.
Ruling:

ATHENA M. SALAS | LABOR CASE DIGEST 2015 149

As a general rule, an employee who has been dismissed for any of the just causes enumerated
under Article 282 of the Labor Code is not entitled to a separation pay. In exceptional cases,
however, the Court has granted separation pay to a legally dismissed employee as an act of
"social justice" or on "equitable grounds." In both instances, it is required that the dismissal (1)
was not for serious misconduct; and (2) did not reflect on the moral character of the employee.
Separation pay is only warranted when the cause for termination is not attributable to the
employee's fault, such as those provided in Articles 283 and 284 of the Labor Code, as well as
in cases of illegal dismissal in which reinstatement is no longer feasible. It is not allowed when
an employee is dismissed for just cause.
In this case, Rivera was dismissed from work because she intentionally circumvented a strict
company policy, manipulated another entity to carry out her instructions without the company's
knowledge and approval, and directed the diversion of funds, which she even admitted doing
under the guise of shortening the laborious process of securing funds for promotional activities
from the head office. These transgressions were serious offenses that warranted her dismissal
from employment and proved that her termination from work was for a just cause. Hence, she is
not entitled to a separation pay.

71. Samr-med Distribution vs. NLRC, et al., G.R. No. 162385, July 15, 2013
Facts:
Gutang was hired by Samar-Med and had the task of supervising the companys sales
personnel and sales agents. Later, Gutang filed a complaint for money claims against SamarMed claiming that he had been illegally dismissed by the later. Samar-Med on the other hand
alleged that there was no dismissal to speak of but abandonment on Gutangs part; and that
this is merely in retaliation to the criminal case filed by Samar-Med againt Gutang for
misappropriating its funds which, during the preliminary investigation thereof, was ruled that
there is probable cause that Gutang is guilty of estafa.
Issue:
WON Gutang was illegally dismissed.
Ruling:
The petition is partly meritorious.
The onus of proving that an employee was not dismissed or, if dismissed, his dismissal was not
illegal fully rests on the employer, and the failure to discharge the onus would mean that the
dismissal was not justified and was illegal. In Gutangs case, Roleda tendered no showing
outside of his mere allegations to substantiate his averment of abandonment by Gutang.
Moreover, although Gutang had undoubtedly stopped working for Samar Med, his doing so had
been for a justifiable reason, consisting in the non-payment of his salary since November 1995
and his being forced to stop working for Samar Med to enable him to seek employment
elsewhere, albeit temporarily, in order to survive.
The dismissal, however, of Gutang was for a just cause. We note that Gutang was a managerial
employee whom Roleda had vested with confidence on delicate matters, such as the custody,

ATHENA M. SALAS | LABOR CASE DIGEST 2015 150

handling, care and protection of Samar Meds properties and funds, as well as its operations
and transactions in Region VIII. Gutang was shown to have failed to account for and to turn over
his sales collections. In that regard, Roledas filing of the criminal case against Gutang and the
public prosecutors finding of a prima facie case for the offense charged after preliminary
investigation amounted to substantial evidence of Gutangs breach of the trust and confidence
reposed in him, a just cause to terminate the employment based on loss of trust and confidence.
Under Article 282(c) of the Labor Code, an employer may terminate an employees employment
on the ground of the latters fraud or willful breach of the trust and confidence reposed in him.
For loss of trust and confidence to constitute a sufficient ground for termination, the employer
must have a reasonable ground to believe, if not to entertain the moral conviction, that the
employee was responsible for the misconduct, and that the nature of his participation therein
rendered him absolutely unworthy of the trust and confidence demanded by his position. Those
requirements were undeniably met in Gutangs case.
The finding of a just cause to dismiss Gutang notwithstanding, we also find that he was not
accorded due process. Roleda as the employer had the obligation to send to him two written
notices before finally dismissing him.
The first written notice would inform Gutang of the particular acts or omissions for which his
dismissal was being sought. The second written notice would notify him of the employers
decision to dismiss him. But the second written notice must not be made until after he was given
a reasonable period after receiving the first written notice within which to answer the charge,
and after he was given the ample opportunity to be heard and to defend himself with the
assistance of his representative, if he so desired. The requirement was mandatory.
Gutangs receipt of the demand letter from Samar-Med to return the amount of P3,302,000.71
was certainly not even a substantial compliance with the twin-notice requirement, because the
purpose of the demand letter was different from those defined for the sending of the required
notices. Nor was he thereby allowed a meaningful opportunity to be heard or to be notified of his
impending termination.
the lack of statutory due process would not nullify the dismissal or render it illegal or ineffectual
when the dismissal was for just cause. But the violation of Gutangs right to statutory due
process clearly warranted the payment of indemnity in the form of nominal damages, whose
amount is addressed to the sound discretion of the Court taking into account the relevant
circumstances. Accordingly, the Court deems the amount of P30,000.00 as nominal damages
sufficient vindication of Gutang's right to due process under the circumstances.

72. Naranjo et al, vs. Biomedica Health Care Inc. G.R. No. 193789, Sept. 19, 2012
Facts:
Petitioners are employees of respondent Biomedica. During the birthday of Biomedicas
President (Motol), petitioners were all absent for various reasons. De Guzman was allegedly
absent due to loose bowel movement, Pimentel for an ophthalmology check-up, Bardaje due to
migraine, Cruz for not feeling well, and Naranjo because he had to attend a meeting at his
childs school. Notably, these are the same employees who filed a letter-complaint against

ATHENA M. SALAS | LABOR CASE DIGEST 2015 151

Biomedica for lack of salary increases, failure to remit Social Security System and Pag-IBIG
contributions, and violation of the minimum wage law, among other grievances.
Later that day, the petitioners were texted to report to work, but upon arriving, they were refused
entry and told to start looking for another job.
Biomedica issued a notice of preventive suspension to petitioners, to wit:
Subject: Notice of Preventive Suspension
& Notice to explain within 24 hours
Effective upon receipt hereof, you are placed under preventive suspension for willfully
organizing and/or engaging in illegal strike on November 7, 2006. Your said illegal act-in
conspiracy with your other coemployees, paralyzed the company operation on that day and
resulted to undue damage and prejudice to the company and is direct violation of Article XI,
Category Four Section 6, 8, 12, 18 & 25 of our Company Policy, which if found guilty, you will be
meted a penalty of dismissal.
Please explain in writing within 24 hours from receipt hereof why you should not be held guilty of
violating the company policy considering further that you committed and timed such act during
the birthday of our Company president.
Thereafter, petitioners filed a complaint for constructive dismissal with NLRC.
Subsequently, Biomedica seved Notices of Termination on petitioners, to wit:
We regret to inform you that since you did not submit the written letter of explanation as
requested in your preventive suspension notice dated November 9, 2006, under Article XI,
Category Four, Section 6, 8, 12, 18 and 25 you are hereby dismissed from service effective
immediately.
Issue:
WON petitoners were illegally dismissed
Held:
The petition is meritorious.
Petitioners were illegally dismissed
It bears pointing out that in the dismissal of an employee, the law requires that due process be
observed. Such due process requirement is twofold, procedural and substantive, that is, the
termination of employment must be based on a just or authorized cause of dismissal and the
dismissal must be effected after due notice and hearing. In the instant case, petitioners were
not afforded both procedural and substantive due process.
Petitioners were not afforded Procedural due process
Art. 277(b) of the Labor Code contains the procedural due process requirements in the
dismissal of an employee.
On the other hand, Rule XIII, Book V, Sec. 2 I (a) of the

ATHENA M. SALAS | LABOR CASE DIGEST 2015 152

Implementing Rules and Regulations of the Labor Code states:


SEC. 2. Standards of due process; requirements of notice.In all cases of termination of
employment, the following standards of due process shall be substantially observed:
I. For termination of employment based on just causes as defined in Article 282 of the Code:
(a) A written notice served on the employee specifying the ground or grounds for termination,
and giving said employee reasonable opportunity within which to explain his side.
(b) A hearing or conference during which the employee concerned, with the assistance of
counsel if he so desires is given opportunity to respond to the charge, present his evidence, or
rebut the evidence presented against him.
(c) A written notice of termination served on the employee, indicating that upon due
consideration of all the circumstances, grounds have been established to justify his termination.
Thus, the Court elaborated in King of Kings Transport, Inc. v.Mama:
x x x Moreover, in order to enable the employees to intelligently prepare their explanation and
defenses, the notice should contain a detailed narration of the facts and circumstances that will
serve as basis for the charge against the employees. A general description of the charge will not
suffice. Lastly, the notice should specifically mention which company rules, if any, are violated
and/or which among the grounds under Art. 282 is being charged against the employees.
Reading from the first notice sent by Biomedica, clearly, petitioners were charged with
conducting an illegal strike, not a mass leave, without specifying the exact acts that the
company considers as constituting an illegal strike or violative of company policies. Such
allegation falls short of the requirement in King of Kings Transport, Inc. of a detailed narration of
the facts and circumstances that will serve as basis for the charge against the employees. A
bare mention of an illegal strike will not suffice.
Further, while Biomedica cites the provisions of the company policy which petitioners
purportedly violated, it failed to quote said provisions in the notice so petitioners can be
adequately informed of the nature of the charges against them and intelligently file their
explanation and defenses to said accusations. The notice is bare of such description of the
company policies. Moreover, it is incumbent upon respondent company to show that petitioners
were duly informed of said company policies at the time of their employment and were given
copies of these policies. No such proof was presented by respondents.
Without a copy of the company policy being presented in the CA or the contents of the pertinent
policies being quoted in the pleadings, there is no way by which one can determine whether or
not there was, indeed, a violation of said company policies.
Moreover, the period of 24 hours allotted to petitioners to answer the notice was severely
insufficient and in violation of the implementing rules of the Labor Code. Under the
implementing rule of Art. 277, an employee should be given reasonable opportunity to file a
response to the notice. King of Kings Transport, Inc. elucidates in this wise:
Reasonable opportunityunder the Omnibus Rules means every kind of assistance that
management must accord to the employees to enable them to prepare adequately for their
defense. This should be construed as a period of at least five (5) calendar days from receipt of

ATHENA M. SALAS | LABOR CASE DIGEST 2015 153

the notice to give the employees an opportunity to study the accusation against them, consult a
union official or lawyer, gather data and evidence, and decide on the defenses they will raise
against the complaint.
Following King of Kings Transport, Inc., the notice sent out by Biomedica in an attempt to
comply with the first notice of the due process requirements of the law was severely deficient.
In addition, Biomedica did not set the charges against petitioners for hearing or conference in
accordance with Sec. 2, Book V, Rule XIII of the Implementing Rules and Regulations of the
Labor Code and in line with ruling in King of Kings Transport, Inc., where the Court explained:
(2) After serving the first notice, the employers should schedule and conduct a hearing or
conference wherein the employees will be given the opportunity to: (1) explain and clarify their
defenses to the charge against them; (2) present evidence in support of their defenses; and (3)
rebut the evidence presented against them by the management. During the hearing or
conference, the employees are given the chance to defend themselves personally, with the
assistance of a representative or counsel of their choice. Moreover, this conference or hearing
could be used by the parties as an opportunity to come to an amicable settlement.
It is incumbent for Biomedica to set the matter for hearing or conference to hear the defenses
and receive evidence of the employees.
Lastly, Biomedica again deviated from the dictated contents of a written notice of termination as
laid down in Sec. 2, Book V, Rule XIII of the Implementing Rules that it should embody the facts
and circumstances to support the grounds justifying the termination. As amplified in King of
Kings Transport, Inc.:
(3) After determining that termination of employment is justified, the employers shall serve the
employees a written notice of termination indicating that: (1) all circumstances involving the
charge against the employees have been considered; and (2) grounds have been established to
justify the severance of their employment.
The November 26, 2006 Notice of Termination issued by Biomedica miserably failed to satisfy
the requisite contents of a valid notice of termination, as it simply mentioned the failure of
petitioners to submit their respective written explanations without discussing the facts and
circumstances to support the alleged violations of Secs. 6, 8, 12, 18 and 25 of Category Four,
Art. XI of the alleged company rules.
Petitioners were denied Substantive due process
In any event, petitioners were dismissed without just or authotized cause.The CA upheld the
dismissal of petitioners on the ground of serious misconduct.
Misconduct has been defined as improper or wrong conduct; the transgression of some
established and definite rule of action, a forbidden act, a dereliction of duty, unlawful in
character implying wrongful intent and not mere error of judgment. The misconduct to be
serious must be of such grave and aggravated character and not merely trivial and unimportant.
To be a just cause for dismissal, such misconduct (a) must be serious; (b) must relate to the
performance of the employees duties; and (c) must show that the employee has become unfit
to continue working for the employer.

ATHENA M. SALAS | LABOR CASE DIGEST 2015 154

Clearly, to justify the dismissal of an employee on the ground of serious misconduct, the
employer must first establish that the employee is guilty of improper conduct, that the employee
violated an existing and valid company rule or regulation, or that the employee is guilty of a
wrongdoing. In the instant case, Biomedica failed to even establish that petitioners indeed
violated company rules, failing to even present a copy of the rules and to prove that petitioners
were made aware of such regulations.

Petitioners did not stage a mass leave


The accusation is for engaging in a mass leave tantamount to an illegal strike.
The term Mass Leave has been left undefined by the Labor Code. Plainly, the legislature
intended that the terms ordinary sense be used. Mass is defined as participated in, attended
by, or affecting a large number of individuals; having a large-scale character. While the term
Leave is defined as an authorized absence or vacation from duty or employment usually with
pay.
Thus, the phrase mass leave may refer to a simultaneous availment of authorized leave
benefits by a large number of employees in a company.
It is undeniable that going on leave or absenting ones self from work for personal reasons when
they have leave benefits available is an employees right.
In the factual milieu at bar, Biomedica did not submit a copy of the CBA or a company
memorandum or circular showing the authorized sick or vacation leaves which petitioners can
avail of. Neither is there any document to show the procedure by which such leaves can be
enjoyed. Absent such pertinent documentary evidence, the Court can only conclude that the
availment of petitioners of their respective leaves was authorized, valid and in accordance with
the company or CBA rules on entitlement to and availment of such leaves. The contention of
Biomedica that the enjoyment of said leaves is in reality an illegal strike does not hold water in
the absence of strong controverting proof to overturn the presumption that a person is innocent
of x x x wrong. Thus, the individual leaves of absence taken by the petitioners are not such
absences that can be regarded as an illegal mass action.
Moreover, a mass leave involves a large number of people or in this case, workers.
Here, the five (5) petitioners were absent that day. The records are bereft of any evidence to
establish how many workers are employed in Biomedica. There is no evidence on record that 5
employees constitute a substantial number of employees of Biomedica. And, as earlier stated, it
is incumbent upon Biomedica to prove that petitioners were dismissed for just causes, this
includes the duty to prove that the leave was large-scale in character and unauthorized. This,
Biomedica failed to prove.
Petitioners did not go on strike
Granting for the sake of argument that the absence of the 5 petitioners is considered a mass
leave, still, their actions cannot be considered a strike.
Art. 212(o) of the Labor Code defines a strike as any temporary stoppage of work by the
concerted action of employees as a result of any industrial or labor dispute.

ATHENA M. SALAS | LABOR CASE DIGEST 2015 155

Concerted is defined as mutually contrived or planned or performed in unison. In the case


at bar, the 5 petitioners went on leave for various reasons. Petitioners were in different places
on that day to attend to their personal needs or affairs. They did not go to the company
premises to petition Biomedica for their grievance. To demonstrate their good faith in availing
their leaves, petitions reported for work and were at the company premises in the afternoon
after they received text messages asking them to do so. This shows that there was NO intent to
go on strike.

Dismissal is not the proper penalty


Even setting aside the facts established above, petitioners still cannot be validly dismissed.
An ordinary striking worker cannot be terminated for mere participation in an illegal strike. There
must be proof that he committed illegal acts during a strike.
In the instant case, Biomedica has not alleged, let alone, proved the commission by petitioners
of any illegal act during the alleged mass leave. There being none, the mere fact that petitioners
conducted an illegal strike cannot be a legal basis for their dismissal.
Petitioners are entitled to separation pay in lieu of
Reinstatement, backwages and nominal damages
The normal consequences of respondents illegal dismissal, then, are reinstatement without loss
of seniority rights, and payment of backwages computed from the time compensation was
withheld up to the date of actual reinstatement. Where reinstatement is no longer viable as an
option, separation pay equivalent to one (1) month salary for every year of service should be
awarded as an alternative. The payment of separation pay is in addition to payment of
backwages.
Petitioners were absent from work on Motols birthday. Respondent Motol, in the course of
denying entry to them uttered harsh, degrading and bad words. Petitioners were terminated in
swift fashion and in gross violation of their right to due process revealing that they are no longer
wanted in the company. The convergence of these facts coupled with the filing by petitioners of
their complaint with the DOLE shows a relationship governed by antipathy and antagonism as to
justify the award of separation pay in lieu of reinstatement. Thus, in addition to backwages,
owing to the strained relations between the parties, separation pay in lieu of reinstatement
would be proper. In Golden Ace Builders, We explained why:
Under the doctrine of strained relations, the payment of separation pay is considered an
acceptable alternative to reinstatement when the latter option is no longer desirable or viable.
On one hand, such payment liberates the employee from what could be a highly oppressive
work environment. On the other hand, it releases the employer from the grossly unpalatable
obligation of maintaining in its employ a worker it could no longer trust.
Strained relations must be demonstrated as a fact, however, to be adequately supported by
evidence substantial evidence to show that the relationship between the employer and the
employee is indeed strained as a necessary consequence of the judicial controversy.

ATHENA M. SALAS | LABOR CASE DIGEST 2015 156

Petitioners are entitled tonominal damages in the amount of PhP 30,000 each for Biomedicas
violation of procedural due process.

73. Manila Jockey Club Inc. vs. Trajano, G.R. No. 160982, June 26, 2013
Facts
MJCI had employed Trajano as a selling teller of betting tickets since November 1989. On April
25, 1998, she reported for work. At around 7:15 p.m., two regular bettors gave her their
respective lists of bets (rota) and money for the bets for Race 14. Although the bettors suddenly
left her, she entered their bets in the selling machine and segregated the tickets for pick up by
the two bettors upon their return. Before closing time, one of the bettors (requesting bettor)
returned and asked her to cancel one of his bets worth P2,000.00. Since she was also operating
the negative machine on that day, she obliged and immediately cancelled the bet as requested.
She gave the remaining tickets and the P2,000.00 to the requesting bettor, the money pertaining
to the canceled bet. When Race 14 was completed, she counted the bets received and the sold
tickets. She found that the bets and the tickets balanced. But then she saw in her drawer the
receipt for the canceled ticket, but the canceled ticket was not inside the drawer. Thinking she
could have given the canceled ticket to the requesting bettor, she immediately looked for him
but could not find him. It was only then that she remembered that there were two bettors who
had earlier left their bets with her. Thus, she went to look for the other bettor (second bettor) to
ask if the canceled ticket was with him. When she located the second bettor, she showed him
the receipt of the canceled ticket to counter-check the serial number with his tickets.
Thereafter, the second bettor returned to Trajano and told her that it was one of his bets that
had been canceled, instead of that of the requesting bettor. To complicate things, it was also the
same bet that had won Race 14. Considering that the bet was for a daily double, the second
bettor only needed to win Race 15 in order to claim dividends. At that point, she realized her
mistake, and explained to the second bettor that the cancellation of his ticket had not been
intentional, but the result of an honest mistake on her part. She offered to personally pay the
dividends should the second bettor win Race 15, which the latter accepted. When Race 15 was
completed, the second bettor lost. She was thus relieved of the obligation to pay any winnings
to the second bettor.
To her surprise, the reliever-supervisor later approached Trajano and told her to submit a written
explanation about the ticket cancellation incident. The next day (April 26, 1998), she submitted
the handwritten explanation to Atty. Joey R. Galit, Assistant Racing Supervisor. She then
resumed her work as a selling teller, until later that day, when she received an inter-office
correspondence signed by Atty. Galit informing her that she was being placed under preventive
suspension effective April 28, 1998, for an unstated period of time. At the end of thirty days of
her suspension, Trajano reported for work. But she was no longer admitted. She then learned
that she had been dismissed when she read a copy of an inter-office correspondence about her
termination posted in a selling station of MJCI.
Trajano instituted a complaint for illegal dismissal against MJCI in the Department of Labor and
Employment (DOLE). She claimed that her dismissal was not based on any of the grounds
enumerated under Article 282 of the Labor Code; that her dismissal on the ground of
unauthorized cancellation of ticket had no basis because she was also the operator of the
negative machine on the day in question with the authority to cancel tickets as requested; that

ATHENA M. SALAS | LABOR CASE DIGEST 2015 157

the cancellation was not intentional on her part but resulted from an honest mistake that did not
amount to dishonesty; that her dismissal was without due process of law because she was not
aware of any justifiable cause of her termination; that she was not notified about or furnished a
copy of the notice of dismissal; that instead, MJCI simply posted copies of the notice in all its
selling stations, an act intended to embarrass and humiliate her by imputing an allegedly
unauthorized cancellation of ticket against her; and that MCJI's acts were tainted with evident
bad faith and malice.
Trajano prayed that she be reinstated to her former position without loss of seniority rights; that
she be paid backwages until she would be fully reinstated; and that she be paid moral and
exemplary damages amounting to P180,000.00 and attorney's fees of 10% of the total award.
On its part, MJCI averred that on April 25, 1998, it received a letter from Jun Carpio, the Field
Officer of the Games and Amusement Board, calling its attention to a complaint against Trajano
brought by a certain bettor named "Tito" who had reported the cancellation of his ticket that had
already won the first leg (Race 14) of the daily double bet; that it acted on the complaint by
placing her under preventive suspension upon her submission of a written explanation and
after the conduct of preliminary investigation on the matter; that on June 5, 1998, it invited her to
a clarificatory meeting in the presence of MJCI Raceday Union President Miguel Altonaga; and
that it terminated her services on the next day "for cause due to unauthorized cancellation of
ticket."
MJCI maintained that Trajano's dismissal was justified because the unauthorized cancellation of
the ticket had constituted a serious violation of company policy amounting to dishonesty; that
her action had also constituted a just cause for terminating her employment under Article 282 of
the Labor Code, particularly paragraph(a) on serious misconduct or willful disobedience and
paragraph (b) on gross and habitual neglect of duty; that the admissions made in her written
explanation left no doubt as to her participation in the unauthorized cancellation of the ticket;
that she was afforded her right to due process by being given the chance to submit her written
explanation and being appraised of the charges against her; that she was accompanied by the
union leaders during the preliminary investigation of her case; and that the non-appeal of the
decision to terminate her indicated that she and the union leaders believed in the merit of the
decision to terminate her.
Issues
1.Whether or not there was just cause when Petitioner (MJCI) dismissed Respondent Aimee O.
Trajano from the service; and
2.Whether or not Petitioner MJCI complied with the due process requirement when it effected
the dismissal of Respondent Trajano.
Ruling
MJCI posits that Trajano held a position of trust and confidence; that the act of canceling the
ticket was unauthorized because it was done without the consent of the bettor; that the CA thus
erred in construing the phrase unauthorized cancellation of ticket as referring to whether or not
she was authorized to cancel the ticket pursuant to company rules; that under the same
premise, the loss of trust and confidence was established because the unauthorized
cancellation of the ticket was a serious misconduct on her part considering that had the bet of
P2,000.00 won the daily double race, the dividend to be paid could have been such a big
amount that she would be unable to pay on her own; that the repercussions of her act to MJCI

ATHENA M. SALAS | LABOR CASE DIGEST 2015 158

would have been disastrous had the bet won, with MJCI being sued by the bettor and being
scandalized in the media; that MJCI would have suffered great loss in both income and
reputation due to such unauthorized cancellation of ticket; and that, consequently, MJCI had the
just cause to dismiss her.
We cannot sustain the position of MJCI.
The valid termination of an employee may either be for just causes under Article 282 or for
authorized causes under Article 283 and Article 284, all of the Labor Code.
Specifically, loss of the employer's trust and confidence is a just cause under Article 282 (c), a
provision that ideally applies only to cases involving an employee occupying a position of trust
and confidence, or to a situation where the employee has been routinely charged with the care
and custody of the employer's money or property. But the loss of trust and confidence, to be a
valid ground for dismissal, must be based on a willful breach of trust and confidence founded on
clearly established facts. "A breach is willful, if it is done intentionally, knowingly and purposely,
without justifiable excuse, as distinguished from an act done carelessly, thoughtlessly,
heedlessly or inadvertently. It must rest on substantial grounds and not on the employer's
arbitrariness, whims, caprices or suspicion; otherwise, the employee would eternally remain at
the mercy of the employer." An ordinary breach is not enough.
Moreover, the loss of trust and confidence must be related to the employee's performance of
duties.
Loss of confidence, as a just cause for termination of employment, is premised on the fact that
the employee concerned holds a position of responsibility, trust and confidence. He must be
invested with confidence on delicate matters such as the custody, handling, care and protection
of the employer's property and/or funds. But in order to constitute a just cause for dismissal, the
act complained of must be "work-related" such as would show the employee concerned to be
unfit to continue working for the employer.
As a selling teller, Trajano held a position of trust and confidence. The nature of her employment
required her to handle and keep in custody the tickets issued and the bets made in her assigned
selling station. The bets were funds belonging to her employer. Although the act complained of
the unauthorized cancellation of the ticket (i.e., unauthorized because it was done without the
consent of the bettor) was related to her work as a selling teller, MJCI did not establish that
the cancellation of the ticket was intentional, knowing and purposeful on her part in order for her
to have breached the trust and confidence reposed in her by MJCI, instead of being only out of
an honest mistake.
Still, to justify the supposed loss of its trust and confidence in Trajano, MJCI contends that the
unauthorized cancellation of the ticket could have greatly prejudiced MJCI for causing damage
to both its income and reputation. CASTDI
We consider the contention of MJCI unwarranted. As the records indicate, MJCI's prejudice
remained speculative and unrealized. To dismiss an employee based on speculation as to the
damage the employer could have suffered would be an injustice. The injustice in the case of
Trajano would be greater if the supposed just cause for her dismissal was not even sufficiently
established. While MJCI as the employer understandably had its own interests to protect, and
could validly terminate any employee for a just cause, its exercise of the power to dismiss

ATHENA M. SALAS | LABOR CASE DIGEST 2015 159

should always be tempered with compassion and imbued with understanding, avoiding its
abuse.
In this regard, we have to stress that the loss of trust and confidence as a ground for the
dismissal of an employee must also be shown to be genuine, for, the loss of confidence should
not be simulated in order to justify what would otherwise be, under the provisions of law, an
illegal dismissal. It should not be used as a subterfuge for causes which are illegal, improper
and unjustified. It must be genuine, not a mere afterthought to justify an earlier action taken in
bad faith."
The foregoing notwithstanding, the Court unavoidably notes that the invocation of loss of trust
and confidence as a ground for dismissing Trajano was made belatedly. In its position paper
dated September 2, 1998, MJCI invoked the grounds under Article 282 (a) and (b) of the Labor
Code to support its dismissal of her, submitting then that the unauthorized cancellation of the
ticket constituted a serious violation of company policy amounting to dishonesty. The first time
that MJCI invoked breach of trust was in its motion for the reconsideration of the decision of the
NLRC. MJCI also thereafter urged the ground of breach of trust in its petition for certiorari in the
CA. Such a belated invocation of loss of confidence broadly hints the ground as a mere
afterthought to buttress an otherwise baseless dismissal of the employee.
Anent compliance with due process, MJCI argues that Trajano's notification of her termination
through the posting in the selling stations should be deemed a substantial if not full compliance
with the due process requirement, considering that she herself even presented a copy of the
posting as evidence; that the rule on giving notice of termination to an employee did not
expressly require the personal service of the notice to the dismissed worker; and that what
mattered was that she was notified in writing of MJCI's decision to terminate her through the
posting in its selling stations.
The argument is bereft of worth and substance.
The procedure to be followed in the termination of employment based on just causes is laid
down in Section 2 (d), Rule I of the Implementing Rules of Book VI of the Labor Code, to wit:
Section 2.Security of Tenure.
xxx xxx xxx
(d)In all cases of termination of employment, the following standards of due process
shall be substantially observed:
For termination of employment based on just causes as defined in Article 282 of the
Labor Code:
(i)A written notice served on the employee specifying the ground or grounds for
termination, and giving said employee reasonable opportunity within which to explain his
side.
(ii)A hearing or conference during which the employee concerned, with the assistance of
counsel if he so desires is given opportunity to respond to the charge, present his
evidence, or rebut the evidence presented against him.

ATHENA M. SALAS | LABOR CASE DIGEST 2015 160

(iii)A written notice of termination served on the employee, indicating that upon due
consideration of all the circumstances, grounds have been established to justify his
termination. In case of termination, the foregoing notices shall be served on the
employee's last known address.
A review of the records warrants a finding that MJCI did not comply with the prescribed
procedure.
There is no question that an illegally dismissed employee is entitled to her reinstatement without
loss of seniority rights and other privileges, and to full backwages, inclusive of allowances and
other benefits or their monetary equivalent.
In case the reinstatement is no longer possible, however, an award of separation pay, in lieu of
reinstatement, will be justified. The Court has ruled that reinstatement is no longer
possible: (a) when the former position of the illegally dismissed employee no longer
exists; or (b) when the employer's business has closed down; or (c)when the employeremployee relationship has already been strained as to render the reinstatement impossible.
The Court likewise considered reinstatement to be non-feasible because a "considerable time"
has lapsed between the dismissal and the resolution of the case. In that regard, a lag of eight
years or ten years is sufficient to justify an award of separation pay in lieu of reinstatement.
Applying the foregoing to this case, the Court concludes that the reinstatement of Trajano is no
longer feasible. More than 14 years have already passed since she initiated her complaint for
illegal dismissal in 1998, filing her position paper on September 3, 1998, before the Court could
finally resolve her case. The lapse of that long time has rendered her reinstatement an
impractical, if not an impossible, option for both her and MJCI. Consequently, an award of
separation pay has become the practical alternative, computed at one month pay for every year
of service.
Anent backwages, Trajano is entitled to full backwages, inclusive of allowances and other
benefits or their monetary equivalent, computed from the time her actual compensation was
withheld on June 6, 1998 up to the finality of this decision (on account of her reinstatement
having meanwhile become non-feasible and impractical). This ruling is consistent with the
legislative intent behind Republic Act No. 6715.
74. Fianza vs. NLRC et al., G.R. No. 163061, June 26, 2013
Facts
On 3 June 1997, petitioner Fianza was employed as Officer for Social Acceptance of
respondent Binga Hydroelectric Plant, Inc. The details of his employment are embodied in
Memorandum 97-10 dated 2 June 1997 issued by Mr. Catalino Tan, the president and
chairperson of the board at that time.
In February 1999, petitioner did not receive his salary of P15,000 for the first 15 days of the
month of February. He was advised not to report for work until his status was officially clarified
by the Manila office.
After petitioner made several other inquiries concerning his status, he was told by a supervisor
to report for work. However, he was also told that the new management committee had to

ATHENA M. SALAS | LABOR CASE DIGEST 2015 161

concur in his reappointment before he could be reinstated in the payroll. It also wanted an
opportunity to determine whether his services would still be necessary to the
company. Meanwhile, the chief of the rehabilitation department of the company recommended
his return.
As the management committee did not act on his inquiries for several months, on 24 May 1999
petitioner filed a Complaint for illegal dismissal before the LA.
Issues
There are, in essence, two important questions to be answered: first, whether petitioner
abandoned his work; and second, whether his employment was regular.
Ruling
In his pleadings, petitioner argues that he was a supervisory employee, as shown by the
evidence he presented and the nature of his work. He further contends that he did not abandon
his work, because he always made sure he followed up the status of his employment, and he
was willing to go back to work once he was re-enrolled in the payroll.
Respondent company asserts in its Memorandum that petitioner was a confidential consultant of
its former president and chairperson Catalino Tan. As such, petitioner's tenure was therefore coterminus with that of Mr. Tan.
At the outset, it is clear that the requisites for a judicial declaration of abandonment are absent
in this case. Suffice it to say that abandonment is a fact that must be proven in accordance with
the standard set by this Court:
It is well-settled in our jurisprudence that "For abandonment to constitute a valid cause
for termination of employment, there must be a deliberate, unjustified refusal of the
employee to resume his employment. This refusal must be clearly shown. Mere absence
is not sufficient, it must be accompanied by overt acts unerringly pointing to the fact that
the employee does not want to work anymore" (Emphasis and italics supplied)
Abandonment as a fact and a defense can only be claimed as a ground for dismissal if the
employer follows the procedure set by law. In line with the burden of proof set by law, the
employer who alleges abandonment "has the burden of proof to show a deliberate and
unjustified refusal of the employee to resume his employment without any intention of
returning." As this Court has stated in Agabon v. National Labor Relations
Commission: ECaTAI
For a valid finding of abandonment, these two factors should be present: (1) the failure
to report for work or absence without valid or justifiable reason; and (2) a clear intention
to sever employer-employee relationship, with the second as the more determinative
factor which is manifested by overt acts from which it may be deduced that the
employees has no more intention to work. The intent to discontinue the employment
must be shown by clear proof that it was deliberate and unjustified.
From the foregoing, it is clear that respondent company failed to prove the necessary elements
of abandonment. Additionally, the NLRC and the CA failed to take into account the strict

ATHENA M. SALAS | LABOR CASE DIGEST 2015 162

requirements set by jurisprudence when they determined the existence of abandonment on the
basis of mere allegations that were contradicted by the evidence shown.
The very act of filing the Complaint for illegal dismissal should have negated any intention on
petitioner's part to sever his employment. In fact, it should already have been sufficient
evidence to declare that there was no abandonment of work. Moreover, petitioner went back to
the company several times to inquire about the status of his employment. The fact that his
inquiries were not answered does not prejudice this position.
Throughout the entire ordeal, petitioner was vigilant in protecting himself from any claim that he
had abandoned his work. The following circumstances evinced his intent to return to work:
1.His continuous inquiry with respondent about the status of his work.
2.His willingness to return to work at any time, subject to the approval of respondent,
and his visits to the plant to apply for work.
3.His filing of an illegal dismissal case.
Considering all these facts, established by the LA and confirmed by the NLRC and the CA, we
conclude that both appellate bodies were remiss in declaring the existence of
abandonment. ECSHAD
Since the first question has been disposed of, the second one now becomes the core issue,
because the existence of an employer-employee relationship in the nature of regular
employment will determine whether or not the company dismissed petitioner illegally.
Respondent company claims that because petitioner was a confidential employee of its former
president, his tenure was co-terminus with that of his employer. To establish this contention,
respondent cites the CA's determination of the facts, as follows:
1.Petitioner directly reported to Mr. Tan, the hiring authority.
2.The hiring did not pass through the existing procedure.
3.The position of officer for social acceptance was absent from the company's table of
organization and position title.
4.Petitioner did not submit any daily time record.
5.Monthly fees received from Mr. Tan were denominated as retainer fees and subjected
to 10% deductions.
6.Petitioner was not included in the payroll.
7.The taxes on the fees were paid by respondent company on behalf of petitioner.
8.Petitioner's name was absent from respondent's records.
These facts allegedly proved that petitioner was the confidential employee of Mr. Tan,
respondent's former president. All of this occurred in the context of a rehabilitation receivership
conducted by the Securities and Exchange Commission Management Committee.
Respondent company failed to realize however that Mr. Tan, being its president, was clothed
with authority to hire employees on its behalf. This was precisely the import of petitioner's
appointment papers, which even carried the letterhead of the company. There is no indication
from the facts that his employment was of a confidential nature. The wording of his appointment
itself does not bear out that conclusion.
Several things stand out in his appointment paper. First, its letterhead is that of respondent
company, indicating the official nature of the document. Second, there is no indication that the

ATHENA M. SALAS | LABOR CASE DIGEST 2015 163

employment is co-terminus with that of the appointing power, or that the position was a
confidential one. In fact, alongside the obligation of petitioner to report to Mr. Tan, is that of
reporting to those whom the latter had designated as well as to the management in case
petitioner had any suggestion. This description evinces a supervisory function, by which the
employee will carry out company policy, but can only give suggestions to management as to the
creation or implementation of a new policy.
Finally, the appointment paper recognizes that the petitioner would initially be on probation
status for two months, at the end of which he would be made a permanent employee should his
services be found satisfactory by respondent. All these circumstances are evident from the
appointment paper itself, which belies the claim of respondent that it had no employer-employee
relationship with petitioner.

75. Pasos vs. Phil National Construction Corp. G.R. No. 192394, July 3, 2013
Facts:
Roy D. Pasos (Pasos) was first hired by Philippine National Construction Corporation (PNCC)
on 26 April 1996. In his employment contract, he was designated as a Project Employee from
26 April 1996 to 25 July 1996 and was assigned to NAIA-II Project as Clerk II (Accounting). His
employment however did not end on 25 July 1996 but was extended until 4 August 1998 (or
more than two years later). His employment contract specifically provided that, If services are
still needed beyond the validity of this contract, the Company shall extend your (Pasos)
services. After services are terminated, the employee shall be under no obligation to re-employ
with the Company nor shall the Company be obliged to re-employ the employee.
Pasos was again hired on 11 November 1998, this time as project employee from 11 November
1998 to 11 February 1999 and assigned to PCSO-Q.I. Project as Accounting Clerk (Reliever).
However, his employment did not actually end on 11 February 1999 but was extended until 19
February 1999.
On 23 February 1999, Pasos was again hired as Accounting Clerk and was assigned to SM
Project. This time though, his employment contract did not specify the date when his
employment will end but it was stated therein that it will be co-terminus with the termination of
the project. Said employment supposedly ended on 19 August 1999. However, it appears that
said employment was extended as Pasos was again appointed as Accounting Clerk for SM
Project (Package II). Again, the employment contract did not state a specific date up to when his
extended employment will be.
Pasos employment was supposedly terminated on 19 October 2000. Pasos however claimed
that his superior instructed him to report for work the following day, intimating to him that he will
again be hired in succeeding SM Projects. For purposes of re-employment, Pasos underwent
medical examination which allegedly revealed that he had pneumonitis. He was advised to take

ATHENA M. SALAS | LABOR CASE DIGEST 2015 164

a 14-day sick leave. After his 14-day leave, he again underwent a medical examination which
revealed that he has Kochs Disease. He was advised to go on a 6-month leave of absence.
After such leave of absence he was declared fit to work, but was denied employment. He was
told that his employment ended on 19 October 2000. PNCC reasoned that it was not obliged to
rehire Pasos, as he was a Project Employee and his employment ended on 19 October 2000.
Hence this case for illegal dismissal with prayer for backwages and reinstatement.
Issues:
1. Is Pasos a regular employee and not a mere project employee and thus can only be
dismissed for cause?
2. Is Pasos entitled to reinstatement?
Ruling:
1) Yes.
In the instant case, the appointments issued to petitioner indicated that he was hired for specific
projects. This Court is convinced however that although he started as a project employee, he
eventually became a regular employee of PNCC.
The principal test used to determine whether employees are project employees is whether or
not the employees were assigned to carry out a specific project or undertaking, the duration or
scope of which was specified at the time the employees were engaged for that project.
In the case at bar, petitioner worked continuously for more than two years after the supposed
three-month duration of his project employment for the NAIA II Project. While his appointment
for said project allowed such extension since it specifically provided that in case his "services
are still needed beyond the validity of the contract, the Company shall extend his services,"
there was no subsequent contract or appointment that specified a particular duration for the
extension.
While for first three months, petitioner can be considered a project employee of PNCC, his
employment thereafter, when his services were extended without any specification of as to the
duration, made him a regular employee of PNCC. And his status as a regular employee was not
affected by the fact that he was assigned to several other projects and there were intervals in
between said projects since he enjoys security of tenure.
A regular employee dismissed for a cause other than the just or authorized causes provided by
law is illegally dismissed. Petitioners regular employment was terminated by PNCC due to
contract expiration or project completion, which are both not among the just or authorized
causes provided in the Labor Code, as amended, for dismissing a regular employee. Thus,
petitioner was illegally dismissed.
2) Yes.
Article 279 of the Labor Code, as amended, provides that an illegally dismissed employee is
entitled to reinstatement, full back wages, inclusive of allowances, and to his other benefits or
their monetary equivalent from the time his compensation was withheld from him up to the time
of his actual reinstatement.

ATHENA M. SALAS | LABOR CASE DIGEST 2015 165

We agree with petitioner that there was no basis for the Labor Arbiters finding of strained
relations and order of separation pay in lieu of reinstatement. This was neither alleged nor
proved. Moreover, it has long been settled that the doctrine of strained relations should be
strictly applied so as not to deprive an illegally dismissed employee of his right to reinstatement.
As held in Globe-Mackay Cable and Radio Corporation v. NLRC:
Obviously, the principle of "strained relations" cannot be applied indiscriminately. Otherwise,
reinstatement can never be possible simply because some hostility is invariably engendered
between the parties as a result of litigation. That is human nature.

Besides, no strained relations should arise from a valid and legal act of asserting ones right;
otherwise an employee who shall assert his right could be easily separated from the service, by
merely paying his separation pay on the pretext that his relationship with his employer had
already become strained.

76 Universal Robina Corp et al., v. Castillo G.R. No. 189686, July 10, 2013
Facts:
Wilfredo Castillo (Castillo) was hired by Universal Robina Corporation (URC) as a truck
salesman on 23 March 1983. He rose to the ranks and eventually became a Regional Sales
Manager until his dismissal on 12 January 2006.
As Regional Sales Manager, he is in tasked to transact, sign and represent the company in all
its dealings with key accounts or customers subject however to his selling expense budget duly
approved by URC Management. Consequently, he is obliged to give an account of all his
dealings or transactions with all his customers to URC. One such customer is Lianas
Supermarket (Lianas).
Sometime in August 2005, URCs Credit and Collection Department Analyst noted an outright
deduction in the amount of Php 72,000 tagged as Gift Certificate. This was unusual and so an
investigation was conducted, which investigation led to the conclusion that Castillo received Gift
Certificates from Lianas worth Php 72 000, which amount remains unresolved in the URC
Account Receivables records. The Corporate Internal Audit, who conducted the investigation,
suspected that Castillo may have committed an act of fraud against the company and Lianas
for his personal gain.
When Lianas Vice President confirmed that Castillo received Gift Certificate worth Php 70 000
from Liana, Castillo was asked was asked to explain in writing why the company should not
institute the appropriate disciplinary action against him.
Further investigation likewise revealed that Castillo signed two (2) blank Charge Invoices of
Lianas Supermarket. Thus, another memo was sent to Castillo directing him to explain why no
administrative sanctions should be meted against him.

ATHENA M. SALAS | LABOR CASE DIGEST 2015 166

Castillo repeatedly denied that he signed two (2) blank Charge Invoices. Clarification inquiries
were held and on 9 January 2006, respondent was served with a written notice of termination.
Thus on 30 May 2006, Castillo filed a case for illegal dismissal against URC. The Labor Arbiter
rendered a decision in Castillos favor and directed URC to pay Backwages, Separation Pay
and Attorneys Fees. URC appealed to NLRC which reversed the Labor Arbiter finding the
dismissal of Castillo as valid. On Certiorari to CA, the dismissal was upheld but the award of
Separation Pay was reinstated, as a form of equitable relief. Hence, this case.
Issues:
Whether Castillo is entitled to Separation Pay
Held:
No.
Why and when separation pay may be awarded or denied, has been the subject of many cases.
We pick out the rulings pertinent to the case at hand.
As the rule now stands, the award of separation pay is authorized in the situations dealt with in
Article 283 and 284 of the Labor Code, but not in terminations of employment based on
instances enumerated in Article 282.18Article 282 states that:
ART. 282. Termination by employer. An employer may terminate an employment for any of the
following causes:
(a) Serious misconduct or willful disobedience by the employee of the lawful orders of his
employer or representative in connection with his work;
(b) Gross and habitual neglect by the employee of his duties;
(c) Fraud or willful breach by the employee of the trust reposed in him by his employer or duly
authorized representative;
(d) Commission of a crime or offense by the employee against the person of his employer or
any immediate member of his family or his duly authorized representatives; and
(e) Other causes analogous to the foregoing.
Central Philippines Bandag Retreaders, Inc. cautioned labor tribunals in indiscriminately
awarding separation pay as a measure of social justice, in this wise:
x x x Labor adjudicatory officials and the CA must demur the award of separation pay based on
social justice when an employees dismissal is based on serious misconduct or willful
disobedience; gross and habitual neglect of duty; fraud or willful breach of trust; or commission
of a crime against the person of the employer or his immediate familygrounds under Art. 282
of the Labor Code that sanction dismissals of employees. They must be most judicious and
circumspect in awarding separation pay or financial assistance as the constitutional policy to
provide full protection to labor is not meant to be an instrument to oppress the employers. The
commitment of the Court to the cause of labor should not embarrass us from sustaining the
employers when they are right, as here. In fine, we should be more cautious in awarding
financial assistance to the undeserving and those who are unworthy of the liberality of the law.

ATHENA M. SALAS | LABOR CASE DIGEST 2015 167

Indeed, respondent has committed acts constituting willful breach of trust and confidence
reposed on him by URC based on the following facts established by the Court of Appeals, thus:
x x x the principal charge against petitioner Castillo was hinged upon "unauthorized
arrangements" which he allegedly entered into. Petitioner Castillos unauthorized dealing with
respect to the changes in the Account Development Agreement is exactly the offending cause of
the host of infractions he committed, i.e., his neglect in signing the blank charge invoices and
his improper receipt of gift certificates for his personal gain. These acts taken together constitute
a breach of the trust and confidence reposed on petitioner Castillo by private respondent URC.
xxx

Nonetheless, the evidence on record negates petitioner Castillos claim of good faith and
furnishes sufficient basis for the breach of trust and loss of confidence reposed on him by
private respondent URC. Petitioner Castillos receipt of the gift certificates is categorically
confirmed by Peter Sy, the Vice President of Marketing of Lianas Supermarket. This piece of
evidence, coming from a disinterested party, speaks eloquently of petitioner Castillos perfidy.
Such an affirmative statement coupled with petitioner Castillo's signatures on the charge
invoices convincingly established the fact that he indeed received the P72,000.00 worth of gift
certificates.
In Bank of the Philippine Islands v. NLRC and Arambulo, we ruled that an employee who has
been dismissed for a just cause under Article 282 of the Labor Code is not entitled to separation
pay. The complainant therein was likewise dismissed on the ground of Joss of trust and
confidence. Applying that rule to the instant case, we here hold that respondent is not entitled to
separation pay.
77. Martinez vs. Central Pangasinan Electric Cooperative, G.R. No. 192306, July 15, 2013
Facts:
In 1991, CENPELCO employed Martinez on a contractual basis and in 1993, was subsequently
regularized as a billing clerk at the former's main office in San Carlos City, Pangasinan. On
January 7, 2002, CENPELCO gave Martinez the position of teller at Area VI in Malasiqui,
Pangasinan.
On April 26, 2002, CENPELCOs Internal Audit Department (IAD) conducted a cash count audit
at its Area VI. Josefina Mandapat (Mandapat), the IAD Officer-in-Charge, analyzed the audit
results and concluded that there was an error in the count of Benjamin Madriaga (Madriaga),
cashier for Area VI, regarding the breakdown of collection turned over by Martinez for April 23,
2002. Specifically, Madriaga erroneously recorded that Martinez remitted 390 pieces of P500bills, instead of the correct number which was just 290, and issued a handwritten temporary
receipt for P406,130.31 instead of P360,447.13. Upon noting that Madriaga issued Official
Receipts Nos. 77365-77367 for the amount of P360,447.13 with corresponding remittance stubs
for Martinezs April 23, 2002 collections, Mandapat concluded that Martinezs overage for the
same day in the amount of P45,682.58 is questionable. Further, Mandapat noted that on April
25, 2002, Martinez committed a shortage in the amount ofP44,846.77, considering that the
latters total accountability for the said date is in the amount of P212,258.56 but his actual cash
count only amounted to P167,411.79.

ATHENA M. SALAS | LABOR CASE DIGEST 2015 168

In view of such audit, Mandapat recommended that Madriaga and Martinez be made to explain
why no disciplinary action should be taken against them. Thus, on May 15, 2002, Martinez filed
his letter-explanation, explaining that he submitted his collections and remittance stubs to
Madriaga who was the one tasked to make the report thereon and who may have mishandled
the proper listing and tallying of the money collected vis--vis the collection stubs. He further
admitted the existence of such shortage and tried to offset the same with his alleged overage on
April 23, 2002.
On June 30 2002, the Companys Grievance Committee, which was commissioned to
investigate the charges imputed to Martinez, submitted its report recommending Martinezs
termination from employment as well as the filing of the appropriate case in court. On November
26, 2002, Martinez was dismissed from service, prompting him to file a complaint for illegal
dismissal with money claims for 13th month pay, service incentive leave pay and allowances, as
well as moral and exemplary damages.
Issue:
The sole issue raised for the Courts resolution is whether Martinezs dismissal on the ground of
loss of trust and confidence is valid.
Ruling:
The petition is bereft of merit.
To validly dismiss an employee on the ground of loss of trust and confidence under Article
296(c) (formerly Article 282[c]) of the Labor Code, the following guidelines must be observed:
(1) the employee concerned must be holding a position of trust and confidence; and (2) there
must be an act that would justify the loss of trust and confidence.
Anent the first requisite, it is noteworthy to mention that there are two classes of positions of
trust, namely: (1) managerial employees whose primary duty consists of the management of the
establishment in which they are employed or of a department or a subdivision thereof, and to
other officers or members of the managerial staff; and (2) fiduciary rank-and-file employees
such as cashiers, auditors, property custodians, or those who, in the normal exercise of their
functions, regularly handle significant amounts of money or property. These employees, though
rank-and-file, are routinely charged with the care and custody of the employers money or
property, and are thus classified as occupying positions of trust and confidence. Being an
employee tasked to collect payments and remit the same to CENPELCO, Martinez belongs to
the latter class and thus, occupies a position of trust and confidence.
Anent the second requisite, the audit report conducted on Martinez's cash count revealed that
he had a shortage in the amount of P44,846. When asked to explain such shortage, Martinez
not only admitted the same but even tried to exculpate himself from liability by attempting to
offset said shortage with his alleged overage on April 23, 2002 in the amount of P45,682.58.
The Court agrees with the CA that this practice should never be countenanced because it would
allow the employees to patch up inaccuracies or even their own wrongdoings and thus, the true
revenues or losses of the company will never be correctly identified. Verily, this irregular practice
would be detrimental to the interests of the employer whose bread and butter depends solely on
realized profits. Perforce, Martinez's failure to properly account for his shortage of such a
significant amount is enough reason for CENPELCO to lose trust and confidence in him.

ATHENA M. SALAS | LABOR CASE DIGEST 2015 169

78. Zuellig Pharma Corp vs. Sibal et al., G.R. No. 173587, July 15, 2013
Facts:
Petitioner Zuellig Pharma Corporation (Zuellig) is a domestic corporation engaged in the
manufacture and distribution of pharmaceutical products. It also distributes pharmaceutical
products manufactured by other companies like Syntex Pharmaceuticals (Syntex). Respondents
(36 in all), on the other hand, were the employees of Zuellig at its Syntex Division.
In 1995, Roche Philippines, Inc. (Roche) purchased Syntex and took over from Zuellig the
distribution of Syntex products. Consequently, Zuellig closed its Syntex Division and terminated
the services of respondents due to redundancy. They were properly notified of their termination
and were paid their respective separation pay in accordance with Section 3(b), Article XIV of the
March 21, 1995 Collective Bargaining Agreement (CBA) for which, respondents individually
signed Release and Quitclaim in full settlement of all claims arising from their employment with
Zuellig.
Controversy arose when respondents filed before the Arbitration Branch of the NLRC separate
Complaints (which were later consolidated) for payment of retirement gratuity and monetary
equivalent of their unused sick leave on top of the separation pay already given them.
Respondents claimed that they are still entitled to retirement benefits and that their receipt of
separation pay and execution of Release and Quitclaim do not preclude pursuing such claim.
Issues:
I
THE COURT OF APPEALS COMMITTED GRAVE ERROR WHEN IT HELD THAT UNDER THE
TERMS AND CONDITIONS OF THE CBA AND THE RETIREMENT AND GRATUITY PLAN X X
X RESPONDENTS COULD AVAIL OF BOTH REDUNDANCY PAY AND RETIRMENT
BENEFITS.
II
THE COURT OF APPEALS COMMITTED GRAVE ERROR IN FINDING THAT RESPONDENTS
ARE ENTITLED TO THE MONETARY EQUIVALENT OF UNUSED SICK LEAVE.
III
THE COURT OF APPEALS COMMITTED GRAVE ERROR IN FAILING TO HOLD THAT
QUITCLAIMS BAR RESPONDENTS FROM CLAIMING FROM PETITIONER ANY MORE
THAN THEY HAVE LAWFULLY RECEIVED.
Ruling:
The Petition is impressed with merit.
I.

The CBA does not allow recovery of both separation pay and retirement gratuity.

In the case of Aquino, the petitioner employees were retrenched after their employer Otis
Elevator Company (Otis) adopted cost-cutting measures and streamlined its operations. They
were thus given separation pay double the amount required by the Labor Code. Subsequently,

ATHENA M. SALAS | LABOR CASE DIGEST 2015 170

however, the employees filed a claim for retirement benefits, alleging entitlement thereto by
virtue of the Retirement Plan. Otis denied the claim by asserting that separation pay and
retirement benefits are mutually exclusive of each other; hence, acceptance of one bars
recovery of the other. When the case reached its final review, this Court held that in the absence
of specific prohibition in the retirement plan or the CBA, retirement benefits and separation pay
are not mutually exclusive of each other and the employees whose services were terminated
without cause are entitled to both separation pay and retirement gratuity.
In the present case, the CBA contains specific provisions which effectively bar the availment of
retirement benefits once the employees have chosen separation pay or vice versa.
Section 2 of Article XIV of the CBA explicitly states that any payment of retirement gratuity shall
be chargeable against separation pay. Clearly, respondents cannot have both retirement
gratuity and separation pay, as selecting one will preclude recovery of the other. To illustrate the
mechanics of how Section 2 of Article XIV bars double recovery, if the employees choose to
retire, whatever amount they will receive as retirement gratuity will be charged against the
separation pay they would have received had their separation from employment been for a
cause which would entitle them to severance pay. These causes are enumerated in Section 3,
Article XIV of the CBA (i.e., retrenchment, closure of business, merger, redundancy, or
installation of labor-saving device). However, if the cause of the termination of their employment
was any of the causes enumerated in said Section 3, they could no longer claim retirement
gratuity as the fund from which the same would be taken had already been used in paying their
separation pay. Put differently, employees who were separated from the company cannot have
both retirement gratuity and separation pay as there is only one fund from which said benefits
would be taken. Inarguably, Section 2 of Article XIV effectively disallows recovery of both
separation pay and retirement gratuity. Consequently, respondents are entitled only to one.
Since they have already chosen and accepted redundancy pay and have executed the
corresponding Release and Quitclaim, they are now barred from claiming retirement gratuity.
In Suarez, Jr. v. National Steel Corporation, the same issue cropped up whether the
retrenched employees are entitled to retirement gratuity even after they have received their
separation pay in accordance with the retrenchment program of the company. In ruling in the
negative, this Court observed that Sections 1 and 3 of Article XIV on Retirement Benefits of the
CBA separately provide for retirement benefits and severance pay for retrenched employees.
Section 1 thereof states, among others, that those retiring with at least 10 years of service
credits are entitled to a retirement pay equivalent to one and one-half months of basic pay for
every year of service, while Section 3 extends two months base pay for every year of service for
laid-off employees pursuant to retrenchment program. This Court elaborated thus:
A perusal of Article XIV of the parties 1994-1996 CBA readily shows that retirement benefits
shall be granted only to those employees who, after rendering at least ten (10) years of
continuous services, would retire upon reaching the mandatory retirement age, or would avail of
optional voluntary retirement. Nowhere can it be deduced from the CBA that those employees
whose employment was terminated through one of the authorized causes are entitled to
retirement benefits. In fact, Section 3 of the afore-quoted Article XIV specifically provides that
retrenched employees shall be given two (2) months pay for every year of service. Section 3
shows the intention of the parties to exclude retrenched employees, like herein petitioners, from
receiving retirement benefits under the existing retirement plan as set forth in Section 1.

ATHENA M. SALAS | LABOR CASE DIGEST 2015 171

Similarly, in this case, there is also nothing in the CBA which would indicate that those
employees whose services were terminated by reason of redundancy are entitled to retirement
gratuity. As in Suarez, Sections 1 and 3 of Article XIV of the CBA of the parties herein separately
provide for the amount of benefits to be received by retired employees on the one hand and
those who were terminated due to retrenchment, closure of business, merger, redundancy, or
installation of labor-saving device on the other. In short, Sections 1 and 3 clearly spell out the
difference in the treatment of employees who retired as provided in Section 1 and those who
were constrained to leave the company due to any of the causes enumerated in Section 3. Such
difference in the treatment, as well as in the corresponding pay or gratuity, indicates the parties
intention to exclude retired employees from receiving separation pay and vice versa. A contrary
construction would distort the clear intent of the parties and render useless the classification
specifically spelled out in the CBA.
II.
Respondents are not entitled to the monetary equivalent of their unused sick leave
credits.
The CAs ruling in effect put something into the CBA that is not written in it, contrary to the old
and familiar Latin maxim of expressio unius est exclusio alterius. The express mention of one
person, thing, act, or consequence excludes all others. Put differently, where the terms are
expressly limited to certain matters, it may not, by interpretation or construction, be extended to
other matters. In this case, Article VIII of the CBA covers only (1) an employee who is 60 years
old and due for compulsory retirement; (2) an employee who retires prior to attaining the
compulsory retirement age but has served at least 25 years; and, (3) an employee who retires
before attaining compulsory retirement age due to illness or disability. Necessarily, the
enumeration cannot be extended to include those who will be leaving the company due to
redundancy, death, merger, installation of labor cost-saving device, retrenchment, or closure of
business as mistakenly ruled by the CA.
As the law between the parties, the CBA must be strictly complied with.
III.

The Release and Quitclaim executed by each of the respondents remains valid.

It is true that quitclaims executed by employees are often frowned upon as contrary to public
policy. But that is not to say that all waivers and quitclaims are invalid as against public policy.
Quitclaims will be upheld as valid if the following requisites are present: "(1) the employee
executes a deed of quitclaim voluntarily; (2) there is no fraud or deceit on the part of any of the
parties; (3) the consideration of the quitclaim is credible and reasonable; and, (4) the contract is
not contrary to law, public order, public policy, morals or good customs or prejudicial to a third
person with a right recognized by law."
In this case, there is no showing that Zuellig coerced or forced respondents to sign the Release
and Quitclaim. In fact, there is no allegation that Zuellig employed fraud or deceit in making
respondents sign the Release and Quitclaim. On the other hand, respondents declared that they
had received the separation pay in full settlement of all claims arising from their employment
with Zuellig. For which reason, they have remised, released and discharged Zuellig.
Notably, the Release and Quitclaim represents a reasonable and fair settlement of respondents
claims. Under Article 283 of the Labor Code, the employers are required to pay employees
separated from employment by reason of redundancy at least one (1) month pay or at least one
(1) month pay for every year of service, whichever is higher. Here, respondents received 100%

ATHENA M. SALAS | LABOR CASE DIGEST 2015 172

of their one (1) month basic pay for every year of service, plus a premium ranging from 20% to
85% of such basic pay for every year of service (depending on the number of years in service),
as separation pay. In Goodrich Manufacturing Corporation, v. Ativo, this Court declared that
It is only where there is clear proof that the waiver was wangled from an unsuspecting or gullible
person, or the terms of settlement are unconscionable on its face, that the law will step in to
annul the questionable transaction. But where it is shown that the person making the waiver did
so voluntarily, with full understanding of what he was doing, and the consideration for the
quitclaim is credible and reasonable, the transaction must be recognized as a valid and binding
undertaking.
79. Zuellig Freight & Cargo System vs. NLRC, G.R. No. 157900, July 22, 2013
The mere change in the corporate name is not considered under the law as the creation of a
new corporation; hence, the renamed corporation remains liable for the illegal dismissal of its
employee separated under that guise.
Facts:
San Miguel was an employee of Zeta Brokerage Corporation from 1985 until dismissed on
1994. On 1994, Zeta Brokerage made amendments of the articles of incorporation of to change
its corporate name, broadening the primary functions, and increasing the capital stock. Zeta
Brokerage changed its name to Zuellig Freight & Cargo System. With the change of its
corporate name, Zuellig contended that Zeta Brokerage now ceased to exist, and that there is
now cessation or closure of establishment, an authorized cause for dismissal of employees, one
of them was San Miguel.
San Miguel brought a complaint for unfair labor practice, illegal dismissal, non-payment of
salaries and moral damages against petitioner, formerly known as Zeta Brokerage Corporation
(Zeta).
On the other hand, petitioner countered that San Miguel's termination from Zeta had been for a
cause authorized by the Labor Code (Article 283 - Closure of establishment and reduction of
personnel)
The Labor Arbiter, NLRC, and Court of Appeals all ruled in favor of San Miguel, ruling that there
was merely a change of business name and primary purpose and upgrading of stocks of the
corporation. Zuellig and Zeta are therefore legally the same person and entity.
Issue:
Whether or not a change in corporate name leads to closure of establishment, and
consequently, to valid dismissal of employees.
Ruling:
It is worthy to point out that the Labor Arbiter, the NLRC, and the CA were united in concluding
that the cessation of business by Zeta was not a bona fide closure to be regarded as a valid
ground for the termination of employment of San Miguel within the ambit of Article 283 of the
Labor Code. The provision pertinently reads:

ATHENA M. SALAS | LABOR CASE DIGEST 2015 173

Article 283.Closure of establishment and reduction of personnel. The employer may also
terminate the employment of any employee due to the installation of labor-saving devices,
redundancy, retrenchment to prevent losses or the closing or cessation of operation of the
establishment or undertaking unless the closing is for the purpose of circumventing the
provisions of this Title, by serving a written notice on the workers and the Department of Labor
and Employment at least one (1) month before the intended date thereof.
Verily, the amendments of the articles of incorporation of Zeta to change the corporate name to
Zuellig Freight and Cargo Systems, Inc. did not produce the dissolution of the former as a
corporation. For sure, the Corporation Code defined and delineated the different modes of
dissolving a corporation, and amendment of the articles of incorporation was not one of such
modes. The effect of the change of name was not a change of the corporate being. "The
changing of the name of a corporation is no more the creation of a corporation than the
changing of the name of a natural person is begetting of a natural person. The act, in both
cases, would seem to be what the language which we use to designate it imports a change
of name, and not a change of being."
A change in the corporate name does not make a new corporation, whether effected by a
special act or under a general law. It has no effect on the identity of the corporation, or on its
property, rights, or liabilities. The corporation, upon such change in its name, is in no sense a
new corporation, nor the successor of the original corporation. It is the same corporation with a
different name, and its character is in no respect changed.
In short, Zeta and petitioner remained one and the same corporation. The change of name did
not give petitioner the license to terminate employees of Zeta like San Miguel without just or
authorized cause. The situation was not similar to that of an enterprise buying the business of
another company where the purchasing company had no obligation to rehire terminated
employees of the latter. 18 Petitioner, despite its new name, was the mere continuation of Zeta's
corporate being, and still held the obligation to honor all of Zeta's obligations, one of which was
to respect San Miguel's security of tenure. The dismissal of San Miguel from employment on the
pretext that petitioner, being a different corporation, had no obligation to accept him as its
employee, was illegal and ineffectual.

80. Abbott Laboratrories Phils et al., Vs. Alcaraz, G.R. No. 192571, July 23, 2013 En banc
Facts:
Alcaraz was hired by Abbott Laboratories as Medical and Regulatory Affairs Manager
(Regulatory Affairs Manager) on a probationary basis. During Alcaraz's pre-employment
orientation, petitioner Allan G. Almazar (Almazar), Hospira's Country Transition Manager, briefed
her on her duties and responsibilities as Regulatory Affairs Manager, stating that: (a) she will
handle the staff of Hospira ALSU and will directly report to Almazar on matters regarding
Hopira's local operations, operational budget, and performance evaluation of the Hospira ALSU
Staff who are on probationary status; (b) she must implement Abbott's Code of Good Corporate
Conduct (Code of Conduct), office policies on human resources and finance, and ensure that
Abbott will hire people who are fit in the organizational discipline; (c) petitioner Kelly Walsh

ATHENA M. SALAS | LABOR CASE DIGEST 2015 174

(Walsh), Manager of the Literature Drug Surveillance Drug Safety of Hospira, will be her
immediate supervisor; (d) she should always coordinate with Abbott's human resource officers
in the management and discipline of the staff; (e) Hospira ALSU will spin off from Abbott in early
2006 and will be officially incorporated and known as Hospira, Philippines. In the interim,
Hospira ALSU operations will still be under Abbott's management, excluding the technical
aspects of the operations which is under the control and supervision of Walsh; and (f) the
processing of information and/or raw material data subject of Hospira ALSU operations will be
strictly confined and controlled under the computer system and network being maintained and
operated from the United States. For this purpose, all those involved in Hospira ALSU are
required to use two identification cards: one, to identify them as Abbott's employees and
another, to identify them as Hospira employees.
On May 23, 2005, Walsh, Almazar, and Bernardo personally handed to Alcaraz a letter stating
that her services had been terminated effective May 19, 2005. The letter detailed the reasons
for Alcaraz's termination particularly, that Alcaraz: (a) did not manage her time effectively; (b)
failed to gain the trust of her staff and to build an effective rapport with them; (c) failed to train
her staff effectively; and (d) was not able to obtain the knowledge and ability to make sound
judgments on case processing and article review which were necessary for the proper
performance of her duties. On May 27, 2005, Alcaraz received another copy of the said
termination letter via registered mail.
Alcaraz felt that she was unjustly terminated from her employment and thus, filed a complaint
for illegal dismissal and damages against Abbott and its officers, namely, Misa, Bernardo,
Almazar, Walsh, Terrible, and Feist. She claimed that she should have already been considered
as a regular and not a probationary employee given Abbott's failure to inform her of the
reasonable standards for her regularization upon her engagement as required under Article 295
of the Labor Code. In this relation, she contended that while her employment contract stated
that she was to be engaged on a probationary status, the same did not indicate the standards
on which her regularization would be based.
On the contrary, petitioners maintained that Alcaraz was validly terminated from her
probationary employment given her failure to satisfy the prescribed standards for her
regularization which were made known to her at the time of her engagement.
Issues:
Whether or not Alcaraz was illegally dismissed.
Whether or not damages may be awarded.
Ruling:
A probationary employee, like a regular employee, enjoys security of tenure. However, in cases
of probationary employment, aside from just or authorized causes of termination, an additional
ground is provided under Article 295 of the Labor Code, i.e., the probationary employee may
also be terminated for failure to qualify as a regular employee in accordance with the
reasonable standards made known by the employer to the employee at the time of the
engagement. Thus, the services of an employee who has been engaged on probationary basis
may be terminated for any of the following: (a) a just or (b) an authorized cause; and (c) when

ATHENA M. SALAS | LABOR CASE DIGEST 2015 175

he fails to qualify as a regular employee in accordance with reasonable standards prescribed by


the employer.
Corollary thereto, Section 6 (d), Rule I, Book VI of the Implementing Rules of the Labor Code
provides that if the employer fails to inform the probationary employee of the reasonable
standards upon which the regularization would be based on at the time of the engagement, then
the said employee shall be deemed a regular employee, viz.:
(d)In all cases of probationary employment, the employer shall make known to the employee the
standards under which he will qualify as a regular employee at the time of his engagement.
Where no standards are made known to the employee at that time, he shall be deemed a
regular employee.
In other words, the employer is made to comply with two (2) requirements when dealing with a
probationary employee: first, the employer must communicate the regularization standards to
the probationary employee; and second, the employer must make such communication at the
time of the probationary employee's engagement. If the employer fails to comply with either, the
employee is deemed as a regular and not a probationary employee.
Keeping with these rules, an employer is deemed to have made known the standards that would
qualify a probationary employee to be a regular employee when it has exerted reasonable
efforts to apprise the employee of what he is expected to do or accomplish during the trial
period of probation. This goes without saying that the employee is sufficiently made aware of his
probationary status as well as the length of time of the probation.
The exception to the foregoing is when the job is self-descriptive in nature, for instance, in the
case of maids, cooks, drivers, or messengers. Also, in Aberdeen Court, Inc. v. Agustin, it has
been held that the rule on notifying a probationary employee of the standards of regularization
should not be used to exculpate an employee who acts in a manner contrary to basic
knowledge and common sense in regard to which there is no need to spell out a policy or
standard to be met. In the same light, an employee's failure to perform the duties and
responsibilities which have been clearly made known to him constitutes a justifiable basis for a
probationary employee's non-regularization.
In this case, petitioners contend that Alcaraz was terminated because she failed to qualify as a
regular employee according to Abbott's standards which were made known to her at the time of
her engagement. Contrarily, Alcaraz claims that Abbott never apprised her of these standards
and thus, maintains that she is a regular and not a mere probationary employee.
The Court finds petitioners' assertions to be well-taken.
A punctilious examination of the records reveals that Abbott had indeed complied with the
above-stated requirements. This conclusion is largely impelled by the fact that Abbott clearly
conveyed to Alcaraz her duties and responsibilities as Regulatory Affairs Manager prior to,
during the time of her engagement, and the incipient stages of her employment.
Verily, basic knowledge and common sense dictate that the adequate performance of one's
duties is, by and of itself, an inherent and implied standard for a probationary employee to be
regularized; such is a regularization standard which need not be literally spelled out or mapped
into technical indicators in every case. In this regard, it must be observed that the assessment
of adequate duty performance is in the nature of a management prerogative which when

ATHENA M. SALAS | LABOR CASE DIGEST 2015 176

reasonably exercised as Abbott did in this case should be respected. This is especially
true of a managerial employee like Alcaraz who was tasked with the vital responsibility of
handling the personnel and important matters of her department.
In fine, the Court rules that Alcaraz's status as a probationary employee and her consequent
dismissal must stand.
A different procedure is applied when terminating a probationary employee; the usual two-notice
rule does not govern. 65 Section 2, Rule I, Book VI of the Implementing Rules of the Labor
Code states that "if the termination is brought about by the . . . failure of an employee to meet
the standards of the employer in case of probationary employment, it shall be sufficient that a
written notice is served the employee, within a reasonable time from the effective date of
termination."
As the records show, Alcaraz's dismissal was effected through a letter dated May 19, 2005
which she received on May 23, 2005 and again on May 27, 2005. Stated therein were the
reasons for her termination, i.e., that after proper evaluation, Abbott determined that she failed
to meet the reasonable standards for her regularization considering her lack of time and people
management and decision-making skills, which are necessary in the performance of her
functions as Regulatory Affairs Manager. Undeniably, this written notice sufficiently meets the
criteria set forth above, thereby legitimizing the cause and manner of Alcaraz's dismissal as a
probationary employee under the parameters set by the Labor Code.
Nonetheless, despite the existence of a sufficient ground to terminate Alcaraz's employment and
Abbott's compliance with the Labor Code termination procedure, it is readily apparent that
Abbott breached its contractual obligation to Alcaraz when it failed to abide by its own procedure
in evaluating the performance of a probationary employee.
Veritably, a company policy partakes of the nature of an implied contract between the employer
and employee.
Once an employer establishes an express personnel policy and the employee continues to work
while the policy remains in effect, the policy is deemed an implied contract for so long as it
remains in effect. If the employer unilaterally changes the policy, the terms of the implied
contract are also thereby changed.
Records show that Abbott's PPSE procedure mandates, inter alia, that the job performance of a
probationary employee should be formally reviewed and discussed with the employee at least
twice: first on the third month and second on the fifth month from the date of employment.
Abbott is also required to come up with a Performance Improvement Plan during the third month
review to bridge the gap between the employee's performance and the standards set, if any. In
addition, a signed copy of the PPSE form should be submitted to Abbott's HRD as the same
would serve as basis for recommending the confirmation or termination of the probationary
employment.
In this case, it is apparent that Abbott failed to follow the above-stated procedure in evaluating
Alcaraz. For one, there lies a hiatus of evidence that a signed copy of Alcaraz's PPSE form was
submitted to the HRD. It was not even shown that a PPSE form was completed to formally
assess her performance. Neither was the performance evaluation discussed with her during the
third and fifth months of her employment. Nor did Abbott come up with the necessary

ATHENA M. SALAS | LABOR CASE DIGEST 2015 177

Performance Improvement Plan to properly gauge Alcaraz's performance with the set company
standards.
While it is Abbott's management prerogative to promulgate its own company rules and even
subsequently amend them, this right equally demands that when it does create its own policies
and thereafter notify its employee of the same, it accords upon itself the obligation to faithfully
implement them. Indeed, a contrary interpretation would entail a disharmonious relationship in
the work place for the laborer should never be mired by the uncertainty of flimsy rules in which
the latter's labor rights and duties would, to some extent, depend.
In this light, while there lies due cause to terminate Alcaraz's probationary employment for her
failure to meet the standards required for her regularization, and while it must be further pointed
out that Abbott had satisfied its statutory duty to serve a written notice of termination, the fact
that it violated its own company procedure renders the termination of Alcaraz's employment
procedurally infirm, warranting the payment of nominal damages.
It was explained that if the dismissal is based on a just cause under Article 282 of the Labor
Code (now Article 296) but the employer failed to comply with the notice requirement, the
sanction to be imposed upon him should be tempered because the dismissal process was, in
effect, initiated by an act imputable to the employee; if the dismissal is based on an authorized
cause under Article 283 (now Article 297) but the employer failed to comply with the notice
requirement, the sanction should bestiffer because the dismissal process was initiated by the
employer's exercise of his management prerogative. Hence, in Jaka, where the employee was
dismissed for an authorized cause of retrenchment as contradistinguished from the employee
in Agabon who was dismissed for a just cause of neglect of duty the Court ordered the
employer to pay the employee nominal damages at the higher amount of P50,000.00.
Evidently, the sanctions imposed in both Agabon and Jaka proceed from the necessity to deter
employers from future violations of the statutory due process rights of employees. In similar
regard, the Court deems it proper to apply the same principle to the case at bar for the reason
that an employer's contractual breach of its own company procedure albeit not statutory in
source has the parallel effect of violating the laborer's rights. Suffice it to state, the contract is
the law between the parties and thus, breaches of the same impel recompense to vindicate a
right that has been violated. Consequently, while the Court is wont to uphold the dismissal of
Alcaraz because a valid cause exists, the payment of nominal damages on account of Abbott's
contractual breach is warranted in accordance with Article 2221 of the Civil Code.
Anent the proper amount of damages to be awarded, the Court observes that Alcaraz's
dismissal proceeded from her failure to comply with the standards required for her
regularization. As such, it is undeniable that the dismissal process was, in effect, initiated by an
act imputable to the employee, akin to dismissals due to just causes under Article 296 of the
Labor Code. Therefore, the Court deems it appropriate to fix the amount of nominal damages at
the amount of P30,000.00, consistent with its rulings in both Agabon and Jaka.

81. Manila Polo Club Employees Union vs. Manila Polo Club, G.R. No. 172846, July 24,
2013
Principle: Standards for a valid retrenchment and closure or cessation of business operations.

ATHENA M. SALAS | LABOR CASE DIGEST 2015 178

Facts:
Petitioner Manila Polo Club Employees Union (MPCEU), which is affiliated with the Federation
of Unions of Rizal (FUR)-TUCP, is a legitimate labor organization duly registered with DOLE,
while respondent Manila Polo Club, Inc. is a non-profit and proprietary membership organization
which provides recreation and sports facilities to its proprietary members, their dependents, and
guests.
The Board of Directors of respondent unanimously resolved to completely terminate the entire
operations of its Food and Beverage (F & B) outlets, except the Last Chukker, and award its
operations to a qualified restaurant operator or caterer. Cited as reasons were as follows:
WHEREAS, the Food and Beverage (F & B) operations has resulted in yearly losses to the Club
in six (6) out of the last eight (8) years with FY 2001 suffering the largest loss at P10,647,981
and that this loss is due mainly to the exceedingly high manpower cost and other management
inefficiencies;
WHEREAS, due to the substantial losses incurred by the Club in both F&B operations and in its
recurring operations, the Board and management had instituted cost and loss-cutting measures;
WHEREAS, the Board recognized the non-viability of the operations of the Food and Beverage
Department and that its continued operations by the Club will result in substantial losses that will
seriously impair the Club's financial health and membership satisfaction;
WHEREAS, the Board recognized the urgent need to act and act decisively and eliminate
factors contributing to substantial losses in the operations of the Club, more particularly the food
and beverage operations. Thus, F & B operations are to cease wholly and totally, subject to
observance and requirements of the law and other rules.
Subsequently, respondent's Board approved the implementation of the retrenchment program of
employees who are directly and indirectly involved with the operations of the F & B outlets.
Issue:
Whether or not the retrenchment of the 117 union members is legal.
Ruling:
It is apparent from the records that this case involves a closure of business undertaking, not
retrenchment. The legal requirements and consequences of these two authorized causes in the
termination of employment are discernible.
We distinguished, in Alabang Country Club, Inc. v. NLRC:
Retrenchment is the reduction of personnel for the purpose of cutting down on costs of
operations in terms of salaries and wages resorted to by an employer because of losses in
operation of a business occasioned by lack of work and considerable reduction in the volume of
business.

Closure of a business or undertaking due to business losses is the reversal of fortune of the
employer whereby there is a complete cessation of business operations to prevent further

ATHENA M. SALAS | LABOR CASE DIGEST 2015 179

financial drain upon an employer who cannot pay anymore his employees since business has
already stopped.
One of the prerogatives of management is the decision to close the entire establishment or to
close or abolish a department or section thereof for economic reasons, such as to minimize
expenses and reduce capitalization.
While the Labor Code provides for the payment of separation package in case of retrenchment
to prevent losses, it does not obligate the employer for the payment thereof if there is closure of
business due to serious losses.
Likewise, the case of Eastridge Golf Club, Inc. v. Eastridge Golf Club, Inc., Labor-Union, Super
stressed the differences:
Retrenchment or lay-off is the termination of employment initiated by the employer, through no
fault of the employees and without prejudice to the latter, during periods of business recession,
industrial depression, or seasonal fluctuations, or during lulls occasioned by lack of orders,
shortage of materials, conversion of the plant for a new production program or the introduction
of new methods or more efficient machinery, or of automation. It is an exercise of management
prerogative which the Court upholds if compliant with certain substantive and procedural
requirements, namely:
1. That retrenchment is necessary to prevent losses and it is proven, by sufficient and
convincing evidence such as the employer's financial statements audited by an independent
and credible external auditor, that such losses are substantial and not merely flimsy and actual
or reasonably imminent; and that retrenchment is the only effective measure to prevent such
imminent losses;
2. That written notice is served on to the employees and the DOLE at least one (1) month prior
to the intended date of retrenchment; and
3. That the retrenched employees receive separation pay equivalent to one (1) month pay or at
least one-half (1/2) month pay for every year of service, whichever is higher.
The employer must prove compliance with all the foregoing requirements. Failure to prove the
first requirement will render the retrenchment illegal and make the employer liable for the
reinstatement of its employees and payment of full backwages. However, were the
retrenchment undertaken by the employer isbona fide, the same will not be invalidated by the
latter's failure to serve prior notice on the employees and the DOLE; the employer will only be
liable in nominal damages, the reasonable rate of which the Court En Banc has set at
P50,000.00 for each employee.
Closure or cessation of business is the complete or partial cessation of the operations and/or
shut-down of the establishment of the employer. It is carried out to either stave off the financial
ruin or promote the business interest of the employer.
Unlike retrenchment, closure or cessation of business, as an authorized cause of termination of
employment, need not depend for validity on evidence of actual or imminent reversal of the
employer's fortune. Article 283 authorizes termination of employment due to business closure,
regardless of the underlying reasons and motivations therefor, be it financial losses or not.

ATHENA M. SALAS | LABOR CASE DIGEST 2015 180

To be precise, closure or cessation of an employer's business operations, whether in whole or in


part, is governed by Article 283 of the Labor Code, as amended. It states:
Article 283.Closure of establishment and reduction of personnel. The employer may also
terminate the employment of any employee due to the installation of labor-saving devices,
redundancy, retrenchment to prevent losses or the closing or cessation of operation of the
establishment or undertaking unless the closing is for the purpose of circumventing the
provisions of this Title, by serving a written notice on the workers and the Ministry of Labor and
Employment at least one (1) month before the intended date thereof. In case of termination due
to the installation of labor-saving devices or redundancy, the worker affected thereby shall be
entitled to a separation pay equivalent to at least his one (1) month pay or to at least one (1)
month pay for every year of service, whichever is higher. In case of retrenchment to prevent
losses and in cases of closures or cessation of operations of establishment or undertaking not
due to serious business losses or financial reverses, the separation pay shall be equivalent to
one (1) month pay or at least one-half (1/2) month pay for every year of service, whichever is
higher. A fraction of at least six (6) months shall be considered one (1) whole year.
In Industrial Timber Corporation v. Ababon, 27the Court explained the above-quoted provision in
this wise:
A reading of the foregoing law shows that a partial or total closure or cessation of operations of
establishment or undertaking may either be due to serious business losses or financial reverses
or otherwise. Under the first kind, the employer must sufficiently and convincingly prove its
allegation of substantial losses, while under the second kind, the employer can lawfully close
shop anytime as long as cessation of or withdrawal from business operations was bona fide in
character and not impelled by a motive to defeat or circumvent the tenurial rights of employees,
and as long as he pays his employees their termination pay in the amount corresponding to
their length of service. Just as no law forces anyone to go into business, no law can compel
anybody to continue the same. It would be stretching the intent and spirit of the law if a court
interferes with management's prerogative to close or cease its business operations just because
the business is not suffering from any loss or because of the desire to provide the workers
continued employment.
In sum, under Article 283 of the Labor Code, three requirements are necessary for a valid
cessation of business operations: (a) service of a written notice to the employees and to the
DOLE at least one month before the intended date thereof; (b) the cessation of business must
be bona fide in character; and (c) payment to the employees of termination pay amounting to
one month pay or at least one-half month pay for every year of service, whichever is higher.
Based on the above and cases of similar import, We summarize:
1. Closure or cessation of operations of establishment or undertaking may either be partial or
total.
2. Closure or cessation of operations of establishment or undertaking may or may not be due to
serious business losses or financial reverses. However, in both instances, proof must be shown
that: (1) it was done in good faith to advance the employer's interest and not for the purpose of
defeating or circumventing the rights of employees under the law or a valid agreement; and (2)
a written notice on the affected employees and the DOLE is served at least one month before
the intended date of termination of employment.

ATHENA M. SALAS | LABOR CASE DIGEST 2015 181

3. The employer can lawfully close shop even if not due to serious business losses or financial
reverses but separation pay, which is equivalent to at least one month pay as provided for by
Article 283 of the Labor Code, as amended, must be given to all the affected employees.
4. If the closure or cessation of operations of establishment or undertaking is due to serious
business losses or financial reverses, the employer must prove such allegation in order to avoid
the payment of separation pay. Otherwise, the affected employees are entitled to separation
pay.
5. The burden of proving compliance with all the above-stated falls upon the employer.
Guided by the foregoing, the Court shall refuse to dwell on the issue of whether respondent was
in sound financial condition when it resolved to stop the operations of its F & B Department. As
stated, an employer can lawfully close shop anytime even if not due to serious business losses
or financial reverses.
The closure of the F & B Department was due to legitimate business considerations, a
resolution which the Court has no business interfering with. We have already resolved that the
characterization of the employee's service as no longer necessary or sustainable, and therefore,
properly terminable, is an exercise of business judgment on the part of the employer; the
determination of the continuing necessity of a particular officer or position in a business
corporation is a management prerogative, and the courts will not interfere with the exercise of
such so long as no abuse of discretion or arbitrary or malicious action on the part of the
employer is shown.
As recognized by both the VA and the CA, evident proofs of respondent's good faith to arrest the
losses which the F & B Department had been incurring since 1994 are: engagement of an
independent consulting firm to conduct manpower audit/organizational development; institution
of cost-saving programs, termination of the services of probationary employees, substantial
reduction of a number of agency staff and personnel, and the retrenchment of eight (8)
managers. After the effective date of the termination of employment relation, respondent even
went on to aid the displaced employees in finding gainful employment by soliciting the
assistance of respondent's members, Makati Skyline, Human Resource Managers of some
companies, and the Association of Human Resource Managers. These were not refuted by
petitioner. Only that, it perceives them as inadequate and insists that the operational losses are
very well covered by the other income of respondent and that less drastic measures could have
been resorted to, like increasing the membership dues and the prices of food and beverage. Yet
the wisdom or soundness of the Management decision is not subject to discretionary review of
the Court for, even the VA admitted, it enjoys a pre-eminent role and is presumed to possess all
relevant and necessary information to guide its business decisions and actions.
Further, unlike in the case of Eastridge Golf Club, Inc., there is nothing on record to indicate that
the closure of respondent's F & B Department was made in bad faith. It was not motivated by
any specific and clearly determinable union activity of the employees; rather, it was truly dictated
by economic necessity. Despite petitioner's allegations, no convincing and credible proofs were
presented to establish the claim that such closure qualifies as an act of union-busting and ULP.
No evidence was shown that the closure is stirred not by a desire to avoid further losses but to
discourage the workers from organizing themselves into a union for more effective negotiations
with the management. Allegations are not proofs and it is incumbent upon petitioner to
substantiate the same. On the contrary, respondent continued to negotiate with petitioner even

ATHENA M. SALAS | LABOR CASE DIGEST 2015 182

after April 30, 2002. In fact, a Memorandum of Agreement was executed before the NCMB
between petitioner and respondent on June 10, 2002 whereby the parties agreed, among
others, to maintain the existing provisions of the CBA, except those pertaining to wage
increases and signing bonus.
Finally, even if the members of petitioner are not considered as illegally dismissed, they are
entitled to separation pay pursuant to Article 283 of the Labor Code, as amended. Per
respondent's information, however, the separation packages of all 117 union members were
already paid during the pendency of the case. Petitioner did not oppose this representation;
hence, We shall treat the fact of receipt of separation pay as having been voluntarily entered
into, with a full understanding of its import, and the amount received as credible and reasonable
settlement that should be respected by the Court as the law between the parties are valid and
binding between them.

82. Canedo vs. Kampilan Security & Detective Agency Inc. et al., G.R. No. 179326, July 31,
2013
Facts:
Respondent agency hired petitioner as security guard and assigned him at the Naga Power
Barge 102 of the National Power Corporation (NPC) at Lutopan, Toledo City.
For not wearing proper uniform while on duty as per report of Allan Alfafara (Alfafara) of the
NPC, petitioner was suspended for a month.
In a letter, NPC informed respondent agency that it was no longer interested in petitioners
services and thus requested for his replacement.
Subsequently, etitioner requested respondent Arquiza to issue a certification in connection with
his intended retirement effective that month. Thus, respondent Arquiza issued the Certification
dated June 25, 2003 (June 25, 2003 Certification):
C E R T I F I C AT I O N
TO WHOM IT MAY CONCERN:
This is to certify that Mr. Luciano ParagosoCaedo whose address is at Lower Bunga, Toledo
City was employed by this agency from November 20, 1996 up to May 7, 2003 as Security
Guard assigned at NPC, Sigpit Substation. He was terminated from his employment by this
agency on May 7, 2003 as per clients request.
Done this 25th day of June 2003 at Cebu City, Philippines.
(Signed)RAMONCITO L. ARQUIZAGeneral ManagerKSDAI
Five days later, petitioner filed before the Labor Arbiter a Complaint for illegal dismissal, illegal
suspension and non-payment of monetary benefits against respondents.
Proceedings before the Labor Arbiter

ATHENA M. SALAS | LABOR CASE DIGEST 2015 183

The Labor Arbiter held that petitioner was illegally dismissed from the service. He also found
petitioners prior suspension illegal and granted him all his monetary claims except for
underpayment of wages
Proceedings before the National Labor Relations Commission
The NLRC opined that petitioners intention to retire as shown by his June 17, 2003 letter
negated his claim of termination. Nevertheless, it maintained that petitioner was suspended
without being notified of his infraction. Thus, he should be paid his salary during the period of
his illegal suspension.
Ruling of the Court of Appeals
The CA denied the Petition after it found no grave abuse of discretion on the part of the NLRC.
In view of the above, the CA concluded that petitioner was merely placed on temporary "offdetail" which is not equivalent to dismissal. However, like the NLRC, the CA found that petitioner
was deprived of due process when he was suspended and thus affirmed his entitlement to his
salary during the period of suspension. It also affirmed the awards for holiday pay and service
incentive leave pay as well as the deduction therefrom of P10,000.00 representing petitioners
cash advance.
Issue:
Whether or not petitioner was dismissed from service.
Ruling:
Petitioner relies on the word "terminated" as used in the June 25, 2003 Certification issued him
by respondent Arquiza and argues that the same is a clear indication that he was dismissed
from service. SC held that petitioner cannot simply rely on this piece of document since the fact
of dismissal must be evidenced by positive and overt acts of an employer indicating an intention
to dismiss.Here, aside from this single document, petitioner proffered no other evidence
showing that he was dismissed from employment. While it is true that he was not allowed to
report for work after the period of his suspension expired, the same was due to NPCs request
for his replacement as NPC was no longer interested in his services. And as correctly argued by
respondents, petitioner from that point onward is not considered dismissed but merely on a
floating status. "Such a floating status is lawful and not unusual for security guards employed in
security agencies as their assignments primarily depend on the contracts entered into by the
agency with third parties."
A floating status can ripen into constructive dismissal only when it goes beyond the six-month
maximum period allowed by law. In this case, petitioner filed the Complaint for illegal dismissal
even before the lapse of the six-month period. Hence, his claim of illegal dismissal lacks basis.
Moreover and as aptly observed by the NLRC, it was in fact petitioner who intended to terminate
his relationship with respondents through his planned retirement. This is further bolstered by his
prayer in his Complaint where he sought for separation pay and not for reinstatement.

83. Ang vs. San Joaquin Jr et al., G.R. No. 185549, Aug. 7, 2013

ATHENA M. SALAS | LABOR CASE DIGEST 2015 184

PRINCIPLE: The employers act of tearing to pieces the employees time card may be
considered an outright not only symbolic termination of the parties employment relationship.
Facts:
Petitioner Ang is the proprietor of Virose Furniture and Glass Supply. Respondents San Joaquin
and Fernandez were regular employees of Virose: San Joaquin as helper, while Fernandez as
driver. Respondents have been continuously in Angs employ without any derogatory record.
Through the years, San Joaquin became a pahinante or delivery helper, and later on an allaround worker of Virose.
On 1999, respondents attended the court hearing relative to the 41 criminal cases filed by
former Virose employee Abrera against Ang for the latters non-remittance of Social Security
System (SSS) contributions. During that hearing, respondents testified against Ang. Previously,
respondents joined Abrera in questioning Angs procedure in remitting their SSS contributions.
After the said hearing Ang began to treat respondents with hostility and antagonism.
On August 28, 1999, Angs wife, Rosa, instructed a Virose salesclerk to find helpers who would
transfer monobloc chairs from the Virose store to her restaurant, Leng-Lengs Foodshop,
located just beside the store. The salesclerk instructed San Joaquin to help, but the latter
refused, saying that he was not an employee of the restaurant but a glass installer of Virose. A
heated argument ensued between San Joaquin on the one hand and Rosa, her son Jonathan,
and the salesclerk on the other. San Joaquin left the store, shouting invectives.
On August 30, 1999, San Joaquin returned to the store, only to find out that Ang had torn his
DTR to pieces that day while the DTR of Fernandez was torn to pieces by Ang immediately after
the hearing in which the respondents testified. On the same day, Fernandez reported for work
and received a memorandum of even date issued by Ang informing him that he was placed on a
one-week suspension for insubordination. The memorandum did not specify the act of
insubordination.
Respondents then filed a case for constructive dismissal in the Labor Arbiter and the latter
decided against them. The former then appealed to the NLRC but it just affirmed the decision of
the Labor Arbiter. Respondent then filed an original Petition for Certiorari to the CA which
reversed the rulings of the LA and the NLRC.
Issue:
Whether or not employers act of tearing to pieces the employees time card may be considered
constructive dismissal or termination of the parties employment relationship.
Ruling:
There is constructive dismissal in this case. "Constructive dismissal exists where there is
cessation of work because continued employment is rendered impossible, unreasonable or
unlikely, as an offer involving a demotion in rank and a diminution in pay." It is a "dismissal in
disguise or an act amounting to dismissal but made to appear as if it were not." Constructive
dismissal may likewise exist if an "act of clear discrimination, insensibility, or disdain by an
employer becomes so unbearable on the part of the employee that it could foreclose any choice
by him except to forego his continued employment." "Constructive dismissal exists when the
employee involuntarily resigns due to the harsh, hostile, and unfavorable conditions set by the

ATHENA M. SALAS | LABOR CASE DIGEST 2015 185

employer." "The test of constructive dismissal is whether a reasonable person in the employees
position would have felt compelled to give up his position under the circumstances."
The CA is correct in its pronouncement that respondents were constructively dismissed from
work.1wphi1 Moreover, by destroying respondents time cards, Ang discontinued and severed
his relationship with respondents. The purpose of a time record is to show an employees
attendance in office for work and to be paid accordingly, taking into account the policy of "no
work, no pay". A daily time record is primarily intended to prevent damage or loss to the
employer, which could result in instances where it pays an employee for no work done; it is a
mandatory requirement for inclusion in the payroll, and in the absence of an employment
agreement, it constitutes evidence of employment. Thus, when Ang tore the respondents time
cards to pieces, he virtually removed them from Viroses payroll and erased all vestiges of
respondents employment; respondents were effectively dismissed from work. The act may be
considered an outright not only symbolic termination of the parties employment relationship.

84. Sanoh Fulton Phils Inc. et al., vs. Bernardo et al., G.R. No. 187214, Aug. 14, 2013
PRINCIPLE: That losses as a valid ground for termination must be supported by sufficient and
convincing evidence.
Facts:
Sanoh is a domestic corporation engaged in the manufacture of automotive parts and wire
condensers for home appliances. Its Wire Condenser Department employed 61 employees.
Respondents belonged to this department.
In view of job order cancellations relating to the manufacture of wire condensers by Matsushita,
Sanyo and National Panasonic, Sanoh decided to phase out the Wire Condenser Department.
On 22 December 2003, the Human Resources Manager of Sanoh informed the 17 employees,
16 of whom belonged to the Wire Condenser Department, of retrenchment effective 22 January
2004. All 17 employees are union members.
A grievance conference was held where the affected employees were informed of the following
grounds for retrenchment:
1) Lack of local market.
2) Competition from imported products.
3) Phasing out of Wire Condenser Department.
Two succeeding conciliation conferences were likewise held but the parties failed to reach an
amicable settlement. Thus, two (2) separate complaints for illegal dismissal.
The Labor Arbiter dismissed the complaint while the NLRC affirmed the formers decision in
toto.
Respondents appealed to the CA which decided in their favor. The petitioners the appealed to
the Supreme Court.
Issues:

ATHENA M. SALAS | LABOR CASE DIGEST 2015 186

Whether or not there was illegal dismissal in this case.


Ruling:
There was illegal dismissal.
Retrenchment to prevent losses and closure not due to serious business losses are two
separate authorized causes for terminating the services of an employee. In J.A.T. General
Services v. NLRC, the Court took the occasion to draw the distinction between retrenchment
and closure, to wit:
Closure of business, on one hand, is the reversal of fortune of the employer whereby there is a
complete cessation of business operations and/or an actual locking-up of the doors of
establishment, usually due to financial losses. Closure of business as an authorized cause for
termination of employment aims to prevent further financial drain upon an employer who cannot
pay anymore his employees since business has already stopped. On the other hand,
retrenchment is reduction of personnel usually due to poor financial returns so as to cut down
on costs of operations in terms of salaries and wages to prevent bankruptcy of the company. It
is sometimes also referred to as down-sizing. Retrenchment is an authorized cause for
termination of employment which the law accords an employer who is not making good in its
operations in order to cut back on expenses for salaries and wages by laying off some
employees. The purpose of retrenchment is to save a financially ailing business establishment
from eventually collapsing.
The respective requirements to sustain their validity are likewise different.
For retrenchment, the three (3) basic requirements are:
(a) proof that the retrenchment is necessary to prevent losses or impending losses;
(b) service of written notices to the employees and to the Department of Labor and Employment
at least one (1) month prior to the intended date of retrenchment; and
(c) payment of separation pay equivalent to one (1) month pay, or at least one-half (1/2) month
pay for every year of service, whichever is higher.
In addition, jurisprudence has set the standards for losses which may justify retrenchment, thus:
(1) the losses incurred are substantial and not de minimis;
(2) the losses are actual or reasonably imminent;
(3) the retrenchment is reasonably necessary and is likely to be effective in preventing the
expected losses; and
(4) the alleged losses, if already incurred, or the expected imminent losses sought to be
forestalled, are proven by sufficient and convincing evidence.
Upon the other hand, in termination, the law authorizes termination of employment due to
business closure, regardless of the underlying reasons and motivations therefor, be it financial
losses or not. However, to put a stamp to its validity, the closure/cessation of business must be
bona fide, i.e., its purpose is to advance the interest of the employer and not to defeat or
circumvent the rights of employees under the law or a valid agreement.

ATHENA M. SALAS | LABOR CASE DIGEST 2015 187

In termination cases either by retrenchment or closure, the burden of proving that the
termination of services is for a valid or authorized cause rests upon the employer. Not every loss
incurred or expected to be incurred by an employer can justify retrenchment. The employer
must prove, among others, that the losses are substantial and that the retrenchment is
reasonably necessary to avert such losses. And to repeat, in closures, the bona fides of the
employer must be proven.
In this case, there was no valid retrenchment. Nor was there a closure of business.
We are mindful of the principle that losses in the operation of the enterprise, lack of work, or
considerable reduction on the volume of business may justify an employer to reduce the work
force. But a lull caused by lack of orders or shortage of materials must be of such nature as
would severely affect the continued business operations of the employer to the detriment of all
and sundry if not properly addressed.
Sanoh asserts that cancelled orders of wire condensers led to the phasing out of the Wire
Condenser Department which triggered retrenchment. Sanoh presented the letters of
cancellation given by Matsushita and Sanyo as evidence of cancelled orders. The evidence
presented by Sanoh barely established the connection between the cancelled orders and the
projected business losses that may be incurred by Sanoh. Sanoh failed to prove that these
cancelled orders would severely impact on their production of wire condensers.
We held in Lambert Pawnbrokers and Jewelry Corporation v. Binamira, that the losses must be
supported by sufficient and convincing evidence and the normal method of discharging this is by
the submission of financial statements duly audited by independent external auditors. It was
aptly observed by the appellate court that no financial statements or documents were presented
to substantiate Sanohs claim of loss of P7 million per month. And a business lull caused by
lack of orders which could have justified retrenchment was not shown by petitioner. As observed
once more by the Court of Appeals, petitioner failed to present proof of the extent of the reduced
order and its contribution to the sustainability of its business.
As the Wire Condenser Department is still in operation and no business losses were proven by
Sanoh, the dismissal of respondents was unlawful.

85. Daabay vs.Coca-Cola Bottlers Phils G.R. No. 199890, Aug. 19, 2013
Facts:
The employment of Daabay with Coca-Cola as Sales Logistics Checker was terminated by the
company in June 2005, following receipt of information from one Cesar Sorin that Daabay was
part of a conspiracy that allowed the pilferage of company property.
The allegations of Sorin were embodied in an affidavit where The losses to the company were
also confirmed by an inventory and audit conducted by Coca-Colas Territory Finance Head,
Silvia Ang. Such losses comprised of cases of assorted softdrinks, empty bottles, missing shells
and missing pallets valued at P20,860,913.00.
Coca-Cola then served upon Daabay a Notice to Explain with Preventive Suspension, which
required him to explain in writing his participation in the scheme that was reported to involve

ATHENA M. SALAS | LABOR CASE DIGEST 2015 188

logistics checkers and gate guards. In compliance therewith, Daabay submitted an Explanation
wherein he denied any participation in the reported.
A formal investigation on the matter ensued. Eventually, Coca-Cola served upon Daabay a
Notice of Termination that cited pilferage, serious misconduct and loss of trust and confidence
as grounds. At the time of his dismissal, Daabay had been a regular employee of Coca-Cola for
eight years, and was receiving a monthly pay of P20,861.00, exclusive of other benefits.
Daabay then filed the subject labor complaint against Coca-Cola and the President and Plant
Logistics Managers of Coca-Cola at the time of the dispute.
Labor Arbiter: Daabay was illegally dismissed because his participation in the alleged
conspiracy was not proved by substantial evidence. In lieu of reinstatement and considering the
already strained relations between the parties, ordered the payment to Daabay of backwages
and separation pay or retirement benefits, as may be applicable. ([P]750,996.00.)
Dissatisfied, Coca-Cola, appealed to the NLRC. Daabay filed a separate appeal to ask for his
reinstatement without loss of seniority rights, the payment of backwages instead of separation
pay or retirement benefits, and an award of litigation expenses, moral and exemplary damages
and attorneys fees.
NLRC: reversed the finding of illegal dismissal. That there was reasonable and well-founded
basis to dismiss [Daabay], not only for serious misconduct, but also for breach of trust or loss of
confidence arising from such company losses. Daabays participation in the conspiracy was
sufficiently established. Several documents such as checkers receipts and sales invoices that
made the fraudulent scheme possible were signed by Daabay. The NLRC also found fault in
Daabay for his failure to detect the pilferage, considering that the timely recording and
monitoring as security control for the outgoing [sic] of company products are necessarily
connected with the functions, duties and responsibilities reposed in him as Sales Logistics
Checker. Notwithstanding its ruling on the legality of the dismissal, the NLRC awarded
retirement benefits in favor of Daabay.
Coca-Colas partial motion for reconsideration to assail the award of retirement benefits was
denied by the NLRC explaining that there was a need to humanize the severe effects of
dismissal and tilt the scales of justice in favor of labor as a measure of equity and
compassionate social justice. Daabay also moved to reconsider, but his motion remained
unresolved by the NLRC. Undaunted, Coca-Cola appealed to the CA.
CA: agreed with Coca-Cola that the award of retirement benefits lacked basis considering that
Daabay was dismissed for just cause. Daabays motion for reconsideration was denied in a
Resolution dated December 9, 2011; hence, this petition.
Issue:
Whether or not Daabay is entitled to retirement benefits.
Ruling:
No.
Daabay was declared by the NLRC to have been lawfully dismissed by Coca-Cola on the
grounds of serious misconduct, breach of trust and loss of confidence. Our pronouncement in

ATHENA M. SALAS | LABOR CASE DIGEST 2015 189

Philippine Airlines, Inc. v. NLRC on the issue of whether an employee who is dismissed for just
cause may still claim retirement benefits equally applies to this case. We held:
At the risk of stating the obvious, private respondent was not separated from petitioners employ
due to mandatory or optional retirement but, rather, by termination of employment for a just
cause. Thus, any retirement pay provided by PALs Special Retirement & Separation Program
dated February 15, 1988 or, in the absence or legal inadequacy thereof, by Article 287 of the
Labor Code does not operate nor can be made to operate for the benefit of private respondent.
Even private respondents assertion that, at the time of her lawful dismissal, she was already
qualified for retirement does not aid her case because the fact remains that private respondent
was already terminated for cause thereby rendering nugatory any entitlement to mandatory or
optional retirement pay that she might have previously possessed.
In ruling against the grant of the retirement benefits, we also take note of the NLRCs lone
justification for the award. Being intended as a mere measure of equity and social justice, the
NLRCs award was then akin to a financial assistance or separation pay that is granted to a
dismissed employee notwithstanding the legality of his dismissal. Jurisprudence on such
financial assistance and separation pay then equally apply to this case.
A contrary rule would, as the petitioner correctly argues, have the effect, of rewarding rather
than punishing the erring employee for his offense. And we do not agree that the punishment is
his dismissal only and that the separation pay has nothing to do with the wrong he has
committed. Of course it has. Indeed, if the employee who steals from the company is granted
separation pay even as he is validly dismissed, it is not unlikely that he will commit a similar
offense in his next employment because he thinks he can expect a like leniency if he is again
found out. This kind of misplaced compassion is not going to do labor in general any good as it
will encourage the infiltration of its ranks by those who do not deserve the protection and
concern of the Constitution.
Clearly, considering that Daabay was dismissed on the grounds of serious misconduct, breach
of trust and loss of confidence, the award based on equity was unwarranted.

Although retirement benefits, where not mandated by law, may still be granted by agreement of
the employees and their employer or as a voluntary act of the employer, there is no proof that
any of these incidents attends the instant case.
86. MZR Industries et al., vs. Colambot, G.R. No. 179001, Aug. 28, 2013
Facts:
Respondent Colambot was a messenger of petitioner MZR Industries whose work
responsibilities include field, messengerial and other liaison work. Colambots work performance
started to deteriorate after 2 years in the company. Petitioner served memoranda to Colambot
for habitual tardiness, negligence, and violations of office policies as well as written warnings for
insubordination and for negligence caused by careless handling of confidential office
documents. Despite the warnings issued, Colambot failed to mend his ways and thereafter in a
Memorandum dated October 25, 2004 issued by petitioner Lea Timbal, Colambot was given a
notice of suspension.

ATHENA M. SALAS | LABOR CASE DIGEST 2015 190

Again, in a Memorandum12 dated November 25, 2004, Colambot was suspended from
November 26, 2004 until December 6, 2004 for insubordination. Petitioners claimed they waited
for Colambot to report back for work on December 7, 2004, but they never heard from him
anymore. Later, petitioners were surprised to find out that Colambot had filed a complaint for
illegal suspension, underpayment of salaries, overtime pay, holiday pay, rest day, service
incentive leave and 13th month pay. On December 16, 2004, the complaint was amended to
illegal dismissal, illegal suspension, underpayment of salaries, holiday pay, service incentive
pay, 13th month pay and separation pay.
Colambot claimed that he was made to choose between resigning from the company or the
company will be the one to terminate his services. He said he refused to resign. Colambot
alleged that Quiroz made him sign a memorandum for his suspension, from November 26 to
December 6, 2004. After affixing his signature, Quiroz told him that effective December 7, 2004,
he is already deemed terminated. Later, on December 2, 2004, respondent went back to the
company to look for Timbal to get his salary. He claimed that Timbal asked him to turn over his
company I.D.
Petitioners maintained that they never terminated Colambot's employment and argues that the
failure to report to work constitutes abandonment of work.
Labor Arbiter: ruled that MZR Industries was guilty of illegal dismissal and ordered to reinstate
complainant to his former position with full backwages from date of dismissal until actual
reinstatement and moral and exemplary damages.
There was no abandonment as there was no deliberate intent on the part of Colambot to sever
the employer-employee relationship. The Labor Arbiter likewise noted that Colambot should
have been notified to return back to work, which petitioner failed to do.
NLRC: Judgment of the Labor Arbiter was set aside and the Complaint is dismissed for lack of
merit.
The NLRC further believed that Colambot abandoned his work due to his refusal to report for
work after his suspension. The failure of MZR to notify Colambot to return back to work is not
tantamount to actual dismissal.
CA: reversed the assailed Decision of the NLRC and ordered the decision of the LA to be
reinstated with modification that in lieu of reinstatement, petitioners were ordered to pay
respondent separation pay equivalent to one (1) month pay for every year of service in addition
to full backwages.
CA based its decision on the grounds that: (1) MZR failed to prove abandonment on the part of
Colambot, and (2) MZR failed to serve Colambot with the required written notices of
dismissal.2007.
Issues:
1.)

W/N there was illegal dismissal.

2.)

W/N there was abandonment of work.

3.)

W/N Respondent is entitled to backwages.

ATHENA M. SALAS | LABOR CASE DIGEST 2015 191

Ruling:
1.) While the rule is that in illegal dismissal cases, the employer bears the burden of proving that
the termination was for a valid or authorized cause, there must be facts and evidence to
establish a prima facie case that the employee was dismissed from employment. Before the
employer must bear the burden of proving that the dismissal was legal, the employee must first
establish by substantial evidence the fact of his dismissal from service.
Colambot's unsubstantiated allegation of having been verbally terminated from his work, there
was no evidence presented to show that he was indeed dismissed from work or was prevented
from returning to his work. In the absence of any showing of an overt or positive act proving that
petitioners had dismissed respondent, the latter's claim of illegal dismissal cannot be sustained.
Neither could the petitioners be blamed for failing to order respondent to return back to
work.1wphi1 Records show that Colambot immediately filed the complaint for illegal dismissal
on December 16, 2004,29 or just a few days when he was supposed to report back to work on
December 7, 2004. For petitioners to order respondent to report back to work, after the latter
had already filed a case for illegal dismissal, would be unsound.
2.) Mere absence or failure to report for work, even after notice to return, is not tantamount to
abandonment. The burden of proof to show that there was unjustified refusal to go back to work
rests on the employer. Abandonment is a matter of intention and cannot lightly be presumed
from certain equivocal acts. To constitute abandonment, there must be clear proof of deliberate
and unjustified intent to sever the employer-employee relationship. Clearly, the operative act is
still the employees ultimate act of putting an end to his employment.
A complaint for illegal dismissal is inconsistent with abandonment of employment. An employee
who takes steps to protest his dismissal cannot logically be said to have abandoned his work.
The filing of such complaint is proof enough of his desire to return to work, thus negating any
suggestion of abandonment
3.) These circumstances, taken together, the lack of evidence of dismissal and the lack of intent
on the part of the respondent to abandon his work, the remedy is reinstatement but without
backwages. However, considering that reinstatement is no longer applicable due to the strained
relationship between the parties and that Colambot already found another employment, each
party must bear his or her own loss, thus, placing them on equal footing.

87. Integrated Microelectronics, Inc. v. Pionilla, G.R. No. 200222, August 28, 2013
Facts:
On November 14, 1996, Pionilla was hired by IMI as its production worker. On May 5, 2005,
Pionilla received a notice from IMI requiring him to explain the incident which occurred the day
before where he was seen escorting a lady to board the company shuttle bus at the Alabang
Terminal. It was reported by the bus marshall that the lady was wearing a company identification
card (ID) which serves as a free pass for shuttle bus passengers even if she was just a job
applicant at IMI. In this regard, Pionilla admitted that he lent his ID to the lady who turned out to

ATHENA M. SALAS | LABOR CASE DIGEST 2015 192

be his relative. He further intimated that he risked lending her his ID to save on their
transportation expenses. Nevertheless, he apologized for his actions.
IMI found Pionilla guilty of violating Article 6.12 of the Company Rules and Regulations (CRR)
which prohibits the lending of one's ID since the same is considered a breach of its security
rules and carries the penalty of dismissal. Subsequently, or on August 17, 2005, Pionilla
received a letter dated August 16, 2005 informing him of his dismissal from service. Three days
after, he filed a complaint for illegal dismissal with damages against IMI.
LA, NLRC and CA ruled that Pionilla was illegally dismissed and ordered IMI to reinstate Pionilla
to his former position and to pay him backwages.
Issue:
WON Pionilla was illegally dismissed and if so, is he entitled to reinstatement and full
backwages.
Ruling:
Yes. The Court ruled that the imposition of the penalty of dismissal was too harsh and
incommensurate to the infraction he committed, this especially considering his admission of the
incident and apologized for it, and his nine years of unblemished service.
As a general rule, an illegally dismissed employee is entitled to reinstatement (or separation
pay, if reinstatement is not viable) and payment of full backwages. In certain cases, however,
the Court has carved out an exception to the foregoing rule and thereby ordered the
reinstatement of the employee without backwages on account of the following: (a) the fact that
dismissal of the employee would be too harsh of a penalty; and (b) that the employer was in
good faith in terminating the employee.
In this case, the Court observes that: (a) the penalty of dismissal was too harsh of a penalty to
be imposed against Pionilla for his infractions; and (b) IMI was in good faith when it dismissed
Pionilla as his dereliction of its policy on ID usage was honestly perceived to be a threat to the
company's security. In this respect, since these concurring circumstances trigger the application
of the exception to the rule on backwages as enunciated in the above-cited cases, the Court
finds it proper to accord the same disposition and consequently directs the deletion of the award
of backwages in favor or Pionilla, notwithstanding the illegality of his dismissal.

88. Asia Brewery, Inc. v. Tunay na Pagkakaisa ng mga Manggagawa sa Asia, G.R. Nos.
171594-96, September 18, 2013
Principle: the financial statements of the employer must be properly audited by an external and
independent auditor in order to be admissible in evidence for purposes of determining the
proper wage award.
Facts:
[Respondent union] and [petitioner corporation] had been negotiating for a new collective
bargaining agreement (CBA) for the years 2003-2006 since the old CBA expired last July 2003.

ATHENA M. SALAS | LABOR CASE DIGEST 2015 193

After about 18 sessions or negotiations, the parties were still unable to reconcile their
differences on their respective positions on most items, particularly on wages and other
economic benefits.
There was a deadlock so the respondent union filed a notice of strike with the NCMB. The union
conducted a strike.
On November 20, 2003, [petitioner corporation] then petitioned the Secretary of the Department
of Labor and Employment (DOLE) to assume jurisdiction over the parties' labor dispute,
invoking Article 263 (g) of the Labor Code.
On December 19, 2003, the public respondent, through Undersecretary/Acting Secretary
Manuel G. Imson, issued an order assuming jurisdiction over the labor dispute between the
[respondent union] and [petitioner corporation].
In the meantime, in a decision dated January 19, 2004, Secretary of Labor Patricia Sto. Tomas
resolved the deadlock between the parties. As summarized in a later resolution, the public
respondent granted the following arbitral awards (which is now in dispute):
(1)WAGE INCREASES as follows:
First Year

P18.00

Second Year =

15.00

Third Year

12.00

Total

P45.00

=====
(2)HEALTH CARE (HMO)
P1,300 premium to be shouldered by Asia Brewery, Inc., for each covered employee and
P1,800 contribution [for each] Union member-dependent.
In modifying the arbitral award of the Secretary of Labor, the CA ruled that: (1) ...XXX... ; (2) The
computation of wage increase should be remanded to the Secretary of Labor because the
computation was based on petitioner corporation's unaudited financial statements, which have
no probative value pursuant to the ruling in Restaurante Las Conchas v. Llego, 6 and was done
in contravention of DOLE Advisory No. 1, Series of 2004, which contained the guidelines in
resolving bargaining deadlocks; and (3) The health benefits should be P1,390.00 per covered
employee because petitioner corporation had already agreed to this amount and the same
cannot be altered or reduced by the Secretary of Labor.
Issue:
Did the Secretary properly consider and appreciate the evidence presented before him?
Petitioners contention: Petitioner corporation admits that what it submitted to the Secretary of
Labor were unaudited financial statements which were then used as one of the bases in fixing
the wage award. However, petitioner corporation argues that these financial statements were
duly signed and certified by its chief financial officer. These statements have also been allegedly
submitted to various government agencies and should, thus, be considered official and public

ATHENA M. SALAS | LABOR CASE DIGEST 2015 194

documents. Moreover, respondent union did not object to the subject financial statements in the
proceedings before the Secretary of Labor and even used the same in formulating its (the
union's) arguments in said proceedings. Thus, petitioner corporation contends that although the
subject financial statements were not audited by an external and independent auditor, the same
should be considered substantial compliance with the order of the Secretary of Labor to produce
the petitioner corporation's complete audited financial statements for the past five years.
Furthermore, the Decision of the Secretary of Labor was not solely based on the subject
financial statements as the CBA history, costing of the proposals, and wages in other similarly
situated bargaining units were considered. Finally, petitioner corporation claims that the
demands of respondent union on wage increase are unrealistic and will cause the former to
close shop.
Ruling:
The contention is untenable.
In Restaurante Las Conchas v. Llego, several employees filed a case for illegal dismissal after
the employer closed its restaurant business. The employer sought to justify the closure through
unaudited financial statements showing the alleged losses of the business. We ruled that such
financial statements are mere self-serving declarations and inadmissible in evidence even if the
employees did not object to their presentation before the Labor Arbiter. Similarly, in Uichico v.
National Labor Relations Commission, the services of several employees were terminated on
the ground of retrenchment due to alleged serious business losses suffered by the employer.
We ruled that by submitting unaudited financial statements, the employer failed to prove the
alleged business losses, viz.:
. . . It is true that administrative and quasi-judicial bodies like the NLRC are not bound by the
technical rules of procedure in the adjudication of cases. However, this procedural rule should
not be construed as a license to disregard certain fundamental evidentiary rules. While the rules
of evidence prevailing in the courts of law or equity are not controlling in proceedings before the
NLRC, the evidence presented before it must at least have a modicum of admissibility for it to
be given some probative value. The Statement of Profit and Losses submitted by Crispa, Inc. to
prove its alleged losses, without the accompanying signature of a certified public accountant or
audited by an independent auditor, are nothing but self-serving documents which ought to be
treated as a mere scrap of paper devoid of any probative value. For sure, this is not the kind of
sufficient and convincing evidence necessary to discharge the burden of proof required of
petitioners to establish the alleged losses suffered by Crispa, Inc. in the years immediately
preceding 1990 that would justify the retrenchment of respondent employees. . . .
While the above-cited cases involve proof necessary to establish losses in cases of business
closure or retrenchment, we see no reason why this rule should not equally apply to the
determination of the proper level of wage award in cases where the Secretary of Labor assumes
jurisdiction in a labor dispute pursuant to Article 263 (g) of the Labor Code.
We take note of the "middle ground" approach employed by the Secretary in this case which we
do not necessarily find to be the best method of resolving a wage dispute. Merely finding the
midway point between the demands of the company and the union, and "splitting the difference"
is a simplistic solution that fails to recognize that the parties may already be at the limits of the
wage levels they can afford. It may lead to the danger too that neither of the parties will engage
in principled bargaining; the company may keep its position artificially low while the union

ATHENA M. SALAS | LABOR CASE DIGEST 2015 195

presents an artificially high position, on the fear that a "Solomonic" solution cannot be avoided.
Thus, rather than encourage agreement, a "middle ground approach" instead promotes a "play
safe" attitude that leads to more deadlocks than to successfully negotiated CBAs.
Thus, we rule that the Secretary of Labor gravely abused her discretion when she relied on the
unaudited financial statements of petitioner corporation in determining the wage award because
such evidence is self-serving and inadmissible. Not only did this violate the December 19, 2003
Order of the Secretary of Labor herself to petitioner corporation to submit its complete audited
financial statements, but this may have resulted to a wage award that is based on an inaccurate
and biased picture of petitioner corporation's capacity to pay one of the more significant
factors in making a wage award. Petitioner corporation has offered no reason why it failed
and/or refused to submit its audited financial statements for the past five years relevant to this
case. This only further casts doubt as to the veracity and accuracy of the unaudited financial
statements it submitted to the Secretary of Labor. Verily, we cannot countenance this procedure
because this could unduly deprive labor of its right to a just share in the fruits of production and
provide employers with a means to understate their profitability in order to defeat the right of
labor to a just wage.
UNION['S] DEMANDS
COMPANY'S OFFERS

this Office awards the following wage increases:

For the FIRST YEAR: P36


For the SECOND YEAR:
For the THIRD YEAR: 36

36
For the First 18 months:

For the Second 18 months: 18


For the SECOND YEAR:

P18

For the FIRST YEAR: P18

15

For the THIRD YEAR:12


TOTAL:
years

P108 for three (3) years

P36 for 36 months

P45

for

three

(3)

As can be seen, the Secretary of Labor failed to indicate the actual data upon which the wage
award was based. It even appears that she utilized the "middle ground approach which we
precisely warned against in Meralco. Factors such as the actual and projected net operating
income, impact of the wage increase on net operating income, the company's previous CBAs,
and industry trends were not discussed in detail so that the precise bases of the wage award
are not discernible on the face of the Decision. The contending parties are effectively precluded
from seeking a review of the wage award, even if proper under our ruling in Meralco, because of
the general but unsubstantiated statement in the Decision that the wage award was based on
factors like the bargaining history, trends of arbitrated and agreed awards, and industry trends.
In fine, there is no way of determining if the Secretary of Labor utilized the proper evidence,
figures or data in arriving at the subject wage award as well as the reasonableness thereof. This
falls short of the requirement of administrative due process obligating the decision-maker to
adjudicate the rights of the parties in such a manner that they can know the various issues
involved and the reasons for the decision rendered.

ATHENA M. SALAS | LABOR CASE DIGEST 2015 196

89. Mcmer Corp., et al., vs. NLRC et al., GR No. 193421, June 4, 2014
Facts:
Private respondent, Libunao is employed as McMers legal department head. He has exhibited
a strong opposition to some company practices resulting in a severe marginal distance between
him and petitioners, General Manager Roque and, company President Alvestir at the workplace.
On About noon of July 20, 2007, private responent was summoned by Roque supposedly to
discuss administrative matter. However, private respondent refused to comply with the Roques
summon. Consequently petitioner Roque went to private respondents office at the height of his
anger with threat to inflict physical harm, shouted a command for private respondent to proceed
to petitioners office. Private respondent was approached sarcastically with commanding voice
by petitioner Roque even in front of some officers and rank-and-file employees and newly-hired
employees.
As a consequence of the foregoing, private respondent elected to discontinue work that
afternoon and immediately proceeded to the Valenzuela Police Headquarters to report on the
incident in the police blotter. Private respondent did not report for work from July 21, 2007 up to
July 30, 2007.
Private respondent filed a complaint for unfair labor practices, constructive illegal dismissal,
non-payment of 13th month pay and separation pay, moral and exemplary damages, as well as
attorneys fees, against petitioners McMer Corporation, Inc., Roque, and Alvestir. Libunao was
informed verbally by petitioner Alvestir that on account of strained relationship brought about by
the institution of a labor case against petitioners, the latter is inclined to dismiss him from office.
Private respondent was, likewise, offered a separation pay in the sum of P55,000.00.
Issues
1: WON private respondent was constructively dismissed
2: WON he is entitled to full backwages, separation pay in lieu of reinstatement, and moral,
exemplary and nominal damages.
Ruling:
1: Yes, there was constructive dismissal.
Constructive dismissal the cessation of work because continued employment is rendered
impossible, unreasonable or unlikely; when there is a demotion in rank or diminution in pay or
both; or when a clear discrimination, insensibility, or disdain by an employer becomes
unbearable to the employee.
It is an act amounting to dismissal but made to appear as if it were not, ergo, a dismissal in
disguise.
As such, the law recognizes and resolves this situation in favor of employees in order to protect
their rights and interests from the coercive acts of the employer.

ATHENA M. SALAS | LABOR CASE DIGEST 2015 197

The test of constructive dismissal is whether a reasonable person in the employees position
would have felt compelled to give up his position under the circumstances. In Aguilar v. Burger
Machine Holdings Corporation the hostile and unreasonable working conditions of petitioner
justified the finding of the Labor Arbiter and the NLRC that petitioner was constructively
dismissed.
Petitioners performance may not have been exceptional as he ranked 14th in the quality food
service control survey for the 1st quarter of 2002. But he was certainly not grossly inefficient as
Burger Machine pictured him to be. In fact, he received several citations and was able to comply
with the directive to reduce his shortages for the month of November 2001. From all indications,
there is really no ground to dismiss petitioner for gross inefficiency. And, as Burger Machine saw
it, the only way to get rid of the latter was to constructively dismiss him.
The employee who is constructively dismissed may be allowed to keep on coming to work.
In Siemens Philippines, Inc. v. Domingo, we have declared that
An employee who is forced to surrender his position through the employer's unfair or
unreasonable acts is deemed to have been illegally terminated and such termination is deemed
to be involuntary.
Constructive dismissal does not always involve forthright dismissal or diminution in rank,
compensation, benefit and privileges.
There may be constructive dismissal if an act of clear discrimination, insensibility or disdain by
an employer becomes so unbearable on the part of the employee that it could foreclose any
choice by him except to forego his continued employment.
In this case, the private respondent was in a position wherein he would have felt compelled to
give up his position under the circumstances because continued employment was just
impossible, unreasonable or unlikely.
The Affidavit executed by Guiao is sufficient to depict the hostile working environment petitioner
McMer maintains. Further, we find her description of petitioner Roques disposition adequate to
support a conclusion that private respondent was caught in the state of humiliation and
embarrassment in the presence of his co-employees as a result thereof. It is also corroborated
another documentary evidence, i.e., the police blotter.
Aside from what transpired on July 20, 2007, various factors were considered in determining the
working environment of petitioner McMer. As may gleaned from the records, what transpired on
July 20, 2007 was not merely an isolated outburst on the part of petitioner Roque. The latters
behaviour towards his employees shows a clear insensibility rendering the working condition of
private respondent unbearable.
Private respondent had reason to dawdle and refuse to comply with the summon of petitioner
Roque out of severe fear that he will be physically harmed. In fact, the same was clearly
manifested by his immediate reaction to the situation by going to the Valenzuela Police to report
the incident.
Roque has displayed harassment and intimidation towards his employees. The fact that none of
the employees complained or brought this to the attention of the appropriate authority does not
validate petitioners actions. Private respondent reasoned that it was difficult for him to look for

ATHENA M. SALAS | LABOR CASE DIGEST 2015 198

another employment, considering that at the time he filed his Position Paper, he was already 58
years old. His eventual decision to leave petitioners due to the agonizing situation at the
workplace cannot, therefore, be discounted.
Private respondent could not have given up a job he has engaged in for eight years unless it
has become so unbearable for him to stay therein. Indeed, private respondent felt compelled to
give up his employment.
Private respondents professional ethic or moral belief was compromised due to certain
business practices of petitioner McMer that were never exposed due to the employees fear of
reprisal, as shown in private respondents Position Paper.
As far as private respondent is concerned, how the working place is being run has caused
inordinate strain on his professional work and moral principles, even stretching to desecration of
dignity in the workplace. The allegation that all of private respondents staff were removed one
by one until finally only the latter was left alone performing managerial and clerical duties is
merely part of the greater scheme brought forth by the insensibility of petitioners in dealing with
the employees.
No employee should be subjected to constant harassment, ridicule and inhumane treatment on
the basis of management prerogative or even for poor performance at work. While we concur
with petitioners that raising ones voice in the workplace as a result of displeasure in the
performance of an employee is not illegal per se, the right to impose disciplinary sanctions upon
an employee for just and valid cause is not without limit. The means does not justify the end;
thus, the same should be in accordance with the norms of due process.
2: Section 279 of the Labor Code explicitly states that an employee who is unjustly dismissed
from work shall be entitled to reinstatement without loss of seniority rights and other privileges
and to his full backwages, inclusive of allowances, and to his other benefits or their monetary
equivalent computed from the time his compensation was withheld from him up to the time of
his actual reinstatement.
As early as Santos v. NLRC, the Supreme Court already explained the underlying basis for the
foregoing provision, to wit
x x x. These twin remedies reinstatement and payment of backwages make the dismissed
employee whole who can then look forward to continued employment. Thus, do these two
remedies give meaning and substance to the constitutional right of labor to security of tenure.
The two forms of relief are distinct and separate, one from the other. Though the grant of
reinstatement commonly carries with it an award of backwages, the inappropriateness or nonavailability of one does not carry with it the inappropriateness or non-availability of the other. x x
x
Generally, a constructively dismissed employee is entitled to the twin remedies of reinstatement
and payment of backwages pursuant to Sec. 279 of the Labor Code. However, strained
relations between the employer and employee is an exception to the rule requiring actual
reinstatement for illegally dismissed employees for the practical reason that the already existing
antagonism will only fester and deteriorate, and will only worsen with possible adverse effects
on the parties if we shall compel reinstatement; thus, the use of a viable substitute that protects
the interests of both parties while ensuring that the law is respected.

ATHENA M. SALAS | LABOR CASE DIGEST 2015 199

In this case, considering that reinstatement is no longer feasible due to the strained relations
between petitioners and private respondent, we find that the payment of separation pay of one
months salary for every year of service is just and reasonable as an alternative of
reinstatement.
Private respondent was unjustly treated in the workplace, and, consequently, bore wounded
feelings and suffered mental anguish during his tenure with petitioner McMer until he was
constructively dismissed from service. Therefore, he is entitled to moral, exemplary and nominal
damages in the aggregate amount of P90,000.00 due to the wanton, oppressive and malevolent
manner by which private respondent was illegally and constructively terminated.

90. Philippine Spring Water Resources Inc vs. Court of Appeals, GR No. 205278, June 11,
2014
Facts:
Mahilum was engaged as the VP for Sales and Marketing for the Bulacan-South Luzon Area of
petitioner Philippine Spring Water Resources, Inc. (PSWRI). Petitioner Danilo Lua (Lua) is the
President and Chief Executive Officer (CEO) of PSWRI.
Sometime in November 2004, the inauguration of PSWRIs Bulacan plant would be celebrated
at the same time with the companys Christmas party. Mahilum was designated as over-all
chairman of the affair to be held on December 19, 2004.
On the inaugural day, Mahilum was not seen around to supervise the program proper as he
entertained some visitors of the company. According to him, he delegated the task to
Evangelista.
Mahilums attention was, however, called when Lua got furious because he was not recognized
during the program. He was not mentioned in the opening remarks or called to deliver his
inaugural speech. Upon inquiry from the emcees of the program, Mahilum learned that they
were not apprised of Luas decision to deliver the speech considering that he previously
declined to have a part in the program as he would be very busy during the affair. Thus, Luas
speech appeared to be optional in the printed program during the affair.
On the following day, Mahilum was required to explain why Lua was not recognized and made
to deliver his speech. At the same time, he was placed under preventive suspension for thirty
(30) days. Mahilum submitted his written explanation. Subsequently, an investigation was
conducted.
When his 30-day suspension ended, Mahilum reported for work but was prevented from
entering the workplace. Sometime in the first week of March 2005, he received a copy of the
Memorandum, dated January 31, 2005, terminating his services effective the next day or on
February 1, 2005. On February 9, 2005, a clearance certificate was issued to Mahilum. He

ATHENA M. SALAS | LABOR CASE DIGEST 2015 200

received the amount of 43,998.56 and was made to execute the Release, Waiver and Quitclaim
in favor of the company and Lua.
Mahilum filed a complaint for illegal dismissal with prayer for reinstatement, payment of back
wages and damages. He argued that he was illegally suspended and, thereafter, dismissed
constructively from the service. He also claimed that he was forced to sign the waiver.
1st Issue:
WON Mahilum was dismissed for a just cause under the ground of loss of trust and confidence.
Ruling:
Yes, Mahilum was illegally dismissed. The charge of loss of trust and confidence had no leg to
stand on, as the act complained of was not work-related. The petitioners were not able to prove
that Mahilum was unfit to continue working for the company.

Mahilums failure to effectively discharge his assignment as the over-all chairman of the
festivities was due to mere inadvertence and the mistaken belief that he had properly delegated
the details of the program to another officer. Further, his designation as the chairman of the
whole affair did not form part of his duty as a supervisor. Mahilum was engaged to supervise the
sales and marketing aspects of PSWRIs Bulacan Plant.
Even as jurisprudence has distinguished the treatment of managerial employees or employees
occupying positions of trust and confidence from that of rank-and-file personnel, insofar as the
application of the doctrine of trust and confidence is concerned, such is inapplicable to the
instant case since as above-stated, private respondents lapse was justified, unintentional,
without deliberate intent and unrelated to the duty for which he was engaged.
The quitclaim executed by Mahilum did not operate to bar a cause of action for illegal dismissal.
That the amounts received by Mahilum were only those owing to him under the law indeed
bolstered the fact that the quitclaim was executed without consideration. Suffice it to say, the
subject quitclaim may not be considered as a valid and binding undertaking.
2nd Issue:
Mahilums entitlement to monetary claims.
Ruling:
Mahilum, as a regular employee at the time of his illegal dismissal, is entitled to separation pay
and backwages, computed from the time of his dismissal up to the finality of the decision.
Reinstatement is no longer viable considering the circumstances of animosity between Mahilum
and Lua.
Article 279 of the Labor Code provides that an employee who is unjustly dismissed from work
shall be entitled to reinstatement without loss of seniority rights and other privileges, to full
backwages, inclusive of allowances, and to other benefits or their monetary equivalent
computed from the time his compensation was withheld from him up to the time of his actual
reinstatement. Due to the strained relations of the parties, however, the payment of separation
pay has been considered an acceptable alternative, when reinstatement is no longer desirable

ATHENA M. SALAS | LABOR CASE DIGEST 2015 201

or viable. On the one hand, such payment liberates the employee from what could be a highly
oppressive work environment. On the other, the payment releases the employer from the
grossly unpalatable obligation of maintaining in its employ a worker it could no longer trust.17
Thus, as an illegally or constructively dismissed employee, the respondent is entitled to: (1)
either reinstatement, if viable, or separation pay, if reinstatement is no longer viable; and (2)
backwages. These two reliefs are separate and distinct from each other and are awarded
conjunctivel
3rd Issue:
Inclusion of commission as part of Mahilums backwages.
Ruling:
The 0.25% commission on cash and delivery sales should not be included in his backwages
because the alleged commissions were profit-sharing payments and had no clear, direct or
necessary relation to the amount of work he actually performed.
Back wages are granted on grounds of equity to workers for earnings lost due to their illegal
dismissal from work.
They are a reparation for the illegal dismissal of an employee based on earnings which the
employee would have obtained, either by virtue of a lawful decree or order, as in the case of a
wage increase under a wage order, or by rightful expectation, as in the case of ones salary or
wage.
The outstanding feature of backwages is thus the degree of assuredness to an employee that
he would have had them as earnings had he not been illegally terminated from his employment.
It is well-established in jurisprudence that the determination of whether or not a commission
forms part of the basic salary depends upon the circumstances or conditions for its payment.
In Phil Duplicators, Inc. v. NLRC, the Court held that commissions earned by salesmen form
part of their basic salary. The salesmens commissions, comprising a pre-determined
percentage of the selling price of the goods sold by each salesman, were properly included in
the term basic salary for purposes of computing the 13th month pay. The salesmens
commissions are not overtime payments, nor profit-sharing payments nor any other fringe
benefit, but a portion of the salary structure which represents an automatic increment to the
monetary value initially assigned to each unit of work rendered by a salesman.
On the other hand, in Boie-Takeda Chemicals, Inc. v. De la Serna, the so-called commissions
paid to or received by medical representatives were excluded from the term basic salary
because these were paid to the medical representatives and rank-and-file employees as
productivity bonuses, which were generally tied to the productivity, or capacity for revenue
production, of a corporation and such bonuses closely resemble profit-sharing payments and
had no clear direct or necessary relation to the amount of work actually done by each individual
employee.
In Mahilums case, Phil. Duplicator cannot be automatically applied without considering his
position as Vice-President for sales and marketing of the PSWRIs Bulacan-South Luzon Area.
This factor constrains the Court to hold that Mahilums 0.25% commission based on the monthly
sales and 0.25% commission for cash payments must be taken to come in the nature of

ATHENA M. SALAS | LABOR CASE DIGEST 2015 202

overriding commission, not sales commission. The latter is not properly includable in the basic
salary as it must be earned by actual market transactions attributable to the claimant. Curiously,
Mahilum did not comment on the petitioners objection to the award. Not being a salesman who
directly effected any sale of a product, the commission embodied in the agreement partook of
the nature of profit-sharing business based on quota.
4th Issue:
Mahilums entitlement to the award of moral and exemplary damages.
Ruling:
Mahilum is only entitled to attorney's fees in the amount of ten percent (10%) of his total
monetary award, having been forced to litigate in order to seek redress of his grievances, as
provided in Article 111 of the Labor Code, as amended, and existing jurisprudence.
He is not entitled to an award of moral or exemplary damages because no evidence presented
to prove the same.Worth reiterating is the rule that moral damages are recoverable where the
dismissal of the employee was attended by bad faith or fraud or constituted an act oppressive to
labor, or was done in a manner contrary to morals, good customs, or public policy. Likewise,
exemplary damages may be awarded if the dismissal was effected in a wanton, oppressive or
malevolent manner.

91. Libcap Marketing Corp. et al., vs. Baquial, GR No. 192011, June 30, 2014
Facts:
Private worked as accounting clerk of petitioner company in Cagayan de Oro city. One day,
petitioner company noticed that there was something amiss with one of the transactions entered
into by petitioner in that for a single deposit to a bank, two separate slips were filled out by the
latter. She tried to explain that two deposits were made that day, but upon further investigation,
it was discovered that there was only one deposit was made. Shortly after the incident,
respondents salary was charged with the payment of the amount involved, as during each
payday, her salary was deducted with staggered amounts to answer for the loss imputed to her.
Respondent then gave notice to petitioner that an investigation would be made in Ilo-ilo City
(she was based in Cagayan de Oro), but due to financial and geographical constraints, she was
not able to participate in the proceedings. She was subsequently terminated from work. In the
Labor Arbiter, NLRC and CA, private respondent was awarded nominal damages due to the
failure of petitioner company to give her due notice before she got terminated. Libcap (petitioner
company), on the other hand, claimed that what is merely required is the opportunity to be
heard, which it claimed to have extended to private respondent but that it was the latter who
failed to attend in the investigations conducted for her alleged infraction.
Issue:
Whether private respondent is entitled to nominal damages, and corollary to answering this
question, whether due notice was given to private respondent.
Ruling:

ATHENA M. SALAS | LABOR CASE DIGEST 2015 203

1. Private respondent is entitled to nominal damages for not being given due process before she
was slapped with an administrative penalty. The CA, the NLRC and the Labor Arbiter are correct
in concluding that respondent was denied due process, but their reasons for arriving at such
conclusion are erroneous. What they seem to have overlooked is that respondent's case has
been pre-judged even prior to the start of the investigation on July 28, 2003. This is evident from
the fact that the amount of P1,437.00 or the amount which petitioners claim was embezzled
was peremptorily deducted each payday from respondent's salary on a staggered basis,
culminating on June 30, 2003, or nearly one month prior to the scheduled investigation on July
28, 2003. In doing so, petitioners have made it clear that they considered respondent as the
individual responsible for the embezzlement; thus, in petitioners' eyes, respondent was
adjudged guilty even before she could be tried the payroll deductions being her penalty and
recompense. By pre-judging respondent's case, petitioners clearly violated her right to due
process from the very beginning, and from then on it could not be expected that she would
obtain a fair resolution of her case. In a democratic system, the infliction of punishment before
trial is fundamentally abhorred. What petitioners did was clearly illegal and improper.
2. There is a difference in separation pay between a just cause (nominal damages) and an
authorized cause. Since in the case of JAKA, the employee was terminated for authorized
causes as the employer was suffering from serious business losses, the Court fixed the
indemnity at a higher amount of P50,000.00. In the case at bar, the cause for termination was
abandonment, thus it is due to the employee's fault. It is equitable under these circumstances to
order the petitioner company to pay nominal damages in the amount of P30,000.00, similar to
the case of Agabon.
3. Nominal damages is for the vindication of a right, not for compensating hours worked and not
paid. Prescinding from the foregoing, we find it necessary to reduce the amount of nominal
damages the CA awarded from P100,000.00 to P30,000.00. We cannot subscribe to the CA's
ratiocination that since respondent rendered overtime work for four years without receiving any
overtime pay, she is entitled to P100,000.00 nominal damages. Nominal damages are awarded
for the purpose of vindicating or recognizing a right and not for indemnifying a loss. Hence, the
CA should have limited the justification of the award of nominal damages to petitioners' violation
of respondent's right to due process in effecting her termination. It should not have considered
the claimed unpaid overtime pay.||| (LIBCAP Marketing Corp. v. Baquial, G.R. No. 192011, June
30, 2014)

92. Ampeloquio vs. Jaka Distribution Inc. GR No. 196936, July 2, 2014
Facts:
Petitioner works as a merchandiser for respondent company. He previously filed a case for
illegal dismissal against respondent company, and obtained a favorable judgment. Sometime in
August 2004 (and after the Labor Arbiter ordered his reinstatement in respondent company), he
resumed work and was designated as a regular merchandiser for JAKA (company). He was
thereafter assigned to other branches outside of Manila, but his wage rate remained the same,
as well as his other allowances. Petitioner, in this case, claims that he is underpaid compared to
how the other merchandisers were paid, plus he also avers that they receive allowances which
he does not receive. Respondent, on the other hand, claims that the new merchandisers are
casual or contractual employees who were contracted from other companies who supplied them

ATHENA M. SALAS | LABOR CASE DIGEST 2015 204

workers, so that the difference in salary between petitioner and them is justified since these
outsourced employees had different employers which gave them benefits based on their own
company practices.

Issues:
1. When an employee is reinstated, does his employment resume as though there was no
interruption or does he start from scratch?
2. Was the difference in salary and benefits entitlement justified between petitioner and other
merchandisers who were seasonally and/or casually employed?

Ruling:
1. The employee's years of service is deemed continuous and never interrupted. Such is
likewise the rationale for reinstatement's twin relief of full backwages. Ampeloquio is correct in
asserting that he is a senior employee compared to the other merchandisers whom he himself
designates as casual or contractual merchandisers. He is likewise senior to other regular
employees subsequently hired by JAKA, specifically two regular messenger employees which
Ampeloquio claims receive wages higher than what he is receiving from JAKA. Attached to the
recognition of seniority rights of a reinstated employee who had been illegally dismissed is the
entitlement to wages appurtenant thereto. As the sole regular merchandiser of JAKA,
Ampeloquio's reinstatement entitles him, at the minimum, to the standard minimum wage at the
time of his employment and to the wages he would have received from JAKA had he not been
illegally dismissed, as if there was no cessation of employment. Ampeloquio is likewise entitled
to any increase which JAKA may have given across the board to all its regular employees. To
repeat, Ampeloquio is not entitled to all benefits or privileges received by other employees
subsequently hired by JAKA just by the fact of his seniority in the service with JAKA.|||
2. Ampeloquio cannot likewise compare his wages to that received by "casual or contractual
merchandisers" or merchandisers who are admittedly outsourced from manpower agencies or
those who are considered seasonal employees hired only during peak season when JAKA is in
need of extra merchandisers. To say the least, these merchandisers are not, strictly speaking,
employees of JAKA, but of a service provider company which has a service contract with JAKA.
The merchandisers in this case simply perform the work at JAKA's outlets, wearing uniforms
approved by JAKA but provided by the service company who is actually their employer. There is
no employer-employee relationship between JAKA and these merchandisers. Receipt by these
merchandisers of a benefit such as transportation or meal allowance is part of the monies they
receive from their employer and embedded in the contract price of the service agreement the
employer has with JAKA.

92. Lim vs. HMR Phils Inc. GR No. 201483, August 4, 2014
Facts:

ATHENA M. SALAS | LABOR CASE DIGEST 2015 205

On February 8, 2001, petitioner Conrado A. Lim (Lim) filed a case for illegal dismissal and
money claims against respondents, HMR Philippines, Inc. (HMR) and its officers, Teresa G.
Santos-Castro, Henry G. Bunag and Nelson S. Camiller. The Labor Arbiter (LA) dismissed the
complaint for lack of merit. On April 11, 2003, the National Labor Relations Commission (NLRC)
in NLRC NCR No. 02-00926-01, reversed the LA and declared Lim to have been illegally
dismissed.
Both Lim and HMR filed their respective petitions for certiorari before the CA, docketed as CAG.R. SP No. 80379 and CA-G.R. SP No. 80630, respectively, which were consolidated. Pending
resolution of the petitions, the CA issued the Temporary Restraining Order (TRO) enjoining the
execution of the NLRC decision.
On February 7, 2007, this Court, in G.R. No. 175950-51, dismissed the petition for certiorari4
filed by HMR assailing the November 15, 2005 CA decision. Entry of judgment was ordered on
July 27, 2007.5cralawred
On September 24, 2007, Lim moved for execution.6 On November 28, 2007, the Computation
and Research Unit (CRU) of the NLRC computed the total award to amount to P2,020,053.46,7
which computed the backwages from February 3, 2001, the date of the illegal dismissal, up to
October 31, 2007, the date of actual reinstatement.
HMR opposed the computation arguing that the backwages should be computed until April 11,
2003 only, the date of promulgation of the NLRC decision, as stated in the dispositive portion of
the NLRC decision, which provided that backwages shall be reckoned from his dismissal on
February 3, 2001 up to the promulgation of this Decision. It also noted that the 10% annual
increase was computed from 1998 to 2007, instead of only from 1998 to 2000 as
decreed.8cralawred
In his Comment, Lim argued that the body of the NLRC decision explictly stated that he was
entitled to full backwages from the time he was illegally dismissed until his actual reinstatement,
which was also in accord with Article 279 of the Labor Code and all prevailing jurisprudence
On April 21, 2009, the LA issued the order10 granting the motion for execution filed by Lim.
Holding that the backwages should be reckoned until April 11, 2003 only in accordance with the
NLRC decision.
Ruling of the NLRC
Lim filed his Motion Ad Cautelam for Reconsideration or Recomputation and Partial Execution
of Monetary Award, insisting that his backwages should be computed up to his actual
reinstatement.12 On August 28, 2009, the NLRC treated the motion as an appeal and sustained
the computation of the LA, explaining that the dispositive portion was clear, and that it could not
alter or amend the amount based on the final decision of the NLRC which was affirmed by both
the CA and this Court.13 Aggrieved, petitioner filed a petition for certiorari before the CA.

Ruling of the CA
In its assailed March 30, 2012 Decision, the CA dismissed the petition. It emphasized that the
April 11, 2003 NLRC decision had long become final and executory after it was affirmed by the
Court and, as such, it may no longer be amended or corrected. While noting that the body of the

ATHENA M. SALAS | LABOR CASE DIGEST 2015 206

NLRC decision stated that petitioner was entitled to backwages until his actual reinstatement,
the CA ruled that when there was a conflict between the dispositive portion and the body of the
decision, the former must prevail as the dispositive portion was the final order, and that it was
the dispositive portion which was the subject of execution. It wrote that the fallo was clear and
unequivocal and could, therefore, be given effect without going to the body of the decision or
further interpretation or construction.
The CA found that although the NLRC had recognized that petitioner was entitled to backwages
until actual reinstatement, nonetheless, it expressly limited the computation of backwages to the
promulgation date of its decision. It wrote that the issue of whether such limitation was lawful or
improper could no longer be ventilated due to the finality of the judgment.
Issues:
1. Whether the computation of backwages should be reckoned until the promulgation of the
NLRC Decision on April 11, 2003 or until actual reinstatement?
2. Whether the petitioner is entitled to the unpaid 10% annual salary increase from 1998-2000?
3. Whether the petitioner is entitled to the 10% annual salary increase after the year 2000?
4. Whether the petitioner is entitled to holiday pay?
5. Whether the petitioner is entitled to sick leave pay?
6. Whether the respondents should be held jointly and severally liable for additional moral and
exemplary damages?
7. Whether the interest in accordance with Eastern Shipping should be awarded?

Ruling:
Backwages
It is beyond question that Lim was illegally dismissed by HMR. All that remains to be settled is
the exact amount owing to petitioner as an illegally dismissed employee.
Article 279 of the Labor Code is clear in providing that an illegally dismissed employee is
entitled to his full backwages computed from the time his compensation was withheld up to the
time of his actual reinstatement, to wit:
Art. 279. Security of tenure. In cases of regular employment, the employer shall not terminate
the services of an employee except for a just cause or when authorized by this Title. An
employee who is unjustly dismissed from work shall be entitled to reinstatement without loss of
seniority rights and other privileges and to his full backwages, inclusive of allowances, and to his
other benefits or their monetary equivalent computed from the time his compensation was
withheld from him up to the time of his actual reinstatement.
In accordance with this provision, the body of the April 11, 2003 NLRC decision expressly
recognizes that Lim is entitled to his full backwages until his actual reinstatement, as follows:

ATHENA M. SALAS | LABOR CASE DIGEST 2015 207

In fine, the act of complainant-appellant herein, do not constitute a serious misconduct as to


justify his dismissal. As such, he is, thus, entitled to reinstatement to his former position as
Assistant Technical Manager, unless such position no longer exists, in which case, he shall be
given a substantially equivalent position without loss of seniority rights. He is, likewise, entitled
to his full backwages from the time he was illegally dismissed until his actual
reinstatement.20cralawred
Nowhere in the body of the NLRC decision was there a discussion restricting the award of
backwages. Nonetheless, the fallo of the said decision limited the computation of the
backwages up to its promulgation on April 11, 2003.
Considering that the judgment decreeing the computation of backwages up to the promulgation
of the NLRC decision has long become final and executory, the key question is whether a
recomputation of backwages up to the date of the actual reinstatement of Lim would violate the
principle of immutability of judgments.

The rule is that it is the dispositive portion that categorically states the rights and obligations of
the parties to the dispute as against each other. Thus, it is the dispositive portion that must be
enforced to ensure the validity of the execution. That a judgment should be implemented
according to the terms of its dispositive portion is a long and well-established rule. A companion
to this rule is the principle of immutability of final judgments. Save for recognized exceptions, a
final judgment may no longer be altered, amended or modified, even if the alteration,
amendment or modification is meant to correct what is perceived to be an erroneous conclusion
of fact or law and regardless of what court renders it. Any attempt to insert, change or add
matters not clearly contemplated in the dispositive portion violates the rule on immutability of
judgments.
The cases of Session Delights Ice Cream and Fast Foods v. Court of Appeals (Session
Delights) and Nacar v. Gallery Frames (Nacar) shed much light on the apparent discrepancy in
the case at hand. As in the present case, both involve labor cases finding that the employees
therein were illegally dismissed. At the LA level, in awarding backwages, a precise computation
was provided from the time of illegal dismissal up to the promulgation of the LA decision.
Additionally, the dispositive portion of the LA decision in Nacar also made a declaration that
separation pay in lieu of reinstatement be computed only up to promulgation of this decision.
The LA decisions in these cases were affirmed by the NLRC and the CA and subsequently
became final and executory. At the execution stage, the computation of backwages came into
issue.
Session Delights made clear that a case for illegal dismissal is one that relates to status, where
the decision or ruling is essentially declaratory of the status and of the rights, obligations and
monetary consequences that flow from the declared status, such as, the payment of separation
pay and backwages. In execution, what is primarily implemented is the declaratory finding on
the status and the rights and obligations of the parties therein; the arising monetary
consequences from the declaration only follow as component of the parties rights and
obligations.27 The precise amount of backwages should ideally be stated in the final decision;
otherwise, the matter is for handling and computation by the LA of origin as the labor official
charged with the implementation of decisions before the NLRC.28cralawred

ATHENA M. SALAS | LABOR CASE DIGEST 2015 208

The Courts disquisition in Session Delights, also referenced with approval in Nacar, is
enlightening:
A source of misunderstanding in implementing the final decision in this case proceeds from the
way the original labor arbiter framed his decision. The decision consists essentially of two parts.
The first is that part of the decision that cannot now be disputed because it has been confirmed
with finality. This is the finding of the illegality of the dismissal and the awards of separation pay
in lieu of reinstatement, backwages, attorneys fees, and legal interests.
The second part is the computation of the awards made. On its face, the computation the labor
arbiter made shows that it was time-bound as can be seen from the figures used in the
computation. This part, being merely a computation of what the first part of the decision
established and declared, can, by its nature, be re-computed. This is the part, too, that the
petitioner now posits should no longer be re-computed because the computation is already in
the labor arbiters decision that the CA had affirmed. The public and private respondents, on the
other hand, posit that a re-computation is necessary because the relief in an illegal dismissal
decision goes all the way up to reinstatement if reinstatement is to be made, or up to the finality
of the decision, if separation pay is to be given in lieu of reinstatement.
Clearly implied from this original computation is its currency up to the finality of the labor
arbiters decision. As we noted above, this implication is apparent from the terms of the
computation itself, and no question would have arisen had the parties terminated the case and
implemented the decision at that point.
However, the petitioner disagreed with the labor arbiters findings on all counts i.e., on the
finding of illegality as well as on all the consequent awards made. Hence, the petitioner
appealed the case to the NLRC which, in turn, affirmed the labor arbiters decision. By law, the
NLRC decision is final, reviewable only by the CA on jurisdictional grounds.
It was at this point that the present case arose. Focusing on the core illegal dismissal portion of
the original labor arbiters decision, the implementing labor arbiter ordered the award recomputed; he apparently read the figures originally ordered to be paid to be the computation
due had the case been terminated and implemented at the labor arbiters level. Thus, the labor
arbiter re-computed the award to include the separation pay and the backwages due up to the
finality of the CA decision that fully terminated the case on the merits. Unfortunately, the labor
arbiters approved computation went beyond the finality of the CA decision (July 29, 2003) and
included as well the payment for awards the final CA decision had deleted specifically, the
proportionate 13th month pay and the indemnity awards. Hence, the CA issued the decision
now questioned in the present petition.
We see no error in the CA decision confirming that a re-computation is necessary as it
essentially considered the labor arbiters original decision in accordance with its basic
component parts as we discussed above. To reiterate, the first part contains the finding of
illegality and its monetary consequences; the second part is the computation of the awards or
monetary consequences of the illegal dismissal, computed as of the time of the labor arbiters
original decision.
To illustrate these points, had the case involved a pure money claim for a specific sum (e.g.
salary for a specific period) or a specific benefit (e.g. 13th month pay for a specific year) made

ATHENA M. SALAS | LABOR CASE DIGEST 2015 209

by a former employee, the labor arbiters computation would admittedly have continuing
currency because the sum is specific and any variation may only be on the interests that may
run from the finality of the decision ordering the payment of the specific sum.
In contrast with a ruling on a specific pure money claim, is a claim that relates to status (as in
this case, where the claim is the legality of the termination of the employment relationship). In
this type of cases, the decision or ruling is essentially declaratory of the status and of the rights,
obligations and monetary consequences that flow from the declared status (in this case, the
payment of separation pay and backwages and attorneys fees when illegal dismissal is found).
When this type of decision is executed, what is primarily implemented is the declaratory finding
on the status and the rights and obligations of the parties therein; the arising monetary
consequences from the declaration only follow as component of the parties rights and
obligations.
In the present case, the CA confirmed that indeed an illegal dismissal had taken place, so that
separation pay in lieu of reinstatement and backwages should be paid. How much that
separation pay would be, would ideally be stated in the final CA decision; if not, the matter is for
handling and computation by the labor arbiter of origin as the labor official charged with the
implementation of decisions before the NLRC.
Consistent with what we discussed above, we hold that under the terms of the decision under
execution, no essential change is made by a re-computation as this step is a necessary
consequence that flows from the nature of the illegality of dismissal declared in that decision. A
re-computation (or an original computation, if no previous computation has been made) is a part
of the law specifically, Article 279 of the Labor Code and the established jurisprudence on this
provision that is read into the decision. By the nature of an illegal dismissal case, the reliefs
continue to add on until full satisfaction, as expressed under Article 279 of the Labor Code. The
re-computation of the consequences of illegal dismissal upon execution of the decision does not
constitute an alteration or amendment of the final decision being implemented. The illegal
dismissal ruling stands; only the computation of monetary consequences of this dismissal is
affected and this is not a violation of the principle of immutability of final judgments.
That the amount the petitioner shall now pay has greatly increased is a consequence that it
cannot avoid as it is the risk that it ran when it continued to seek recourses against the labor
arbiters decision. Article 279 provides for the consequences of illegal dismissal in no uncertain
terms, qualified only by jurisprudence in its interpretation of when separation pay in lieu of
reinstatement is allowed. When that happens, the finality of the illegal dismissal decision
becomes the reckoning point instead of the reinstatement that the law decrees. In allowing
separation pay, the final decision effectively declares that the employment relationship ended so
that separation pay and backwages are to be computed up to that point.
Although the NLRC decision in the present case did not provide a precise computation, the
principles enunciated in Session Delights still equally apply. In Session Delights, the
computation of the LA was found to be time-bound, which implied the currency of the
computation up to the finality of the LA decision. In the present case, the NLRC declared
backwages to be reckoned up to the promulgation of its decision, which was an express
declaration of the currency of the computation up to the finality of the NLRC decision, especially
considering that HMR was ordered to reinstate immediately petitioner Lim. The decisions in

ATHENA M. SALAS | LABOR CASE DIGEST 2015 210

both cases are premised on their immediate execution, in that no question would have arisen
had the parties terminated the case and the decision implemented at that point.
As discussed above, no essential change is being made by a recomputation because such is a
necessary consequence which flows from the nature of the illegality of the dismissal. To
reiterate, a recomputation, or an original computation, if no previous computation was made, as
in the present case, is a part of the law that is read into the decision, namely, Article 279 of the
Labor Code and established jurisprudence.31 Article 279 provides for the consequences of
illegal dismissal, one of which is the payment of full backwages until actual reinstatement,
qualified only by jurisprudence when separation pay in lieu of reinstatement is allowed, where
the finality of the illegal dismissal decision instead becomes the reckoning point.
The nature of an illegal dismissal case requires that backwages continue to add on until full
satisfaction. The computation required to reflect full satisfaction does not constitute an alteration
or amendment of the final decision being implemented as the illegal dismissal ruling stands.
Thus, in the present case, a computation of backwages until actual reinstatement is not a
violation of the principle of immutability of final judgments.
The respondents aver that the recoverable backwages cannot go beyond December 26, 2007,
the date HMR offered to reinstate Lim, who allegedly refused to be reinstated and abandoned
his job.
HMR sent the petitioner a letter, dated December 22, 2007, directing him to report for work on
December 26, 2007, with an offer of separation pay in the amount of P150,000.00 in lieu of
reinstatement which he could avail of not later than December 26, 2007. Lim replied in a letter,
dated December 24, 2007, requesting for a meeting in January 2008, considering that his
counsel was out of the country; that the NLRC was still in the process of computing the amount
of the award which was necessary to consider the offer of separation pay; and that a writ of
execution had not yet been issued. HMR never responded to the petitioners request, and up to
the present, the latter has yet to be reinstated.
From the above, it is apparent that the petitioner cannot be deemed to have refused
reinstatement or to have abandoned his job. HMRs offer of reinstatement appeared superficial
and insincere considering that it never replied to the petitioners letter. It did not make any
further attempt to reinstate the petitioner either. The recoverable backwages, thus, continue to
run, and must be reckoned up until the petitioners actual reinstatement.

10% annual salary increase


Petitioner Lim argues that the LA completely failed to include in its computation the unpaid 10%
annual increase in his salary from 1998 to 2000, as stated in the fallo of the NLRC decision, and
the 10% salary increase per annum in backwages until actual reinstatement.
The pertinent portion of the fallo of the NLRC decision reads:
The Computation and Research Unit (CRU) of this Commission is hereby directed to compute
the backwages and the 10% annual increase from 1998 to 2000.
In awarding the 10% annual salary increase from 1998 to 2000, the body of the NLRC decision
explained:

ATHENA M. SALAS | LABOR CASE DIGEST 2015 211

We see no reason, therefore, why complainant-appellant herein, being a regular employee,


should be deprived of what he is entitled to under Company policy. As such, he should be paid
his unpaid 10% annual increase for the years 1998, 1999 and 2000.37cralawred
Lim is, thus, entitled to be paid his unpaid 10% annual salary increase for the years 1998-2000.
A reading of the assailed order of the LA would reveal that it made the following adjustment in
connection to the 10% annual salary increase:
2) that the base rate applicable is his salary as of February 3, 2003 inclusive of the ten percent
adjustment due at the time, or P12,500.00 plus ten percent (10%) or
P13,750.00;38chanrobleslaw
This is incorrect on two counts. First, the LA failed to include the actual unpaid 10% annual
increase from 1998-2000. The first computation of the LA, as well as the suggested computation
of respondent HMR itself, gave the correct computation of the unpaid salary increase from
1998-2000

93. Benson Industries Employees Union et al. vs. Benson Industries, Inc. GR No. 200746,
August 6, 2014
FACTS:
Respondent Benson Industries, Inc. (Benson) is a domestic corporation engaged in the
manufacturing of green coils with the brand name Lion-Tiger Mosquito Killer. On February 12,
2008, Benson sent its employees, including herein petitioners, a notice5 informing them of their
intended termination from employment, to be effected on March 15, 2008 on the ground of
closure and/or cessation of business operations. In consequence, the majority of Bensons
employees resigned.6 Meanwhile, petitioners, through Benson Industries Employees UnionALU-TUCP (Union), filed a notice of strike, claiming that the companys supposed closure was
merely a ploy to replace the union members with lower paid workers, and, as a result, increase
its profit at their expense.7 The strike did not, however, push through due to the parties
amicable settlement during the conciliation proceedings before the NCMB, whereby petitioners
accepted Bensons payment of separation pay, computed at 15 days for every year of service,
as per the parties Memorandum of Agreement8 dated April 9, 2008.9cr

This notwithstanding, petitioners proffered a claim for the payment of additional separation pay
at the rate of four (4) days for every year of service. As basis, petitioners invoked Section 1,
Article VIII of the existing collective bargaining agreement (CBA) executed by and between the
Union and Benson which states that [Benson] shall pay to any employee/laborer who is
terminated from the service without any fault attributable to him, a Separation Pay equivalent to
not less than nineteen (19) days pay for every year of service based upon the latest rate of pay
of the employee/laborer concerned.10 Benson opposed petitioners claim, averring that the
separation pay already paid to them was already more than what the law requires. Reaching an
impasse on the conflict, the parties referred the issue to voluntary arbitration, wherein the
validity of Bensons closure was brought up as well.11cralawred
VA Ruled in favor of the employees.

ATHENA M. SALAS | LABOR CASE DIGEST 2015 212

CA reversed the VA.

ISSUE:
Whether or not the employees are entitled to additional separation pay of 4 days as stated in the
CBA

RULING
The company was made to pay the 4 day additional separation pay. VA Decision was reinstated.
Closure of business may be considered as a reversal of an employers fortune whereby there is
a complete cessation of business operations and/or an actual locking-up of the doors of the
establishment, usually due to financial losses. Under the Labor Code, it is treated as an
authorized cause for termination, aimed at preventing further financial drain upon an employer
who cannot anymore pay its employees since business has already stopped. As a form of
recompense, the employer is required to pay its employees separation benefits, except when
the closure is due to serious business losses.
While serious business losses generally exempt the employer from paying separation benefits,
it must be pointed that the exemption only pertains to the obligation of the employer under
Article 297 of the Labor Code. This is because of the laws express parameter that mandates
payment of separation benefits in case of closures or cessation of operations of establishment
or undertaking not due to serious business losses or financial reverses. The policy distinction
underlying Article 297 that is, the distinction between closures due to serious business losses
and those which are not was deftly discussed by the Court in the case of Cama v. Jonis Food
Services, Inc.,21 as follows:chanRoblesvirtualLawlibrary
For a similar exemption to obtain against a contract, such as a CBA, the tenor of the parties
agreement ought to be similar to the laws tenor. When the parties, however, agree to deviate
therefrom, and unqualifiedly covenant the payment of separation benefits irrespective of the
employers financial position, then the obligatory force of that contract prevails and its terms
should be carried out to its full effect. Verily, it is fundamental that obligations arising from
contracts have the force of law between the contracting parties and thus should be complied
with in good faith.
As in all contracts, the parties in a CBA may establish such stipulations, clauses, terms and
conditions as they may deem convenient provided these are not contrary to law, morals, good
customs, public order or public policy. Thus, where the CBA is clear and unambiguous, it
becomes the law between the parties and compliance therewith is mandated by the express
policy of the law.28

In this case, it is undisputed that a CBA was forged by the employer, Benson, and its
employees, through the Union, to govern their relations effective July 1, 2005 to June 30, 2010.
It is equally undisputed that Benson agreed to and was thus obligated under the CBA to pay its

ATHENA M. SALAS | LABOR CASE DIGEST 2015 213

employees who had been terminated without any fault attributable to them separation benefits
at the rate of 19 days for every year of service.
The postulation that Benson had closed its establishment and ceased operations due to serious
business losses cannot be accepted as an excuse to clear itself of any liability since the ground
of serious business losses is not, unlike Article 297 of the Labor Code, considered as an
exculpatory parameter under the aforementioned CBA.

94. Montinola vs. Philippine Airlines, GR No. 198656, Sept. 8, 2014


FACTS
Montinola was employed as a flight attendant of Philippine Airlines (PAL) since 1996. On
January 29, 2008, Montinola and other flight crew members were subjected to custom searches
in Honolulu, Hawaii, USA. Items from the airline were recovered from the flight crew by customs
officials. Nancy Graham (Graham), US Customs and Border Protection Supervisor, sent an
email to PAL regarding the search. The email7 contained a list of PAL flight crew members.
PAL conducted an investigation. Montinola was among those implicated because she was
mentioned in Grahams email.9 On February 1, 2008, PALs Cabin Services Sub-Department
required Montinola to comment on the incident. She gave a handwritten explanation three days
after, stating that she did not take anything from the aircraft. She also committed to give her full
cooperation should there be any further inquiries on the matter.
During the hearing, Montinola admitted that in Honolulu, US customs personnel conducted a
search of her person. At that time, she had in her possession only the following food items:
cooked camote, 3-in-1 coffee packs, and Cadbury hot chocolate.
PAL, through Senior Assistant Vice President for Cabin Services Sub-Department Sylvia C.
Hermosisima, found Montinola guilty of 11 violations of the companys Code of Discipline and
Government Regulation. She was meted with suspension for one (1) year without pay.
Montinola asked for a reconsideration. Hermosisima, however, denied her motion for
reconsideration a month after.
Labor Arbiter: Suspension was illegal.
In addition, the Labor Arbiter awarded moral damages in the amount of P100,000.00 and
exemplary damages amounting to P100,000.00 for the following reasons:
The office observes that the records are replete with substantial evidence that the
circumstances leading to complainants one-year suspension without pay are characterized by
arbitrariness and bad faith on the part of respondents.
The Labor Arbiter also awarded attorneys fees to Montinola because she was forced to litigate
and incur expenses to protect [her] rights.
NLRC Affirmed. CA AFFIRMED. Court of Appeals deleted the moral and exemplary damages
and attorneys fees stating that: to the award of moral damages, not every employee who is
illegally dismissed or suspended is entitled to damages.

ATHENA M. SALAS | LABOR CASE DIGEST 2015 214

ISSUE:
Whether or not Montinola is entitled to moral and exemplary damages awarded by the LA.

RULING:
Montinola is entitled to moral and exemplary damages. She is also entitled to attorneys fees.
The Labor Code provides:
Art. 279. Security of Tenure In cases of regular employment, the employer shall not terminate
the services of an employee except for a just cause or when authorized by this Title. An
employee who is unjustly dismissed from work shall be entitled to reinstatement without loss of
seniority rights and other privileges and to full backwages, inclusive of allowances, and to his
other benefits or their monetary equivalent computed from the time his compensation was
withheld from him up to the time of his actual reinstatement.
Security of tenure of workers is not only statutorily protected, it is also a constitutionally
guaranteed right.61 Thus, any deprivation of this right must be attended by due process of
law.62 This means that any disciplinary action which affects employment must pass due
process scrutiny in both its substantive and procedural aspects.
Suspension from work is prima facie a deprivation of this right. Thus, termination and
suspension from work must be reasonable to meet the constitutional requirement of due
process of law. It will be reasonable if it is based on just or authorized causes enumerated in the
Labor Code.65cralawred
The procedure can be summarized in this manner. First, the employer must furnish the
employee with a written notice containing the cause for termination. Second, the employer must
give the employee an opportunity to be heard. This could be done either through a position
paper or through a clarificatory hearing.66 The employee may also be assisted by a
representative or counsel. Finally, the employer must give another written notice apprising the
employee of its findings and the penalty to be imposed against the employee, if any.67 In labor
cases, these requisites meet the constitutional requirement of procedural due process, which
contemplates notice and opportunity to be heard before judgment is rendered, affecting ones
person or property.68cralawred
In this case, PAL complied with procedural due process as laid out in Article 277, paragraph (b)
of the Labor Code. PAL issued a written notice of administrative charge, conducted a
clarificatory hearing, and rendered a written decision suspending Montinola. However, we
emphasize that the written notice of administrative charge did not serve the purpose required
under due process. PAL did not deny her allegation that there would be a waiver of the
clarificatory hearing if she insisted on a specific notice of administrative charge. With Montinola
unable to clarify the contents of the notice of administrative charge, there were irregularities in
the procedural due process accorded to her.
Moreover, PAL denied Montinola substantial due process.
Just cause has to be supported by substantial evidence. Substantial evidence, or such relevant
evidence as a reasonable mind might accept as adequate to support a conclusion,69 is the

ATHENA M. SALAS | LABOR CASE DIGEST 2015 215

quantum of evidence required in administrative bodies such as the National Labor Relations
Commission.
PAL, however, merely relied on these pieces of information in finding administrative liability
against Montinola:
1) A list of offenses found in PALs Code of Discipline that Montinola allegedly violated;

2) A list of flight crew members that were checked at the Honolulu airport; and

3) A list of all items confiscated from all these flight crew members.
The lists are not sufficient to show the participation of any of the flight crew members, least of all
Montinola. None of the evidence presented show that the customs officials confiscated any of
these items from her.
The employee is entitled to moral damages when the employer acted a) in bad faith or fraud; b)
in a manner oppressive to labor; or c) in a manner contrary to morals, good customs, or public
policy.
Bad faith implies a conscious and intentional design to do a wrongful act for a dishonest
purpose or moral obliquity.73Cathay Pacific Airways v. Spouses Vazquez74 established that
bad faith must be proven through clear and convincing evidence.75 This is because [b]ad faith
and fraud . . . are serious accusations that can be so conveniently and casually invoked, and
that is why they are never presumed. They amount to mere slogans or mudslinging unless
convincingly substantiated by whoever is alleging them.76 Here, there was clear and
convincing evidence of bad faith adduced in the lower tribunals.
PALs actions in implicating Montinola and penalizing her for no clear reason show bad faith.
PALs denial of her request to clarify the charges against her shows its intent to do a wrongful
act for moral obliquity. If it were acting in good faith, it would have gathered more evidence from
its contact in Honolulu or from other employees before it started pointing fingers. PAL should not
have haphazardly implicated Montinola and denied her livelihood even for a moment.

II.
Montinola is also entitled to exemplary damages.
Under Article 2229 of the Civil Code, [e]xemplary or corrective damages are imposed, by way
of example or correction for the public good, in addition to the moral, temperate, liquidated or
compensatory damages. As this court has stated in the past: Exemplary damages are
designed by our civil law to permit the courts to reshape behaviour that is socially deleterious in
its consequence by creating negative incentives or deterrents against such
behaviour.83cralawred

ATHENA M. SALAS | LABOR CASE DIGEST 2015 216

It is socially deleterious for PAL to suspend Montinola without just cause in the manner suffered
by her. Hence, exemplary damages are necessary to deter future employers from committing
the same acts.
III.
Montinola is also entitled to attorneys fees.
Article 2208 of the Civil Code enumerates the instances when attorneys fees can be awarded.
This case qualifies for the first, second, and seventh reasons why attorneys fees are awarded
under the Civil Code.

First, considering that we have awarded exemplary damages in this case, attorneys fees can
likewise be awarded.
Second, PALs acts and omissions compelled Montinola to incur expenses to protect her rights
with the National Labor Relations Commission and the judicial system. She went through four
tribunals, and she was assisted by counsel. These expenses would have been unnecessary if
PAL had sufficient basis for its decision to discipline Montinola.
Finally, the action included recovery for wages. To bring justice to the illegal suspension of
Montinola, she asked for backwages for her year of suspension.

95. NORTHWEST AIRLINES, INC., vs. MA. CONCEPCION M. DEL ROSARIO, G.R. No.
157633. September 10, 2014

FACTS:
Respondent Del Rosario was one of the Manila-based flight attendants for Northwest Airlines
and was assigned at the Business Class Section of the Airline. Gamboa, one of the flight
attendants assigned at the first class section of the same airline had a blunt wine bottle opener
and asked her runner to borrow one from the other flight attendants who happen to be the
respondent. Del Rosario told the runner that a flight attendant who was not ready with a wine
bottle oepener has no business being working at the First Class Section. Unfortunately, this
comment reached the ears of Gamboa and resulted to a heated exchange of words between
the two. This happened while the passengers were already on board the aircraft. Escano,
another flight attendant tried to pacify but to no avail, resorted to call the attention of Morales,
the Assistant Base Manager to pacify them. Morales tried to pacify them and ordered that they
take the argument somewhere else where there are no passengers. Morales further told them
that they are allowed to continue to fly on the same flight on the condition that they would have
to stay away from each other during the entire flight. Del Rosario was not willing to commit
herself to do so that she decided not to allow both of them on the same flight and furnished
them a Notive of Removal from Service.

ATHENA M. SALAS | LABOR CASE DIGEST 2015 217

An investigation was conducted. Thereafter, Del Rosario was informed of her termination from
service on the ground that although there was no physical contact between her and Gamboa,
fighting was strictly prohibited by Northwest Airline that would entail dismissal from service even
committed for the first time. Del Rosario filed a complaint before the Labor Arbiter for Illegal
Dismissal against Nortwest Airline. The decision of the Labor Arbiter was in favor of Northwest.
Del Rosario appealed to the NLRC. The decision of the Labor Arbiter was reversed by the
NLRC in its decision declaring that the incident between her and Gamboa could not be
considered as synonymous with fighting as the activity prohibited by Northwest's Rules of
Conduct. Northwest elevated the decision to the CA alleging. CA sustained the decision of the
NLRC and ruled that Del Rosarios conduct does not amount to serious misconduct because the
heated exchange of words between Gamboa and respondend did not come withing the
definition of the word fighting. Hence, this petition.

ISSUE:
Whether or not the dismissal of Del Rosario was valid.

HELD:
The Court AFFIRMS the decision of the CA.
As provided in Article 282 of the Labor Code, an employer may terminate an employee for a just
cause, to wit:
Art. 282. TERMINATION BY EMPLOYER.
An employer may terminate an employee for any of the following causes:
(a) Serious misconduct or willful disobedience by the employee of the lawful orders of his
employer or representative in connection with his work;
(b) Gross and habitual neglect by the employee of his duties;
(c) Fraud or willful breach by the employee of the trust reposed in him by his employer or duly
authorized representative;
(d) Commission of a crime or offense by the employee against the person of his employer or
any immediate member of his family or his duly authorized representative; and
(e) Other causes analogous to the foregoing.

Misconduct or improper behavior, to be a just cause for termination of employment, must: (a) be
serious; (b) relate to the performance of the employee's duties; and (c) show that the employee
has become unfit to continue working for the employer. There is no doubt that the last two
elements of misconduct were present in the case of Del Rosario. The cause of her dismissal
related to the performance of her duties as a flight attendant, and she became unfit to continue
working for Northwest. However, the incident involving Del Rosario and Gamboa could not be
justly considered as akin to the fight contemplated by Northwest. In the eyes of the NLRC, Del

ATHENA M. SALAS | LABOR CASE DIGEST 2015 218

Rosario and Gamboa were arguing but not fighting. The understanding of fight as one that
required physical combat was absent during the incident. Moreover, even assuming arguendo
that the incident was the kind of fight prohibited by Northwest's Rules of Conduct, the same
could not be considered as of such seriousness as to warrant Del Rosario's dismissal from the
service. The gravity of the fight, which was not more than a verbal argument between them, was
not enough to tarnish or diminish Northwest's public image.

96. Mount Carmel College Employees Union et al., vs. Mount Carmel College Inc. GR No.
187621, Sept. 24, 2014

FACTS:
Petitioners are high school academic and non-academic personnel of the Mount Carmel College
Inc. They were informed of their retrenchment due to the closure of the elementary and high
school department of the school. The petitioners contend that such closure was merely a
subterfuge of their termination due to their union activities. The petitioners contend that it was
motivated by ill-will since the school reopened the elementary and high school departments 2
months after with newly hired teachers, thus claiming for the remaining separation pay
differentials. The respondents denied committing any unfair labor practice as the closure of their
elementary and high school department was due to financial losses it suffered due to the
decline of its enrollment, coupled with the demand of increase in salaried from the teachers
which gave the school no other choice but to close their elementary and high-school
departments.
The Labor Arbiter ruled that the petitioners were illegally dismissed, as the losses of the
respondent were not serious enough to warrant a closure of business and their financial
statements even showed a net surplus, awarding petitioners with separation pay in lieu of
reinstatement and attorneys fees. It further ruled that the respondents committed unfair labor
practice. The respondent appealed the same to the NLRC. The NLRC ruled that the petitioners
were not illegally dismissed; that the retrenchment of the petitioners was an exercise of
management prerogative as its financial status justifies the same. The petitioners questioned
the bond posted by the respondents however the NLRC ruled that the failure to a copy of the
appeal bond and other documents to the Appeal Memorandum furnished to the petitioners is a
minor defect, that they acted in good faith when the procured the bond from CBIC, which was
blacklisted but thereafter was accredited again by the SC, making its bond good. Alleging grave
abuse of discretion, the CA sustained the ruling of the NLRC, that there is no factual basis in the
allegation that the school closed down for the purpose of union busting and that the school
cannot be compelled to continue to operate at a loss as shown in its financial statements.
ISSUE:
1. Whether or not the CA correctly ruled that the NLRC did not commit any grave abuse of
discretion when it allowed the respondent's appeal despite the blacklisting of CBIC at the time it
issued the appeal bond.
2. Whether or not the CA committed grave abuse of discretion when it sustained the NLRCs
ruling of valid retrenchment.

ATHENA M. SALAS | LABOR CASE DIGEST 2015 219

HELD:
1. In this regard, the Court has ruled that in a judgment involving a monetary award, the appeal
shall be perfected only upon: (1) proof of payment of the required appeal fee; (2) posting of a
cash or surety bond issued by a reputable bonding company; and (3) filing of a memorandum of
appeal.
In this case, it was not disputed that at the time CBIC issued the appeal bond, it was already
blacklisted by the NLRC. Good faith, however, is not an excuse for setting aside the mandatory
and jurisdictional requirement of the law. The condition of posting a cash or surety bond is not a
meaningless requirement it is meant to assure the workers that if they prevail in the case,
they will receive the money judgment in their favor upon the dismissal of the former's appeal.
Such aim is defeated if the bond issued turned out to be invalid due to the surety company's
expired accreditation. Much more in this case where the bonding company was blacklisted at
the time it issued the appeal bond.
2. Retrenchment, as an authorized cause for the dismissal of employees, finds basis in Article
283 24 of the Labor Code, which states:
Art. 283. Closure of establishment and reduction of personnel. The employer may also
terminate the employment of any employee due to the installation of labor-saving devices,
redundancy, retrenchment to prevent losses or the closing or cessation of operation of the
establishment or undertaking unless the closing is for the purpose of circumventing the
provisions of this Title, by serving a written notice on the workers and the Ministry of Labor and
Employment at least one (1) month before the intended date thereof. . . . . In case of
retrenchment to prevent losses and in cases of closures or cessation of operations of
establishment or undertaking not due to serious business losses or financial reverses, the
separation pay shall be equivalent to one (1) month pay or at least one-half (1/2) month pay for
every year of service, whichever is higher. A fraction of at least six (6) months shall be
considered one (1) whole year.
Standards have been laid down by the Court in order to prevent its abuse by an employer, to
wit:
(1) That retrenchment is reasonably necessary and likely to prevent business losses which, if
already incurred, are not merely de minimis, but substantial, serious, actual and real, or if only
expected, are reasonably imminent as perceived objectively and in good faith by the employer;
(2) That the employer served written notice both to the employees and to the Department of
Labor and Employment at least one month prior to the intended date of retrenchment;
(3) That the employer pays the retrenched employees separation pay equivalent to one (1)
month pay or at least one-half (1/2) month pay for every year of service, whichever is higher;
(4) That the employer exercises its prerogative to retrench employees in good faith for the
advancement of its interest and not to defeat or circumvent the employees' right to security of
tenure; and
(5) That the employer used fair and reasonable criteria in ascertaining who would be dismissed
and who would be retained among the employees, such as status, efficiency, seniority, physical
fitness, age, and financial hardship for certain workers.

ATHENA M. SALAS | LABOR CASE DIGEST 2015 220

The burden of proving that the termination of services is for a valid or authorized cause rests
upon the employer. In termination by retrenchment, not every loss incurred or expected to be
incurred by an employer can justify retrenchment. The employer must prove, among others, that
the losses are substantial and that the retrenchment is reasonably necessary to avert such
losses.
In this case, while the respondent may have presented its Financial Statements, the
respondent, nevertheless, failed to establish with reasonable certainty that the proportion of its
revenues are largely expended for its elementary and high school personnel salaries, wages
and other benefits. The expenses for the elementary and high school departments were not set
out in detail and instead, were lumped together with the college department. Such detail
becomes material in the light of the respondent's claim that the personnel expenses for the
elementary and high school departments were "eating into" the portion of its budget allocated
for other purposes. There could be no practical basis from which the respondent's claim finds
support. Aside from this, the respondent failed to present any proof establishing how the
continued operations of the elementary and high school departments has become
impracticable. Finally, on the petitioners' allegation that the closure and their retrenchment
amounted to unfair labor practice, suffice it to say that the petitioners failed to discharge its
burden of proving that the retrenchment was motivated by ill will, bad faith or malice, or that it
was aimed at interfering with their right to self-organize.

97. Radio Mindanao Network, Inc. vs. Amurao III, GR No. 167225, October 22, 2014
Facts:
On February 16, 1989, petitioner Radio Mindanao Network, Inc. (RMN) hired respondent
Michael Maximo R. Amurao III (Michael) as a radio broadcaster for its DWKC-FM station and
production manager for its metropolitan radio operations at a monthly salary of P28,400.00.
Years later, RMN decided to reformat and restructure the programming of its DWKC-FM station
to meet the demands of the broadcasting industry. On April 25, 2002, the president of RMN met
with Michael and other personnel of the station to inform them of the managements decision,
advising them that the reformatting and restructuring of the stations programs would
necessarily affect their employment; but assuring that they would be paid their retirement pay
and other benefits. To formalize the discussions had in their meeting, RMN furnished Michael
and other personnel separate letters dated May 14, 2002. However, Michael and the other
personnel refused to sign in receipt when the letters were served on them. Not long after,
however, they accepted the offer of RMN and executed affidavits relinquishing all their claims
against the employer. 5 months after receiving his benefits and his execution of the quitclaim,
Michael filed a complaint against RMN for illegal dismissal with money claims in the National
Labor Relations Commission (NLRC).
Labor Arbiter rendered a decision declaring the dismissal of Michael as illegal, holding the
quitclaim Michael signed as void because it was not voluntarily executed.
RMN appealed to the NLRC, contending that the quitclaim signed in its favor was valid and
binding because it represented a voluntary and reasonable settlement of Michaels claims; and

ATHENA M. SALAS | LABOR CASE DIGEST 2015 221

that Michael was estopped from filing the illegal dismissal case against it. NLRC held that the
quitclaim was null and void for not being voluntarily executed. RMN then appealed to the CA,
and the CA denied due course to the petition and dismissed it for lack of merit.
Issue:
Whether or not the Affidavit of Release/Quitclaim executed by Michael was valid and binding;
and
Ruling:
The Court finds and considers the CAs ruling unfounded.
Not all quitclaims are per se invalid or against public policy. A quitclaim is invalid or contrary to
public policy only: (1) where there is clear proof that the waiver was wrangled from an
unsuspecting or gullible person; or (2) where the terms of settlement are unconscionable on
their face. In instances of invalid quitclaims, the law steps in to annul the questionable waiver.
Indeed, there are legitimate waivers that represent the voluntary and reasonable settlements of
laborers claims that should be respected by the Court as the law between the parties. Where
the party has voluntarily made the waiver, with a full understanding of its terms as well as its
consequences, and the consideration for the quitclaim is credible and reasonable, the
transaction must be recognized as a valid and binding undertaking, and may not later be
disowned simply because of a change of mind. A waiver is essentially contractual.
In our view, the requisites for the validity of Michaels quitclaim were satisfied. Firstly, Michael
acknowledged in his quitclaim that he had read and thoroughly understood the terms of his
quitclaim and signed it of his own volition. Being a radio broadcaster and production manager,
he occupied a highly responsible position in the company. It would be implausible to hold,
therefore, that he could be easily duped into simply signing away his rights. Secondly, the
settlement pay of P311,922.00 was credible and reasonable considering that Michael did not
even assail such amount as unconscionably low, or even state that he was entitled to a higher
amount. Thirdly, that he was required to sign the quitclaim as a condition to the release of the
settlement pay did not prove that its execution was coerced. Having agreed to part with a
substantial amount of money, RMN took steps to protect its interest and obtain its release from
all obligations once it paid Michael his settlement pay, which it did in this case.
With the quitclaim having been freely and voluntarily signed, RMN was released and absolved
from any liability in favor of Michael. Suffice it to say that the quitclaim is ineffective in barring
recovery of the full measure of an employees rights only when the transaction is shown to be
questionable and the consideration is scandalously low and inequitable. Such is not true here.

98. Imasen Philippine Manufacturing Corp., vs. Alcon et al., GR No. 194884, October 22,
2014
Facts:
Petitioner Imasen Philippine Manufacturing Corporation is a domestic corporation engaged in
the manufacture of auto seat-recliners and slide-adjusters. It hired the respondents as manual
welders in 2001. On October 5, 2002, the respondents reported for work on the second shift from 8:00 pm to 5:00 am of the following day. At around 12:40 am, Cyrus A. Altiche, Imasen's

ATHENA M. SALAS | LABOR CASE DIGEST 2015 222

security guard on duty, went to patrol and inspect the production plant's premises. When Altiche
reached Imasen's Press Area, he heard the sound of a running industrial fan. Intending to turn
the fan off, he followed the sound that led him to the plant's "Tool and Die" section. At the "Tool
and Die" section, Altiche saw the respondents having sexual intercourse on the floor, using a
piece of carton as mattress. Altiche immediately went back to the guard house and relayed what
he saw to Danilo S. Ogana, another security guard on duty.
On October 14, 2002, Imasen issued the respondents separate interoffice memoranda
informing them of Altiche's report on the October 5, 2002 incident and directing them to submit
their individual explanation. The respondents complied with the directive; they claimed that they
were merely sleeping in the "Tool and Die" section at the time of the incident. They also claimed
that other employees were near the area, making the commission of the act charged
impossible.
On December 4, 2002, Imasen issued the respondents separate interoffice memoranda
terminating their services. It found the respondents guilty of the act charged which it considered
as "gross misconduct contrary to the existing policies, rules and regulations of the company."
On December 5, 2002, the respondents filed before the LA the complaint for illegal dismissal.
The respondents maintained their version of the incident. The LA found the respondents'
dismissal valid, i.e., for the just cause of gross misconduct and with due process. The LA
additionally pointed out that the respondents did not show any ill motive or intent on the part; of
Altiche and Ogano sufficient to render their accounts of the incident suspicious.
The CA nullified the NLRC's ruling. The CA disagreed with the conclusion that the respondents'
sexual intercourse inside company premises constituted serious misconduct that the Labor
Code considers sufficient to justify the penalty of dismissal. The CA pointed out that the
respondents' act, while provoked by "reckless passion in an inviting environment and time," was
not done with wrongful intent or with the grave or aggravated character that the law requires. To
the CA, the penalty of dismissal is not commensurate to the respondents' act, considering
especially that the respondents had not committed any infraction in the past. Accordingly, the
CA reduced the respondents' penalty to a three-month suspension.
Issue:
The sole issue for this Court's resolution is whether the respondents' infraction engaging in
sexual intercourse inside company premises during work hours amounts to serious
misconduct within the terms of Article 282 (now Article 296) of the Labor Code justifying their
dismissal.
Ruling:
The just causes for dismissing an employee are provided under Article 282 (now Article 296) of
the Labor Code. Under Article 282(a), serious misconduct by the employee justifies the
employer in terminating his or her employment.
Misconduct is defined as an improper or wrong conduct. It is a transgression of some
established and definite rule of action, a forbidden act, a dereliction of duty, willful in character,
and implies wrongful intent and not mere error in judgment. To constitute a valid cause for the
dismissal within the text and meaning of Article 282 of the Labor Code, the employee's

ATHENA M. SALAS | LABOR CASE DIGEST 2015 223

misconduct must be serious, i.e., of such grave and aggravated character and not merely trivial
or unimportant.
Additionally, the misconduct must be related to the performance of the employee's duties
showing him to be unfit to continue working for the employer. Further, and equally important
and required, the act or conduct must have been performed with wrongful intent.c
To summarize, for misconduct or improper behavior to be a just cause for dismissal, the
following elements must concur: (a) the misconduct must be serious; (b) it must relate to the
performance of the employee's duties showing that the employee has become unfit to continue
working for the employer; and (c) it must have been performed with wrongful intent.
After due consideration, we find the NLRC legally correct and well within its jurisdiction when it
affirmed the validity of the respondents' dismissal on the ground of serious misconduct. Sexual
acts and intimacies between two consenting adults belong, as a principled ideal, to the realm of
purely private relations. Whether aroused by lust or inflamed by sincere affection, sexual acts
should be carried out at such place, time and circumstance that, by the generally accepted
norms of conduct, will not offend public decency nor disturb the generally held or accepted
social morals. Under these parameters, sexual acts between two consenting adults do not have
a place in the work environment.

Indisputably, the respondents engaged in sexual intercourse inside company premises and
during work hours. These circumstances, by themselves, are already punishable misconduct.
Added to these considerations, however, is the implication that the respondents did not only
disregard company rules but flaunted their disregard in a manner that could reflect adversely on
the status of ethics and morality in the company. Under these factual premises and in the
context of legal parameters we discussed, we cannot help but consider the respondents'
misconduct to be of grave and aggravated character so that the company was justified in
imposing the highest penalty available dismissal. Their infraction transgressed the bounds of
socially and morally accepted human public behavior, and at the same time showed brazen
disregard for the respect that their employer expected of them as employees. By their
misconduct, the respondents, in effect, issued an open invitation for others to commit the same
infraction, with like disregard for their employer's rules, for the respect owed to their employer,
and for their co-employees' sensitivities. Taken together, these considerations reveal a depraved
disposition that the Court cannot but consider as a valid cause for dismissal.

ATHENA M. SALAS | LABOR CASE DIGEST 2015 224

15. SUSPENSION OF BUSINESS OPERATIONS


99. Mindanao Terminal & Brokerage Service Inc et al., vs. Nagkahiusang Mamumuo sa
Minterbro-Southern Phils Federation of Labor, G.R. No. 174300, December 5, 2012

Facts:
Minterbro, the employer-company, is engaged in arrastre and stevedoring services. On April 14,
1997, it temporarily ceased its operation because its port was in a bad shape then. On August 1,
1997, the employer formally informed DOLE of the temporary cessation of operation up to
December 16, 1997; repair works on the pier were commenced on the same day.
The employees, thru its union, complained of constructive dismissal and consequently sought
for separation benefits. They argued that Minterbro did not operate for more than six months
which constitutes dismissal thereby entitling them to their separation claims.
In denying complainants their separation benefits, the Executive Labor Arbiter considered the
period embraced within August 1, 1997, when Minterbro formally informed the DOLE of the
temporary cessation of operation up to December 16, 1997, when it was issued a certificate
declaring the wharf safe and ready for operations and December 22-28, 1997, when the
employer-company serviced a vessel MV Uranus which obviously did not exceed six (6)
months, thus denying complainants their monetary benefits. Incidentally, the period reckoned is
incorrect.
Issue:
WON the suspension of business operations was valid?

Ruling:
No, because the suspension of business operations herein exceeded six months.
Under Article 286 of the Labor Code, the bona fide suspension of the operation of a business or
undertaking for a period not exceeding six months shall not terminate employment.
Consequently, when the bona fide suspension of the operation of a business or undertaking
exceeds six months, then the employment of the employee shall be deemed terminated. By the
same token and applying said rule by analogy, if the employee was forced to remain without
work or assignment for a period exceeding six months, then he is in effect constructively
dismissed.
The union members/employees were not given work starting April 14, 1997 and that more than
six months have elapsed after the union members were laid off when the next vessel was
serviced at the Minterbro pier on December 22 to 28, 1997.
A lay-off, used interchangeably with "retrenchment," is a recognized prerogative of
management. When a lay-off is temporary, the employment status of the employee is not
deemed terminated, but merely suspended. Article 286 of the Labor Code provides, in part, that
the bona fide suspension of the operation of the business or undertaking for a period not
exceeding six months does not terminate employment.

ATHENA M. SALAS | LABOR CASE DIGEST 2015 225

100. Leopard Security & Investigation Agency vs. Quitoy, G.R. No. 186344, Feb. 20, 2013

Facts:
Quitoy, et. al (respondents) were assigned to the different branches of its only client in Cebu
City (Union Bank) by petitioner Leopard Security & Investigation Agency (LSIA). On April 1,
2005, Union Bank decided to terminate the services of LSIA, effective at the end of the same
month. LSIA only informed the guards a day before the effectivity of said termination made by
Union Bank. On May 10, 2005 LSIA ordered the guards to report to its main office in
Mandaluyong City.
The guards did not heed to the said order. Few days prior however, the guards had already filed
a complaint for illegal dismissal (May 3, 2005).
Issue:
WON the floating status of the guards per se amounted to illegal dismissal.
Ruling:
No.
Applying Article 286 of the Labor Code of the Philippines by analogy, this Court has repeatedly
recognized that security guards may be temporarily sidelined by their security agency as their
assignments primarily depend on the contracts entered into by the latter with third parties.
Temporary "off-detail" or "floating status" is the period of time when security guards are in
between assignments or when they are made to wait after being relieved from a previous post
until they are transferred to a new one. It takes place when, as here, the security agencys
clients decide not to renew their contracts with the agency, resulting in a situation where the
available posts under its existing contracts are less than the number of guards in its roster. For
as long as such temporary inactivity does not continue for a period exceeding six months, it has
been ruled that placing an employee on temporary "off-detail" or "floating status" is not
equivalent to dismissal.

In the case at bench, respondents were informed on 29 April 2005 that they were going to be
relieved from duty as a consequence of the 30 April 2005 expiration of the security service
contract between Union Bank and LSIA. While respondents lost no time in immediately filing
their complaint, the record equally shows that they were directed by LSIA to report for work at its
Mandaluyong City office on 10 May 2005 or a mere ten days from the time the former were
effectively sidelined. Considering that a security guard is only considered illegally dismissed
from service when he is sidelined from duty for a period exceeding six months we find that the

ATHENA M. SALAS | LABOR CASE DIGEST 2015 226

CA correctly upheld the NLRCs ruling that respondents were not illegally dismissed by LSIA.
Parenthetically, said ruling is binding on respondents who did not appeal either the decision
rendered by the NLRC or the CA in line with the entrenched procedural rule in this jurisdiction
that a party who did not appeal cannot assign such errors as are designed to have the judgment
modified.

101. SKM Art Craft Corp., vs. Bauca, et al., GR No. 171282, November 27, 2013

Facts:
The 23 respondents in G.R. No. 171282 were employed by petitioner SKM Art Craft Corporation
which is engaged in the handicraft business. On April 18, 2000, around 1:12 a.m., a fire
occurred at the inspection and receiving/repair/packing area of petitioner's premises in
Intramuros, Manila. The fire investigation report stated that the structure and the beach rubber
building were totally damaged. Also burned were four container vans and a trailer truck. The
estimated damage was P22 million. On May 8, 2000, petitioner informed respondents that it will
suspend its operations for six months, effective May 9, 2000.
On May 16, 2000, only eight days after receiving notice of the suspension of petitioner's
operations, the 23 respondents (and other co-workers) filed a complaint for illegal dismissal.
They alleged that there was discrimination in choosing the workers to be laid off and that
petitioner had discovered that most of them were members of a newly-organized union.
Petitioner denied the claim of illegal dismissal and said that Article 286 of the Labor Code allows
the bona fide suspension of a business or undertaking for a period not exceeding six months.
Petitioner claimed that the fire cost it millions in losses and that it is impossible to resume its
normal operations for a significant period of time.
The Labor Arbiter ruled that the employees were illegally dismissed, on the ground that the
petitioner failed to admit them back to work after the expiration of the 6-month period, and
ordered petitioner to pay them backwages amounting to P59,918 each. The NLRC reversed the
decision of the LA on the ground that it was premature to file a case for illegal dismissal only 8
days into the suspension of operations considering the 6-month period had not yet lapsed. It
ordered reinstatement but without payment of backwages. On appeal to the CA, the CA ruled
that the petitioner failed to prove that the suspension of its operations was bona fide.
Issues:
(1) WON the employees were illegally dismissed considering the following:

ATHENA M. SALAS | LABOR CASE DIGEST 2015 227

a.) The complaint for illegal dismissal was filed only 8 days after notice of suspension of
operations
b.) The petitioner failed to admit the respondents back to work after the lapse of the 6-month
period
(2) WON the suspension of business operations of SKM Art Craft was bona fide
Ruling:
On the issue of illegal dismissal, while we agree with the NLRC that the suspension of
petitioner's operation is valid, the Labor Arbiter and the CA are correct that respondents were
illegally dismissed since they were not recalled after six months, after the bona fide suspension
of petitioner's operations.
It is admitted that petitioner's premises was burned on April 18, 2000. Petitioner also submitted
pictures of its premises after the fire, the certification by the Barangay Chairman that petitioner's
factory was burned, and the fire investigation report of the Bureau of Fire Protection. To prove
the damages, petitioner submitted a list of burned machines, its inventory for April 2000 and the
fire investigation report which stated that the estimated damage is P22 million.
We therefore agree with the NLRC that petitioner's suspension of operations is valid because
the fire caused substantial losses to petitioner and damaged its factory. On this point, we
disagree with the CA that petitioner failed to prove that its suspension of operations is bona fide.
The list of materials burned was not the only evidence submitted by petitioner. It was
corroborated by pictures and the fire investigation report, and they constitute substantial
evidence of petitioner's losses.
Under Article 286 of the Labor Code, the bona fide suspension of the operations of a business
or undertaking for a period not exceeding six months shall not terminate employment. Article
286 provides,
ART. 286.When employment not deemed terminated. The bona fide suspension of the
operations of a business or undertaking for a period not exceeding six (6) months, or the
fulfillment by the employee of a military or civic duty shall not terminate employment.
In all such cases, the employer shall reinstate the employee to his former position without loss
of seniority rights if he indicates his desire to resume his work not later than one (1) month from
the resumption of operations of his employer or from his relief from the military or civic duty.
The NLRC correctly noted that the complaint for illegal dismissal filed by respondents was
premature since it was filed only eight days after petitioner announced that it will suspend its
operations for six months. In Nippon Housing Phil., Inc. v. Leynes, we said that a complaint for
illegal dismissal filed prior to the lapse of said six months is generally considered as prematurely
filed.
In this case, however, we agree with the Labor Arbiter and the CA that respondents were
already considered illegally dismissed since petitioner failed to recall them after six months,
when its bona fide suspension of operations lapsed. We stress that under Article 286 of the
Labor Code, the employment will not be deemed terminated if the bona fide suspension of
operations does not exceed six months. But if the suspension of operations exceeds six
months, the employment will be considered terminated.

ATHENA M. SALAS | LABOR CASE DIGEST 2015 228

WHEREFORE, we DENY the petition in G.R. No. 171282 and AFFIRM the Decision dated
November 9, 2005 and Resolution dated January 24, 2006 of the Court of Appeals in CA-G.R.
SP No. 76670, subject to the settlement agreements and quitclaims signed by almost all of the
respondents.

102. Navotas Shipyard Corp., vs. Montallana et al., GR No. 190053, March 24, 2014
Facts:
The case arose when respondents Innocencio Montallana, Alfredo Bautista, Teodoro Judloman,
Guillermo Bongas, Rogelio Bongas, Diosdado Busante, Emiliano Badu and Rosendo SubingSubing filed a complaint for illegal (constructive) dismissal, with money claims, against the
petitioners, Navotas Shipyard Corporation (company) and its President/General Manager, Jesus
Villaflor.
The respondents alleged that on October 20, 2003, the company's employees (about 100) were
called to a meeting where Villaflor told them: "Magsasara na ako ng negosyo, babayaran ko na
lang kayo ng separation pay dahil wala na akong pangsweldo sa inyo. Marami akong mga
utang sa krudo, yelo, at iba pa."4 Since then, they were not allowed to report for work but
Villaflor's promise to give them separation pay never materialized despite their persistent
demands and follow-ups. ScaHDT
The petitioners, on the other hand, claimed that due to the "seasonal lack of fish caught and
uncollected receivables[,]" 5 the company suffered financial reverses. It was thus constrained to
temporarily cease operations. They projected that the company could resume operations before
the end of six months or on April 22, 2004. It reported the temporary shutdown to the
Department of Labor and Employment, National Capital Region (DOLE-NCR) and filed an
Establishment Termination Report.
The Labor Arbiter initially denied the complaint but awarded the complainants 13th month pay
and SIL for the year 2003. The LA argued that the respondents could not have been illegally
dismissed as the temporary shutdown of operations merely suspends the employment
relationship. The NLRC affirmed the LA decision in toto. On appeal to the CA, the appellate
court ruled that the closure was intended to be permanent and that in the absence of proper
notice given by the petitioners, the respondents should be considered illegally dismissed.
Petitioners then appealed the case pursuant to Rule 45 questioning the propriety of the award
for separation pay and backwages notwithstanding the closure of the companys business
operations.
Issues:
(1)

What is the applicable law to the case, Art. 283 or Art. 286?

(2)

WON the respondents were illegally dismissed

(3)

WON the respondents are entitled to backwages/nominal damages

(4)
WON the respondents are entitled to separation pay, service incentive leave and 13th
month pay

ATHENA M. SALAS | LABOR CASE DIGEST 2015 229

Ruling:
To place the case in perspective, we first examine the applicable law in view of the
disagreement between the petitioners and the respondents in that respect. According to the CA,
the "[p]etitioners anchor their arguments mainly on Article 283 of the Labor Code, stating that
private respondents resorted to retrenchment and permanent closure of business, while private
respondents maintain that what is applicable is Article 286 . . . as the closure of business was
merely temporary." Articles 283 and 286 of the Labor Code provide:
ART. 283. Closure of establishment and reduction of personnel. The employer may also
terminate the employment of any employee due to the installation of labor-saving devices,
redundancy, retrenchment to prevent losses or the closing or cessation of operation of the
establishment or undertaking unless the closing is for the purpose of circumventing the
provisions of this Title, by serving a written notice on the workers and the [Department of Labor]
and Employment at least one (1) month before the intended date thereof. In case of termination
due to the installation of labor-saving devices or redundancy, the worker affected thereby shall
be entitled to a separation pay equivalent to at least his one (1) month pay or to at least one (1)
month pay for every year of service, whichever is higher. In case of retrenchment to prevent
losses and in cases of closures and cessation of operations of establishment or undertaking not
due to serious business losses or financial reverses, the separation pay shall be equivalent to
one (1) month pay or to at least one-half (1/2) month pay for every year of service, whichever is
higher. A fraction of at least six (6) months shall be considered one (1) whole year.
ART. 286. When employment not deemed terminated. The bona-fide suspension of the
operation of a business or undertaking for a period not exceeding six (6) months, or the
fulfillment by the employee of a military or civic duty shall not terminate employment. In all such
cases, the employer shall reinstate the employee to his former position without loss of seniority
rights if he indicates his desire to resume his work not later than one (1) month from the
resumption of operations of his employer or from his relief from the military or civic duty.
As we earlier stated, the petitioners undertook a temporary shutdown. In fact, the company
notified the DOLE of the shutdown and filed an Establishment Termination Report containing the
names of the affected employees. The petitioners expected the company to recover before the
end of the six-month shutdown period, but unfortunately, no recovery took place. Thus, the
shutdown became permanent. According to the petitioners, they gave the company's employees
their separation pay.
We disagree with the company's position that it resorted to a retrenchment under Article 283 of
the Labor Code; it was a temporary shutdown under Article 286 where the employees are
considered on floating status or whose employment is temporarily suspended.
Under the circumstances, we cannot say that the company's employees were illegally
dismissed; rather, they lost their employment because the company ceased operations after
failing to recover from their financial reverses. The CA itself recognized what happened to the
company when it observed: "The temporary shutdown has ripened into a closure or cessation of
operations. In this situation[,] private respondents are definitely entitled to the corresponding
benefits of separation." Even the respondents had an inkling of the company's fate when they
claimed before the LA that on October 20, 2003, they were called, together with all the other
employees of the company, by Villaflor; the latter allegedly told them that he would be closing
the company, but would give them their separation pay. He also disclosed to them the reason

ATHENA M. SALAS | LABOR CASE DIGEST 2015 230

he could no longer pay their salaries due to the company's unsettled financial obligations on fuel
and ice and other indebtedness.
Since there was no illegal dismissal, the respondents are not entitled to backwages. The term
"backwages" presupposes illegal termination of employment. It is restitution of earnings unduly
withheld from the employee because of illegal termination. Hence, where there is no illegal
termination, there is no basis for claim or award of backwages.
The lack of basis for backwages notwithstanding, we note that the respondents claimed that
they were not given individual written notices of the company's temporary shutdown or of its
closure. The records support the respondents' position. Other than the Establishment
Termination Report submitted by the company to the DOLE-NCR when it temporarily shut down
its operations and which included the respondents' names, there is no evidence (other than the
petitioner's informal talk with its employees, which did not strictly comply with the legal
requirement) that they were served individual written notices at least thirty (30) days before the
effectivity of the termination, as required under Section 1 (iii), Rule I, Book VI of the Omnibus
Rules Implementing the Labor Code. Pursuant to existing jurisprudence, if the dismissal is by
virtue of a just or authorized cause, but without due process, the dismissed workers are entitled
to an indemnity in the form of nominal damages.
Under Article 283 of the Labor Code quoted earlier, the employer may terminate the
employment of any employee due to, among other causes, the closure or cessation of
operations of the establishment or undertaking. In such an eventuality, the employee may or
may not be entitled to separation pay. On this point, Article 283 provides: in cases of closures or
cessation of operations of establishment or undertaking not due to serious business losses or
financial reverses, the separation pay shall be equivalent to one (1) month pay or to at least
one-half (1/2) month pay for every year of service, whichever is higher. A fraction of least six
months shall be considered one (1) whole year.
Considering that the company's closure was due to serious financial reverses, it is not legally
bound to give the separated employees separation pay. In Reahs Corporation v. NLRC,the
Court explained that "[t]he grant of separation pay, as an incidence of termination of
employment under Article 283, is a statutory obligation on the part of the employer and a
demandable right on the part of the employee, except only where the closure or cessation of
operations was due to serious business losses or financial reverses and there is sufficient proof
of this fact or condition."
We note, however, that in his meeting with the employees, including the respondents, on
October 20, 2003, Villaflor told them that he would be giving them separation pay as a
consequence of the company's closure. He should now honor his undertaking to the
respondents and grant them separation pay. Except for the petitioners' claim that "they gave the
separation pays of their employees," they failed to present proof of actual payment. In this light,
Villaflor's grant of separation pay to the respondents has still to be fulfilled.
Finally, the petitioners did not appeal the LA's award of service incentive leave pay and 13th
month pay for the year 2003 to the respondents. Accordingly, the award stands.

ATHENA M. SALAS | LABOR CASE DIGEST 2015 231

103. Emeritus Security & Maintenance Systems Inc., vs. Dailig, GR No. 204761, April 2,
2014
Facts:
Respondent Dailig is one of the security guards of petitioner agency, who was assigned to one
of its clients (Panasonic) in Calamba, Laguna. On December 10, 2005 respondent was relieved
from his post. On June 16, 2006 he filed a complaint for illegal dismissal claiming that on
various dates in December 2005 to January to May 2006, he went to petitioners office to followup his next assignment, but was still not given one. He argued that if an employee is on floating
status for more than 6 months, such an employee is deemed illegally dismissed.
Petitioner countered that he never dismissed respondent, and even sent notices to him to report
to work on the day he received the order of relief and once on January 27, 2006. Despite these
notices, respondent failed to report for work, which meant according to petitioner that he was no
longer interested to continue his employment.
The Labor Arbiter, NLRC, and CA ruled for respondent saying that he was illegally dismissed,
and gave him separation pay instead of reinstatement, because of the strained relations
doctrine. Petitioner questions the finding of illegal dismissal, and separation pay because it
had already reinstated respondent with its sister company, Emme.
Issues:
A. Does a floating status of a security guard lasting for more than 6 months constitute
constructive dismissal?
B. Is separation pay still proper when an employee is reinstated with a sister company?
Ruling:
A. Yes, in this case for 2 reasons:
Petitioner's allegation of sending respondent a notice sometime in January 2006, requiring him
to report for work, is unsubstantiated, and thus, self-serving. The Court agrees with the ruling of
the Labor Arbiter, NLRC and Court of Appeals that a floating status of a security guard, such as
respondent, for more than six months constitutes constructive dismissal. In Nationwide Security
and Allied Services, Inc. v. Valderama, 8 the Court held: . . . the temporary inactivity or "floating
status" of security guards should continue only for six months. Otherwise, the security agency
concerned could be liable for constructive dismissal. The failure of petitioner to give respondent
a work assignment beyond the reasonable six-month period makes it liable for constructive
dismissal. . . . .
Further, the Court notes that the Labor Arbiter, NLRC, and Court of Appeals unanimously found
that respondent was illegally dismissed by petitioner. Factual findings of quasi-judicial bodies
like the NLRC, if supported by substantial evidence, are accorded respect and even finality by
this Court, more so when they coincide with those of the Labor Arbiter. 10 Such factual findings
are given more weight when the same are affirmed by the Court of Appeals
B. Yes

ATHENA M. SALAS | LABOR CASE DIGEST 2015 232

Article 279 of the Labor Code of the Philippines mandates the reinstatement of an illegally
dismissed employee, to wit:
Security of Tenure. . . . An employee who is unjustly dismissed from work shall be entitled to
reinstatement without loss of seniority rights and other privileges and to his full back wages,
inclusive of allowances, and to his other benefits or their monetary equivalent computed from
the time his compensation was withheld from him up to the time of his actual reinstatement.
Thus, reinstatement is the general rule, while the award of separation pay is the exception. The
circumstances warranting the grant of separation pay, in lieu of reinstatement, are laid down by
the Court in Globe-Mackay Cable and Radio Corporation v. National Labor Relations
Commission, thus:
Over time, the following reasons have been advanced by the Court for denying reinstatement
under the facts of the case and the law applicable thereto; that reinstatement can no longer be
effected in view of the long passage of time (22 years of litigation) or because of the realities of
the situation; or that it would be 'inimical to the employer's interest;' or that reinstatement may
no longer be feasible; or, that it will not serve the best interests of the parties involved; or that
the company would be prejudiced by the workers' continued employment; or that it will not serve
any prudent purpose as when supervening facts have transpired which make execution on that
score unjust or inequitable or, to an increasing extent, due to the resultant atmosphere of
'antipathy and antagonism' or 'strained relations' or 'irretrievable estrangement' between the
employer and the employee.
Petitioner counters that Emeritus and Emme are sister companies with the same Board of
Directors and officers, arguing that Emeritus and Emme are in effect one and the same
corporation.
Considering petitioner's undisputed claim that Emeritus and Emme are one and the same, there
is no basis in respondent's allegation that he was not reinstated to his previous employment.
Besides, respondent assails the corporate personalities of Emeritus and Emme only in his
Comment filed before this Court. Further, respondent did not appeal the Labor Arbiter's
reinstatement order.
Contrary to the Court of Appeals' ruling, there is nothing in the records showing any strained
relations between the parties to warrant the award of separation pay. There is neither allegation
nor proof that such animosity existed between petitioner and respondent. In fact, petitioner
complied with the Labor Arbiter's reinstatement order.
Considering that (1) petitioner reinstated respondent in compliance with the Labor Arbiter's
decision, and (2) there is no ground, particularly strained relations between the parties, to justify
the grant of separation pay, the Court of Appeals erred in ordering the payment thereof, in lieu of
reinstatement.

104. Lopez vs. Irvine Construction Corp. GR No. 207253, August 20, 2014
Facts:
Petitioner is a construction worker of respondent construction firm who sometimes acts as a
guard at one of respondents warehouses in Cavite. On December 18 2005 he was purportedly

ATHENA M. SALAS | LABOR CASE DIGEST 2015 233

laid off work, where he was told ikaw ay lay-off muna. Thus he filed a case for illegal dismissal.
Respondent denied this by saying that he was not illegally dismissed but was temporarily laid off
when the project in Cavite was finished, and that he was sent a return to work order within the 6
month period for bona fide suspension of the operation of a business or undertaking in the
Labor Code.
The LA and NLRC found petitioner to have been illegally dismissed, but the CA reversed the
ruling dismissing the case for illegal dismissal.
Issue:
Granting that petitioner is a regular employee, was there a proper temporary lay-off due to
bona-fide suspension of business operations?
Ruling:
Note: At the outset, the Court found petitioner to be a regular employee and not a project
employee, entitled to security of tenure. The discussion is limited to suspension of business
operations.
NO, THERE WAS NO PROPER LAY-OFF DUE TO BONA-FIDE SUSPENSION OF BUSINESS
Among the authorized causes for termination under Article 283 of the Labor Code is
retrenchment, or what is sometimes referred to as a "lay-off":
Art. 283. Closure of Establishment and Reduction of Personnel. The employer may also
terminate the employment of any employee due to the installation of labor-saving devices,
redundancy, retrenchment to prevent losses or the closing or cessation of operation of the
establishment or undertaking unless the closing is for the purpose of circumventing the
provisions of this Title, by serving a written notice on the workers and the Ministry of Labor and
Employment at least one (1) month before the intended date thereof.
It is defined as the severance of employment, through no fault of and without prejudice to the
employee, resorted to by management during the periods of business recession, industrial
depression, or seasonal fluctuations, or during lulls caused by lack of orders, shortage of
materials, conversion of the plant to a new production program or the introduction of new
methods or more efficient machinery, or of automation. Elsewise stated, lay-off is an act of the
employer of dismissing employees because of losses in the operation, lack of work, and
considerable reduction on the volume of its business, a right recognized and affirmed by the
Court. However, a lay-off would be tantamount to a dismissal only if it is permanent. When a
lay-off is only temporary, the employment status of the employee is not deemed terminated, but
merely suspended.
Pursuant to Article 286 of the Labor Code, the suspension of the operation of business or
undertaking in a temporary lay-off situation must not exceed six (6) months:
ART. 286. When Employment not Deemed Terminated. The bona-fide suspension of the
operation of a business or undertaking for a period not exceeding six (6) months.
Within this six-month period, the employee should either be recalled or permanently retrenched.
Otherwise, the employee would be deemed to have been dismissed, and the employee held
liable therefor.

ATHENA M. SALAS | LABOR CASE DIGEST 2015 234

Notably, in both a permanent and temporary lay-off, jurisprudence dictates that the one-month
notice rule to both the DOLE and the employee under Article 283 of the Labor Code, as above
cited, is mandatory. Also, in both cases, the lay-off, being an exercise of the employer's
management prerogative, must be exercised in good faith that is, one which is intended for
the advancement of employers' interest and not for the purpose of defeating or circumventing
the rights of the employees under special laws or under valid agreements. Instructive on the
nature of a lay-off as a management prerogative is the following excerpt from the case of
Industrial Timber Corporation v. NLRC:
Closure or [suspension] of operations for economic reasons is, therefore, recognized as a valid
exercise of management prerogative. The determination to cease [or suspend] operations is a
prerogative of management, which the State does not usually interfere with, as no business or
undertaking [is] required to continue operating at a loss simply because it has to maintain its
workers in employment. Such an act would be tantamount to a taking of property without due
process of law.
As the NLRC correctly ruled in this case, Lopez, who, as earlier discussed was a regular
employee of Irvine, was not merely temporarily laid off from work but was terminated from his
employment without any valid cause therefor; thus, the proper disposition is to affirm the LA's
ruling that Lopez had been illegally dismissed.
Although the NLRC did not expound on the matter, it is readily apparent that the supposed layoff of Lopez was hardly justified considering the absence of any causal relation between the
cessation of Irvine's project in Cavite with the suspension of Lopez's work. To repeat, Lopez is a
regular and not a project employee. Hence, the continuation of his engagement with Irvine,
either in Cavite, or possibly, in any of its business locations, should not have been affected by
the culmination of the Cavite project alone. In light of the well-entrenched rule that the burden to
prove the validity and legality of the termination of employment falls on the employer, 47 Irvine
should have established the bona fide suspension of its business operations or undertaking that
would have resulted in the temporary lay-off of its employees for a period not exceeding six (6)
months in accordance with Article 286 of the Labor Code.
In this case, Irvine failed to prove compliance with the parameters of Article 286 of the Labor
Code. As the records would show, it merely completed one of its numerous construction projects
which does not, by and of itself, amount to a bona fide suspension of business operations or
undertaking. In invoking Article 286 of the Labor Code, the paramount consideration should be
the dire exigency of the business of the employer that compels it to put some of its employees
temporarily out of work. 51 This means that the employer should be able to prove that it is faced
with a clear and compelling economic reason which reasonably forces it to temporarily shut
down its business operations or a particular undertaking, incidentally resulting to the temporary
lay-off of its employees.
Due to the grim economic consequences to the employee, case law states that the employer
should also bear the burden of proving that there are no posts available to which the employee
temporarily out of work can be assigned.
Verily, Irvine cannot conveniently suspend the work of any of its employees in the guise of a
temporary lay-off when it has not shown compliance with the legal parameters under Article 286
of the Labor Code. With Irvine failing to prove such compliance, the resulting legal conclusion is
that Lopez had been constructively dismissed; and since the same was effected without any

ATHENA M. SALAS | LABOR CASE DIGEST 2015 235

valid cause and due process, the NLRC properly affirmed the LA's ruling that Lopez's dismissal
was illegal.

105. Exocet Security & Allied Services Corp., vs. Serrano, GR No. 198538, Sept. 29, 2014
Facts:
Petitioner Exocet Security and Allied Services Corporation (Exocet) is engaged in the provision
of security personnel to its various clients or principals. By virtue of its contract with JG Summit
Holdings Inc. (JG Summit), Exocet assigned respondent Armando D. Serrano (Serrano) on
September 24, 1994 as close-insecurity personnel for one of JG Summits corporate officers,
Johnson Robert L. Go. After eight years, Serrano was re-assigned as close-in security for Lance
Gokongwei, and then to his wife, Mary Joyce Gokongwei. As close-in security, records show
that Serrano was receiving a monthly salary of P11,274.30.
On August 15, 2006, Serrano was relieved by JG Summit from his duties. For more than six
months after he reported back to Exocet, Serrano was without any reassignment. On March 15,
2007, Serrano filed a complaint for illegal dismissal against Exocet with the National Labor
Relations Commission (NLRC).
For its defense, Exocet denied dismissing Serrano alleging that, after August 15, 2006, Serrano
no longer reported for duty assignment as VIP security for JG Summit, and that on September
2006, he was demanding for VIP Security detail to another client. However, since, at that time,
Exocet did not have clients in need of VIP security assignment, Serrano was temporarily
assigned to general security service.6 Exocet maintained that it was Serrano who declined the
assignment on the ground that he is not used to being a regular security guard. Serrano, Exocet
added, even refused to report for immediate duty, as he was not given a VIP security
assignment.
Considering the parties respective allegations, the Labor Arbiter ruled that Serrano was illegally
dismissed. In its June 30, 2008 Decision, the Labor Arbiter found that Serrano, while not actually
dismissed, was placed on a floating status for more than six months and so, was deemed
constructively dismissed. Thus, the Labor Arbiter ordered Exocet to pay Serrano separation pay.
Not satisfied with the award, Serrano appealed the Labor Arbiters Decision to the NLRC. In its
March 5, 2009 Resolution, the NLRC initially affirmed the ruling of the Labor Arbiter, but
modified the monetary award to include the payment of backwages for six months that Serrano
was not given a security assignment.
Acting on Exocets motion for reconsideration, however, the NLRC, in its September 2, 2009
Resolution, further modified its earlier decision by removing the award for backwages.11 The
NLRC deviated from its earlier findings and ruled that Serrano was not constructively dismissed,
as his termination was due to his own fault, stubborn refusal, and deliberate failure to accept a
re-assignment.12 Nevertheless, the NLRC proceeded to affirm in toto the decision of the Labor
Arbiter on the ground that Exocet did not interpose the appeal.
On January 22, 2010, the NLRC issued another Resolution denying Serranos motion for
reconsideration.14 Hence, not satisfied with the NLRCs ruling, Serrano filed a petition for
certiorari with the CA assailing the September 2, 2009 Resolution of the NLRC. Serrano insisted

ATHENA M. SALAS | LABOR CASE DIGEST 2015 236

that he was constructively dismissed and, thus, is entitled to reinstatement without loss of
seniority rights and to full backwages from the time of the alleged dismissal up to the time of the
finality of the Decision.
On March 31, 2011, the appellate court rendered a Decision in Serranos favor, reversing and
setting aside the NLRCs September 2, 2009 Resolution and ordering Exocet to pay Serrano
separation pay and backwages.15 In so ruling, the CA found that Serrano was constructively
dismissed, as Exocet failed to re-assign him within six months after placing him on floating
status.Petitioner Exocets Motion for Reconsideration was denied by the appellate court in its
September 7, 2011 Resolution.17
Issue:
The sole issue for resolution is whether or not Serrano was constructively dismissed.
Ruling:
The petition has merit.
The crux of the controversy lies on the consequence of the lapse of the six-month period, during
which respondent Serrano was placed on a floating status and petitioner Exocet could not
assign him to a position he wants. The appellate court was of the view that Serrano was
constructively dismissed. The Court maintains otherwise.
As the circumstance is generally outside the control of the security agency or the employer, the
Court has ruled that when a security guard is placed on a floating status, he or she does not
receive any salary or financial benefit provided by law.
Verily, a floating status requires the dire exigency of the employers bona fidesuspension of
operation of a business or undertaking. In security services, this happens when the security
agencys clients which do not renew their contracts are more than those that do and the new
ones that the agency gets. Also, in instances when contracts for security services stipulate that
the client may request the agency for the replacement of the guards assigned to it even for want
of cause, the replaced security guard may be placed on temporary off-detail if there are no
available posts under respondents existing contracts.
When a security guard is placed on a floating status, he does not receive any salary or
financial benefit provided by law. Due to the grim economic consequences to the employee, the
employer should bear the burden of proving that there are no posts available to which the
employee temporarily out of work can be assigned.
It must be emphasized, however, that although placing a security guard on floating status or a
temporary off-detail is considered a temporary retrenchment measure, there is similarly no
provision in the Labor Code which treats of a temporary retrenchment or lay-off. Neither is there
any provision which provides for its requisites or its duration.22 Nevertheless, since an
employee cannot be laid-off indefinitely, the Court has applied Article 292 (previously Article
286) of the Labor Code by analogy to set the specific period of temporary lay-off to a maximum
of six (6) months.
In the controversy now before the Court, there is no question that the security guard, Serrano,
was placed on floating status after his relief from his post as a VIP security by his security
agencys client. Yet, there is no showing that his security agency, petitioner Exocet, acted in bad

ATHENA M. SALAS | LABOR CASE DIGEST 2015 237

faith when it placed Serrano on such floating status. What is more, the present case is not a
situation where Exocet did not recall Serrano to work within the six-month period as required by
law and jurisprudence. Exocet did, in fact, make an offer to Serrano to go back to work. It is just
that the assignmentalthough it does not involve a demotion in rank or diminution in salary,
pay, benefits or privilegeswas not the security detail desired by Serrano.
Clearly,Serranos lack of assignment for more than six months cannot be attributed to petitioner
Exocet. On the contrary, records show that, as early as September 2006, or one month after
Serrano was relieved as a VIP security, Exocet had already offered Serrano a position in the
general security service because there were no available clients requiring positions for VIP
security. Notably, even though the new assignment does not involve a demotion in rank or
diminution in salary, pay, or benefits, Serrano declined the position because it was not the post
that suited his preference, as he insisted on being a VIP Security.
Anent the clients action, respondent agency had no recourse but to assign complainant to a
new posting. However, complainant, having had a taste of VIP detail and perhaps the perks that
come with such kind of assignment, vaingloriously assumed that he can only be assigned to VIP
close-in posting and that he would accept nothing less. In fact, after his relief and tardy
appearance at respondents office, he was offered re-assignment albeit to general security
services which he refused. Respondents clearly made known to him that as of the moment no
VIP detail was vacant or sought by other clients but complainant was adamant in his refusal.
Complainant even had the nerve to assert that he just be informed if there is already a VIP
detail available for him and that he will just report for re-assignment by then. It is also well to
note that to these allegations, complainant made no denial.29
Thus, it is manifestly unfair and unacceptable to immediately declare the mere lapse of the sixmonth period of floating status as a case of constructive dismissal, without looking into the
peculiar circumstances that resulted in the security guards failure to assume another post. This
is especially true in the present case where the security guards own refusal to accept a non-VIP
detail was the reason that he was not given an assignment within the six-month period. The
security agency, Exocet, should not then be held liable.
Indeed, from the facts presented, Serrano was guilty of wilful disobedience to a lawful order of
his employer in connection with his work, which is a just cause for his termination under Art.288
(previously Art. 282)of the Labor Code.31 Nonetheless, Exocet did not take Serranos wilful
disobedience against him. Hence, Exocet is considered to have waived its right to terminate
Serrano on such ground.
Petitioner Exocet Security and Allied Services Corporation is neither guilty of illegal dismissal
nor constructive dismissal. Petitioner is hereby ORDERED to look for a security assignment for
respondent within a period of thirty (30) days from finality of judgment. If one is available,
petitioner is ordered to notify respondent Armando D. Serrano to report to such available guard
position within ten (10) days from notice. If respondent fails to report for work within said time
period, he shall be deemed to have abandoned his employment with petitioner.In such case,
respondent Serrano is notentitled to any backwages, separation pay, or similar benefits.
If no security assignment is available for respondent within a period of thirty (30) days from
finality of judgment, petitioner Exocet should comply with the requirements of DOLE Department
Order No. 14, Series of 2001, in relation to Art. 289 of the Labor Code, and serve a written
notice on respondent Serrano and the DOLE one (1) month before the intended date of

ATHENA M. SALAS | LABOR CASE DIGEST 2015 238

termination; and pay Serrano separation pay equivalent to half month pay for every year of his
service.

16.DISEASE AS GROUND FOR TERMINATION

106. Deoferio vs. Intel Technology Phils., GR No. 202996, June 18, 2014
Facts:
On February 1, 1996, respondent Intel Technology Philippines, Inc. (Intel) employed Deoferio as
a product quality and reliability engineer with a monthly salary of P9,000.00. In July2001, Intel
assigned him to the United States as a validation engineer for an agreed period of two years
and with a monthly salary of US$3,000.00. On January 27, 2002, Deoferio was repatriated to
the Philippines after being confined at Providence St. Vincent Medical Center for major
depression with psychosis.4 In the Philippines, he worked as a product engineer with a monthly
salary of P23,000.00.
After several consultations, Dr. Lee issued a psychiatric report dated January 17, 2006
concluding and stating that Deoferios psychotic symptoms are not curable within a period of six
months and "will negatively affect his work and social relation with his co-worker[s]." Pursuant to
these findings, Intel issued Deoferio a notice of termination on March 10, 2006.
Deoferio responded to his termination of employment by filing a complaint for illegal dismissal
with prayer for money claims against respondents Intel and Mike Wentling (respondents). He
denied that he never had mental illness and insisted that he satisfactorily performed his duties
as a product engineer. He argued that Intel violated his statutory right to procedural due process
when it summarily issued a notice of termination. He further claimed that he was entitled to a
salary differential equivalent to the pre-terminated period of his assignment in the United States
minus the base pay that he had already received. Deoferio also prayed for backwages,
separation pay, moral and exemplary damages, as well as attorneys fees.
In defense, the respondents argued that Deoferios dismissal was based on Dr. Lees
certification that: (1) his schizophrenia was not curable within a period of six months even with
proper medical treatment; and (2) his continued employment would be prejudicial to his and to
the other employees health. The respondents also insisted that Deoferios presence at Intels
premises would pose an actual harm to his co-employees as shown by his previous acts. On
May 8, 2003, Deoferio emailed an Intel employee with this message: "All souls day back to
work Monday WW45." On January 18, 2005, he cut the mouse cables, stepped on the
keyboards, and disarranged the desks of his co-employees. The respondents also highlighted
that Deoferio incurred numerous absences from work due to his mental condition, specifically,
from January 31, 2002 until February 28, 2002, from August 2002 until September 2002, and
from May 2003 until July 2003. Deoferio also took an administrative leave with pay from
January 2005 until December 2005.
The respondents further asserted that the twin-notice requirement in dismissals does not apply
to terminations under Article 284 of the Labor Code. They emphasized that the Labor Codes
implementing rules (IRR) only requires a competent public health authoritys certification to

ATHENA M. SALAS | LABOR CASE DIGEST 2015 239

effectively terminate the services of an employee.They insisted that Deoferios separation and
retirement payments for P247,517.35 were offset by his company car loan which amounted to
P448,132.43. He was likewise not entitled to moral and exemplary damages, as well as
attorneys fees, because the respondents faithfully relied on Dr. Lees certification that he was
not fit to work as a product engineer.
The Labor Arbitration Ruling
In a decision dated March 6, 2008,the Labor Arbiter (LA) ruled that Deoferio had been validly
dismissed. The LA gave weight to Dr. Lees certification that Deoferio had been suffering from
schizophrenia and was not fit for employment. The evidence on record shows that Deoferios
continued employment at Intel would pose a threat to the health of his co-employees. The LA
further held that the Labor Code and its IRR do not require the employer to comply with the
twin-notice requirement in dismissals due to disease. The LA also found unmeritorious
Deoferios money claims against Intel.
On appeal by Deoferio, the National Labor Relations Commission (NLRC) wholly affirmed the
LAs ruling.The NLRC also denied Deoferios motion for reconsideration, prompting him to seek
relief from the CA through a petition for certiorari under Rule 65 of the Rules of Court.
The CAs Ruling
On February 24, 2012, the CA affirmed the NLRC decision. It agreed with the lower tribunals
findings that Deoferio was suffering from schizophrenia and that his continued employment at
Intel would be prejudicial to his health and to those of his co-employees. It ruled that the only
procedural requirement under the IRR is the certification by a competent public health authority
on the non-curability of the disease within a period of six months even with proper medical
treatment. It also concurred with the lower tribunals that Intel was justified in not paying Deoferio
separation pay as required by Article 284 of the Labor Code because this obligation had already
been offset by the matured car loan that Deoferio owed Intel.
Deoferio filed the present petition after the CA denied his motion for reconsideration.
The Petition
In the present petition before the Court, Deoferio argues that the uniform finding that he was
suffering from schizophrenia is belied by his subsequent employment at Maxim Philippines
Operating Corp. and Philips Semiconductors Corp., which both offered him higher
compensations. He also asserts that the Labor Code does not exempt the employer from
complying with the twin-notice requirement in terminations due to disease.
The Respondents Position
In their Comment, the respondents posit that the petition raises purely questions of fact which a
petition for review on certiorari does not allow. They submit that Deoferios arguments have
been fully passed upon and found unmeritorious by the lower tribunals and by the CA. They
additionally argue that Deoferios subsequent employment in other corporations is irrelevant in
determining the validity of his dismissal; the law merely requires the non-curability of the
disease within a period of six months even with proper medical treatment.
The respondents also maintain that Deoferios claim for salary differential is already barred by
prescription under Article 291 of the Labor Code. Even assuming that the claim for salary

ATHENA M. SALAS | LABOR CASE DIGEST 2015 240

differential has been timely filed, the respondents assert that the parties expressly agreed in the
International Assignment Relocation Agreement that "the assignment length is only an estimate
and not a guarantee of employment for any particular length of time." Moreover, his assignment
in the United States was merely temporary and did not change his salary base, an amount
which he already received.

Issues:
This case presents to us the following issues:
(1) Whether Deoferio was suffering from schizophrenia and whether his continued employment
was prejudicial to his health, as well as to the health of his co-employees;
(2) Whether the twin-notice requirement in dismissals applies to terminations due to disease;
and
As part of the second issue, the following issues are raised:
(a) Whether Deoferio is entitled to nominal damages for violation of his right to statutory
procedural due process; and
(b) Whether the respondents are solidarily liable to Deoferio for nominal damages.
(3) Whether Deoferio is entitled to salary differential, backwages, separation pay, moral and
exemplary damages, as well as attorneys fees.
Ruling:
We find the petition partly meritorious. Intel had an authorized cause to dismiss Deoferio from
employment Concomitant to the employers right to freely select and engage an employee is the
employers right to discharge the employee for just and/or authorized causes.
The present case involves termination due to disease an authorized cause for dismissal under
Article 284 of the Labor Code. As substantive requirements, the Labor Code and its IRR require
the presence of the following elements:
(1) An employee has been found to be suffering from any disease.
(2) His continued employment is prohibited by law or prejudicial to his health, as well as to the
health of his co-employees.
(3) A competent public health authority certifies that the disease is of such nature or at such a
stage that it cannot be cured within a period of six months even with proper medical treatment.
With respect to the first and second elements, the Court liberally construed the phrase
"prejudicial to his health as well as to the health of his co-employees" to mean "prejudicial to his
health or to the health of his co-employees." We did not limit the scope of this phrase to
contagious diseases for the reason that this phrase is preceded by the phrase "any disease"
under Article 284 of the Labor Code, to wit:
The third element substantiates the contention that the employee has indeed been suffering
from a disease that: (1) is prejudicial to his health as well as to the health of his co-employees;
and (2) cannot be cured within a period of six months even with proper medical treatment.

ATHENA M. SALAS | LABOR CASE DIGEST 2015 241

Without the medical certificate, there can be no authorized cause for the employees dismissal.
The absence of this element thus renders the dismissal void and illegal.
Simply stated, this requirement is not merely a procedural requirement, but a substantive one.
The certification from a competent public health authority is precisely the substantial evidence
required by law to prove the existence of the disease itself, its non-curability within a period of
six months even with proper medical treatment, and the prejudice that it would cause to the
health of the sick employee and to those of his co-employees.
In the current case, we agree with the CA that Dr. Lees psychiatric report substantially proves
that Deoferio was suffering from schizophrenia, that his disease was not curable within a period
of six months even with proper medical treatment, and that his continued employment would be
prejudicial to his mental health. This conclusion is further substantiated by the unusual and
bizarre acts that Deoferio committed while at Intels employ.
The twin-notice requirement applies to terminations under Article 284 of the Labor Code. The
Labor Code and its IRR are silent on the procedural due process required in terminations due to
disease. Despite the seeming gap in the law, Section 2, Rule 1, Book VI of the IRR expressly
states that the employee should be afforded procedural due process in all cases of dismissals.
In Sy v. Court of Appeals and Manly Express, Inc. v. Payong, Jr., promulgated in 2003 and 2005,
respectively, the Court finally pronounced the rule that the employer must furnish the employee
two written notices in terminations due to disease, namely: (1) the notice to apprise the
employee of the ground for which his dismissal is sought; and (2) the notice informing the
employee of his dismissal, to be issued after the employee has been given reasonable
opportunity to answer and to be heard on his defense. These rulings reinforce the State policy
of protecting the workers from being terminated without cause and without affording them the
opportunity to explain their side of the controversy.
From these perspectives, the CA erred in not finding that the NLRC gravely abused its
discretion when it ruled that the twin-notice requirement does not apply to Article 284 of the
Labor Code. This conclusion is totally devoid of any legal basis; its ruling is wholly unsupported
by law and jurisprudence. In other words, the NLRCs unprecedented, whimsical and arbitrary
ruling, which the CA erroneously affirmed, amounted to a jurisdictional error.
Deoferio is entitled to nominal damages for violation of his right to statutory procedural due
process.
We award Deoferio the sum of P30,000.00 as nominal damages for violation of his statutory
right to procedural due process. In so ruling, we take into account Intels faithful compliance with
Article 284 of the Labor Code and Section 8, Rule 1, Book 6 of the IRR. We also note that
Deoferios separation pay equivalent to one-half month salary for every year of service45 was
validly offset by his matured car loan. Under Article 1278 of the Civil Code, in relation to Article
1706 of the Civil Code46 and Article 113(c) of the Labor Code, compensation shall take place
when two persons are creditors and debtors of each other in their own right. We likewise
consider the fact that Intel exhibited real concern to Deoferio when it financed his medical
expenses for more than four years. Furthermore, prior to his termination, Intel liberally allowed
Deoferio to take lengthy leave of absences to allow him to attend to his medical needs.

ATHENA M. SALAS | LABOR CASE DIGEST 2015 242

Intel shall be solely liable to Deoferio for the satisfaction of nominal damages. Wentling, as a
corporate officer, cannot be held liable for acts done in his official capacity because a
corporation, by legal fiction, has a personality separate and distinct from its officers,
stockholders, and members. There is also no ground for piercing the veil of corporate fiction
because Wentling acted in good faith and merely relied on Dr. Lees psychiatric report in
carrying out the dismissal.48
Deoferio is not entitled to salary differential, backwages, separation pay, moral and exemplary
damages, as well as attorney's fees.
Deoferio's claim for salary differential is already barred by prescription. Under Article 291 of the
Labor Code, all money claims arising from employer-employee relations shall be filed within
three years from the time the cause of action accrued. In the current case, more than four years
have elapsed from the pre-termination of his assignment to the United States until the filing of
his complaint against the respondents. We thus see no point in further discussing this matter.
SO ORDERED.

17.OTHER CAUSES OF SEVERANCE OF EMPLOYMENT RELATION

107. Gan vs. Galderma Philippines, Inc. G.R. No. 177167, January 17, 2013
Facts:
Respondent Galderma Philippines, Inc. (Galderma), a wholly-owned subsidiary of Galderma
Pharma S.A., is engaged in the business of selling, marketing, and distribution of Cetaphil
Brand Product Lines (CBPL) that include Cetaphil liquid and bar cleansers, and pharmaceutical
products, such as Locetar, Benzac and other prescription drugs. CBPL, which are over-thecounter products sold and/or distributed through supermarkets and health and beauty outlets,
are handled by Galderma's Consumer Products Division, while pharmaceutical products, which
are mostly prescription drugs sold and/or distributed through drug stores, are handled by its
Ethical Products Division.

ATHENA M. SALAS | LABOR CASE DIGEST 2015 243

On February 9, 2001, petitioner Nelson B. Gan (Gan) was hired by Galderma as Product
Manager for its Consumer Products Division to handle the marketing of CBPL effective March 1,
2001.
Gan was initially under the immediate supervision of Sales and Marketing Manager, Stephen C.
Peregrino (Peregrino). Starting September 1, 2001, however, in view of Peregrinos resignation,
he directly reported to Galderma's President and General Manager, respondent Rosendo C.
Veneracion (Veneracion).
With his satisfactory performance during the first year, Gan was acknowledged and rewarded by
Galderma through positive performance appraisal, salary and benefits increases, and informal
notations on his marketing reports. Pursuant to its intention to give him additional product
management responsibilities, Galderma provided Gan with product knowledge training on
Benzac and Locetar brands in December 2001. Thereafter, Gans incentive program was
revised and took effect in April 2002.
On April 11, 2002, Gan severed his employment ties with Galderma, and the company
responded positively, making his severance effective on July 15, 2002, which is three months
allowance for the company to seek replacement.
Three months passed, on July 25, 2002, Gan filed a Complaint for illegal constructive dismissal,
full backwages, separation pay, damages, attorneys fees, and cost of suit against respondents
Galderma and Veneracion, alleging that the resignation was not voluntary due to series of
incidents where he said that Veneracion made several acts that made him feel humiliated and
embarrassed, alleging multiple acts of harassment by Veneracion due to the new
responsibilities given to him.
The respondents, on the other hand, denied sunch allegations saying that Galderma's senior
managers noticed that Gan had a change of attitude from the time the management decided to
include the Benzac and Locetar brands under his responsibility. Despite the fact that the
company provided Gan with product knowledge training on the said brands, he initially refused
to accept the additional assignment. The company had to remind Gan that the assignment was
part of his Job Description, which allowed the company to assign him to undertake additional
tasks as may be deemed necessary by operations. Thereafter transpired series of activities
necessary for the job and a number of meetings happened. Later on, Gan asked the company
that he wanted to resign and even asked for his arrangement of favorable terms.
Issue:
whether or not there was constructive dismissal of Gan.
Ruling:
To begin with, constructive dismissal is defined as quitting or cessation of work because
continued employment is rendered impossible, unreasonable or unlikely; when there is a
demotion in rank or a diminution of pay and other benefits. It exists if an act of clear
discrimination, insensibility, or disdain by an employer becomes so unbearable on the part of the
employee that it could foreclose any choice by him except to forego his continued employment.
There is involuntary resignation due to the harsh, hostile, and unfavorable conditions set by the
employer. The test of constructive dismissal is whether a reasonable person in the employee's
position would have felt compelled to give up his employment/position under the circumstances.

ATHENA M. SALAS | LABOR CASE DIGEST 2015 244

On the other hand, "resignation is the voluntary act of an employee who is in a situation where
one believes that personal reasons cannot be sacrificed in favor of the exigency of the service,
and one has no other choice but to dissociate oneself from employment. It is a formal
pronouncement or relinquishment of an office, with the intention of relinquishing the office
accompanied by the act of relinquishment. As the intent to relinquish must concur with the overt
act of relinquishment, the acts of the employee before and after the alleged resignation must be
considered in determining whether he or she, in fact, intended to sever his or her employment."
Since Gan submitted a resignation letter, it is incumbent upon him to prove with clear, positive,
and convincing evidence that his resignation was not voluntary but was actually a case of
constructive dismissal; that it is a product of coercion or intimidation. He has to prove his
allegations with particularity.
Gan could not have been coerced. Coercion exists when there is a reasonable or well-grounded
fear of an imminent evil upon a person or his property or upon the person or property of his
spouse, descendants or ascendants. Neither do the facts of this case disclose that Gan was
intimidated
The instances of "harassment" alleged by Gan are more apparent than real. Aside from the
need to treat his accusations with caution for being self-serving due to lack of substantial
documentary or testimonial evidence to corroborate the same, the acts of "harassment," if true,
do not suffice to be considered as "peculiar circumstances" material to the execution of the
subject resignation letter.
What the records of this case reveal is that Gan deliberately wrote and filed a resignation letter
that is couched in a clear, concise, and categorical language. Its content confirmed his
unmistakable intent to resign. The resignation letter indicates that he was resigning "to pursue
the establishment of his own business or explore opportunities with other companies." The
reasons stated for relinquishing his position are but logical options for a person of his
experience and standing.
He was a managerial employee holding a responsible position and receiving more than the
mandated minimum wage. He also appears to have a good professional track record that
highlights his marketability. At the time he resigned, he had more than a decade of experience
in sales and marketing with expertise in product management. Indeed, it would be absurd to
assume that he did not understand the full import of the words he used in his resignation letter
and the consequences of executing the same.
What is evident, therefore, is that Gan's resignation is NOT "a case of adherence, not of
choice," but was a product of a mutually beneficial arrangement. We agree with respondents
that the result of the negotiation leading to Gan's resignation is a "win-win" solution for both
parties. On one hand, Gan was able to obtain a favorable severance pay while getting flexible
working hours to implement his post-resignation career options. On the other hand, Galderma
was able to cut its relation with an employee perceived to be unwilling to perform additional
product responsibilities while being given ample time to look for an alternative to hire and train.
Indeed, Gan voluntarily resigned from Galderma for a valuable consideration. He negotiated for
an improvement of the resignation package offered and he managed to obtain an acceptable
one. As opposed to the case of San Miguel Corporation v. NLRC, Gan was not tricked or was
"morally and psychologically hoodwinked" to draft, sign, and tender his resignation letter. It was

ATHENA M. SALAS | LABOR CASE DIGEST 2015 245

not made without proper discernment and time to reflect; nor was it a knee-jerk reaction that left
him with no alternative but to accede.

108. Padillo vs. Rural bank of Nabunturan Inc. G.R. No. 199338, January 21, 2013.
Facts:
On October 1, 1977, petitioner, the late Eleazar Padillo (Padillo), was employed by respondent
Rural Bank of Nabunturan, Inc. (Bank) as its SA Bookkeeper. Due to liquidity problems which
arose sometime in 2003, the Bank took out retirement/insurance plans with Philippine American
Life and General Insurance Company (Philam Life) for all its employees in anticipation of its
possible closure and the concomitant severance of its personnel. In this regard, the Bank
procured Philam Plan Certificate of Full Payment No. 88204, Plan Type 02FP10SC, Agreement
No. PP98013771 (Philam Life Plan) in favor of Padillo for a benefit amount of P100,000.00 and
which was set to mature on July 11, 2009.
On October 14, 2004, respondent Mark S. Oropeza (Oropeza), the President of the Bank,
bought majority shares of stock in the Bank and took over its management which brought about
its gradual rehabilitation. The Bank's finances improved and eventually, its liquidity was
regained.
During the latter part of 2007, Padillo suffered a mild stroke due to hypertension which
consequently impaired his ability to effectively pursue his work. In particular, he was diagnosed
with Hypertension S/P CVA (Cerebrovascular Accident) with short term memory loss, the nature
of which had been classified as a total disability. On September 10, 2007, he wrote a letter
addressed to respondent Oropeza expressing his intention to avail of an early retirement
package. Despite several follow-ups, his request remained unheeded.
On October 3, 2007, Padillo was separated from employment due to his poor and failing health
as reflected in a Certification dated December 4, 2007 issued by the Bank. Not having received
his claimed retirement benefits, Padillo filed on September 23, 2008 with the NLRC Regional
Arbitration Branch No. XI of Davao City a complaint for the recovery of unpaid retirement
benefits. He asserted, among others, that the Bank had adopted a policy of granting its aging
employees early retirement packages, pointing out that one of his co-employees, Nenita Lusan
(Lusan), was accorded retirement benefits in the amount of P348,672.72 when she retired at
the age of only fifty-three (53). The Bank and Oropeza (respondents) countered that the claim of
Padillo for retirement benefits was not favorably acted upon for lack of any basis to grant the
same.
Issue:
Was it legal for the employer Bank to withhold Padillos retirement benefits considering that it
was he who separated himself from employment due to failing health and that he was not
terminated by the Bank?
Is the grant to Luson of an early retirement package considered a company practice such as
would entitle Padillo to the same benefit?
Did the Bank act in bad faith and hence, liable for damages in discriminating against Padillo?

ATHENA M. SALAS | LABOR CASE DIGEST 2015 246

Ruling:
At the outset, it must be maintained that the Labor Code provision on termination on the ground
of disease under Article 297 does not apply in this case, considering that it was the petitioner
and not the Bank who severed the employment relations. As borne from the records, the clear
import of Padillo's September 10, 2007 letter and the fact that he stopped working before the
foregoing date and never reported for work even thereafter show that it was Padillo who
voluntarily retired and that he was not terminated by the Bank.
Article 297 of the Labor Code contemplates a situation where the employer, and not the
employee, initiates the termination of employment on the ground of the latter's disease or
sickness, viz.:
A plain reading of the [Article 297 of the Labor Code] clearly presupposes that it is the employer
who terminates the services of the employee found to be suffering from any disease and whose
continued employment is prohibited by law or is prejudicial to his health as well as to the health
of his co-employees. It does not contemplate a situation where it is the employee who severs
his or her employment ties. This is precisely the reason why Section 8, Rule 1, Book VI of the
Omnibus Rules Implementing the Labor Code, directs that an employer shall not terminate the
services of the employee unless there is a certification by a competent public health authority
that the disease is of such nature or at such a stage that it cannot be cured within a period of six
(6) months even with proper medical treatment. (Emphasis, underscoring and words in brackets
supplied)
Thus, given the inapplicability of Article 297 of the Labor Code to the case at bar, it necessarily
follows that petitioners' claim for separation pay anchored on such provision must be denied.
What is applicable, however, is the Labor Code provision on retirement. In particular, Article 300
of the Labor Code as amended by Republic Act Nos. 7641 32 and 8558 33 partly provides:
Art. 300. Retirement. Any employee may be retired upon reaching the retirement age
established in the collective bargaining agreement or other applicable employment contract.
In case of retirement, the employee shall be entitled to receive such retirement benefits as he
may have earned under existing laws and any collective bargaining agreement and other
agreements: Provided, however, that an employee's retirement benefits under any collective
bargaining and other agreements shall not be less than those provided herein.

In the absence of a retirement plan or agreement providing for retirement benefits of employees
in the establishment, an employee upon reaching the age of sixty (60) years or more, but not
beyond sixty-five (65) years which is hereby declared the compulsory retirement age, who has
served at least five (5) years in the said establishment, may retire and shall be entitled to
retirement pay equivalent to at least one-half (1/2) month salary for every year of service, a
fraction of at least six (6) months being considered as one whole year.
Unless the parties provide for broader inclusions, the term one half (1/2) month salary shall
mean fifteen (15) days plus one-twelfth (1/12) of the 13th month pay and the cash equivalent of
not more than five (5) days of service incentive leaves.

ATHENA M. SALAS | LABOR CASE DIGEST 2015 247

Simply stated, in the absence of any applicable agreement, an employee must (1) retire when
he is at least sixty (60) years of age and (2) serve at least (5) years in the company to entitle
him/her to a retirement benefit of at least one-half (1/2) month salary for every year of service,
with a fraction of at least six (6) months being considered as one whole year. Notably, these age
and tenure requirements are cumulative and non-compliance with one negates the employee's
entitlement to the retirement benefits under Article 300 of the Labor Code altogether.
In this case, it is undisputed that there exists no retirement plan, collective bargaining
agreement or any other equivalent contract between the parties which set out the terms and
condition for the retirement of employees, with the sole exception of the Philam Life Plan which
premiums had already been paid by the Bank.
Neither was it proven that there exists an established company policy of giving early retirement
packages to the Bank's aging employees. In the case of Metropolitan Bank and Trust Company
v. National Labor Relations Commission, it has been pronounced that to be considered a
company practice, the giving of the benefits should have been done over a long period of time,
and must be shown to have been consistent and deliberate. In this relation, petitioners' bare
allegation of the solitary case of Lusan cannot assuming such fact to be true sufficiently
establish that the Bank's grant of an early retirement package to her (Lusan) evolved into an
established company practice precisely because of the palpable lack of the element of
consistency. As such, petitioners' reliance on the Lusan incident cannot bolster their claim.
All told, in the absence of any applicable contract or any evolved company policy, Padillo should
have met the age and tenure requirements set forth under Article 300 of the Labor Code to be
entitled to the retirement benefits provided therein. Unfortunately, while Padillo was able to
comply with the five (5) year tenure requirement as he served for twenty-nine (29) years
he, however, fell short with respect to the sixty (60) year age requirement given that he was only
fifty-five (55) years old when he retired. Therefore, without prejudice to the proceeds due under
the Philam Life Plan, petitioners' claim for retirement benefits must be denied.
Nevertheless, the Court concurs with the CA that financial assistance should be awarded but at
an increased amount. With a veritable understanding that the award of financial assistance is
usually the final refuge of the laborer, considering as well the supervening length of time which
had sadly overtaken the point of Padillo's death an employee who had devoted twenty-nine
(29) years of dedicated service to the Bank the Court, in light of the dictates of social justice,
holds that the CA's financial assistance award should be increased from P50,000.00 to
P75,000.00, still exclusive of the P100,000.00 benefit receivable by the petitioners under the
Philam Life Plan which remains undisputed.
Finally, the Court finds no bad faith in any of respondents' actuations as they were within their
right, absent any proof of its abuse, to ignore Padillo's misplaced claim for retirement benefits.
Respondents' obstinate refusal to accede to Padillo's request is precisely justified by the fact
that there lies no basis under any applicable agreement or law which accords the latter the right
to demand any retirement benefits from the Bank. While the Court mindfully notes that damages
may be recoverable due to an abuse of right under Article 21 in conjunction with Article 19 of
the Civil Code of the Philippines, 36 the following elements must, however, obtain: (1) there is a
legal right or duty; (2) exercised in bad faith; and (3) for the sole intent of prejudicing or injuring
another. Records reveal that none of these elements exists in the case at bar and thus, no
damages on account of abuse of right may be recovered.

ATHENA M. SALAS | LABOR CASE DIGEST 2015 248

Neither can the grant of an early retirement package to Lusan show that Padillo was unfairly
discriminated upon. Records show that the same was merely an isolated incident and
petitioners have failed to show that any bad faith or motive attended such disparate treatment
between Lusan and Padillo. Irrefragably also, there is no showing that other Bank employees
were accorded the same benefits as that of Lusan which thereby dilutes the soundness of
petitioners' imputation of discrimination and bad faith. Verily, it is axiomatic that bad faith can
never be presumed it must be proved by clear and convincing evidence. This petitioners
were unable to prove in the case at bar.

109 .Grace Christian High School vs. Lavandera, GR No. 177845, August 20, 2014
Facts:
Filipinas was employed by petitioner Grace Christian High School (GCHS) as high school
teacher since June 1977, with a monthly salary of P18,662.00 as of May 31, 2001.
On August 30, 2001, Filipinas filed a complaint for illegal (constructive) dismissal, non-payment
of service incentive leave (SIL) pay, separation pay, service allowance, damages, and attorneys
fees against GCHS and/or its principal, Dr. James Tan. She alleged that on May 11, 2001, she
was informed that her services were to be terminated effective May 31, 2001, pursuant to
GCHS retirement plan which gives the school the option to retire a teacher who has rendered at
least 20 years of service, regardless of age. The retirement pay would be one-half () month for
every year of service. At that time, Filipinas was only 58 years old and still physically fit to work.
She pleaded with GCHS to allow her to continue teaching but her services were terminated,
contrary to the provisions of RA 7641 (Retirement Pay Law.)
GCHS denied that they illegally dismissed Filipinas. They asserted that the latter was
considered retired on May 31, 1997 after having rendered 20 years of service pursuant to
GCHS retirement plan and that she was duly advised that her retirement benefits in the amount
of P136,210.00 based on her salary at the time of retirement, i.e., P13,621.00, had been
deposited to the trustee-bank in her name. Nonetheless, her services were retained on a yearly
basis until May 11, 2001 when she was informed that her year-to-year contract would no longer
be renewed.
LA Ruling:
LA ruled that Filipinas was not terminated but was considered as retired and thus, eligible to
receive retirement pay. But denied Filipinas claims for service allowance, salary increase, and
damages for lack of sufficient bases.
NLRC Ruling:
NLRC set aside the LAs award, and held that under Article 287 of the Labor Code, as amended
by RA 7641, the retirement package consists of 15 days salary, plus13th month pay and SIL pay
pro-rated to their one-twelfth (1/12) equivalent.
CA Ruling:

ATHENA M. SALAS | LABOR CASE DIGEST 2015 249

CA affirmed with modification the NLRCs Decision. It held the computation of one-half month
salary by equating it to 22.5 days which is arrived at after adding 15 days plus 2.5 days
representing one-twelfth of the 13th month pay, plus 5 days of service incentive leave
Issue:
Whether or not the CA committed reversible error in using the multiplier 22.5 days in
computing the retirement pay differentials of Filipinas.
Ruling:
No.
RA 7641 provides for the rules on retirement pay to qualified private sector employees in the
absence of any retirement plan in the establishment. The said law states that an employees
retirement benefits under any CBA and other agreements shall not be less than those provided
under the same that is, at least one-half () month salary for every year of service, a fraction
of at least six (6) months being considered as one whole year and that [u]nless the parties
provide for broader inclusions, the term one-half () month salary shall mean fifteen (15) days
plus one-twelfth (1/12) of the 13th month pay and the cash equivalent of not more than five (5)
days of service incentive leaves.
In this case, GCHS has a retirement plan for its faculty and non-faculty members, which gives it
the option to retire a teacher who has rendered at least 20 years of service, regardless of age,
with a retirement pay of one-half () month for every year of service. However, that GCHS
computed Filipinas retirement pay without including one-twelfth (1/12) of her 13th month pay
and the cash equivalent of her five (5) days SIL, both the NLRC and the CA correctly ruled that
Filipinas retirement benefits should be computed in accordance with Article 287 of the Labor
Code, as amended by RA 7641, being the more beneficent retirement scheme.

Thus, the term one-half month salary shall include all the following:

(a) Fifteen (15) days salary of the employee based on his latest salary rate
(b) The cash equivalent of not more than five (5) days of service incentive leave;
(c) One-twelfth of the 13th month pay due the employee.
(d) All other benefits that the employer and employee may agree upon that should be included
in the computation of the employees retirement pay.

110. Intel Technology Phils Inc. vs. NLRC et al., GR No. 200575, February 5, 2014
Facts:
Cabiles was initially hired by Intel Phil. on April 16, 1997 as an Inventory Analyst. He was
subsequently promoted several times over the years and was also assigned at Intel Arizona and

ATHENA M. SALAS | LABOR CASE DIGEST 2015 250

Intel Chengdu. He later applied for a position at Intel Semiconductor Limited Hong Kong (Intel
HK).
Cabiles was offered the position of Finance Manager by Intel HK. On January 31, 2007, Cabiles
signed the job offer and started working on February 1, 2007. On September 8, 2007, after
seven (7) months of employment, Cabiles resigned from Intel HK.
About two years thereafter, Cabiles filed a complaint for non-payment of retirement benefits and
for moral and exemplary damages with the NLRC Regional Arbitration Branch-IV. He insisted
that he was employed by Intel for 10 years and 5 months from April 1997 to September 2007
a period which included his seven (7) month stint with Intel HK. Thus, he believed he was
qualified to avail of the benefits under the companys retirement policy allowing an employee
who served for 10 years or more to receive retirement benefits.
Petitioner Position:
It contends that he is disqualified to receive the benefits for his failure to complete the required
minimum ten (10) years of service as he resigned to assume new responsibilities with Intel HK
effective February 1, 2007.
Respondents Position:
Cabiles submits for his entitlement to retirement pay as he was under the employ of Intel Phil.
for more than ten (10) years in accordance with the prevailing retirement policy. Cabiles views
his employment with Intel HK as a continuation of his service with Intel Phil. alleging that it was
but an assignment by his principal employer, similar to his assignments to Intel Arizona and Intel
Chengdu. Having rendered 9.5 years of service with Intel Phil. and an additional seven months
with Intel HK, he claims that he had completed the required 10 year continuous service with
Intel Phil., thus, qualifying him for retirement benefits.
Labor Arbiters Ruling
On March 18, 2010, the LA found that Cabiles did not sever his employment with Intel Phil.
when he moved to Intel HK, similar to the instances when he was assigned at Intel Arizona and
Intel Chengdu, thus entitled to Retirement pay.
NLRC Ruling
NLRC determined that his decision to move to Intel HK was not definitive proof of permanent
severance of his ties with Intel Phil. It treated his transfer to Hong Kong as akin to his overseas
assignments in Arizona and Chengdu.
Issue:
WON there was severance of relation between Cabiles and Intel Philippines when he assumed
new responsibilities with Intel HK, thus disqualified to receive retirement benefits for failure to
complete the required 10 years of service
Ruling:
Cabiles Resigned from Intel Philippines

ATHENA M. SALAS | LABOR CASE DIGEST 2015 251

Resignation is the formal relinquishment of an office, the overt act of which is coupled with an
intent to renounce. This intent could be inferred from the acts of the employee before and after
the alleged resignation.
In this case, Cabiles, while still on a temporary assignment in Intel Chengdu, was offered by
Intel HK the job of a Finance Manager. His acceptance of the offer meant letting go of the
retirement benefits he now claims as he was informed through email correspondence that his
9.5 years of service with Intel Phil. would not be rounded off in his favor. All these are indicative
of the clearest intent of Cabiles to sever ties with Intel Phil. He chose to forego his tenure with
Intel Phil., with all its associated benefits, in favor of a more lucrative job for him and his family
with Intel HK.
Undoubtedly, Cabiles decision to move to Hong Kong required the abandonment of his
permanent position with Intel Phil. in order for him to assume a position in an entirely different
company. Clearly, the "transfer" was more than just an assignment. It constituted a severance of
Cabiles relationship with Intel Phil., for the assumption of a position with a different employer,
rank, compensation and benefits.

111. Sutherland & Global Services Phils Inc., vs. Labrador, GR No. 193107, March 24, 2014
Facts:
Petitioner Sutherland Global Services (Philippines), Inc. (Sutherland)is engaged in the business
of process outsourcing and technology consultingservices for international clients.In August
2006, Sutherland hired Labrador as one of its call center agents with the main responsibility
ofanswering various queries and complaints through phoned-in calls.
In his two years of working at Sutherland, Labrador committed several infractions.But it was only
on June 17, 2008 that Labrador was finally charged with violation for transgressing the NonCompliance Sale Attribute policy clause stated in the Employee Handbook. Allegedly, on May
13, 2008, one of Sutherlands customers complained that Labrador
Initially asked for her credit card account, but only for purposes of verification. As it turned out, a
second account was created and a new order was placed under the same customers name.
Thus, two sets of packages were shipped to the customer who had to pay twice for the same
product.
Under Sutherlands Employee Handbook, Labradors action is classified as an act of dishonesty
or fraud.6 On May 24, 2008, Sutherland sent Labrador a Notice to Explain7 in writing why he
should not be held administratively liable.
After investigation, a recommendation was issued finding Labrador guilty of violating the
Employee Handbook due to gross or habitual neglect of duty.The recommendation further
stated:
With (sic) the request of Mr. Larry Labrador (Customer ServiceRepresentative UOLIB Sales)
for resignation instead of termination, due to humanitarian purposes and his stay and
contribution to the account, SGS Management allows his request of resigning from the
company, ergo: he shall resign from the company effective immediately.

ATHENA M. SALAS | LABOR CASE DIGEST 2015 252

On June 17, 2008, Labrador submitted his resignation letter.


Issue:
Whether the CA erred in ruling that Labrador was illegally terminated and did not voluntarily
resign
Rulings:
We have consistently ruled that the power to dismiss an employee is a recognized prerogative
inherent in the employer's right to freely manage and regulate his business. The law, however,
in protecting the rights of the laborers, authorizes neither oppression nor self-destruction of the
employer. The worker's right to security of tenure is not an absolute right, for the law provides
that he may be dismissed for cause. Furthermore, Article 282 of the Labor Code provides that
an employee may be terminated from the service on either of the following just causes:
Art. 282. Termination by employer. - An employer may terminate an employment for any of the
following causes:
xxxxxxxxxx
2. Gross and habitual neglect by the employee of his duties;
xxxxxxxxx
5. Other causes analogous to the foregoing.

The failure to faithfully comply with the company rules and regulations is considered to be a just
cause in terminating ones employment, depending on the nature, severity and circumstances of
non-compliance. An employer has the right to regulate, according to its discretion and best
judgment, all aspects of employment, including work assignment, working methods, processes
to be followed, working regulations, transfer of employees, work supervision, lay-off of workers
and the discipline, dismissal and recall of workers.
Thus, it was within Sutherlands prerogative to terminate Labradors employment when he
committed a serious infraction and, despite a previous warning, repeated it. To reiterate, he
opened another client account without the latters consent, with far-reaching and costly effects
on the company.
For one, the repeated past infractions would have resulted in negative feedbacks on
Sutherlands performance and reputation. It would likewise entail additional administrative
expense since Sutherland would have to address the complaints an effort that would entail
investigation costs and the return of the doubly-delivered merchandise. As a rule, an employer
cannot be compelled to continue with the employment of workers when continued employment
will prove inimical to the employer's interests.
To Sutherlands credit, it duly complied with the procedural requirement in dismissing an
employee; it clearly observed both substantive and procedural due process. Its action was
based on a just and authorized cause, and the dismissal was effected after due notice and
hearing.29 After Labradors subsequent infraction, Sutherland sent him a Notice to Explain and
an administrative hearing was thereafter conducted. During the hearing,Labrador himself

ATHENA M. SALAS | LABOR CASE DIGEST 2015 253

admitted his faults. These incidents were properlyrecorded and were properly discussed in
Sutherland's recommendation Butbefore Sutherland could finally pronounce its verdict,
Labrador submittedhis resignation letter, impelled no doubt, as Sutherland alleged, by the
needto protect his reputation and his future employment chances. To be sure,Sutherland's
explanation was not remote, far-fetched or unbelievable eventhe undisputable evidence on
record of infractions.

112. Chiang Kai Shek College et al., vs. Torres, GR No. 189456, April 2, 2014
Facts:
Petitioner Chiang Kai Shek College is a private educational institution that offers elementary to
college education to the public. Individual petitioner Carmelita Espino is the Vice-President of
the school. Respondent had been employed as a grade school teacher of the school from July
1970 until 31 May 2003. The manner of her severance from employment is the matter at hand.
Respondent was accused of leaking a copy of a special quiz given to Grade 5 students of
HEKASI (HEKASI 5). HEKASI stands for Heograpiya, Kasaysayan at Sibika(Geography, History
and Civics). Petitioners came to know about the leakage from one of the teachers of HEKASI 5,
Aileen Benabese (Ms. Benabese). Ms. Benabese narrated that after giving a special quiz, she
borrowed the book of one of her students, Aileen Regine M. Anduyan (Aileen), for the purpose
of making an answer key. When she opened Aileens book, a piece of paper fell. Said paper
turned out to be a copy of the same quiz she had just given and the same already contained
answers.
Ms. Benabese informed the schools Assistant Supervisor Mrs. Gloria Caneda (Mrs. Caneda)
about the incident. Mrs. Caneda conferred with Assistant Supervisor Encarnacion Koo (Mrs.
Koo), who was in charge of the HEKASI area, and Supervisor LuningningTibi (Ms. Tibi). Mrs.
Koo confronted respondent, who had initially denied leaking the test paper but later on admitted
that she gave the test paper to Mrs. TeresitaAnduyan (Mrs. Anduyan), her co-teacher and the
mother of Aileen. Respondent and Mrs. Anduyan were both directed to submit their written
statement on the incident.

According to petitioners, their Investigating Committee had actually decided to terminate


respondent and had in fact prepared a memorandum of termination,10 but respondent allegedly
pleaded for a change of punishment in a short letter dated 5 September 2002, to wit:
Request for change of punishment from termination to suspensionand I am resigning at the end
of the school year.
Mrs. Rosalinda M. Torres
On 10 June 2003, respondent filed a complaint for constructive dismissal and illegal suspension
with the Labor Arbiter.
Issue:

ATHENA M. SALAS | LABOR CASE DIGEST 2015 254

Whether or not in this case the schools act of imposing the penalty of suspension instead of
immediate dismissal from service at the behest of the erring employee, in exchange for the
employees resignation at the end of the school year, constitutes constructive dismissal.
Rulings:
Resignation is the voluntary act of an employee who is in a situation where one believes that
personal reasons cannot be sacrificed for the favor of employment, and opts to leave rather
than stay employed. It is a formal pronouncement or relinquishment of an office, with the
intention of relinquishing the office accompanied by the act of relinquishment. As the intent to
relinquish must concur with the overt act of relinquishment, the acts of the employee before and
after the alleged resignation must be considered in determining whether, he or she, in fact,
intended to sever his or her employment.19
Respondent had admitted to leaking a copy of the HEKASI 5 special quiz. She reluctantly made
the admission and apologized to Mrs. Koo when the latter confronted her. She admitted during
the 28 August 2002 hearing of executing two (2) contradictory statements. On 30 August 2002,
the Investigating Committee found respondent guilty of leaking a copy of the special quiz. Based
on this infraction alone, Chiang Kai Shek College would have been justified to validly terminate
respondent from service. As Associate Justice Antonio T. Carpio emphasized, academic
dishonesty is the worst offense a teacher can make because teachers caught committing
academic dishonesty lose their credibility as educators and cease to be role models for their
students. More so that under Chiang Kai Shek College Faculty Manual, leaking and selling of
test questions is classified as a grave offense punishable by dismissal/termination.
On 5 September 2002, respondent was verbally informed by Mrs. Caneda, Mrs. Carmelita
Espino and Ms. Tibi that she was being dismissed from service. Before the Investigating
Committee could formalize respondents dismissal, respondent handwrote a letter requesting
that the penalty be lowered from dismissal to suspension in exchange for respondents
resignation at the end of the school year.
Given the indications of voluntary resignation, we rule that there is no constructive dismissal in
this case. There is constructive dismissal when there is cessation of work, because continued
employment is rendered impossible, unreasonable or unlikely, as an offer involving a demotion
in rank or a diminution in pay and other benefits. Aptly called a dismissal in disguise or an act
amounting to dismissal but made to appear as if it were not, constructive dismissal may,
likewise, exist if an act of clear discrimination, insensibility, or disdain by an employer becomes
so unbearable on the part of the employee that it could foreclose any choice by him except to
forego his continued employment. There was here no discrimination committed by petitioners.
While respondent did not tender her resignation wholeheartedly, circumstances of her own
making did not give her any other option. With due process, she was found to have committed
the grave offense of leaking test questions. Dismissal from employment was the justified
equivalent penalty. Having realized that, she asked for, and was granted, not just a deferred
imposition of, but also an acceptable cover for the penalty.

113. Goodyear Philippines Inc. vs. Angus, GR No. 185449, November 12, 2014

ATHENA M. SALAS | LABOR CASE DIGEST 2015 255

Principle: In the absence of an express or implied prohibition against it, collection of both
retirement benefits and separation pay upon severance from employment is allowed. This is
grounded on the social justice policy that doubts should always be resolved in favor of labor
rights.
FACTS:
Angus was employed by Goodyear on November 16, 1966 and occupied the position of
Secretary to the Manager of Quality and Technology. In order to maintain the viability of its
operations in the midst of economic reversals, Goodyear implemented a cost saving measure.
Consequently, Angus received from the Human Resources Director of Goodyear a letter
terminating her services since her position is redundant and no longer necessary for its effective
operation.
Angus was given by Goodyear her early retirement benefit having served the company for
almost 35 years and have reached the required minimum age of 55 to qualify for early
retirement to which Angus replied accepting her termination however demanding that she be
given her separation pay as mandated by law, separate from her early retirement benefit,
arguing that nothing in the company's Retirement Plan under the CBA, the CBA itself or the
Employment Contract prohibits the grant of more than one land of separation pay.
Both the Labor Arbiter and the NLRC ruled in favor of Goodyear, that the grant of both
separation pay and retirement benefit is not allowed under the retirement Plan/CBA. Court of
Appeals reversed hence Goodyear appealed to the Supreme Court.
ISSUE:
Whether or not Angus is entitled to both retirement benefits and separation pay
RULING:
Yes.
In Aquino v. National Labor Relations Commission,33 citing Batangas Laguna Tayabas Bus
Company v. Court of Appeals and University of the East v. Hon. Minister of Labor35 the Court
held that an employee is entitled to recover both separation pay and retirement benefits in the
absence of a specific prohibition in the Retirement Plan or CBA. Concomitantly, the Court ruled
that an employee's right to receive separation pay in addition to retirement benefits depends
upon
the
provisions
of
the
company's
Retirement
Plan
and/or
CBA.
Angus presented the parties' 2001-2004 CBA and upon examination of the same, the Court
agrees with her that it does not contain any restriction on the availment of benefits under the
company's Retirement Plan and of separation pay.
Moreover, the Court agrees with the CA that the amount Angus received from petitioners
represented only her retirement pay and not separation pay. A cursory reading of petitioners'
September 18, 2001 letter notifying Angus of her termination from employment shows that they
granted her early retirement benefits pegged at 47 days' pay per year of service. In fact,
petitioners were even explicit in stating in the said letter that the amount she was to receive
would come from the company's Pension Fund, which, as correctly asserted by Angus, was
created to cover retirement benefit payment of employees. In addition, the document showing a
detailed account of Angus' termination benefits speaks for itself as the same is entitled
"Summary
of
Retirement
Pay
and
other
Company
Benefits."

ATHENA M. SALAS | LABOR CASE DIGEST 2015 256

It is worthy to mention at this point that retirement benefits and separation pay are not mutually
exclusive. Retirement benefits are a form of reward for an employee's loyalty and service to an
employer and are earned under existing laws, CBAs, employment contracts and company
policies.On the other hand, separation pay is that amount which an employee receives at the
time of his severance from employment, designed to provide the employee with the wherewithal
during the period that he is looking for another employment and is recoverable only in instances
enumerated under Articles 283 and 284 of the Labor Code or in illegal dismissal cases when
reinstatement is not feasible. In the case at bar, Article 283 clearly entitles Angus to separation
pay apart from the retirement benefits she received from petitioners.

18.PRESCRIPTION OF CLAIMS
19.JURISDICTION OF THE LABOR ARBITER

114. Portillo vs. Rudolf Lietz, Inc. et al., G.R. No. 196539, October 10, 2012

FACTS:

ATHENA M. SALAS | LABOR CASE DIGEST 2015 257

Portillo was employed by Lietz Inc. Intheir letter agreement, one of the terms and conditions is
upon termination of Portillos employment, she will not engaged, for a period of 3 years, directly
or indirectly as employee, manager, proprietor, or solicitor for yourself or others in a similar or
competitive business or the same character of work which she was employed by Lietz Inc. The
agreement is contained in a Goodwill Clause.
The employment relation of the two parties ended however Lietz Inc. learned that Portillo had
been hired by Ed Keller Philippines to head its Pharma Raw Material Department. Ed Keller is a
direct competitor of Lietz. As a consequence of which, Lietz Inc. did not release Portillos
salaries and commission.
Portillo then filed a complaint with the NLRC. Lietz Inc. admitted liability for Portillos money
claim however raised the defense of legal compensation for Portillos alleged breached of the
Goodwill Clause.
Labor Arbiter ruled in favor of Portillo. CA however ruled that while Portillo is entitled to her
claim, legal compensation is proper relying on Paragraph 4 of Article 217 of the Labor Code that
there is "causal connection between [Portillos] monetary claims against [respondents] and the
latters claim from liquidated damages against the former."
ISSUE:
Whether or not the legal compensation is proper on the premise set by CA that the claim of
Portillo and the claim of Lietz arises from employer-employee relation

RULING:
The Labor Arbiter has no jurisdiction.
Art. 217. Jurisdiction of Labor Arbiters and the Commission. (a) Except as otherwise
provided under this code, the Arbiters shall have original and exclusive jurisdiction to hear and
decide, within thirty (30) calendar days after the submission of the case by the parties for
decision without extension, even in the absence of stenographic notes, the following case
involving all workers, whether agricultural or nonagricultural:
x xxx
4. Claims for actual, moral, exemplary and other forms of damages arising from the employeremployee relations; (Underscoring supplied)
Evidently, the Court of Appeals is convinced that the claim for liquidated damages emanates
from the "Goodwill Clause of the employment contract and, therefore, is a claim for damages
arising from the employer-employee relations."
We thereafter ruled that the "reasonable causal connection with the employer-employee
relationship" is a requirement not only in employees money claims against the employer but is,
likewise, a condition when the claimant is the employer.

There is no causal connection between the petitioner employees claim for unpaid wages and
the respondent employers claim for damages for the alleged "Goodwill Clause" violation.
Portillos claim for unpaid salaries did not have anything to do with her alleged violation of the
employment contract as, in fact, her separation from employment is not "rooted" in the alleged

ATHENA M. SALAS | LABOR CASE DIGEST 2015 258

contractual violation. She resigned from her employment. She was not dismissed. Portillos
entitlement to the unpaid salaries is not even contested. Indeed, Lietz Inc.s argument about
legal compensation necessarily admits that it owes the money claimed by Portillo.
The alleged contractual violation did not arise during the existence of the employer-employee
relationship. It was a post-employment matter, a post-employment violation.
In Dai-Chi Electronics Manufacturing Corporation v. Villarama, Jr., which reiterated the San
Miguel ruling and allied jurisprudence, we pronounced that a non-compete clause, as in the
"Goodwill Clause" referred to in the present case, with a stipulation that a violation thereof
makes the employee liable to his former employer for liquidated damages, refers to postemployment relations of the parties, hence falls under the jurisdiction of the civil courts.
We reiterated that Article 217, paragraph 4 does not automatically cover all disputes between an
employer and its employee(s). We noted that the cause of action was within the realm of Civil
Law, thus, jurisdiction over the controversy belongs to the regular courts. At bottom, we
considered that the stipulation referred to post-employment relations of the parties.
It is clear, therefore, that while Portillos claim for unpaid salaries is a money claim that arises
out of or in connection with an employer-employee relationship, Lietz Inc.s claim against Portillo
for violation of the goodwill clause is a money claim based on an act done after the cessation of
the employment relationship. And, while the jurisdiction over Portillos claim is vested in
the labor arbiter, the jurisdiction over Lietz Inc.s claim rests on the regular courts.
As it is, petitioner does not ask for any relief under the Labor Code. It merely seeks to recover
damages based on the parties' contract of employment as redress for respondent's breach
thereof. Such cause of action is within the realm of Civil Law, and jurisdiction over the
controversy belongs to the regular courts. More so must this be in the present case, what with
the reality that the stipulation refers to the postemployment relations of the parties.
In the case at bar, the difference in the nature of the credits that one has against the other,
conversely, the nature of the debt one owes another, which difference in turn results in the
difference of the forum where the different credits can be enforced, prevents the application of
compensation. Simply, the labor tribunal in an employees claim for unpaid wages is
without authority to allow the compensation of such claims against the post-employment
claim of the former employer for breach of a post-employment condition. The labor
tribunal does not have jurisdiction over the civil case of breach of contract.

115. Ace Navigation Co. Inc. et al., vs. Fernandez, G.R. No. 197309, October 10, 2012
Facts:
Seaman Teodorico Fernandez, filed with the NLRC a complaint for disability benefits, with
prayer for moral and exemplary damages, plus attorneys fees, against Ace Navigation Co., Inc.,
Vela International Marine Ltd., and/or Rodolfo Pamintuan.
The petitioners moved to dismiss the complaint, contending that the labor arbiter had no
jurisdiction over the dispute. They argued that exclusive original jurisdiction is with the voluntary
arbitrator or panel of voluntary arbitrators, pursuant to Section 29 of the POEA Standard
Employment Contract (POEA-SEC), since the parties are covered by the AMOSUP-TCC or
AMOSUP-VELA (as later cited by the petitioners) collective bargaining agreement (CBA). Under

ATHENA M. SALAS | LABOR CASE DIGEST 2015 259

Section 14 of the CBA, a dispute between a seafarer and the company shall be settled through
the grievance machinery and mandatory voluntary arbitration.
Fernandez opposed the motion. He argued that inasmuch as his complaint involves a money
claim, original and exclusive jurisdiction over the case is vested with the labor arbiter.
Issue:
WON the labor arbiter has jurisdiction over the dispute.
Ruling:
Section 3, Article XIII (on Social Justice and Human Rights) of the Constitution declares:
xxxx
The State shall promote the principle of shared responsibility between workers and employers
and the preferential use of voluntary modes in settling disputes, including conciliation, and shall
enforce their mutual compliance therewith to foster industrial peace.
Article 260 of the Labor Code (Grievance machinery and voluntary arbitration) states:
The parties to a Collective Bargaining Agreement shall include therein provisions that will
ensure the mutual observance of its terms and conditions. They shall establish a machinery for
the adjustment and resolution of grievances arising from the interpretation or implementation of
their Collective Bargaining Agreement and those arising from the interpretation or enforcement
of company personnel policies.
Article 261 of the Labor Code (Jurisdiction of Voluntary Arbitrators or panel of Voluntary
Arbitrators), on the other hand, reads in part:
The Voluntary Arbitrator or panel of Voluntary Arbitrators shall have original and exclusive
jurisdiction to hear and decide all unresolved grievances arising from the interpretation or
implementation of the Collective Bargaining Agreement and those arising from the interpretation
or enforcement of company personnel policies[.]

Article 262 of the Labor Code (Jurisdiction over other labor disputes) declares:
The Voluntary Arbitrator or panel of Voluntary Arbitrators, upon agreement of the parties, shall
also hear and decide all other labor disputes including unfair labor practices and bargaining
deadlocks.
Further, the POEA-SEC, which governs the employment of Filipino seafarers, provides in its
Section 29 on Dispute Settlement Procedures:
In cases of claims and disputes arising from this employment, the parties covered by a
collective bargaining agreement shall submit the claim or dispute to the original and exclusive
jurisdiction of the voluntary arbitrator or panel of voluntary arbitrators.
xxxx

ATHENA M. SALAS | LABOR CASE DIGEST 2015 260

Under the above-quoted constitutional and legal provisions, the voluntary arbitrator or panel of
voluntary arbitrators has original and exclusive jurisdiction over Fernandezs disability claim.
There is no dispute that the claim arose out of Fernandezs employment with the petitioners and
that their relationship is covered by a CBA. The CBA provides for a grievance procedure for the
resolution of grievances or disputes which occur during the employment relationship and, like
the grievance machinery created under Article 261 of the Labor Code, it is a two-tiered
mechanism, with voluntary arbitration as the last step.
Since the parties used unequivocal language in their CBA for the submission of their disputes to
voluntary arbitration (a condition laid down in Vivero for the recognition of the submission to
voluntary arbitration of matters within the original and exclusive jurisdiction of labor arbiters), we
find that the CA committed a reversible error in its ruling; it disregarded the clear mandate of the
CBA between the parties and the POEA-SEC for submission of the present dispute to voluntary
arbitration.
It bears stressing at this point that we are upholding the jurisdiction of the voluntary arbitrator or
panel of voluntary arbitrators over the present dispute, not only because of the clear language of
the parties CBA on the matter; more importantly, we so uphold the voluntary arbitrators
jurisdiction, in recognition of the States express preference for voluntary modes of dispute
settlement, such as conciliation and voluntary arbitration as expressed in the Constitution, the
law and the rules.
In closing, we quote with approval a most recent Court pronouncement on the same issue, thus

It is settled that when the parties have validly agreed on a procedure for resolving grievances
and to submit a dispute to voluntary arbitration then that procedure should be strictly observed.

116. Cosare vs. Broadcom Asia, Inc. GR No. 201298, February 5, 2014, citing 2010 Matling
Industrial and Commercial Corp et al.,
Facts:
The case stems from a complaint for constructive dismissal, illegal suspension and monetary
claims filed with the National Capital Region Arbitration Branch of the National Labor Relations
Commission (NLRC) by Cosare against the respondents. Cosare was named an incorporator of
Broadcom and was also its Assistant Vice President for Sales (AVP for Sales) and Head of the
Technical Coordination.
Issue:
WON the controversy falls under the jurisdiction of the LA or the regular courts.
Ruling:
It is the LA, and not the regular courts, which has the original jurisdiction over the subject
controversy. An intra-corporate controversy, which falls within the jurisdiction of regular courts,
has been regarded in its broad sense to pertain to disputes that involve any of the following
relationships: (1) between the corporation, partnership or association and the public; (2)
between the corporation, partnership or association and the state in so far as its franchise,

ATHENA M. SALAS | LABOR CASE DIGEST 2015 261

permit or license to operate is concerned; (3) between the corporation, partnership or


association and its stockholders, partners, members or officers; and (4) among the
stockholders, partners or associates, themselves. Settled jurisprudence, however, qualifies that
when the dispute involves a charge of illegal dismissal, the action may fall under the jurisdiction
of the LAs upon whose jurisdiction, as a rule, falls termination disputes and claims for damages
arising from employer-employee relations as provided in Article 217 of the Labor Code.
Consistent with this jurisprudence, the mere fact that Cosare was a stockholder and an officer of
Broadcom at the time the subject controversy developed failed to necessarily make the case an
intra-corporate dispute.
In Matling Industrial and Commercial Corporation v. Coros, the Court distinguished between a
"regular employee" and a "corporate officer" for purposes of establishing the true nature of a
dispute or complaint for illegal dismissal and determining which body has jurisdiction over it.
Succinctly, it was explained that "[t]he determination of whether the dismissed officer was a
regular employee or corporate officer unravels the conundrum" of whether a complaint for illegal
dismissal is cognizable by the LA or by the RTC. "In case of the regular employee, the LA has
jurisdiction; otherwise, the RTC exercises the legal authority to adjudicate.
Applying the foregoing to the present case, the LA had the original jurisdiction over the
complaint for illegal dismissal because Cosare, although an officer of Broadcom for being its
AVP for Sales, was not a "corporate officer" as the term is defined by law. We emphasized in
Real v. Sangu Philippines, Inc. the definition of corporate officers for the purpose of identifying
an intra-corporate controversy. Citing Garcia v. Eastern Telecommunications Philippines, Inc.,
we held:
" Corporate officers in the context of Presidential Decree No. 902-A are those officers of the
corporation who are given that character by the Corporation Code or by the corporations bylaws. There are three specific officers whom a corporation must have under Section 25 of the
Corporation Code. These are the president, secretary and the treasurer. The number of officers
is not limited to these three. A corporation may have such other officers as may be provided for
by its by-laws like, but not limited to, the vice-president, cashier, auditor or general manager.
The number of corporate officers is thus limited by law and by the corporations by-laws."
As may be deduced from the foregoing, there are two circumstances which must concur in order
for an individual to be considered a corporate officer, as against an ordinary employee or officer,
namely: (1) the creation of the position is under the corporations charter or by-laws; and (2) the
election of the officer is by the directors or stockholders. It is only when the officer claiming to
have been illegally dismissed is classified as such corporate officer that the issue is deemed an
intra-corporate dispute which falls within the jurisdiction of the trial courts.
Finally, the mere fact that Cosare was a stockholder of Broadcom at the time of the cases filing
did not necessarily make the action an intra- corporate controversy. "Not all conflicts between
the stockholders and the corporation are classified as intra-corporate. There are other facts to
consider in determining whether the dispute involves corporate matters as to consider them as
intra-corporate controversies." Time and again, the Court has ruled that in determining the
existence of an intra-corporate dispute, the status or relationship of the parties and the nature of
the question that is the subject of the controversy must be taken into account. Considering that
the pending dispute particularly relates to Cosares rights and obligations as a regular officer of
Broadcom, instead of as a stockholder of the corporation, the controversy cannot be deemed

ATHENA M. SALAS | LABOR CASE DIGEST 2015 262

intra-corporate. This is consistent with the "controversy test" explained by the Court in Reyes v.
Hon. RTC, Br. 142, to wit:
Under the nature of the controversy test, the incidents of that relationship must also be
considered for the purpose of ascertaining whether the controversy itself is intra-corporate. The
controversy must not only be rooted in the existence of an intra-corporate relationship, but must
as well pertain to the enforcement of the parties correlative rights and obligations under the
Corporation Code and the internal and intra-corporate regulatory rules of the corporation. If the
relationship and its incidents are merely incidental to the controversy or if there will still be
conflict even if the relationship does not exist, then no intra-corporate controversy exists.

117. 2011 Real Amecos Innovations Inc. et al., vs. Lopez, GR No. 178055, July 2, 2014
Facts:
Amecos received a Subpoena from the Office of the City Prosecutor of Quezon City in
connection with a complaint filed by the Social Security System (SSS) for alleged delinquency in
the remittance of SSS contributions. Amecos attributed its failure to remit the SSS contributions
to herein respondent Eliza R. Lopez (respondent). Amecos claimed respondent refused to
provide Amecos with her SSS Number and to be deducted her contributions; that on the basis of
the foregoing, Amecos no longer enrolled respondent with the SSS and did not deduct her
corresponding contributions.
Amecos eventually settled its obligations with the SSS. Thereafter, petitioners sent a demand
letter to respondent for P27,791.65 representing her share in the SSS contributions but to no
avail. Thus, petitioners filed the instant Complaint for sum of money and damages against
respondent before the Metropolitan Trial Court (MeTC) of Caloocan City.
Respondent filed her Answer with Motion to Dismiss and averred that the regular courts do not
have Jurisdiction over the instant case as it arose out of their employer-employee relationship.

Issue:
WHETHER THE REGULAR CIVIL COURT AND NOT THE LABOR ARBITER OR THE
NATIONAL LABOR RELATIONS COMMISSION HAS JURISDICTION OVER CLAIM[S] FOR
REIMBURSEMENT ARISING FROM EMPLOYER-EMPLOYEE RELATIONS.

Ruling:
This Court holds that as between the parties, Article 217(a)(4) of the Labor Code is applicable.
Said provision bestows upon the Labor Arbiter original and exclusive jurisdiction over claims for
damages arising from employer-employee relations. The observation that the matter of SSS
contributions necessarily flowed from the employer-employee relationship between the parties is
correct; thus, petitioners claims should have been referred to the labor tribunals. In this
connection, it is noteworthy to state that the Labor Arbiter has jurisdiction to award not only the
reliefs provided by labor laws, but also damages governed by the Civil Code.

ATHENA M. SALAS | LABOR CASE DIGEST 2015 263

At the same time, it cannot be assumed that since the dispute concerns the payment of SSS
premiums, petitioners claim should be referred to the Social Security Commission (SSC). As far
as SSS is concerned, there is no longer a dispute, with respect to petitioners accountability to
the System; petitioners already settled their pecuniary obligations to it.

118. Indophil Textile Mills Inc. vs. Engr. Adviento, GR No. 171212, August 4, 2014

Facts:
Petitioner hired respondent Engr. Salvador Adviento as Civil Engineer to maintain its facilities.
Respondent consulted a physician due to recurring weakness and dizziness. He was diagnosed
with Chronic Poly Sinusitis, and thereafter, with moderate, severe and persistent Allergic
Rhinitis. Accordingly, respondent was advised by his doctor to totally avoid house dust mite and
textile dust as it will transmute into health problems.
Distressed, respondent filed a complaint against petitioner with the National Labor Relations
Commission for alleged illegal dismissal and for the payment of backwages, separation pay,
actual damages and attorneys fees.
Subsequently, respondent filed another Complaint with the Regional Trial Court (RTC) alleging
that he contracted such occupational disease by reason of the gross negligence of petitioner to
provide him with a safe, healthy and workable environment.
In reply, petitioner filed a Motion to Dismiss on the ground that: (1) the RTC has no jurisdiction
over the subject matter of the complaint because the same falls under the original and exclusive
jurisdiction of the Labor Arbiter (LA) under Article 217(a)(4) of the Labor Code; and (2) there is
another action pending with the Regional Arbitration Branch III of the NLRC involving the same
parties for the same cause.

Issue:
Whether or not the RTC has jurisdiction over the subject matter of respondents complaint
praying for moral damages, exemplary damages, compensatory damages, anchored on
petitioners alleged gross negligence in failing to provide a safe and healthy working
environment for respondent

Ruling:
The RTC has jurisdiction over respondents complaint.
We have recognized that not all claims involving employees can be resolved solely by our labor
courts, specifically when the law provides otherwise. For this reason, we have formulated the
reasonable causal connection rule, wherein if there is a reasonable causal connection
between the claim asserted and the employer-employee relations, then the case is within the

ATHENA M. SALAS | LABOR CASE DIGEST 2015 264

jurisdiction of the labor courts; and in the absence thereof, it is the regular courts that have
jurisdiction.
We ruled in the recent case of Portillo v. Rudolf Lietz, Inc. that not all disputes between an
employer and his employees fall within the jurisdiction of the labor tribunals such that when the
claim for damages is grounded on the "wanton failure and refusal" without just cause of an
employee to report for duty despite repeated notices served upon him of the disapproval of his
application for leave of absence, the same falls within the purview of Civil Law. As early as
Singapore Airlines Limited v. Pao, we differentiated between abandonment per se and the
manner and consequent effects of such abandonment and ruled that the first, is a labor case,
while the second, is a civil law case:
...petitioner's claim for damages is grounded on the "wanton failure and refusal" without just
cause of private respondent Cruz to report for duty despite repeated notices served upon him of
the disapproval of his application for leave of absence without pay. This, coupled with the further
averment that Cruz "maliciously and with bad faith" violated the terms and conditions of the
conversion training course agreement to the damage of petitioner, removes the present
controversy from the coverage of the Labor Code and brings it within the purview of Civil Law.
In the case at bench, we find that such reasonable causal connection is nil. True, the
maintenance of a safe and healthy workplace is ordinarily a subject of labor cases. More, the
acts complained of appear to constitute matters involving employee-employer relations since
respondent used to be the Civil Engineer of petitioner. However, it should be stressed that
respondents claim for damages is specifically grounded on petitioners gross negligence to
provide a safe, healthy and workable environment for its employees - a case of quasi-delict.
In the case at bar, respondent alleges that due to the continued and prolonged exposure to
textile dust seriously inimical to his health, he suffered work-contracted disease which is now
irreversible and incurable, and deprived him of job opportunities. Clearly, injury and damages
were allegedly suffered by respondent, an element of quasi-delict. Secondly, the previous
contract of employment between petitioner and respondent cannot be used to counter the
element of no pre-existing contractual relation since petitioners alleged gross negligence in
maintaining a hazardous work environment cannot be considered a mere breach of such
contract of employment since the negligence is direct, substantive and independent.
It also bears stressing that respondent is not praying for any relief under the Labor Code of the
Philippines. He neither claims for reinstatement nor backwages or separation pay resulting from
an illegal termination. The cause of action herein pertains to the consequence of petitioners
omission which led to a work-related disease suffered by respondent, causing harm or damage
to his person. Such cause of action is within the realm of Civil Law, and jurisdiction over the
controversy belongs to the regular courts.
Further, it cannot be gainsaid that the claim for damages occurred after the employer-employee
relationship of petitioner and respondent has ceased. Given that respondent no longer demands
for any relief under the Labor Code, Article 217(a)(4) of the Labor Code is inapplicable to the
instant case.
20.2011 NLRC RULES OF PROCEDURE OF THE NLRC

ATHENA M. SALAS | LABOR CASE DIGEST 2015 265

119. Aujero vs. Phil Communications Satellite Corp. G.R. No. 193484, January 18, 2012
Facts:
It was in 1967 that the petitioner started working for respondent Philippine Communications
Satellite Corporation (Philcomsat) as an accountant in the latters Finance Department. On
August 15, 2001 or after 34 years of service, the petitioner applied for early retirement. His
application for retirement was approved, effective September 15, 2001, entitling him to receive
retirement benefits at a rate equivalent to one and a half of his monthly salary for every year of
service. At that time, the petitioner was Philcomsats Senior Vice-President with a monthly salary
of P274,805.00.
On September 12, 2001, the petitioner executed a Deed of Release and Quitclaim in
Philcomsats favor, following his receipt from the latter of a check in the amount of
P9,439,327.91.
Almost three (3) years thereafter, the petitioner filed a complaint for unpaid retirement benefits,
claiming that the actual amount of his retirement pay is P14,015,055.00 and the P9,439,327.91
he received from Philcomsat as supposed settlement for all his claims is unconscionable, which
is more than enough reason to declare his quitclaim as null and void. According to the petitioner,
he had no choice but to accept a lesser amount as he was in dire need thereof and was all set
to return to his hometown and he signed the quitclaim despite the considerable deficiency as no
single centavo would be released to him if he did not execute a release and waiver in
Philcomsats favor.
The petitioner claims that his right to receive the full amount of his retirement benefits, which is
equivalent to one and a half of his monthly salary for every year of service, is provided under the
Retirement Plan that Philcomsat created on January 1, 1977 for the benefit of its employees.

Issue:
Whether the delay in the filing of Philcomsats appeal and posting of surety bond is inexcusable.
Whether the quitclaim executed by the petitioner in Philcomsats favor is valid, thereby
foreclosing his right to institute any claim against Philcomsat.

Ruling:
Procedural rules may be relaxed to give way to the full determination of a case on its merits.
The emerging trend in our jurisprudence is to afford every party-litigant the amplest opportunity
for the proper and just determination of his cause free from the constraints of technicalities.24
Far from having gravely abused its discretion, the NLRC correctly prioritized substantial justice
over the rigid and stringent application of procedural rules. This, by all means, is not a case of
grave abuse of discretion calling for the issuance of a writ of certiorari.
While the law looks with disfavor upon releases and quitclaims by employees who are inveigled
or pressured into signing them by unscrupulous employers seeking to evade their legal
responsibilities, a legitimate waiver representing a voluntary settlement of a laborers claims

ATHENA M. SALAS | LABOR CASE DIGEST 2015 266

should be respected by the courts as the law between the parties.Considering the petitioners
claim of fraud and bad faith against Philcomsat to be unsubstantiated, this Court finds the
quitclaim in dispute to be legitimate waiver.
While the petitioner bewailed as having been coerced or pressured into signing the release and
waiver, his failure to present evidence renders his allegation self-serving and inutile to invalidate
the same. That no portion of his retirement pay will be released to him or his urgent need for
funds does not constitute the pressure or coercion contemplated by law.
120. Sarona vs. NLRC G.R. No. 185280, January 18, 2012

Facts:
Petitioner, a security guard in Sceptre since April 1976, was asked by Sceptres operations
manager on June 2003, to submit a resignation letter as a requirement for an application in
Royale and to fill up an employment application form for the said company. He was then
assigned at Highlight Metal Craft Inc. from July 29 to August 8, 2003 and was later transferred
to Wide Wide World Express Inc. On September 2003, he was informed that his assignment at
WWWE Inc. was withdrawn because Royale has been allegedly replaced by another security
agency which he later discovered to be untrue. Nevertheless, he was once again assigned at
Highlight Metal sometime in September 2003 and when he reported at Royales office on
October 1, 2003, he was informed that he would no longer be given any assignment as
instructed by Sceptres general manager.
He thus filed a complaint for illegal dismissal. The LA ruled in petitioners favor as he found him
illegally dismissed and was not convinced by the respondents claim on petitioners
abandonment. Respondents were ordered to pay back wages computed from the day he was
dismissed up to the promulgation of his decision on May 11, 2005.The LA also ordered for the
payment of separation pay but refused to pierce Royales corporate veil.
Respondents appealed to the NLRC claiming that the LA acted with grave abuse of discretion
upon ruling on the illegal dismissal of petitioner. NLRC partially affirmed the
LAs decision with regard to petitioners illegal dismissal and separation pay but modified the
amount of backwages and limited it to only 3 months of his last month salary reducing P95, 600
to P15, 600 since he worked for Royale for only 1 month and 3 days.
Petitioner did not appeal but raised the validity of LAs findings on piercing Royales corporate
personality and computation of his separation pay and such petition was dismissed by the
NLRC. Petitioner elevated NLRCs decision to the CA on a petition for certiorari, and the CA
disagreed with the NLRCs decision of not proceeding to review the evidence for determining if
Royale is Sceptres alter ego that would warrant the piercing of its corporate veil.

Issue:
Whether Petitions under Rule 45 should only be confined to questions of law.

ATHENA M. SALAS | LABOR CASE DIGEST 2015 267

Whether NLRC committed grave abuse of discretion when it limited itself to reviewing and
deciding only the issues that were elevated on appeal.
Whether Royales corporate fiction should be pierced for the purpose of compelling it to
recognize the petitioners length of service with Sceptre and for holding it liable for the benefits
that have accrued to him arising from his employment with Sceptre.

Ruling:
By way of exception to the general rule, this Court will scrutinize the facts if only to rectify the
prejudice and injustice resulting from an incorrect assessment of the evidence presented.
A Rule 45 Petition should be confined to questions of law. Nevertheless, this Court has the
power to resolve a question of fact, such as whether a corporation is a mere alter ego of another
entity or whether the corporate fiction was invoked for fraudulent or malevolent ends, if the
findings in assailed decision is not supported by the evidence on record or based on a
misapprehension of facts. The question of whether one corporation is merely an alter ego of
another is purely one of fact. So is the question of whether a corporation is a paper company, a
sham or subterfuge or whether the petitioner adduced the requisite quantum of evidence
warranting the piercing of the veil of the respondents corporate personality.
Nevertheless, this Court will not hesitate to deviate from what are clearly procedural guidelines
and disturb and strike down the findings of the CA and those of the labor tribunals if there is a
showing that they are unsupported by the evidence on record or there was a patent
misappreciation of facts. Indeed, that the impugned decision of the CA is consistent with the
findings of the labor tribunals does not per se conclusively demonstrate the correctness thereof.
NLRC cannot be accused of grave abuse of discretion because under the NLRC Rules of
Procedure, the NLRC shall limit itself to reviewing and deciding only the issues that were
elevated on appeal. The NLRC, while not totally bound by technical rules of procedure, is not
licensed to disregard and violate the implementing rules it implemented. In all instances,
however, technicalities were not allowed to stand in the way of equitably and completely
resolving the labor disputes. Technical rules are not binding in labor cases and are not to be
applied strictly if the result would be detrimental to the working man.
Consistent with the mandates of the Labor Code and prevailing jurisprudence notwithstanding,
the NLRC in adjudicating labor cases choose not to encumber itself with technicalities and
limitations consequent to procedural rules if such will only serve as a hindrance to its duty to
decide cases judiciously and in a manner that would put an end with finality to all existing labor
conflicts between the employer and the employee.
The respondents scheme reeks of bad faith and fraud and compassionate justice dictates that
Royale and Sceptre be merged as a single entity, compelling Royale to credit and recognize the
petitioners length of service with Sceptre. The respondents cannot use the legal fiction of a
separate corporate personality for ends subversive of the policy and purpose behind its creation
or which could not have been intended by law to which it owed its being.

121. (a) Salenga et al., vs. Court of Appeals, G.R. No. 174941, February 1, 2012

ATHENA M. SALAS | LABOR CASE DIGEST 2015 268

FACTS:
Petitioner Salenga filed a Complaint for illegal dismissal with a claim for reinstatement and
payment of back wages, benefits, and moral and exemplary damages against respondent Clark
Develoment Corporation (CDC) and Colayco. Earlier, petitioner was informed that the position
of head executive assistant, which he held, was declared redundant by the CDC board of
directors. Thus his employment was to be terminated.Respondents CDC were represented by
the Office of the Government Corporate Counsel (OGCC).
Labor ArbiterDarlucio issued a Decision in favor of petitioner Salenga. At the time the above
Decision was rendered, respondent CDC was already under the leadership of Sergio T. Naguiat.
He subsequently instructed the manager of the Corporate and Legal Services Department and
concurrent corporate board secretary, not to appeal the Decision and to so inform the OGCC.
Despite these instructions, two separate appeals were filed before LA Darlucio. One appeal was
filed by former CDC President/CEO RufoColayco. Another appeal was from the OGCC on
behalf of respondent CDC and RufoColayco. The Memorandum of Appeal was verified and
certified by HilanaTimbol-Roman, the executive vice president of respondent CDC. TimbolRoman and OGCC lawyer Roy Christian Mallari also executed a Joint Affidavit of Declaration
wherein they swore that they were the "respective authorized representative and counsel" of
respondentcorporation. However, the Memorandum of Appeal and the Joint Affidavit of
Declaration were not accompanied by a board resolution from respondent's board of
directors authorizing either Timbol-Roman or Atty. Mallari, or both, to pursue the case or
to file the appeal on behalf of respondent.
Petitioner Salana thereafter opposed the two appeals on the grounds that both appellants,
respondent CDC as allegedly represented by Timbol-Roman and Atty. Mallari and Rufo
Colayco had failed to observe Rule VI, Sections 4 to 6 of the NLRC Rules of Procedure; and
that appellants had not been authorized by respondent's board of directors to represent the
corporation and, thus, they were not the "employer" whom the Rules referred to.
The First Division of the NLRC upheld LA Darlucio's ruling. However, the NLRC held that the
award of exemplary and moral damages were unsubstantiated. Moreover, it also dropped
Colayco as a respondent to the case, since LA Darlucio had failed to provide any ground on
which to anchor the former's solidary liability.
Petitioner Salenga thereafter moved for a partial reconsideration of the above-mentioned
Decision. He sought the reinstatement of the award of exemplary and moral damages. He
likewise insisted that the NLRC should not have entertained the appeal. Petitioner's theory
revolved on the fact that neither Timbol-Roman nor Colayco was authorized to represent
the corporation, so the corporation itself did not appeal LA Darlucio's Decision. As a
result, that Decision should be considered as final and executory.His second Motion for
Reconsideration reiterated his claim that the NLRC should not have entertained the imperfect
appeal, absent a proper verification and certification against forum-shopping from the duly
authorized representative of respondent CDC. Without that authority, neither could the OGCC
act on behalf of the corporation.
The parties underwent several hearings before the NLRC First Division. During these times,
petitioner Salenga demanded from the OGCC to present a board resolution authorizing it or any
other person to represent the corporation in the proceedings. This, the OGCC failed to do.
The First Division of the NLRC held that, absent a board resolution authorizing TimbolRoman to file the appeal on behalf of respondent CDC, the appeal was not perfected and

ATHENA M. SALAS | LABOR CASE DIGEST 2015 269

was thus a mere scrap of paper. In other words, the NLRC had no jurisdiction over the
appeal filed before it.
Respondent CDC filed with the CA Petition for Certiorari. However, the Petition still lacked a
board resolution from the board of directors of respondent corporation authorizing its then
President Angeles to verify and certify the Petition on behalf of the board.
Petitioner Salenga maintains that the CA had no jurisdiction to entertain CDCs petition as a
remedy for an appeal that had actually not been filed, absent a board resolution allowing the
appeal. He claims the CA acted with grave abuse of discretion when it failed to dismiss the
original and supplemental Petitions despite the lack of a board resolution authorizing the filing
thereof and the absence of a proper verification and certification against non-forum shopping.
ISSUE:
Was it proper for the NLRC and CA to entertain respondent CDCs appeal of the decision
rendered by LA Darlucio?
RULING:
The NLRC had no jurisdiction to entertain the appeal filed by Timbol-Roman and former CDC
CEO Colayco.
LA Darlucio rendered a Decision in favor of petitioner. The OGCC, representing respondent
CDC and former CEO Colayco separately appealed from said Decision. Both alleged that they
had filed the proper bond to cover the award granted by LA Darlucio. It is clear from the NLRC
Rules of Procedure that appeals must be verified and certified against forum-shopping by the
parties-in-interest themselves. In the case at bar, the parties-in-interest are petitioner Salenga,
as the employee, and respondent Clark Development Corporation as the employer.
A corporation can only exercise its powers and transact its business through its board of
directors and through its officers and agents when authorized by a board resolution or its
bylaws. The power of a corporation to sue and be sued is exercised by the board of directors.
The physical acts of the corporation, like the signing of documents, can be performed only by
natural persons duly authorized for the purpose by corporate bylaws or by a specific act of the
board. The purpose of verification is to secure an assurance that the allegations in the pleading
are true and correct and have been filed in good faith.
Thus, we agree with petitioner that, absent the requisite board resolution, neither TimbolRoman nor Atty. Mallari, who signed the Memorandum of Appeal and Joint Affidavit of
Declaration allegedly on behalf of respondent corporation, may be considered as the
"appellant" and "employer" referred to by Rule VI, Sections 4 to 6 of the NLRC Rules of
Procedure, which state:

SECTION 4. REQUISITES FOR PERFECTION OF APPEAL. (a) The Appeal


shall be filed within the reglementary period as provided in Section 1 of this
Rule; shall be verified by appellant himself in accordance with Section 4,
Rule 7 of the Rules of Court, with proof of payment of the required appeal fee
and the posting of a cash or surety bond as provided in Section 6 of this Rule;
shall be accompanied by memorandum of appeal in three (3) legibly
typewritten copies which shall state the grounds relied upon and the arguments
in support thereof; the relief prayed for; and a statement of the date when the
appellant received the appealed decision, resolution or order and a certificate

ATHENA M. SALAS | LABOR CASE DIGEST 2015 270

of non-forum shopping with proof of service on the other party of such appeal.
A mere notice of appeal without complying with the other requisites aforestated
shall not stop the running of the period for perfecting an appeal.
Xxx
The OGCC failed to produce any valid authorization from the board of directors despite
petitioner Salenga's repeated demands. In fact, many of its pleadings, representations, and
submissions lacked board authorization.
We cannot agree with the OGCC's attempt to downplay this procedural flaw by claiming that, as
the statutorily assigned counsel for GOCCs, it does not need such authorization. In
Constantino-David v. Pangandaman-Gania, we exhaustively explained why it was necessary for
government agencies or instrumentalities to execute the verification and the certification against
forum-shopping through their duly authorized representatives. We ruled thereon as follows:
But the rule is different where the OSG is acting as counsel of record for a
government agency. For in such a case it becomes necessary to determine
whether the petitioning government body has authorized the filing of the
petition and is espousing the same stand propounded by the OSG. Verily,
it is not improbable for government agencies to adopt a stand different
from the position of the OSG since they weigh not just legal
considerations but policy repercussions as well. They have their
respective mandates for which they are to be held accountable, and the
prerogative to determine whether further resort to a higher court is
desirable and indispensable under the circumstances.
The verification of a pleading, if signed by the proper officials of the client
agency itself, would fittingly serve the purpose of attesting that the
allegations in the pleading are true and correct and not the product of the
imagination or a matter of speculation, and that the pleading is filed in
good faith. Of course, the OSG may opt to file its own petition as a "People's
Tribune" but the representation would not be for a client office but for its own
perceived best interest of the State.
The case of Commissioner of Internal Revenue v. S.C. Johnson and Son, Inc.,
is not also a precedent that may be invoked at all times to allow the OSG to
sign the certificate of non-forum shopping in place of the real party-in-interest.
The ruling therein mentions merely that the certification of non-forum shopping
executed by the OSG constitutes substantial compliance with the rule since
"the OSG is the only lawyer for the petitioner, which is a government agency
mandated under Section 35, Chapter 12, Title III, Book IV, of the 1987
Administrative Code (Reiterated under Memorandum Circular No. 152 dated
May 17, 1992) to be represented only by the Solicitor General."
By its very nature, "substantial compliance" is actually inadequate observance
of the requirements of a rule or regulation which are waived under equitable
circumstances to facilitate the administration of justice there being no damage
or injury caused by such flawed compliance. This concept is expressed in the
statement "the rigidity of a previous doctrine was thus subjected to an inroad
under the concept of substantial compliance." In every inquiry on whether to
accept "substantial compliance," the focus is always on the presence of

ATHENA M. SALAS | LABOR CASE DIGEST 2015 271

equitable conditions to administer justice effectively and efficiently without


damage or injury to the spirit of the legal obligation.
Xxxxxxxxx
The fact that the OSG under the 1987 Administrative Code is the only
lawyer for a government agency wanting to file a petition, or complaint for
that matter, does not operate per se to vest the OSG with the authority to
execute in its name the certificate of non-forum shopping for a client
office. For, in many instances, client agencies of the OSG have legal
departments which at times inadvertently take legal matters requiring
court representation into their own hands without the intervention of the
OSG. Consequently, the OSG would have no personal knowledge of the
history of a particular case so as to adequately execute the certificate of
non-forum shopping; and even if the OSG does have the relevant
information, the courts on the other hand would have no way of
ascertaining the accuracy of the OSG's assertion without precise
references in the record of the case. Thus, unless equitable
circumstances which are manifest from the record of a case prevail, it
becomes necessary for the concerned government agency or its
authorized representatives to certify for non-forum shopping if only to be
sure that no other similar case or incident is pending before any other
court. ASTIED
We recognize the occasions when the OSG has difficulty in securing the
attention and signatures of officials in charge of government offices for the
verification and certificate of non-forum shopping of an initiatory pleading. This
predicament is especially true where the period for filing such pleading is nonextendible or can no longer be further extended for reasons of public interest
such as in applications for the writ of habeas corpus, in election cases or where
sensitive issues are involved. This quandary is more pronounced where public
officials have stations outside Metro Manila.
But this difficult fact of life within the OSG, equitable as it may seem, does not
excuse it from wantonly executing by itself the verification and certificate of
non-forum shopping. If the OSG is compelled by circumstances to verify and
certify the pleading in behalf of a client agency, the OSG should at least
endeavor to inform the courts of its reasons for doing so, beyond instinctively
citing City Warden of the Manila City Jail v. Estrella and Commissioner of
Internal Revenue v. S.C. Johnson and Son, Inc.
Henceforth, to be able to verify and certify an initiatory pleading for nonforum shopping when acting as counsel of record for a client agency, the
OSG must (a) allege under oath the circumstances that make signatures
of the concerned officials impossible to obtain within the period for filing
the initiatory pleading; (b) append to the petition or complaint such
authentic document to prove that the party-petitioner or complainant
authorized the filing of the petition or complaint and understood and
adopted the allegations set forth therein, and an affirmation that no action
or claim involving the same issues has been filed or commenced in any
court, tribunal or quasi-judicial agency; and, (c) undertake to inform the
court promptly and reasonably of any change in the stance of the client
agency.

ATHENA M. SALAS | LABOR CASE DIGEST 2015 272

Anent the document that may be annexed to a petition or complaint under


letter (b) hereof, the letter-endorsement of the client agency to the OSG,
or other correspondence to prove that the subject-matter of the initiatory
pleading had been previously discussed between the OSG and its client,
is satisfactory evidence of the facts under letter (b) above. In this
exceptional situation where the OSG signs the verification and certificate
of non-forum shopping, the court reserves the authority to determine the
sufficiency of the OSG's action as measured by the equitable
considerations discussed herein. (Emphasis ours, italics provided)
The ruling cited above may have pertained only to the Office of the Solicitor General's
representation of government agencies and instrumentalities, but we see no reason why this
doctrine cannot be applied to the case at bar insofar as the OGCC is concerned.
While in previous decisions we have excused transgressions of these rules, it has always been
in the context of upholding justice and fairness under exceptional circumstances. In this case,
though, respondent failed to provide any iota of rhyme or reason to compel us to relax these
requirements. Instead, what is clear to us is that the so-called appeal was done against the
instructions of then President/CEO Naguiat not to file an appeal. Timbol-Roman, who signed the
Verification and the Certification against forum-shopping, was not even an authorized
representative of the corporation. The OGCC was equally remiss in its duty. It ought to have
advised respondent corporationthe proper procedure for pursuing an appeal. Instead, it
maintained the appeal and failed to present any valid authorization from respondent corporation
even after petitioner had questioned OGCC's authority all throughout the proceedings. Thus, it
is evident that the appeal was made in bad faith.
Time and again, we have said that the perfection of an appeal within the period prescribed by
law is jurisdictional, and the lapse of the appeal period deprives the courts of jurisdiction to alter
the final judgment. Thus, there is no other recourse but to respect the findings and ruling of the
labor arbiter. Clearly, therefore, the CA committed grave abuse of discretion in entertaining the
Petition filed before it after the NLRC had dismissed the case based on lack of jurisdiction. The
assailed CA Decision did not even resolve petitioner Salenga's consistent and persistent claim
that the NLRC should not have taken cognizance of the appeal in the first place, absent a board
resolution. Thus, LA Darlucio's Decision with respect to the liability of the corporation still stands.

121. (b) Lockheed Detective & Watchman Agency, G.R. No. 185918, April 18, 2012
FACTS:
Lockheed Detective and Watchman Agency, Inc. entered into a contract for security services
with University of the Philippines. Separate claims were filed against them by security guards for
payment of underpaid wages, among others.
Lockheed and UP as job contractor and principal, respectively, were held solidarily liable for all
the claims. Further, UP, as third-party respondent, was held liable to Lockheed, as third-party
complainant and cross-claimant, for the unpaid legislated salary increases of the security
guards.
A writ of execution was later issued, with NLRC directing that the satisfaction of the judgment
award in favor of Lockheed will be only against the funds of UP which are not identified as
public funds. Consequently, Lockheed filed and was granted its motion for the issuance of an

ATHENA M. SALAS | LABOR CASE DIGEST 2015 273

alias writ of execution. A Notice of Garnishment was issued to Philippine National Bank UP
Diliman Branch for the satisfaction of the award.

UP filed an Urgent Motion to Quash Garnishment. UP contended that the funds being subjected
to garnishment at PNB are government/public funds. The funds are under the name of UP
System Trust Receipts, earmarked for Student Guaranty Deposit, Scholarship Fund, Student
Fund, etc. UP argued that as public funds, the subject PNB account cannot be disbursed except
pursuant to an appropriation required by law. The Labor Arbiter, however, dismissed the urgent
motion for lack of merit. The amount was withdrawn by the sheriff from UP's PNB account.
Upon UPs Petition for Certiorari, the CA did not depart from its findings that the funds covered
in the savings account sought to be garnished do not fall within the classification of public funds.
However, it reconsidered UPs Motion to Quash Garnishment, in light of the ruling in the case of
National Electrification Administration v. Moraleswhich mandates that all money claims against
the government must first be filed with the Commission on Audit (COA).
Lockheed contends that UP has its own separate and distinct juridical entity from the national
government and has its own charter. Thus, it can be sued and be held liable. Lastly, Lockheed
contends that UP cannot anymore seek the quashal of the writ of execution and notice of
garnishment as they are already fait accompli.
ISSUE:
Was the garnishment proper?
RULING:
We agree with UP that there was no point for Lockheed in discussing the doctrine of state
immunity from suit as this was never an issue in this case. Clearly, UP consented to be sued
when it participated in the proceedings below. What UP questions is the hasty garnishment of its
funds in its PNB account.
This Court finds that the CA correctly applied the NEA case. Like NEA, UP is a juridical
personality separate and distinct from the government and has the capacity to sue and be sued.
Thus, also like NEA, it cannot evade execution, and its funds may be subject to
garnishment or levy. However, before execution may be had, a claim for payment of the
judgment award must first be filed with the COA. Under Commonwealth Act No. 327, as
amended by Section 26 of P.D. No. 1445, it is the COA which has primary jurisdiction to
examine, audit and settle "all debts and claims of any sort" due from or owing the Government
or any of its subdivisions, agencies and instrumentalities, including government-owned or
controlled corporations and their subsidiaries. With respect to money claims arising from the
implementation of Republic Act No. 6758, their allowance or disallowance is for COA to decide,
subject only to the remedy of appeal by petition for certiorari to this Court.
We cannot subscribe to Lockheed's argument that NEA is not similarly situated with UP
because the COA's jurisdiction over the latter is only on post-audit basis. A reading of the
pertinent Commonwealth Act provision clearly shows that it does not make any distinction as to
which of the government subdivisions, agencies and instrumentalities, including governmentowned or controlled corporations and their subsidiaries whose debts should be filed before the
COA.
As to the fait accompli argument of Lockheed, contrary to its claim that there is nothing that can
be done since the funds of UP had already been garnished, since the garnishment was

ATHENA M. SALAS | LABOR CASE DIGEST 2015 274

erroneously carried out and did not go through the proper procedure (the filing of a claim with
the COA), UP is entitled to reimbursement of the garnished funds plus interest of 6% per
annum, to be computed from the time of judicial demand to be reckoned from the time UP filed
a petition for certiorari before the CA which occurred right after the withdrawal of the garnished
funds from PNB.

122. 3rd Alert Security & Detective Services Inc. vs. Navia, G.r. No. 200653, June
13, 2012
FACTS:
The labor arbiter, NLRC and CA were unanimous in deciding that Navia was illegally dismissed
by 3rd Alert Security and Detective Services. 3 rd Alerts appeals and motions for reconsideration
with NLRC and CA were all denied.
In the meantime, the NLRC issued an Entry of Judgment certifying that its resolution declaring
Navias dismissal illegal has become final and executory. Thus, Navia filed with the labor arbiter
an ex-parte motion for recomputation of back wages and an ex-parte motion for execution
based on the recomputed back wages. The labor arbiter issued a writ of execution to enforce
the recomputed monetary awards.
3rd Alert appealed the recomputed amount stated in the writ of execution to the NLRC. 3rd Alert
also alleged that the writ was issued with grave abuse of discretion since there was already a
notice of reinstatement sent to Navia.
The NLRC dismissed the appeal, ruling that 3rd Alert is guilty of bad faith since there was no
earnest effort to reinstate Navia. The NLRC also ruled that there was no notice or reinstatement
sent to Navia's counsel. The CA also found the petition without merit because Navia had not
been reinstated either physically or in the payroll.

ISSUE:
Was the writ of execution issued with grave abuse of discretion?
RULING:
Article 223 of the Labor Code provides that in case there is an order of reinstatement, the
employer must admit the dismissed employee under the same terms and conditions, or merely
reinstate the employee in the payroll. The order shall be immediately executory. Thus, 3rd
Alert cannot escape liability by simply invoking that Navia did not report for work. The law states
that the employer must still reinstate the employee in the payroll. Where reinstatement is no
longer viable as an option, separation pay equivalent to one (1) month salary for every year of
service could be awarded as an alternative.
Since the proceedings below indicate that 3rd Alert failed to adduce additional evidence to show
that it tried to reinstate Navia, either physically or in the payroll, we adopt as correct the finding
that there was no earnest effort to reinstate Navia. The CA was correct in affirming the judgment

ATHENA M. SALAS | LABOR CASE DIGEST 2015 275

of the NLRC in this regard.To reiterate, no indication exists showing that 3rd Alert exerted any
efforts to reinstate Navia; worse, 3rd Alert's lame excuse of having sent a notice of
reinstatement to a certain "Biznar" only compounded the intent to mislead the courts.
Also, the main issue of this case, finding Navia to have been illegally dismissed, has already
attained finality. Litigation must end and terminate sometime and somewhere, and it is essential
for an effective and efficient administration of justice that, once a judgment has become final,
the winning party be not deprived of the fruits of the verdict. The order is to reinstate Navia;
sadly, the mere execution of this judgment has to even reach the highest court of the land,
thereby frustrating the entire judicial process.
"It is settled that in actions for recovery of wages or where an employee was forced to litigate
and incur expenses to protect his right and interest, he is entitled to an award of attorney's
fees." Navia, having been compelled to litigate due to 3rd Alert's failure to satisfy his valid claim,
is also entitled to attorney's fees of ten percent (10%) of the total award at the time of actual
payment, following prevailing jurisprudence.

123. Estate of Dulay vs. Aboitiz Jebsen Maritime, Inc. G.R. No. 172642, June 13, 2012

Facts:
Nelson R. Dulay (Nelson, for brevity) was employed by [herein respondent] General Charterers
Inc. (GCI), a subsidiary of co-petitioner [herein co-respondent] Aboitiz Jebsen Maritime Inc.
since 1986. He initially worked as an ordinary seaman and later as bosun on a contractual
basis. From September 3, 1999 up to July 19, 2000, Nelson was detailed in petitioners vessel,
the MV Kickapoo Belle.
On August 13, 2000, or 25 days after the completion of his employment contract, Nelson died
due to acute renal failure secondary to septicemia. At the time of his death, Nelson was a bona
fide member of the Associated Marine Officers and Seamans Union of the Philippines
(AMOSUP), GCIs collective bargaining agent. Nelsons widow, Merridy Jane, thereafter claimed
for death benefits through the grievance procedure of the Collective Bargaining Agreement
(CBA) between AMOSUP and GCI. However, on January 29, 2001, the grievance procedure
was "declared deadlocked" as petitioners refused to grant the benefits sought by the widow.
On March 5, 2001, Merridy Jane filed a complaint with the NLRC Sub-Regional Arbitration
Board in General Santos City against GCI for death and medical benefits and damages.
Herein respondents, on the other hand, asserted that the NLRC had no jurisdiction over the
action on account of the absence of employer-employee relationship between GCI and Nelson
at the time of the latters death.
Issue:
Whether or not the CA committed error in ruling that the Labor Arbiter has no jurisdiction over
the case.

ATHENA M. SALAS | LABOR CASE DIGEST 2015 276

Ruling:
It is true that R.A. 8042 is a special law governing overseas Filipino workers. However, a careful
reading of this special law would readily show that there is no specific provision thereunder
which provides for jurisdiction over disputes or unresolved grievances regarding the
interpretation or implementation of a CBA. Section 10 of R.A. 8042, which is cited by petitioner,
simply speaks, in general, of "claims arising out of an employer-employee relationship or by
virtue of any law or contract involving Filipino workers for overseas deployment including claims
for actual, moral, exemplary and other forms of damages." On the other hand, Articles 217(c)
and 261 of the Labor Code are very specific in stating that voluntary arbitrators have jurisdiction
over cases arising from the interpretation or implementation of collective bargaining
agreements. Stated differently, the instant case involves a situation where the special statute
(R.A. 8042) refers to a subject in general, which the general statute (Labor Code) treats in
particular.5 In the present case, the basic issue raised by Merridy Jane in her complaint filed
with the NLRC is: which provision of the subject CBA applies insofar as death benefits due to
the heirs of Nelson are concerned. The Court agrees with the CA in holding that this issue
clearly involves the interpretation or implementation of the said CBA. Thus, the specific or
special provisions of the Labor Code govern.

124. Prudential Guarantee and Assurance Employee labor Union vs. NLRC, et al., G.R.
No. 185335,

Facts:
June 13, 2012Vallota commenced his employment with respondent Prudential Guarantee and
Assurance, Inc. (PGAI) on May 16, 1995 as a Junior Programmer assigned to the Electronic
Data Processing (EDP) Department. He reported directly to Gerald Dy Victory, then head of the
EDP, until his replacement by respondent Jocelyn Retizos (Retizos)sometime in 1997.
In August of 2005, Vallota was elected to the Board of Directors of the Union.
On November 11, 2005, PGAIs Human Resource Manager, Atty. Joaquin R. Rillo (Atty. Rillo),
invited Union President, Mike Apostol (Apostol) to his office. Atty. Rillo informed Apostol that
PGAI was going to conduct an on-the-spot security check in the Information and Technology (IT)
Department. Atty. Rillo also requested that Union representatives witness the inspection to
which Apostol agreed.
The inspection team proceeded to the IT Department, and the EDP head, through PGAI
network administrator Angelo Gutierrez (Gutierrez), initiated the spot check of IT Department
computers, beginning with the one assigned to Vallota. After exploring the contents of all the

ATHENA M. SALAS | LABOR CASE DIGEST 2015 277

folders and subfolders in the "My Documents" folder, Gutierrez apparently did not find anything
unusual with Vallotas computer and said "Wala naman, saan dito?" Retizos insisted, "Nandyan
yan," and took over the inspection until she found a folder named "MAA." She then exclaimed,
"Heto oh! Ano to? Bakit may MAA dito?" Retizos asked Vallota, "Are you working for MAA?"
Vallota replied, "Hindi po, MAA mutual life po yan na makikita po sa internet." Gutierrez saved a
copy of the contents of the MAA folder in a floppy disk.3
Sensing that Vallota was being singled out, Apostol insisted that all the computers in the IT
Department, including that of Retizos, be also subjected to a spot security check. Later, at
Retizos office, and in the presence of Atty. Rillo, Vallota was informed that Retizos and Atty.
Rillo would print the files found in his computer under the folder "MAA." Vallota did not object.
After the files were printed, Vallota and the Union Secretary were asked to sign each page of
the printout. Vallota, however, was not given a copy of the printed file.
On November 14, 2005, Vallota received a memorandum4 directing him to explain within 72
hours why highly confidential files were stored in his computer. Vallota responded in writing on
November 21, 2005.6
On December 21, 2005, Vallota was given a notice of termination of his employment effective
January 10, 2006 on the ground of loss of trust and confidence. The decision (AC-05-02) was
embodied in a memorandum12 dated December 21, 2005.
Thus, the petitioners filed a complaint for illegal dismissal with claims for full backwages, moral
and exemplary damages, and attorneys fees.

Issue:
WHETHER OR NOT THE HONORABLE COURT OF APPEALS COMMITTED GRAVE ABUSE
OF DISCRETION IN GIVING LIBERALITY TO PRIVATE RESPONDENTS['] FOUR BLATANT
VIOLATIONS OF THE NLRC RULES OF PROCEDURE

Ruling:
The allegation of grave abuse of discretion is misplaced, as this is an issue appropriate for a
petition for certiorari under Rule 65, not a petition for review on certiorari under Rule 45. There
is no question that grave abuse of discretion or errors of jurisdiction may be corrected only by
the special civil action of certiorari. Such special remedy does not avail in instances of error of
judgment which can be corrected by appeal or by a petition for review. Because the petitioners
availed of the remedy under Rule 45, recourse to Rule 65 cannot be allowed either as an addon or as a substitute for appeal.

125. Radio Philippines Network Inc et al., vs. Yap et al., G.R. No. 187713, August 1, 2012

Facts:

ATHENA M. SALAS | LABOR CASE DIGEST 2015 278

Respondents were employees of RPN and former members of the RPN Employees Union
(RPNEU) whom the RPNEU's Grievance and Investigation Committee recommended for
expulsion from the union. Thereafter, RPN notified the respondents that their employment would
be terminated, whereupon the respondents filed with the LA a complaint for illegal dismissal and
non-payment of benefits.
LA rendered a decision ordering the reinstatement of the respondents with payment of
backwages and full benefits and without loss of seniority rights.
Petitioner submitted a Manifestation that it has complied with the reinstatement of the
complainants. RPN General Manager Linao informed respondents that they had been
reinstated, but only in the payroll. Four (4) days later, the respondents returned to RPN to collect
their salaries, it being a payday; but they were barred entry and the guards manhandled them,
pulled them by the hair and arms and pushed them back to the street.
LA then cited the petitioners for indirect contempt for "committing disobedience to lawful order."
When brought before the NLRC, the body dismissed the same and also denied the petitioners'
motion for reconsideration. It was appealed to the CA which also dismissed the petition as well
as the motion for reconsideration, for failure to submit the documents it enumerated in its
Resolution.
Issue:
Whether or not thepayroll reinstatement of the employees availed by RPN is proper.
Ruling:
Yes. The manner of reinstating a dismissed employee in the payroll generally involves an
exercise of management prerogative.
In the case of Pioneer Texturizing Corp. v. NLRC, it was held that an order reinstating a
dismissed employee is immediately self-executory without need of a writ of execution, in
accordance with the third paragraph of Article 223 of the Labor Code. The article states that the
employee entitled to reinstatement "shall either be admitted back to work under the same terms
and conditions prevailing prior to his dismissal or separation or, at the option of the employer,
merely reinstated in the payroll." Thus, even if the employee is able and raring to return to work,
the option of payroll reinstatement belongs to the employer.
The new NLRC Rules of Procedure, which took effect on January 7, 2006, now requires the
employer to submit a report of compliance within ten (10) calendar days from receipt of the LA's
decision, disobedience to which clearly denotes a refusal to reinstate. The employee need no
longer file a motion for issuance of a writ of execution, since the LA shall thereafter motu
proprio issue the writ. With the new rules, there will be no difficulty in determining the employer's
intransigence in immediately complying with the order.
It has been held that in case of strained relations or non-availability of positions, the employer is
given the option to reinstate the employee merely in the payroll, precisely in order to avoid the
intolerable presence in the workplace of the unwanted employee. The Court explained
in Maranaw Hotel Resort Corporation v. NLRC, thus:

ATHENA M. SALAS | LABOR CASE DIGEST 2015 279

This option [to reinstate a dismissed employee in the payroll] is based on practical
considerations. The employer may insist that the dismissal of the employee was for a just and
valid cause and the latter's presence within its premises is intolerable by any standard; or such
presence would be inimical to its interest or would demoralize the co-employees. Thus, while
payroll reinstatement would in fact be unacceptable because it sanctions the payment of
salaries to one not rendering service, it may still be the lesser evil compared to the intolerable
presence in the workplace of an unwanted employee.
The circumstances of the present case have more than amply shown that the physical
restoration of the respondents to their former positions would be impractical and would hardly
promote the best interest of both parties. Respondents have accused the petitioners of being
directly complicit in the plot to expel them from the union and to terminate their employment,
while petitioners have charged the respondents with trying to sabotage the peace of the
workplace in "furthering their dispute with the union." The resentment and enmity between the
parties have so strained their relationship and even provoked antipathy and antagonism, as
amply borne out by the physical clashes that had ensued every time the respondents attempted
to enter the RPN compound, that respondents' presence in the workplace will not only be
distracting but even disruptive, to say the least.
The proposal to pay the respondents' salaries through ATM cards, now a wide practice cannot
be said to be prejudicial or oppressive since it would not entail any unusual effort by the
respondents to collect their money. As to the respondents' demand to be paid their salaries on
the 15th and 30th of the month along with the other employees, instead of on the 5th and 20th
days, petitioners reason that the salaries must be staggered due to RPN's erratic cash flows.
The law only requires that the fortnightly intervals be observed.
126. Ace Navigation Co., Inc. vs. Fernandez, et al., G.r. No. 197309, October 10, 2012
Facts:
Seaman Teodorico Fernandez, assisted by his wife Glenita, filed with the NLRC a complaint for
disability benefits.
The petitioners moved to dismiss the complaint, contending that the labor arbiter had no
jurisdiction over the dispute. They argued that exclusive original jurisdiction is with the voluntary
arbitrator or panel of voluntary arbitrators, pursuant to Section 29 of the POEA Standard
Employment Contract, since the parties are covered by the AMOSUP-TCC or AMOSUP-VELA.
Under Section 14 of the CBA, a dispute between a seafarer and the company shall be settled
through the grievance machinery and mandatory voluntary arbitration.
The LA denied the motion to dismiss, holding that under Section 10 of RA 8042, the labor arbiter
has original and exclusive jurisdiction over money claims arising out of an employer-employee
relationship or by virtue of any law or contract, notwithstanding any provision of law to the
contrary. NLRC agreed with the LA that the case involves a money claim and is within the
jurisdiction of the labor arbiter.
CA also rejected the petitioners' submission and stressed that the jurisdiction of voluntary
arbitrators is limited to the seafarers' claims which do not fall within the labor arbiter's original
and exclusive jurisdiction or even in cases where the labor arbiter has jurisdiction, the parties
have agreed in unmistakable terms (through their CBA) to submit the case to voluntary
arbitration.

ATHENA M. SALAS | LABOR CASE DIGEST 2015 280

Issue:
Who has the original and exclusive jurisdiction over Fernandez's disability claim the labor
arbiter under Section 10 of R.A. No. 8042, as amended, or the voluntary arbitration mechanism
as prescribed in the parties' CBA and the POEA-SEC?
Ruling:
The answer lies in the State's labor relations policy laid down in the Constitution and fleshed out
in the enabling statute, the Labor Code. Section 3, Article XIII (on Social Justice and Human
Rights) of the Constitution declares:
The State shall promote the principle of shared responsibility between workers and employers
and the preferential use of voluntary modes in settling disputes, including conciliation, and shall
enforce their mutual compliance therewith to foster industrial peace.
Article 260 of the Labor Code (Grievance machinery and voluntary arbitration) states:
The parties to a Collective Bargaining Agreement shall include therein provisions that will
ensure the mutual observance of its terms and conditions. They shall establish a machinery for
the adjustment and resolution of grievances arising from the interpretation or implementation of
their Collective Bargaining Agreement and those arising from the interpretation or enforcement
of company personnel policies.
Article 261 of the Labor Code (Jurisdiction of Voluntary Arbitrators or panel of Voluntary
Arbitrators), on the other hand, reads in part:
The Voluntary Arbitrator or panel of Voluntary Arbitrators shall have original and exclusive
jurisdiction to hear and decide all unresolved grievances arising from the interpretation or
implementation of the Collective Bargaining Agreement and those arising from the interpretation
or enforcement of company personnel policies[.]
Article 262 of the Labor Code (Jurisdiction over other labor disputes) declares:
The Voluntary Arbitrator or panel of Voluntary Arbitrators, upon agreement of the parties, shall
also hear and decide all other labor disputes including unfair labor practices and bargaining
deadlocks.
Further, the POEA-SEC, which governs the employment of Filipino seafarers, provides in its
Section 29 on Dispute Settlement Procedures:
In cases of claims and disputes arising from this employment, the parties covered by a
collective bargaining agreement shall submit the claim or dispute to the original and
exclusive jurisdiction of the voluntary arbitrator or panel of voluntary arbitrators. If the
parties are not covered by a collective bargaining agreement, the parties may at their option
submit the claim or dispute to either the original and exclusive jurisdiction of the National Labor
Relations Commission (NLRC), pursuant to Republic Act (RA) 8042 otherwise known as the
Migrant Workers and Overseas Filipinos Act of 1995 or to the original and exclusive jurisdiction
of the voluntary arbitrator or panel of voluntary arbitrators. If there is no provision as to the
voluntary arbitrators to be appointed by the parties, the same shall be appointed from the
accredited voluntary arbitrators of the National Conciliation and Mediation Board of the
Department of Labor and Employment. [emphasis ours]
Under the above-quoted constitutional and legal provisions, the voluntary arbitrator or panel of
voluntary arbitrators has original and exclusive jurisdiction over Fernandez's disability claim.

ATHENA M. SALAS | LABOR CASE DIGEST 2015 281

There is no dispute that the claim arose out of Fernandez's employment with the petitioners and
that their relationship is covered by a CBA the AMOSUP/TCC or the AMOSUP-VELA CBA.
The CBA provides for a grievance procedure for the resolution of grievances or disputes which
occur during the employment relationship and, like the grievance machinery created under
Article 261 of the Labor Code, it is a two-tiered mechanism, with voluntary arbitration as the last
step.
Contrary to the CA's reading of the CBA's Article 14, there is unequivocal or unmistakable
language in the agreement which mandatorily requires the parties to submit to the grievance
procedure any dispute or cause of action they may have against each other. The relevant
provisions of the CBA state:
14.6Any Dispute, grievance, or misunderstanding concerning any ruling, practice, wages
or working conditions in the COMPANY or any breach of the Contract of Employment, or
any dispute arising from the meaning or application of the provisions of this Agreement
or a claim of violation thereof or any complaint or cause of action that any such Seaman
may have against the COMPANY, as well as complaints which the COMPANY may have
against such Seaman shall be brought to the attention of the GRIEVANCE RESOLUTION
COMMITTEE before either party takes any action, legal or otherwise. Bringing such a
dispute to the Grievance Resolution Committee shall be unwaivable prerequisite or
condition precedent for bringing any action, legal or otherwise, in any forum and the
failure to so refer the dispute shall bar any and all legal or other actions.
14.7a)If by reason of the nature of the Dispute, the parties are unable to amicably settle
the dispute, either party may refer the case to a MANDATORY ARBITRATION
COMMITTEE. The MANDATORY ARBITRATION COMMITTEE shall consist of one
representative to be designated by the UNION, and one representative to be designated by the
COMPANY and a third member who shall act as Chairman and shall be nominated by mutual
choice of the parties. . . .
h)Referral of all unresolved disputes from the Grievance Resolution Committee to the
Mandatory Arbitration Committee shall be unwaivable prerequisite or condition
precedent for bringing any action, claim, or cause of action, legal or otherwise, before
any court, tribunal, or panel in any jurisdiction. The failure by a party or seaman to so
refer and avail oneself to the dispute resolution mechanism contained in this action shall
bar any legal or other action. All parties expressly agree that the orderly resolution of all
claims in the prescribed manner served the interests of reaching settlements or claims in
an orderly and uniform manner, as well as preserving peaceful and harmonious labor
relations between seaman, the Union, and the Company.
What might have caused the CA to miss the clear intent of the parties in prescribing a grievance
procedure in their CBA is, as the petitioners' have intimated, the use of the auxiliary verb "may"
in Article 14.7 (a) of the CBA which, to reiterate, provides that "[i]f by reason of the nature of
the Dispute, the parties are unable to amicably settle the dispute, either party may refer
the case to a MANDATORY ARBITRATION COMMITTEE."
While the CA did not qualify its reading of the subject provision of the CBA, it is reasonable to
conclude that it viewed as optional the referral of a dispute to the mandatory arbitration
committee when the parties are unable to amicably settle the dispute.
We find this a strained interpretation of the CBA provision. The CA read the provision separately,
or in isolation of the other sections of Article 14, especially 14.7 (h), which, in clear, explicit
language, states that the "referral of all unresolved disputes from the Grievance
Resolution Committee to the Mandatory Arbitration Committee shall be unwaivable
prerequisite or condition precedent for bringing any action, claim, or cause of action,

ATHENA M. SALAS | LABOR CASE DIGEST 2015 282

legal or otherwise, before any court, tribunal, or panel in any jurisdiction" and that the
failure by a party or seaman to so refer the dispute to the prescribed dispute resolution
mechanism shall bar any legal or other action.
Read in its entirety, the CBA's Article 14 (Grievance Procedure) unmistakably reflects the
parties' agreement to submit any unresolved dispute at the grievance resolution stage to
mandatory voluntary arbitration under Article 14.7 (h) of the CBA. And, it should be added that,
in compliance with Section 29 of the POEA-SEC which requires that in cases of claims and
disputes arising from a seafarer's employment, the parties covered by a CBA shall submit the
claim or dispute to the original and exclusive jurisdiction of the voluntary arbitrator or panel of
voluntary arbitrators.
Consistent with this finding, Fernandez's contention that his complaint for disability benefits is
a money claim that falls within the original and exclusive jurisdiction of the labor arbiter under
Section 10 of R.A. No. 8042 is untenable. We likewise reject his argument that he never
referred his claim to the grievance machinery (so that no unresolved grievance exists as
required under Article 261 of the Labor Code), and that the parties to the case are not the union
and the employer. Needless to state, no such distinction exists in the parties' CBA and the
POEA-SEC.
It bears stressing at this point that we are upholding the jurisdiction of the voluntary arbitrator or
panel of voluntary arbitrators over the present dispute, not only because of the clear language of
the parties' CBA on the matter; more importantly, we so uphold the voluntary arbitrator's
jurisdiction, in recognition of the State's express preference for voluntary modes of dispute
settlement, such as conciliation and voluntary arbitration as expressed in the Constitution, the
law and the rules.
In closing, we quote with approval a most recent Court pronouncement on the same issue, thus
It is settled that when the parties have validly agreed on a procedure for resolving
grievances and to submit a dispute to voluntary arbitration then that procedure should
be strictly observed.

127. Portillo vs. Lietz Inc. et al., G.R. No. 196539, October 10, 2012
Facts:
In a letter agreement dated 3 May 1991, signed by individual respondent Rudolf Lietz (Rudolf)
and conformed to by Portillo, the latter was hired by the former under the following terms and
conditions:

A copy of [Lietz, Inc.'s] work rules and policies on personnel is enclosed and an
inherent part of the terms and conditions of employment.

We acknowledge your proposal in your application specifically to the effect that


you will not engage in any other gainful employment by yourself or with any
other company either directly or indirectly without written consent of [Lietz,
Inc.], and we hereby accept and henceforth consider your proposal an

ATHENA M. SALAS | LABOR CASE DIGEST 2015 283

undertaking on your part, a breach of which will render you liable to [Lietz, Inc.]
for liquidated damages.

On her tenth (10th) year with Lietz, Inc., specifically on 1 February 2002, Portillo was promoted
to Sales Representative and received a corresponding increase in basic monthly salary and
sales quota. In this regard, Portillo signed another letter agreement containing a "Goodwill
Clause:"

It remains understood and you agreed that, on the termination of your


employment by act of either you or [Lietz, Inc.], and for a period of three (3)
years thereafter, you shall not engage directly or indirectly as employee,
manager, proprietor, or solicitor for yourself or others in a similar or competitive
business or the same character of work which you were employed by [Lietz,
Inc.] to do and perform. Should you breach this good will clause of this
Contract, you shall pay [Lietz, Inc.] as liquidated damages the amount of 100%
of your gross compensation over the last 12 months, it being agreed that this
sum is reasonable and just.

Three (3) years thereafter, on 6 June 2005, Portillo resigned from Lietz, Inc. During her exit
interview, Portillo declared that she intended to engage in business a rice dealership, selling
rice in wholesale.

Subsequently, Lietz, Inc. learned that Portillo had been hired by Ed Keller Philippines, Limited to
head its Pharma Raw Material Department. Ed Keller Limited is purportedly a direct competitor
of Lietz, Inc.

On 14 September 2005, Portillo filed a complaint with the National Labor Relations Commission
(NLRC) for non-payment of 1 1/2 months' salary, two (2) months' commission, 13th month pay,
plus moral, exemplary and actual damages and attorney's fees.

In its position paper, Lietz, Inc. admitted liability for Portillo's money claims in the total amount of
P110,662.16. However, Lietz, Inc. raised the defense of legal compensation: Portillo's money
claims should be offset against her liability to Lietz, Inc. for liquidated damages in the amount of
P869,633.09 7 for Portillo's alleged breach of the "Goodwill Clause" in the employment contract
when she became employed with Ed Keller Philippines, Limited.
Issue:
Simply, the issue is whether Portillo's money claims for unpaid salaries may be offset against
respondents' claim for liquidated damages.
Ruling:

ATHENA M. SALAS | LABOR CASE DIGEST 2015 284

Paragraph 4 of Article 217 of the Labor Code appears to have caused the reliance by the Court
of Appeals on the "causal connection between [Portillo's] monetary claims against [respondents]
and the latter's claim from liquidated damages against the former."

Art. 217. Jurisdiction of Labor Arbiters and the Commission. (a) Except
as otherwise provided under this code, the Arbiters shall have original and
exclusive jurisdiction to hear and decide, within thirty (30) calendar days after
the submission of the case by the parties for decision without extension, even
in the absence of stenographic notes, the following case involving all workers,
whether agricultural or non-agricultural:
xxx xxx xxx
4. Claims for actual, moral, exemplary and other forms of damages
arising from the employer-employee relations;

That the "Goodwill Clause" in this case is likewise a post-employment issue should brook no
argument. There is no dispute as to the cessation of Portillo's employment with Lietz, Inc. She
simply claims her unpaid salaries and commissions, which Lietz, Inc. does not contest. At that
juncture, Portillo was no longer an employee of Lietz, Inc. The "Goodwill Clause" or the "NonCompete Clause" is a contractual undertaking effective after the cessation of the employment
relationship between the parties. In accordance with jurisprudence, breach of the undertaking is
a civil law dispute, not a labor law case.

It is clear, therefore, that while Portillo's claim for unpaid salaries is a money claim that arises
out of or in connection with an employer-employee relationship, Lietz, Inc.'s claim against
Portillo for violation of the goodwill clause is a money claim based on an act done after the
cessation of the employment relationship. And, while the jurisdiction over Portillo's claim is
vested in the labor arbiter, the jurisdiction over Lietz, Inc.'s claim rests on the regular courts.

In the case at bar, the difference in the nature of the credits that one has against the other,
conversely, the nature of the debt one owes another, which difference in turn results in the
difference of the forum where the different credits can be enforced, prevents the application of
compensation. Simply, the labor tribunal in an employee's claim for unpaid wages is without
authority to allow the compensation of such claims against the post employment claim of the
former employer for breach of a post employment condition. The labor tribunal does not have
jurisdiction over the civil case of breach of contract.

We are aware that in Baez v. Hon. Valdevilla, we mentioned that:

Whereas this Court in a number of occasions had applied the jurisdictional


provisions of Article 217 to claims for damages filed by employees [citation
omitted], we hold that by the designating clause "arising from the employeremployee relations" Article 217 should apply with equal force to the claim of

ATHENA M. SALAS | LABOR CASE DIGEST 2015 285

an employer for actual damages against its dismissed employee, where the
basis for the claim arises from or is necessarily connected with the fact of
termination, and should be entered as a counterclaim in the illegal dismissal
case.

While on the surface, Baez supports the decision of the Court of Appeals, the facts beneath
premise an opposite conclusion. There, the salesman-employee obtained from the NLRC a final
favorable judgment of illegal dismissal. Afterwards, the employer filed with the trial court a
complaint for damages for alleged nefarious activities causing damage to the employer.
Explaining further why the claims for damages should be entered as a counterclaim in the illegal
dismissal case, we said:

Even under Republic Act No. 875 (the 'Industrial Peace Act,' now completely
superseded by the Labor Code), jurisprudence was settled that where the
plaintiff's cause of action for damages arose out of, or was necessarily
intertwined with, an alleged unfair labor practice committed by the union, the
jurisdiction is exclusively with the (now defunct) Court of Industrial Relations,
and the assumption of jurisdiction of regular courts over the same is a nullity. To
allow otherwise would be "to sanction split jurisdiction, which is prejudicial to
the orderly administration of justice." Thus, even after the enactment of the
Labor Code, where the damages separately claimed by the employer were
allegedly incurred as a consequence of strike or picketing of the union, such
complaint for damages is deeply rooted from the labor dispute between the
parties, and should be dismissed by ordinary courts for lack of jurisdiction. As
held by this Court in National Federation of Labor vs. Eisma, 127 SCRA 419:

Certainly, the present Labor Code is even more committed to the view
that on policy grounds, and equally so in the interest of greater
promptness in the disposition of labor matters, a court is spared the
often onerous task of determining what essentially is a factual matter,
namely, the damages that may be incurred by either labor or
management as a result of disputes or controversies arising from
employer-employee relations.

There is no causal connection between the petitioner employees' claim for unpaid wages and
the respondent employers' claim for damages for the alleged "Goodwill Clause" violation.
Portillo's claim for unpaid salaries did not have anything to do with her alleged violation of the
employment contract as, in fact, her separation from employment is not "rooted" in the alleged
contractual violation. She resigned from her employment. She was not dismissed. Portillo's
entitlement to the unpaid salaries is not even contested. Indeed, Lietz, Inc.'s argument about
legal compensation necessarily admits that it owes the money claimed by Portillo.
The alleged contractual violation did not arise during the existence of the employer-employee
relationship. It was a post-employment matter, a post-employment violation. Reminders are apt.
That is provided by the fairly recent case of Yusen Air and Sea Services Phils., Inc. v.
Villamor, which harked back to the previous rulings on the necessity of "reasonable causal

ATHENA M. SALAS | LABOR CASE DIGEST 2015 286

connection" between the tortious damage and the damage arising from the employer-employee
relationship. Yusen proceeded to pronounce that the absence of the connection results in the
absence of jurisdiction of the labor arbiter. Importantly, such absence of jurisdiction cannot be
remedied by raising before the labor tribunal the tortious damage as a defense.

128. Gonzales vs. Solid Cement Corp., G.R. No. 198423, October 23, 2012
Facts:

The current petition arose from the execution of the final and executory judgment in the parties'
illegal dismissal dispute (referred to as "original case," docketed in this Court as G.R. No.
165330 and entitled Solid Cement Corporation, et al. v. Leo Gonzales). The Labor
Arbiter (LA) resolved the case at his level on December 12, 2000. Since the LA found that an
illegal dismissal took place, the company reinstated petitioner Gonzales in the payroll on
January 22, 2001.

In the meanwhile, the parties continued to pursue the original case on the merits. The case was
appealed to the National Labor Relations Commission (NLRC) and from there to the Court of
Appeals (CA) on a petition for certiorari under Rule 65 of the Rules of Court. The LA's ruling of
illegal dismissal was largely left undisturbed in these subsequent recourses. The original
case eventually came to this Court. In our Resolutions of March 9, 2005 and June 8, 2005, we
denied the petition of respondent Solid Cement Corporation (Solid Cement) for lack of merit.
Our ruling became final and entry of judgment took place on July 12, 2005.

Soon after its finality, the original case was remanded to the LA for execution. The LA decision
dated December 12, 2000 declared the respondents guilty of illegal dismissal and ordered the
reinstatement of Gonzales to his former position "with full backwages and without loss of
seniority rights and other benefits[.]" Under this ruling, as modified by the NLRC ruling on
appeal, Gonzales was awarded the following:

(1)Backwages in the amount of P636,633.33;


(2)Food and Transportation Allowance in the amount of P18,080.00;
(3)Moral damages in the amount of P100,000.00;
(4)Exemplary damages in the amount of P50,000.00; and
(5)Ten percent (10%) of all sums owing to the petitioner as attorney's fees.

Actual reinstatement and return to work for Gonzales (who had been on payroll
reinstatement since January 22, 2001) came on July 15, 2008.

ATHENA M. SALAS | LABOR CASE DIGEST 2015 287

When Gonzales moved for the issuance of an alias writ of execution on August 4, 2008, he
included several items as components in computing the amount of his backwages. Acting on the
motion, the LA added P57,900.00 as rice allowance and P14,675.00 as medical reimbursement
(with the company's apparent conformity), and excluded the rest of the items prayed for in the
motion, either because these items have been paid or that, based on the records of the case,
Gonzales was not entitled thereto. Under the LA's execution order dated August 18, 2009,
Gonzales was entitled to a total of P965,014.15.
The NLRC, in its decision dated February 19, 2010 and resolution dated May 18, 2010, modified
the LA's execution order by including the following amounts as part of the judgment award:

Additional backwages from Dec. 13, 2000 to Jan. 21, 2001


Salary differentials from year 2000 until August 2008

P50,800.00 9
617,517.48

13th month pay differential

51,459.79

13th month pay for years 2000 and 2001

80,000.00

12% interest from July 12, 2005

878,183.42

This ruling increased Gonzales' entitlement to P2,805,698.04.

On a petition for certiorari under Rule 65 of the Rules of Court, the CA set aside the NLRC's
decision and reinstated the LA's order, prompting Gonzales to come to the Court via a petition
for review on certiorari (docketed as G.R. No. 198423) under Rule 45 of the Rules of Court. In
our Minute Resolutions, we denied Gonzales' Rule 45 petition. At this point came the two
motions now under consideration.
LA ruling:
WHEREFORE, premises considered, respondents are hereby declared guilty of
ILLEGAL DISMISSAL and ordered to reinstate complainant to his former position with
full backwages and without loss of seniority rights and other benefits which to date
amounts (sic)to Six Hundred Thirty Six Thousand and Six Hundred Thirty Three Pesos
and Thirty Three Centavos (P636,633.33).
Further, respondents are jointly and severally liable to pay the following:
1.P18,080 as reimbursement for food and transportation allowance;
2.Five Hundred Thousand (P500,000.00) Pesos as moral damages;
3.Two Hundred Fifty Thousand (P250,000.00) Pesos as exemplary damages; and
4.10% of all sums owing to complainant as attorney's fees.

NLRC Ruling:

ATHENA M. SALAS | LABOR CASE DIGEST 2015 288

WHEREFORE, premises considered, the decision under review is hereby, MODIFIED by


REDUCING the amount of moral and exemplary damages due the complainant to the
sum of P100,000.00 and P50,000.00, respectively.
Further, joint and several liability for the payment of backwages, food and transportation
allowance and attorney's fees as adjudged in the appealed decision is hereby imposed
only upon respondents Allen Querubin and Solid Cement Corporation, the latter having a
personality which is distinct and separate from its officers.
The relief of reinstatement is likewise, AFFIRMED.

CA Ruling:

IN VIEW OF ALL THE FOREGOING, the instant petition is hereby dismissed for lack of
merit. Accordingly, the decision of the Second Division of the NLRC dated 26 March
2002 in NLRC CA No. 027452-01 is hereby AFFIRMED.

Issue:
whether the CA was legally correct in finding that the NLRC acted outside its jurisdiction
when it modified the LA's execution order

Ruling:

Thus, even without proof of nonpayment, the NLRC was right in requiring the payment of the
13th month pay and the salaries due after the LA's decision until the illegally dismissed
petitioner was reinstated in the payroll, i.e., from December 13, 2000 to January 21, 2001. It
follows that the CA was wrong when it concluded that the NLRC acted outside its jurisdiction
by including these monetary awards as items for execution.

These amounts are not excluded from the concept of backwages as the salaries fell due after
Gonzales should have been reinstated, while the 13th month pay fell due for the same period by
legal mandate. These are entitlements that cannot now be glossed over if the final decision on
the merits in this case were to be respected.

The Legal Obstacle: the prohibition


on 2nd motion for reconsideration

The above discussions unavoidably lead to the conclusion that the Court's Minute Resolutions
denying Gonzales' petition were not properly issued and are tainted by the nullity of the CA
decision these Resolutions effectively approved. We do not aim to defend these actions,
however, by mechanically and blindly applying the principle of immutability of judgment, nor by

ATHENA M. SALAS | LABOR CASE DIGEST 2015 289

tolerating the CA's inappropriate application of this principle. The immutability principle, rather
than being absolute, is subject to well-settled exceptions, among which is its inapplicability when
a decision claimed to be final is not only erroneous, but null and void.
We cannot also be oblivious to the legal reality that the matter before us is no longer the validity
of Gonzales' dismissal and the legal consequences that follow matters long laid to rest and
which we do not and cannot now disturb. Nor is the matter before us the additional monetary
benefits that Gonzales claims in his petition, since these essentially involve factual matters that
are beyond a Rule 45 petition to rule upon and correct.

The matter before us in the Rule 45 petition questioning the CA's Rule 65 determination is
the scope of the benefits awarded by the LA, as modified on appeal and ultimately
affirmed by this Court, which ruling has become final and which now must be implemented as
a matter of law.

Given these considerations, to reopen this case on second motion for reconsideration would not
actually embroil the Court with changes in the decision on the merits of the case, but would
confine itself solely to the issue of the CA's actions in the course of determining lack or excess
of jurisdiction or the presence of grave abuse of discretion in reviewing the NLRC's ruling on the
execution aspect of the case.

Additionally, while continued consideration of a case on second motion for reconsideration very
strongly remains an exception, our action in doing so in this case is not without sound legal
justification. An order of execution that varies the tenor of a final and executory judgment is null
and void. This was what the CA effectively did it varied the final and executory judgment of
the LA, as modified on appeal and ultimately affirmed by the Court. We would simply be
enforcing our own Decision on the merits of the original case by nullifying what the CA did.

Viewed in these lights, the recognition of, and our corrective action on, the nullity of the CA's
ruling on the current petition is a duty this Court is under obligation to undertake pursuant to
Section 1, Article VIII of the Constitution. We undertake this corrective action by restoring what
the CA should have properly recognized to be covered by the Decision on the merits of the
original case.

129. Martos et al., vs. New San Jose Builders, G.R. No. 192650, October 24, 2012

FACTS:

New San Jose Builders, Inc. is a corporation engaged in the construction of road, bridges, etc.
San Jose was constrained to slow down and suspend most of the works on projects due to lack
of funds. Thus, the workers were informed that many of them would be laid off and the rest

ATHENA M. SALAS | LABOR CASE DIGEST 2015 290

would be reassigned to other projects. Thus several complaints were filed with the Labor Arbiter
for illegal dismissal and other monetary benefits.

The LA handed down a decision declaring, among others, that petitioner Felix Martoswas
illegally dismissed and entitled to separation pay, backwages and other monetary benefits; and
dismissing, without prejudice, the complaints/claims of the other complainants.

In the NLRC, the petitioner-workers appealed their dismissal without prejudice. Respondent San
Jose also appealed the part where Martos was declared regular and illegally dismissed. The
NLRC ruled in favor of some of the workers.

In the certiorari to the CA, it upheld the ruling of the LA. The CA explained that the NLRC
committed grave abuse of discretion in reviving the complaints of petitioners despite their failure
to verify the same. Out of the 102 complainants, only Martos verified the position paper and his
counsel never offered any explanation for his failure to secure the verification of the others.

Petitioners insist that the lack of verification of a position paper is only a formal and not a
jurisdictional defect.
Respondent says that while it is true that the NLRC Rules must be liberally construed and that
the NLRC is not bound by the technicalities of law and procedure, it should not be the first to
arbitrarily disregard specific provisions of the rules which are precisely intended to assist the
parties in obtaining just, expeditious and inexpensive settlement of labor disputes. It was only
Felix Martos who verified their position paper and their memorandum of appeal. It was only he
alone who was vigilant in looking after his interest and enforcing his rights. Petitioners should be
considered to have waived their rights and interests in the case for their consistent neglect and
passive attitude.

ISSUES:
1] whether or not the CA was correct in dismissing the complaints filed by those petitioners who
failed to verify their position papers; and

2] whether or not Martos should be reinstated.

RULING:

1.The absence of a proper verification is cause to treat the pleading as unsigned and
dismissible.

ATHENA M. SALAS | LABOR CASE DIGEST 2015 291

The lone signature of Martos would have been sufficient if he was authorized by his copetitioners to sign for them. Unfortunately, petitioners failed to adduce proof that he was so
authorized. The liberal construction of the rules may be invoked in situations where there may
be some excusable formal deficiency or error in a pleading, provided that the same does not
subvert the essence of the proceeding and it at least connotes a reasonable attempt at
compliance with the rules.

Considering that the dismissal of the other complaints by the LA was without prejudice, the other
complainants should have taken the necessary steps to rectify their procedural mistake after the
decision of the LA was rendered. They should have corrected this procedural flaw by
immediately filing another complaint with the correct verification this time. Surprisingly, they did
not even attempt to correct this technical blunder. Worse, they committed the same procedural
error when they filed their appeal with the NLRC.

2. As to Martos, the Court agrees that the reinstatement being sought by him was no longer
practicable because of strained relation between the parties. Indeed, he can no longer question
this fact. Thus, the Court deems it fair to award separation pay in lieu of reinstatement. In
addition to his separation pay, Martos is also entitled to payment of full backwages, 13th month
pay, service incentive leave pay, and attorney's fees

130. Building Care Corp. vs. Macaraeg, G.R. No. 198357, December 10, 2012
FACTS:
Petitioners are in the business of providing security services to their clients. They hired
respondent on 1996. However she was relieved of her post and was reassigned on 2008, but
after said period, she was allegedly no longer given any assignment. Thus the filing of a
complaint for illegal dismissal, underpayment of salaries, non-payment of separation pay and
refund of cash bond. Conciliation and mediation proceedings failed, so the parties were ordered
to submit their respective position papers.

On May 13, 2009, the LA decided the case as wanting of merit but ordered the agency to pay
P5000 as financial assistance.
Respondent then filed a Notice of Appeal with the National Labor Relations Commission
(NLRC), but in a Decision dated October 23, 2009, the NLRC dismissed the appeal for
having been filed out of time, thereby declaring that the Labor Arbiter's Decision had become
final and executory on June 16, 2009.
In a petition for certiorari elevated to the CA, the CA awarded reinstatement and backwages.
ISSUE:
Whether the CA erred in liberally applying the rules of procedure and ruling that respondent's
appeal should be allowed and resolved on the merits despite having been filed out of time

RULING:

ATHENA M. SALAS | LABOR CASE DIGEST 2015 292

The Court cannot sustain the CA's Decision.


It should be emphasized that the resort to a liberal application, or suspension of the application
of procedural rules, must remain as the exception to the well-settled principle that rules must be
complied with for the orderly administration of justice.

In this case, the justifications given by the CA for its liberalitywas the fault of respondent's
former counsel. Note, however, that neither respondent nor her former counsel gave any
explanation or reason citing extraordinary circumstances for her lawyer's failure to abide by the
rules for filing an appeal.

It is, however, an oft-repeated ruling that the negligence and mistakes of counsel bind the client.
A departure from this rule would bring about never-ending suits. The only exception would be,
where the lawyer's gross negligence would result in the grave injustice of depriving his client of
the due process of law.

It should also be borne in mind that the right of the winning party to enjoy the finality of the
resolution of the case is also an essential part of public policy and the orderly administration of
justice. Hence, such right is just as weighty or equally important as the right of the losing party
to appeal or seek reconsideration within the prescribed period.

In sum, the Court cannot countenance relaxation of the rules absent the showing of
extraordinary circumstances to justify the same. In this case, no compelling reasons can be
found to convince this Court that the CA acted correctly by according respondent such liberality.

131. Kapisanang Pangkaunlaran ng Kababaihang Potrero, Inc. (KPKPI) v.


Barreno,Ametin, Nonay, Dionisio and Casio, G.R. No. 175900, June 10, 2013
Facts:
On November 1997, the Technology and Livelihood Resource Center tapped KPKI to
participate in its microlending program and was granted a loan for microfinance or re-lending for
the poor. As such, KPKPI hired respondents for its KPKPI Mile Program

ATHENA M. SALAS | LABOR CASE DIGEST 2015 293

On September 20, 2001, respondents filed a Complaint before DOLE-NCR for underpayment of
wages, non-payment of labor standard benefits, namely, legal/special holiday pay, 13th month
pay and service incentive leave pay, and non-coverage with the Social Security System and
Home Development Mutual Fund against KPKPI and its Program Manager (DOLE CASE).
During its pendency, however, respondents were informed of their termination. This prompted
the filing of another Complaint against petitioners, this time for illegal dismissal with prayer for
reinstatement and payment of their money claims before the NLRC (NLRC CASE).
LA: NO forum shopping; the subsequent dismissal of the respondents affected the jurisdiction of
the DOLE-NCR since illegal dismissal cases are beyond the latter's jurisdiction. Necessarily
therefore, the case for money claims pending before the DOLE-NCR had to be consolidated
with the illegal dismissal case before the NLRC.
NLRC: set aside the LA's ruling; respondents guilty of forum shopping in filing the same
complaint against petitioners in two (2) fora, namely the DOLE and the NLRC.
CA: agreed with the NLRC that respondents committed forum shopping in seeking their money
claims before the DOLE and the NLRC. Nonetheless, it declared that the ends of justice would
be better served if respondents would be given the opportunity to be heard on their complaint
for illegal dismissal. Accordingly, CA ordered the remand of the case to the NLRC for further
proceedings on the matter of illegal dismissal, separation pay, damages, and attorney's fees.
Issue: Whether the CA erred in ordering the reinstatement and remand of the NLRC CASE to
the NLRC despite its finding of forum shopping.
Ruling: Respondents are not guilty of forum shopping. SC concurs in the result arrived at
by the CA in remanding the cases for illegal dismissal to the NLRC for resolution of the appeal.
Forum shopping exists "when one party repetitively avails of several judicial remedies in
different courts, simultaneously or successively, all substantially founded on the same
transactions and the same essential facts and circumstances, and all raising substantially the
same issues either pending in, or already resolved adversely, by some other court." What is
truly important to consider in determining whether it exists or not is the vexation caused the
courts and parties-litigants by a party who asks different courts and/or administrative agencies
to rule on the same or related causes and/or grant the same or substantially the same reliefs, in
the process creating the possibility of conflicting decisions being rendered by different fora upon
the same issues.
In the case at bar, there is no identity of causes of action between the cases pending with the
DOLE and the NLRC. The DOLE CASE involved violations of labor standard provisions where
an employer-employee relationship exists. On the other hand, the NLRC CASE questioned the
propriety of respondents' dismissal. No less than the Labor Code provides for these two (2)
separate remedies for distinct causes of action. More importantly, at the time the DOLE CASE
was initiated, respondents' only cause of action was petitioners' violation of labor standard laws
which falls within the jurisdiction of the DOLE. It was only after the same was filed that
respondents were dismissed from employment, prompting the filing of the NLRC CASE, which
is within the mantle of the NLRC's jurisdiction. Under the foregoing circumstances, respondents
had no choice but to avail of different fora.

In cases where the complaint for violation of labor standard laws preceded the termination of the
employee and the filing of the illegal dismissal case, it would not be in consonance with justice

ATHENA M. SALAS | LABOR CASE DIGEST 2015 294

to charge the complainants with engaging in forum shopping when the remedy available to them
at the time their causes of action arose was to file separate cases before different fora.

132. Philippine Transmarine Carriers, Inc. (PTCI) v. Legaspi, G.R. No. 202791, June 10,
2013
Facts: Legaspi was employed as Utility Pastry on board the vessel "Azamara Journey" under
the employment of petitioner PTCI. His employment was covered by a CBA where it was agreed
that the company shall pay a maximum disability compensation of up to US$60,000.00 only.

While on board, Legaspi suffered "Cardiac Arrest S/P ICD Insertation." He was checked by the
ships doctor, prescribed medications and then repatriated for further medical treatment and
exam. The company-designated physician assessed his condition to be Disability Grade 2.

Not satisfied, Legaspi filed a complaint for full and permanent disability compensation against
PTCI before the Labor Arbiter. LA ruled in favor of Legaspi. LA awarded US$80,000.00 based
on the ITF Cruise Ship Model Agreement for Catering Personnel, not on the CBA. Unsatisfied,
PTCI appealed before NLRC. NLRC affirmed the decision of the LA. On September 5, 2010,
NLRC issued the Entry of Judgment stating that its resolution affirming the LA decision had
become final and executory.

On October 22, 2010, during the hearing on the motion for execution before the NLRC, PTCI
agreed to pay Legaspi US$81,320.00. The terms and conditions of which were embodied in the
Receipt of Judgment Award with Undertaking, wherein Legaspi acknowledged receipt of the
said amount and undertook to return it to PTCI in the event the latter's petition
for certiorari would be granted, without prejudice to Legaspis right to appeal. It was also agreed
upon that the remaining balance would be given on the next scheduled conference.

On November 8, 2010, PTCI timely filed a petition for certiorari with the CA. In the meantime, on
March 2, 2011, LA issued a writ of execution which noted PTCIs payment of the amount of
US$81,320.00. In compliance, PTCI tendered to NLRC Cashier the US$8,132.00 attorneys
fees and P3,042.95 execution fees. LA ordered their release to Legaspi.

Unaware of the entry of judgment of the NLRC, payment of US$81,320.00 and the writ of
execution issued by LA, CA partially granted the petition for certiorari and modified the assailed
resolutions of the NLRC, awarding only US$60,000.00 pursuant to the CBA.

PTCI filed its Manifestation with Motion to Amend the Dispositive Portion, submitting to the CA
the writ of execution issued by the LA in support of its motion. PTCI contended that since it had
already paid the total amount of US$89,452.00, it was entitled to the return of the excess
payment in the amount of US$29,452.00. CA denied it and ruled that the petition should have
been dismissed for being moot and academic because the assailed decision of the NLRC had

ATHENA M. SALAS | LABOR CASE DIGEST 2015 295

become final and executory and that the said judgment had been satisfied even before the filing
of the petition for certiorari.

Issues: Whether the petition for certiorari has become moot and academic.

Ruling: The petition for certiorari is not moot.


Section 14, Rule VII of the 2011 NLRC Rules of Procedure provides that decisions, resolutions
or orders of the NLRC shall become final and executory after ten (10) calendar days from
receipt thereof by the parties, and entry of judgment shall be made upon the expiration of the
said period. In St. Martin Funeral Home v. NLRC, however, it was ruled that judicial review of
decisions of the NLRC may be sought via a petition for certiorari before the CA under Rule 65 of
the Rules of Court; and under Section 4 thereof, petitioners are allowed sixty (60) days from
notice of the assailed order or resolution within which to file the petition. Hence, in cases where
a petition for certiorari is filed after the expiration of the 10-day period under the 2011 NLRC
Rules of Procedure but within the 60-day period under Rule 65 of the Rules of Court, the CA
can grant the petition and modify, nullify and reverse a decision or resolution of the NLRC.

Accordingly, in this case, although the petition for certiorari was not filed within the 10-day
period, PTCI timely filed it before the CA within the 60-day reglementary period under Rule 65. It
has, thus, been held that the CA's review of the decisions or resolutions of the NLRC under
Rule 65, particularly those which have already been executed, does not affect their statutory
finality, considering that Section 4, Rule XI of the 2011 NLRC Rules of Procedure, provides that
a petition for certiorari filed with the CA shall not stay the execution of the assailed decision
unless a restraining order is issued.

In the present case, the Receipt of the Judgment Award with Undertaking was fair to both the
employer and the employee. It stipulated that Legaspi should return the amount to PTCI if the
petition for certiorari would be granted but without prejudice to Legaspis right to appeal. The
agreement, thus, provided available remedies to both parties. It is clear that PTCI paid Legaspi
subject to the terms and conditions stated in the Receipt of the Judgment Award with
Undertaking. Both parties signed the agreement. Legaspi neither refuted the agreement nor
claimed that he was forced to sign it against his will. Therefore, the petition for certiorari was not
rendered moot despite PTCIs satisfaction of the judgment award, as Legaspi had obliged
himself to return the payment if the petition would be granted.

Legaspi agreed to the stipulation that he would return the amount paid to him in the event that
the petition for certiorari would be granted. Since the petition was indeed granted by the CA,
albeit partially, Legaspi must comply with the condition to return the excess amount. To allow
now Legaspi to retain the excess money judgment would amount to his unjust enrichment to the
prejudice of PTCI. Legaspi is ORDERED to return the excess amount of payment in the sum of
US$29,452.00 to PTCI, the amount shall earn interest at the rate of 12% per annum from the
finality of this judgment

ATHENA M. SALAS | LABOR CASE DIGEST 2015 296

133. Pasos vs. Phil National Construction Corp. G.R. No. 192394, July 3, 2013
Facts:
Petitioner contends that PNCCs appeal from the Labor Arbiters decision should not have been
allowed since the appeal bond filed was insufficient. He likewise argues that the appellate court
erred in heavily relying in the case of Cagayan Valley Drug Corporation v. Commissioner of
Internal Revenue23 which enumerated the officials and employees who can sign the verification
and certification without need of a board resolution. He contends that in said case, there was
substantial compliance with the requirement since a board resolution was submitted albeit
belatedly unlike in the instant case where no board resolution was ever submitted even
belatedly.
As to the substantive issue, petitioner submits that the CA erroneously concluded that he was a
project employee when there are indicators which point otherwise. He contends that even if he
was just hired for the NAIA 2 Project from April 26, 1996 to July 25, 1996, he was made to work
until August 4, 1998. He also avers the DOLE had certified that he was not among the
employees listed in the termination reports submitted by PNCC which belies the photocopies of
termination reports attached by PNCC to its pleadings listing petitioner as one of the affected
employees. Petitioner points out that said termination reports attached to PNCCs pleadings are
mere photocopies and were not even certified by the DOLE-NCR as true copies of the originals
on file with said office. Further, he argues that in violation of the requirement of Department
Order No. 19 that the duration of the project employment is reasonably determinable, his
contracts for the SM projects did not specify the date of completion of the project nor was the
completion determinable at the time that petitioner was hired.
PNCC counters that documentary evidence would show that petitioner was clearly a project
employee and remained as such until his last engagement. It argues that the repeated rehiring
of petitioner as accounting clerk in different projects did not make him a regular employee. It
also insists that it complied with the reportorial requirements and that it filed and reported the
termination of petitioner upon every completion of project to which he was employed.
Issues:
(1) Should an appeal be dismissed outright if the appeal bond filed is less than the adjudged
amount?
(2) Can the head of the personnel department sign the verification and certification on behalf of
the corporation sans any board resolution or secretarys certificate authorizing such officer to do
the same? and
(3) Is petitioner a regular employee and not a mere project employee and thus can only be
dismissed for cause?
Ruling:
1) Not necessarily. While perfection of an appeal within the reglementary period and in the
manner prescribed by law is jurisdictional, the bond requirement on appeals involving
monetary awards has been relaxed in certain cases. This can only be done where there
was substantial compliance of the Rules or where the appellants, at the very least,

ATHENA M. SALAS | LABOR CASE DIGEST 2015 297

exhibited willingness to pay by posting a partial bond. In the instant case, the Labor
Arbiter in his decision ordered PNCC to pay petitioner back wages amounting
toP422,630.41 and separation pay of P37,662 or a total of P460,292.41. When PNCC
filed an appeal bond amounting to P422,630.41 or at least 90% of the adjudged amount,
there is no question that this is substantial compliance with the requirement that allows
relaxation of the rules.
2) Yes, the head of the personnel department may sign the verification and certification
because he was in a position to assure that the allegations were true and correct and on
the basis of liberality in applying the rules.
We have held that the following officials or employees of the company can sign the
verification and certification without need of a board resolution: (1) the Chairperson of
the Board of Directors, (2) the President of a corporation, (3) the General Manager or
Acting General Manager, (4) Personnel Officer, and (5) an Employment Specialist in a
labor case.
While the above cases do not provide a complete listing of authorized signatories to the
verification and certification required by the rules, the determination of the sufficiency of
the authority was done on a case to case basis. The rationale applied in the foregoing
cases is to justify the authority of corporate officers or representatives of the corporation
to sign the verification or certificate against forum shopping, being "in a position to verify
the truthfulness and correctness of the allegations in the petition."
While we agree with petitioner that in Cagayan Valley, the requisite board resolution was
submitted though belatedly unlike in the instant case, this Court still recognizes the
authority of Mr. Erece, Jr. to sign the verification and certification on behalf of PNCC
sans a board resolution or secretarys certificate as we have allowed in Pfizer, Inc. v.
Galan, one of the cases cited in Cagayan Valley. In Pfizer, the Court ruled as valid the
verification signed by an employment specialist as she was in a position to verify the
truthfulness and correctness of the allegations in the petition despite the fact that no
board resolution authorizing her was ever submitted by Pfizer, Inc. even belatedly. We
believe that like the employment specialist in Pfizer, Mr. Erece, Jr. too, as head of the
Personnel Services Department of PNCC, was in a position to assure that the
allegations in the pleading have been prepared in good faith and are true and correct.
3) Petitioner was a regular employee and enjoys the right to security of tenure.
While for first three months, petitioner can be considered a project employee of PNCC,
his employment thereafter, when his services were extended without any specification of
as to the duration, made him a regular employee of PNCC. And his status as a regular
employee was not affected by the fact that he was assigned to several other projects
and there were intervals in between said projects since he enjoys security of tenure.
134. Nacar vs. Gallery Frames, G.R. No. 189871, August 13, 2013, En banc
Facts:

ATHENA M. SALAS | LABOR CASE DIGEST 2015 298

Petitioner argues that notwithstanding the fact that there was a computation of backwages in the
Labor Arbiters decision, the same is not final until reinstatement is made or until finality of the
decision, in case of an award of separation pay. Petitioner maintains that considering that the
October 15, 1998 decision of the Labor Arbiter did not become final and executory until the April
17, 2002 Resolution of the Supreme Court in G.R. No. 151332 was entered in the Book of
Entries on May 27, 2002, the reckoning point for the computation of the backwages and
separation pay should be on May 27, 2002 and not when the decision of the Labor Arbiter was
rendered on October 15, 1998. Further, petitioner posits that he is also entitled to the payment
of interest from the finality of the decision until full payment by the respondents.
On their part, respondents assert that since only separation pay and limited backwages were
awarded to petitioner by the October 15, 1998 decision of the Labor Arbiter, no more
recomputation is required to be made of said awards. Respondents insist that since the decision
clearly stated that the separation pay and backwages are "computed only up to [the]
promulgation of this decision," and considering that petitioner no longer appealed the decision,
petitioner is only entitled to the award as computed by the Labor Arbiter in the total amount
ofP158,919.92. Respondents added that it was only during the execution proceedings that the
petitioner questioned the award, long after the decision had become final and executory.
Respondents contend that to allow the further recomputation of the backwages to be awarded
to petitioner at this point of the proceedings would substantially vary the decision of the Labor
Arbiter as it violates the rule on immutability of judgments.
Issue:
Whether or not a re-computation in the course of execution of the labor arbiter's original
computation of the awards made, pegged as of the time the decision was rendered and
confirmed with modification by a final CA decision, is legally proper
Ruling:
Yes it was legally proper. A re-computation is necessary as it essentially considered the labor
arbiter's original decision in accordance with its basic component parts as we discussed above.
To reiterate, the first part contains the finding of illegality and its monetary consequences; the
second part is the computation of the awards or monetary consequences of the illegal dismissal,
computed as of the time of the labor arbiter's original decision.
The instant case is similar to the case of Session Delights Ice Cream and Fast Foods v. Court of
Appeals (Sixth Division), wherein the issue submitted to the Court for resolution was the
propriety of the computation of the awards made, and whether this violated the principle of
immutability of judgment. Like in the present case, it was a distinct feature of the judgment of
the Labor Arbiter in the above-cited case that the decision already provided for the computation
of the payable separation pay and backwages due and did not further order the computation of
the monetary awards up to the time of the finality of the judgment. Also in Session Delights, the
dismissed employee failed to appeal the decision of the labor arbiter.
A source of misunderstanding in implementing the final decision in this case proceeds from the
way the original labor arbiter framed his decision. The decision consists essentially of two parts.

ATHENA M. SALAS | LABOR CASE DIGEST 2015 299

The first is that part of the decision that cannot now be disputed because it has been confirmed
with finality. This is the finding of the illegality of the dismissal and the awards of separation pay
in lieu of reinstatement, backwages, attorney's fees, and legal interests.
The second part is the computation of the awards made. On its face, the computation the labor
arbiter made shows that it was time-bound as can be seen from the figures used in the
computation. This part, being merely a computation of what the first part of the decision
established and declared, can, by its nature, be re-computed. This is the part, too, that the
petitioner now posits should no longer be re-computed because the computation is already in
the labor arbiter's decision that the CA had affirmed. The public and private respondents, on the
other hand, posit that a re-computation is necessary because the relief in an illegal dismissal
decision goes all the way up to reinstatement if reinstatement is to be made, or up to the finality
of the decision, if separation pay is to be given in lieu reinstatement.
That the labor arbiter's decision, at the same time that it found that an illegal dismissal had
taken place, also made a computation of the award, is understandable in light of Section 3, Rule
VIII of the then NLRC Rules of Procedure which requires that a computation be made. This
Section in part states:
[T]he Labor Arbiter of origin, in cases involving monetary awards and at all events, as far as
practicable, shall embody in any such decision or order the detailed and full amount awarded.
Clearly implied from this original computation is its currency up to the finality of the labor
arbiter's decision. As we noted above, this implication is apparent from the terms of the
computation itself, and no question would have arisen had the parties terminated the case and
implemented the decision at that point.
However, the petitioner disagreed with the labor arbiter's findings on all counts - i.e., on the
finding of illegality as well as on all the consequent awards made. Hence, the petitioner
appealed the case to the NLRC which, in turn, affirmed the labor arbiter's decision. By law, the
NLRC decision is final, reviewable only by the CA on jurisdictional grounds.
The petitioner appropriately sought to nullify the NLRC decision on jurisdictional grounds
through a timely filed Rule 65 petition for certiorari. The CA decision, finding that NLRC
exceeded its authority in affirming the payment of 13th month pay and indemnity, lapsed to
finality and was subsequently returned to the labor arbiter of origin for execution.
It was at this point that the present case arose. Focusing on the core illegal dismissal portion of
the original labor arbiter's decision, the implementing labor arbiter ordered the award recomputed; he apparently read the figures originally ordered to be paid to be the computation
due had the case been terminated and implemented at the labor arbiter's level. Thus, the labor
arbiter re-computed the award to include the separation pay and the backwages due up to the
finality of the CA decision that fully terminated the case on the merits. Unfortunately, the labor
arbiter's approved computation went beyond the finality of the CA decision (July 29, 2003) and
included as well the payment for awards the final CA decision had deleted - specifically, the
proportionate 13th month pay and the indemnity awards. Hence, the CA issued the decision
now questioned in the present petition.
Consequently, from the above disquisitions, under the terms of the decision which is sought to
be executed by the petitioner, no essential change is made by a recomputation as this step is a
necessary consequence that flows from the nature of the illegality of dismissal declared by the
Labor Arbiter in that decision. A recomputation (or an original computation, if no previous
computation has been made) is a part of the law specifically, Article 279 of the Labor Code

ATHENA M. SALAS | LABOR CASE DIGEST 2015 300

and the established jurisprudence on this provision that is read into the decision. By the nature
of an illegal dismissal case, the reliefs continue to add up until full satisfaction, as expressed
under Article 279 of the Labor Code. The recomputation of the consequences of illegal dismissal
upon execution of the decision does not constitute an alteration or amendment of the final
decision being implemented. The illegal dismissal ruling stands; only the computation of
monetary consequences of this dismissal is affected, and this is not a violation of the principle of
immutability of final judgments.
That the amount respondents shall now pay has greatly increased is a consequence that it
cannot avoid as it is the risk that it ran when it continued to seek recourses against the Labor
Arbiter's decision. Article 279 provides for the consequences of illegal dismissal in no uncertain
terms, qualified only by jurisprudence in its interpretation of when separation pay in lieu of
reinstatement is allowed. When that happens, the finality of the illegal dismissal decision
becomes the reckoning point instead of the reinstatement that the law decrees. In allowing
separation pay, the final decision effectively declares that the employment relationship ended so
that separation pay and backwages are to be computed up to that point.
135. Castells vs. Saudi Arabia Airlines, G.R. No. 188514, Aug 28, 2013
Facts:
Respondent SAUDIA AIRLINES issued a memorandum transferring the petitioners from Manila
to Jeddah due to operational requirements. Centi-Mandanas complied with the transfer order
while Castiled did not. Upon arrival of Cent-Mandanas, she was told that her contract would no
longer be renewed and that she was asked to sign a pre-typed resignation letter. She averred
that while she never wished to resign, SAUDIA left her with no other viable choice as it would
terminate her services anyway. Thus, she filled out the resignation form handed to her. Castells
alleged that upon her non-compliance with the transfer order, she prepared a resignation letter
stating that she felt she was being forced to resign. She then alleged that the SAUDIA Manila
Office Manager told her to amend the same to state that she was voluntarily resigning; this she
reluctantly followed.

Petitioners filed this complaint for illegal dismissal against the respondent with the LA, with
prayer for reinstatement, full backwages, moral and exemplary damages, and attorneys fees.
They alleged that the Jeddah-based flight attendants aged 39 to 40 years old, were already
processing their respective resignations and that the transfer order was made so that they
would be terminated upon their arrival in Jeddah. Respondent maintained that the resignations
were intelligently and voluntarily made and that they were penned and duly signed by them. It
asserted further that the petitioners voluntarily executed an undertaking acknowledging receipt
of various sums of money and irrevocably and unconditionally releasing the respondent from
any claim or demand which they may gave in connection with their employment.

LA ruled in favor of the petitioners and held the respondent guilty of illegal dismissal and
ordered to pay them full; backwages less the amount they already received. It founded that
petitioners did not voluntarily resigned but they were forced to do so because of old age under
pain of actual termination. Further held that the subject undertaking was akin to a quitclaim,
cannot bar the petitioners from filing the case against the respondents. Respondent appealed to
NLRC, and the latter reversed the decision of LA and dismissed the illegal dismissal case

ATHENA M. SALAS | LABOR CASE DIGEST 2015 301

against the petitioners and ruled that there was no coercion on employed by the respondent and
that the undertaking executed by petitioners was valid. Petitioner filed their Motion for
Reconsideration which was subsequently denied by the NLRC in its resolution dated October
26, 2007. .
Aggrieved, the petitioners elevated the matter to the CA. On January 16, 2008, petitioners filed
with the CA a Motion for Extension to File a Petition for Certiorari, praying that they be given a
period of 15 days from January 18, 2008, or until February 2, 2008, within which to file the
subject petition. The said motion was granted in a Resolution dated January 29, 2008.
SAUDIA filed a Motion for Reconsideration, primarily contending that A.M. No. 07-7-12-SC,
which took effect on December 27, 2007, no longer allowed the filing of an extension of time to
file a petition for certiorari; thus, the CA should not have admitted the subject petition. In a
Resolution dated August 28, 2008, the CA reconsidered its earlier resolution and granted
SAUDIAs motion. It deemed the subject petition not admitted due to petitioners non-compliance
with the reglementary period prescribed by Section 4, Rule 65 of the Rules of Court (Rules), as
amended by A.M. No. 07-7-12-SC. Hence, it considered the case closed and terminated.
Petitioners filed a Motion for Reconsideration dated September 26, 2008, which was, however,
denied in a Resolution dated June 16, 2009, prompting them to institute the instant petition.
Issue: Whether or not the CA correctly refused admission of the subject petition
Ruling:
It is well-settled that procedural rules should be treated with utmost respect and due regard,
since they are designed to facilitate the adjudication of cases to remedy the worsening problem
of delay in the resolution of rival claims and in the administration of justice. However, Court has
recognized exceptions to the strict application of such rules, but only for the most compelling
reasons where stubborn obedience to the Rules would defeat rather than serve the ends of
justice.24 These exceptions, as enumerated in the case of Labao v. Flores, are as follows:
(1) most persuasive and weighty reasons;
(2) to relieve a litigant from an injustice not commensurate with his failure to comply with
the prescribed procedure;
(3) good faith of the defaulting party by immediately paying within a reasonable time
from the time of the default;
(4) the existence of special or compelling circumstances;
(5) the merits of the case;
(6) a cause not entirely attributable to the fault or negligence of the party favored by the
suspension of the rules;
(7) a lack of any showing that the review sought is merely frivolous and dilatory;
(8) the other party will not be unjustly prejudiced thereby;
(9) fraud, accident, mistake, or excusable negligence without appellants fault;
(10) peculiar legal and equitable circumstances attendant to each case;
(11)
in the name of substantial justice and fair play;
(12) importance of the issues involved; and
(13) Exercise of sound discretion by the judge guided by all the attendant
circumstances.
Despite the rigid wording of Section 4, Rule 65 of the Rules, as amended by A.M. No.
07-7-12-SC which now disallows an extension of the 60-day reglementary period to file a

ATHENA M. SALAS | LABOR CASE DIGEST 2015 302

petition for certiorari courts may nevertheless extend the same, subject to its sound
discretion.

General rule is that a petition for certiorari must be filed within sixty (60) days from notice
of the judgment, order, or resolution sought to be assailed. Under exceptional
circumstances, however, and subject to the sound discretion of the Court, said period may
be extended.

In this case, the CA had already exercised its sound discretion in granting the extension
to file the subject petition thru a Resolution dated January 29, 2008. Consequently, it could
not renege on such grant by rendering another issuance almost seven months later, i.e.,
Resolution dated August 28, 2008, which resulted in the refusal to admit the same petition.
136. Dongon vs. Rapid Movers and Forwarders Co Inc., et al., G.R. No. 163431, Aug. 28,
2013
Facts:
Rapid is engaged in the hauling and trucking business while private respondent Nathaniel T.
Dongon is a former truck helper leadman. Private respondents area of assignment is the
Tanduay Otis Warehouse where he has a job of facilitating the loading and unloading
petitioners trucks. On 23 April 2001, private respondent and his driver, Vicente Villaruz, were in
the vicinity of Tanduay as they tried to get some goods to be distributed to their clients.
Tanduays security guard called the attention of private respondent as to the fact that Mr. Villaruz
was not wearing an Identification Card (I.D. Card). Private respondent, then, assured the guard
that he will secure a special permission from the management to warrant the orderly release of
goods. Instead of complying with his compromise, private respondent lent his I.D. Card to
Villaruz; and by reason of such misrepresentation , private respondent and Mr. Villaruz got a
clearance from Tanduay for the release of the goods. However, the security guard, who saw the
misrepresentation committed by private respondent and Mr. Villaruz, accosted them and
reported the matter to the management of Tanduay. On 23 May 2001, after conducting an
administrative investigation, private respondent was dismissed from the petitioning Company.
On 01 June 2001, private respondent filed a Complaint for Illegal Dismissal.
September I 0, 2001, LA ruled against the private respondent and ruled that Rapid Movers
rightly exercised its prerogative to dismiss the petitioner considering that: he had admitted
lending his company ID to the driver; his act had constituted mental dishonesty and deceit
amounting to breach of trust; Rapid Movers relationship with Tanduay had been jeopardized by
his act; and that he had been banned from all the warehouses of Tanduay as a result, leaving
Rapid Movers with no available job for him. June 17, 2002, NLRC reversed the LA ruling and
held the Rapid Movers had not discharged its burden to prove the validity of petitioners
dismissal. It opined that Rapid Movers did not suffered any pecuniary damage and that the
dismissal was a penalty disproportionate to the act of the petitioner. It awarded separation pay
and damages.
The case was elevated to the CA averring grave abuse of discretion on the part of NLRC.
October 24, 2003, the Ca reinstated the decision of the LA and upheld the right of Rapid Movers

ATHENA M. SALAS | LABOR CASE DIGEST 2015 303

to discipline its workers. Petitioner file a motion for reconsideration but the same was denied on
March 22, 2004. Hence, the case was appealed with the SC.
Petitioner argued that his dismissal was illegal; that the CA gravely abused its discretion in
disregarding his showing the he did not violate Rapid Movers rules and regulation but simply
performed work in line with his duties and it did not appreciate his good faith and lack of any
intention to willfully disobey the company rules; that the penalty of dismissal is too harsh and
disproportionate to his supposed violation. Respondent Rapid movers, prays that the petition for
certiorari be dismissed for being an improper remedy for a lost appeal; it insist that the CA did
not commit grave abuse of discretion.
Issues:
1. Whether or not the petition should be dismissed for being an improper remedy for a lost
appeal.
2. Whether or not the petitioner was guilty of willful disobedience which will justify his
dismissal.
Ruling:

The petition should not be dismissed. It has been clarified that a party seeking the review of
decisions of the NLRC should file his petition for certiorari in the CA under rule 65 on the ground
of grave abuse of discretion amounting to lack or excess of jurisdiction on the part of NLRC.
Thereafter, the remed y of the aggrieved party from the CA decision is petition for review on
certiorari under rule 45.
The petition filed here is self-styled as a petition for review on certiorari, but Rapid Movers
points out that the petition was really one for certiorari under Rule 65 of the Rules of Court due
to its basis being the commission by the CA of a grave abuse of its discretion and because the
petition was filed beyond the reglementary period of appeal under Rule 45.
The Court deems it proper to allow due course to the petition as one for certiorari under Rule 65
in the broader interest of substantial justice, particularly because the NLRCs appellate
adjudication was set aside by the CA, and in order to put at rest the doubt that the CA, in so
doing, exercised its judicial authority oppressively. Whether the petition was proper or not
should be of less importance than whether the CA gravely erred in undoing and setting aside
the determination of the NLRC as a reviewing forum vis-vis the Labor Arbiter. We note in this
regard that the NLRC had declared the dismissal of petitioner to be harsh and not
commensurate to the infraction committed. Given the spirit and intention underlying our labor
laws of resolving a doubtful situation in favor of the working man, we will have to review the
judgment of the CA to ascertain whether the NLRC had really committed grave abuse of its
discretion. This will settle the doubts on the propriety of terminating petitioner, and at the same
time ensure that justice is served to the parties.
For willful disobedience to be a ground, it is required that: (a) the conduct of the employee must
be willful or intentional; and (b) the order the employee violated must have been reasonable,
lawful, made known to the employee, and must pertain to the duties that he had been engaged
to discharge. Willfulness must be attended by a wrongful and perverse mental attitude rendering
the employees act inconsistent with proper subordination. In any case, the conduct of the
employee that is a valid ground for dismissal under the Labor Code constitutes harmful behavior
against the business interest or person of his employer. It is implied that in every act of willful

ATHENA M. SALAS | LABOR CASE DIGEST 2015 304

disobedience, the erring employee obtains undue advantage detrimental to the business
interest of the employer. Under the foregoing standards, the disobedience attributed to petitioner
could not be justly characterized as willful within the contemplation of Article 296 of the Labor
Code. He neither benefitted from it, nor thereby prejudiced the business interest of Rapid
Movers. His explanation that his deed had been intended to benefit Rapid Movers was credible.
There could be no wrong or perversity on his part that warranted the termination of his
employment based on willful disobedience.

Although we recognize the inherent right of the employer to discipline its employees, we should
still ensure that the employer exercises the prerogative to discipline humanely and
considerately, and that the sanction imposed is commensurate to the offense involved and to
the degree of the infraction. The discipline exacted by the employer should further consider the
employees length of service and the number of infractions during his employment. The
employer should never forget that always at stake in disciplining its employee are not only his
position but also his livelihood, and that he may also have a family entirely dependent on his
earnings. Considering that petitioners motive in lending his company ID to Villaruz was to
benefit Rapid Movers as their employer by facilitating the loading of goods at the Tanduay Otis
Warehouse for distribution to Rapid Movers clients, and considering also that petitioner had
rendered seven long unblemished years of service to Rapid Movers, his dismissal was plainly
unwarranted. The NLRCs reversal of the decision of the Labor Arbiter by holding that penalty
too harsh and disproportionate to the wrong attributed to him was legally and factually justified,
not arbitrary or whimsical.

137. Malayang Manggagawa ng Stayfast Phils Inc vs. NLRC et al., G.r. No. 155306, Aug.
28, 2013

FACTS:
The Labor Arbiter and the National Labor Relations Commission (NLRC) made similar findings
of fact. Petitioner MMSP and NLMS-Olalia) sought to be the exclusive bargaining agent of the
employees of respondent company, Stayfast Philippines, Inc. A certification election was
conducted and out of the 223 valid votes cast, petitioner garnered 109 votes while NLMS-Olalia
received 112 votes and 2 votes were for "No Union." NLMS-Olalia was certified by the MedArbiter as the sole and exclusive bargaining agent of all rank and file employees of respondent
company.
Petitioner appealed the Order of the Med-Arbiter to the Secretary of Labor and Employment.
The Secretary of Labor and Employment initially set aside the Order of the Med-Arbiter and
called for run-off election between petitioner and NLMS-Olalia. On motion of NLMS-Olalia,

ATHENA M. SALAS | LABOR CASE DIGEST 2015 305

however, the Secretary of Labor and Employment reconsidered his earlier decision and restored
the Med-Arbiters Order dated January 9, 1996.
Meanwhile, NLMS-Olalia demanded to collectively bargain with respondent company. The latter
rejected petitioners demand, insisting that it would negotiate a collective bargaining agreement
only with whichever union is finally certified as the sole and exclusive bargaining agent of the
workers. Nevertheless, NLMS-Olalia went on strike on April 1, 1997 until it was temporarily
restrained eight days later.
Subsequently, Petitioner filed its own notice of strike in the National Conciliation and Mediation
Board (NCMB). Respondent company opposed petitioners move and filed a motion to dismiss
on the ground that petitioner was not the certified bargaining agent and therefore lacked
personality to file a notice of strike. Thereafter, the parties were able to make concessions
during the conciliation-mediation stage in the NCMB which led petitioner to withdraw its notice of
strike.
In this connection, the NCMB issued a Certification dated July 31, 1997 which stated that the
notice of strike filed by MMSP-Independent on June 1997 against Stayfast is dropped. It further
certified that there was no new Notice of Strike filed by the same union.
On July 21, 1997, however, petitioners members staged a "sit-down strike" to dramatize their
demand for a fair and equal treatment as respondent company allegedly continued to
discriminate against them. Respondent company issued a memorandum requiring the alleged
participants in the "sit-down strike" to explain within 24 hours why they should not be terminated
or suspended from work for infraction of company rules and regulations pertaining to
unauthorized work stoppage, acts inimical to company interest, and disregard of instruction of
immediate supervisor to perform assigned task.
As no one complied with the memorandum within the 24-hour deadline, respondent company
promptly terminated the service of the participants in the "sit-down strike" on July 22, 1997.
Consequently, on July 23, 1997, petitioner staged a strike and filed a complaint for unfair labor
practice, union busting and illegal lockout against respondent company and its General
Manager, Maria Almeida, in the NLRC.
Petitioner alleged that respondents had committed acts of discrimination, such as the denial of
the use of the canteen to conduct strike votes, or denial of petitioners leave to attend hearings
in labor cases, among others.
For its part, respondent company claimed that petitioner lacked legal authority to go on strike
since it is a minority union. Petitioner committed illegal acts during the strike and obstructed the
free ingress and egress from respondent companys premises.
On April 27, 1999, the Labor Arbiter rendered a Decision which ruled that, while petitioner may
file a notice of strike on behalf of its members, petitioner failed to cite any instance of
discrimination or harassment when it filed its notice of strike on June 5, 1997 and the incidents
mentioned as discriminatory occurred after the filing of the said notice. Moreover, assuming the
strike was legal at the beginning, it became illegal when petitioner committed acts prohibited
under Article 264(e) of the Labor Code, such as acts of violence, coercion and intimidation and
obstruction of free ingress and egress.

ATHENA M. SALAS | LABOR CASE DIGEST 2015 306

Petitioner appealed but, NLRC upheld the Labor Arbiters Decision. According to the NLRC, the
actuations of petitioner were patently illegal because the sit-down strike staged on July 21, 1997
was made barely a week after petitioner withdrew its notice of strike, with prejudice, on account
of the concessions agreed upon by the parties. Petitioner filed no new notice of strike that could
have supported its charges of discriminatory acts and unfair labor practice. Moreover, no
evidence was presented to establish such charges. Also, petitioners members were given the
opportunity to explain their violation of respondent companys rules on unauthorized work
stoppage, acts inimical to company interest and disregard of instruction of immediate supervisor
to perform assigned task.
Hence, this petition for certiorari under Rule 65 of the Rules of Court.

ISSUES:
1. That the CA committed grave abuse of discretion amounting to lack or excess of jurisdiction
when they upheld the rulings of the NLRC and disregarded the constitutional protection of labor
as well as Article 248 (e) and Article 263 of the Labor Code.
2. That the CA committed grave abuse of discretion amounting to lack or excess of jurisdiction
when they upheld the decision of the NLRC that the termination of complainants/appellants
were valid and corollary thereto no reinstatement[,] backwages, damages and attorneys fees
were awarded.
Petitioner claims that the discriminatory acts of respondent company and its General Manager
against petitioners members constituted unfair labor practice under Article 248(e) of the Labor
Code, as amended. The termination of employment of petitioners 127 officers and members
constituted union-busting and unlawful lockout. As the said officers and members were
unlawfully dismissed from employment, they are entitled to reinstatement with full backwages.
The arbitrary action of respondent company and its General Manager wantonly disregarded the
legal rights of petitioners officers and members thereby entitling said officers and members to
damages and attorneys fees.

RULING:
The petition fails for many reasons.
1. Certiorari is the wrong remedy.
For purposes of appeal, the Decision dated July 1, 2002 of the Court of Appeals was a final
judgment as it denied due course to, and dismissed, the petition. Thus, the Decision disposed of
the petition of petitioner in a manner that left nothing more to be done by the Court of Appeals in
respect to the said case. Thus, petitioner should have filed an appeal by petition for review on
certiorari under Rule 45, not a petition for certiorari under Rule 65, in this Court.
Moreover, certiorari is not and cannot be made a substitute for an appeal where the latter
remedy is available but was lost through fault or negligence. In this case, petitioner received the
Decision dated July 1, 2002 on August 2, 2002 and, under the rules, had until August 19, 2002

ATHENA M. SALAS | LABOR CASE DIGEST 2015 307

to file an appeal by way of a petition for review in this Court. Petitioner let this period lapse
without filing an appeal and, instead, filed this petition for certiorari on October 1, 2002.
2. Petitioner unable to establish allegation of grave abuse of discretion on the part of CA.
Where a petition for certiorari under Rule 65 of the Rules of Court alleges grave abuse of
discretion, the petitioner should establish that the respondent court or tribunal acted in a
capricious, whimsical, arbitrary or despotic manner in the exercise of its jurisdiction as to be
equivalent to lack of jurisdiction
The term "grave abuse of discretion" has a specific meaning. An act of a court or tribunal can
only be considered as with grave abuse of discretion when such act is done in a "capricious or
whimsical exercise of judgment as is equivalent to lack of jurisdiction." The abuse of discretion
must be so patent and gross as to amount to an "evasion of a positive duty or to a virtual refusal
to perform a duty enjoined by law, or to act at all in contemplation of law, as where the power is
exercised in an arbitrary and despotic manner by reason of passion and hostility."
In this case, nowhere in the petition did petitioner show that the issuance of the Decision dated
July 1, 2002 of the Court of Appeals was patent and gross that would warrant striking it down
through a petition for certiorari. Aside from a general statement in the Jurisdictional Facts
portion of the petition and the sweeping allegation of grave abuse of discretion in the general
enumeration of the grounds of the petition, petitioner failed to substantiate its imputation of
grave abuse of discretion on the part of the Court of Appeals. No argument was advanced to
show that the Court of Appeals exercised its judgment capriciously, whimsically, arbitrarily or
despotically by reason of passion and hostility. Petitioner did not even discuss how or why the
conclusions of the Court of Appeals were made with grave abuse of discretion. Instead,
petitioner limited its discussion on its version of the case, which had been already rejected both
by the Labor Arbiter and the NLRC. Thus, petitioner failed in its duty to demonstrate with
definiteness the grave abuse of discretion that would justify the proper availment of a petition for
certiorari under Rule 65 of the Rules of Court.
3. Even on the merits, petition must fail.
Petitioners case rests on the alleged discriminatory acts of respondent company against
petitioners officers and members. However, both the Labor Arbiter and the NLRC held that
there was no sufficient proof of respondent companys alleged discriminatory acts. Thus,
petitioners unfair labor practice, union-busting and unlawful lockout claims do not hold water.
Moreover, the established facts as found by the NLRC are as follows: the "sit-down strike" made
by petitioners officers and members on July 21, 1997 was in violation of respondent companys
rules, and petitioners officers and members ignored the opportunity given by respondent
company for them to explain their misconduct, which resulted in the termination of their
employment.
138. Borra et al., vs. Court of Appeals, et al., G.R. No. 167484, Sept. 9, 2013

FACTS:
On September 12, 1997, herein petitioners filed with the National Labor Relations Commission
(NLRC) Regional Arbitration Branch No. VIin Bacolod City two separate complaints against

ATHENA M. SALAS | LABOR CASE DIGEST 2015 308

herein private respondent alone, while RAB Case No. 06-09-10699-97 impleaded herein private
respondent and a certain Fela Contractor as respondents. In RAB Case No. 06-09-10698-97,
petitioners asked that they be recognized and confirmed as regular employees of herein private
respondent and further prayed that they be awarded various benefits received by regular
employees for three (3) years prior to the filing of the complaint, while in RAB Case No. 06-0910699-97,herein petitioners sought for payment of unpaid wages, holiday pay, allowances, 13th
month pay, service incentive leave pay, moral and exemplary damages also during the three (3)
years preceding the filing of the complaint.
On October 16, 1997, private respondent filed a Motion to Consolidate the above mentioned
cases, but the Labor Arbiter in charge of the case denied the said Motion.
On January 9, 1998, private respondent filed a Motion to Dismiss RAB Case No. 06-09-0698-97
on the ground of res judicata. Private respondent cited an earlier decided case entitled "
Humphrey Perez, et al. v. Hawaiian Philippine Co. et al. "(Perez case) and docketed as RAB
Case No.06-04-10169-95, which was an action for recovery of 13th month pay and service
incentive leave pay, and it includes herein petitioners among the complainants and herein
private respondent and one Jose Castillon (Castillon) as respondents. Private respondent
contended that the Perez case, which has already become final and executory, as no appeal
was taken there from, serves as a bar to the litigation of RAB Case No. 06-09-10698-97,
because it was ruled therein that petitioners are not employees of private respondent but of
Castillon.
In an Order dated July 9, 1998, the Labor Arbiter granted private respondent's Motion to
Dismiss.
Petitioners appealed to the NLRC which set aside the Order of the Labor Arbiter, reinstated the
complaint in RAB Case No. 06-09-10698-97and remanded the same for further proceedings.
Private respondent appealed to the CA. On January 12, 2001, the CA rendered judgment,
affirming the Decision of the NLRC and denied the subsequent motion for reconsideration.
Aggrieved, private respondent filed a petition for review on certiorari before this Court. The case
was entitled as "Hawaiian Philippine Company v. Borra" and docketed as G.R. No. 151801. On
November 12, 2002, this Court rendered its Decision denying the petition and affirming the
Decision of the CA.
On June 22, 2004, the CA rendered its questioned Decision, the dispositive portion of which
reads, thus:
WHEREFORE, foregoing premises considered, the petition is GRANTED. Accordingly, the
Order dated August 12, 2003 of public respondent is hereby ANNULLED and SET ASIDE. RAB
Case No. 09-10698-97 is ordered DISMISSED.
SO ORDERED.
Petitioners filed a Motion for Reconsideration, but the CA denied it in its Resolution dated
January 14, 2005.
Hence, the present petition for certiorari based on the following grounds:

ATHENA M. SALAS | LABOR CASE DIGEST 2015 309

ISSUES:
1. Does the CA have jurisdiction to entertain the petition for certiorari? Yes.
2. Is there an employer-employee relationship between petitioners and private respondent? No.

RULING:
1. This Court is not persuaded by petitioners' argument that the CA has no jurisdiction over
private respondent's petition for certiorari because this Court, in G.R. No. 151801, lodged
jurisdiction in the Labor Arbiter by directing the remand of RAB Case No. 06-09-10698-97
thereto for further proceedings.
It is settled that jurisdiction over the subject matter is conferred by law and it is not within the
courts, let alone the parties, to themselves determine or conveniently set aside.
In this regard, Rule 41 of the Rules of Court, which is applied in a suppletory character to cases
covered by the NLRC Rules, provides that in all the instances enumerated under the said Rule,
where the judgment or final order is not appealable, the aggrieved party may file an appropriate
special civil action under Rule 65.
On the basis of the foregoing, it is clear that the CA has jurisdiction over the special civil action
for certiorari filed by private respondent as the latter was able to allege and establish that the
denial of its motion to dismiss was tainted with grave abuse of discretion. Petitioners are wrong
to argue that this Court's directive in G.R. No. 151801 to remand RAB Case No. 06-09-1069897 to the Labor Arbiter for further proceedings deprives the CA of its jurisdiction over private
respondent's petition for certiorari.
2. As earlier mentioned, this issue has already been settled. In the already final and executory
decision of the Labor Arbiter in RAB Case No.06-09-10699-97, it was ruled therein that no
employer-employee relationship exists between private respondent and petitioners because the
latter's real employer is Fela Contractor. Thus, insofar as the question of employer and
employee relations between private respondent and petitioners is concerned, the final judgment
in RAB Case No. 06-09-10699-97 has the effect and authority of res judicata by conclusiveness
of judgment.
Hence, there is no point in determining the main issue raised in RAB Case No. 06-09-10698-97,
whether petitioners may be considered regular employees of private respondent, because, in
the first place, they are not even employees of the latter. As such, the CA correctly held that the
Labor Arbiter committed grave abuse of discretion in denying private respondent's motion to
dismiss RAB Case No. 06-09-10698-97.
139. McBurnie vs. Ganzon et al., GR Nol. 178034 & 178117 & 186984-85, Oct. 17, 2013, (En
banc)
Facts:
On October 4, 2002, McBurnie, an Australian national, instituted a complaint for illegal dismissal
and other monetary claims against the respondents. McBurnie claimed that on May 11, 1999, he
signed a five-year employment agreement with the company EGI as an Executive VicePresident who shall oversee the management of the company's hotels and resorts within the

ATHENA M. SALAS | LABOR CASE DIGEST 2015 310

Philippines. He performed work for the company until sometime in November 1999, when he
figured in an accident that compelled him to go back to Australia while recuperating from his
injuries. While in Australia, he was informed by respondent Ganzon that his services were no
longer needed because their intended project would no longer push through.

The respondents opposed the complaint, contending that their agreement with McBurnie was to
jointly invest in and establish a company for the management of hotels. They did not intend to
create an employer-employee relationship, and the execution of the employment contract that
was being invoked by McBurnie was solely for the purpose of allowing McBurnie to obtain an
alien work permit in the Philippines. At the time McBurnie left for Australia for his medical
treatment, he had not yet obtained a work permit.

The LA declared McBurnie as having been illegally dismissed from employment, and thus
entitled to receive from the respondents the following amounts: (a) US$985,162.00 as salary
and benefits for the unexpired term of their employment contract, (b) P2,000,000.00 as moral
and exemplary damages, and (c) attorney's fees equivalent to 10% of the total monetary award.

Feeling aggrieved, the respondents appealed the LA's Decision to the NLRC. On November 5,
2004, they filed their Memorandum of Appeal and Motion to Reduce Bond, and posted an
appeal bond in the amount of P100,000.00. The respondents contended in their Motion to
Reduce Bond, inter alia, that the monetary awards of the LA were null and excessive, allegedly
with the intention of rendering them incapable of posting the necessary appeal bond.

Issue:

The crucial issue in this case concerns the sufficiency of the appeal bond that was posted by
the respondents.

Ruling:

To clarify, the prevailing jurisprudence on the matter provides that the filing of a motion to reduce
bond, coupled with compliance with the two conditions emphasized in Garcia v. KJ
Commercial for the grant of such motion, namely, (1) a meritorious ground, and (2) posting of a
bond in a reasonable amount, shall suffice to suspend the running of the period to perfect
an appeal from the labor arbiter's decision to the NLRC. To require the full amount of the
bond within the 10-day reglementary period would only render nugatory the legal provisions
which allow an appellant to seek a reduction of the bond.

A serious error of the NLRC was its outright denial of the motion to reduce the bond, without
even considering the respondents' arguments and totally unmindful of the rules and

ATHENA M. SALAS | LABOR CASE DIGEST 2015 311

jurisprudence that allow the bond's reduction. Instead of resolving the motion to reduce the
bond on its merits, the NLRC insisted on an amount that was equivalent to the monetary award.

Although the general rule provides that an appeal in labor cases from a decision involving a
monetary award may be perfected only upon the posting of a cash or surety bond, the Court has
relaxed this requirement under certain exceptional circumstances in order to resolve
controversies on their merits. These circumstances include: (1) the fundamental consideration
of substantial justice; (2) the prevention of miscarriage of justice or of unjust enrichment; and (3)
special circumstances of the case combined with its legal merits, and the amount and the issue
involved.

The bond requirement in appeals involving monetary awards has been and may be relaxed in
meritorious cases, including instances in which (1) there was substantial compliance with the
Rules, (2) surrounding facts and circumstances constitute meritorious grounds to reduce the
bond, (3) a liberal interpretation of the requirement of an appeal bond would serve the desired
objective of resolving controversies on the merits, or (4) the appellants, at the very least,
exhibited their willingness and/or good faith by posting a partial bond during the reglementary
period.

As the Court, nonetheless, remains firm on the importance of appeal bonds in appeals from
monetary awards of LAs, we stress that the NLRC, pursuant to Section 6, Rule VI of the NLRC
Rules of Procedure, shall only accept motions to reduce bond that are coupled with the posting
of a bond in a reasonable amount.

At the time of a motion to reduce appeal bond's filing, the question of what constitutes "a
reasonable amount of bond" that must accompany the motion may be subject to differing
interpretations of litigants. The judgment of the NLRC which has the discretion under the law to
determine such amount cannot as yet be invoked by litigants until after their motions to reduce
appeal bond are accepted.

It is in this light that the Court finds it necessary to set a parameter for the litigants' and the
NLRC's guidance on the amount of bond that shall hereafter be filed with a motion for a bond's
reduction. To ensure that the provisions of Section 6, Rule VI of the NLRC Rules of Procedure
that give parties the chance to seek a reduction of the appeal bond are effectively carried out,
without however defeating the benefits of the bond requirement in favor of a winning litigant, all
motions to reduce bond that are to be filed with the NLRC shall be accompanied by the posting
of a cash or surety bond equivalent to 10% of the monetary award that is subject of the
appeal, which shall provisionally be deemed the reasonable amount of the bond in the
meantime that an appellant's motion is pending resolution by the Commission. In conformity
with the NLRC Rules, the monetary award, for the purpose of computing the necessary appeal
bond, shall exclude damages and attorney's fees. Only after the posting of a bond in the
required percentage shall an appellant's period to perfect an appeal under the NLRC Rules be
deemed suspended.

ATHENA M. SALAS | LABOR CASE DIGEST 2015 312

Notably, in the present case, following the CA's rendition of its Decision which allowed a
reduced appeal bond, the respondents have posted a bond in the amount of P10,000,000.00.

Given the circumstances in this case and the merits of the respondents' arguments before the
NLRC, the Court holds that the respondents had posted a bond in a "reasonable amount", and
had thus complied with the requirements for the perfection of an appeal from the LA's decision.
Moreover, in all cases, the reduction of the appeal bond shall be justified by meritorious grounds
and accompanied by the posting of the required appeal bond in a reasonable amount.
The requirement on the existence of a "meritorious ground" delves on the worth of the parties'
arguments, taking into account their respective rights and the circumstances that attend the
case.

140. Lepanto Consolidated Mining Corp., vs. Icao, GR No. 196047, January 15, 2014
Facts:
The instant petition stemmed from a complaint for illegal dismissal and damages filed by private
respondent Belio C. Icao [Icao] against petitioners Lepanto Consolidated Mining Company
(LCMC) and its Chief Executive Officer [CEO] Felipe U. Yap [Yap] before the Arbitration Branch
of the NLRC.

The Labor Arbiter ruled that petitioner and its CEO liable for illegal dismissal and ordering them
to pay respondent Icao P345,879.45, representing his full backwages and separation pay.

The petitioner and its CEO filed an Appearance with Memorandum of Appeal before the NLRC.
Instead of posting the required appeal bond in the form of a cash bond or a surety bond in an
amount equivalent to the monetary award of P345,879.45 adjudged in favor of Icao, they filed a
Consolidated Motion for Release of Cash Bond and to Apply Bond Subject for Release As
Payment for Appeal Bond (Consolidated Motion). They requested therein that the NLRC
release the cash bond of P401,610.84, which they had posted in the separate case Dangiw
Siggaao v. LCMC, and apply that same cash bond to their present appeal bond liability. They
reasoned that since this Court had already decided Dangiw Siggaao in their favor, and that the
ruling therein had become final and executory, the cash bond posted therein could now be
released. They also cited financial difficulty as a reason for resorting to this course of action
and prayed that, in the interest of justice, the motion be granted.

The NLRC First Division dismissed the appeal of petitioner and the latter's CEO for nonperfection. It found that they had failed to post the required appeal bond equivalent to the
monetary award of P345,879.45. It explained that their Consolidated Motion for the release of
the cash bond in another case (Dangiw Siggaao), for the purpose of applying the same bond to
the appealed case before it, could not be considered as compliance with the requirement to post
the required appeal bond.

ATHENA M. SALAS | LABOR CASE DIGEST 2015 313

Issue:

The sole issue before the Court is whether or not petitioner complied with the appeal bond
requirement under the Labor Code and the NLRC Rules by filing a Consolidated Motion to
release the cash bond it posted in another case, which had been decided with finality in its
favor, with a view to applying the same cash bond to the present case.

Ruling:

While it is true that the procedure undertaken by petitioner is not provided under the Labor Code
or in the NLRC Rules, we consider such procedure compliant with the appeal bond requirement
under the Labor Code.

When the law does not clearly provide a rule or norm for the tribunal to follow in deciding a
question submitted, but leaves to the tribunal the discretion to determine the case in one way or
another, the judge must decide the question in conformity with justice, reason and equity, in
view of the circumstances of the case. Applying this doctrine, we rule that petitioner substantially
complied with the mandatory requirement of posting an appeal bond for the reasons explained
below.

First, there is no question that the appeal was filed within the 10-day reglementary
period. Except for the alleged failure to post an appeal bond, the appeal to the NLRC was
therefore in order.

Second, it is also undisputed that petitioner has an unencumbered amount of money in the form
of cash in the custody of the NLRC. To reiterate, petitioner had posted a cash bond of
P401,610.84 in the separate case Dangiw Siggaao, which was earlier decided in its favor. As
claimed by petitioner and confirmed by the Judgment Division of the Judicial Records Office of
this Court, the Decision of the Court in Dangiw Siggaao had become final and executory as of
28 April 2008, or more than seven months before petitioner had to file its appeal in the present
case. This fact is shown by the Entry of Judgment on file with the aforementioned office. Hence,
the cash bond in that case ought to have been released to petitioner then.

Under the Rule VI, Section 6 of the 2005 NLRC Rules, "[a] cash or surety bond shall be valid
and effective from the date of deposit or posting, until the case is finally decided, resolved or
terminated, or the award satisfied." Hence, it is clear that a bond is encumbered and bound to a
case only for as long as 1) the case has not been finally decided, resolved or terminated; or 2)
the award has not been satisfied. Therefore, once the appeal is finally decided and no award
needs to be satisfied, the bond is automatically released. Since the money is now
unencumbered, the employer who posted it should now have unrestricted access to the cash
which he may now use as he pleases as appeal bond in another case, for instance. This is
what petitioner simply did.

ATHENA M. SALAS | LABOR CASE DIGEST 2015 314

Third, the cash bond in the amount of P401,610.84 posted in Dangiw Siggaao is more than
enough to cover the appeal bond in the amount of P345,879.45 required in the present case.
Fourth, this ruling remains faithful to the spirit behind the appeal bond requirement which is to
ensure that workers will receive the money awarded in their favor when the employer's appeal
eventually fails. There was no showing at all of any attempt on the part of petitioner to evade
the posting of the appeal bond. On the contrary, petitioner's move showed a willingness to
comply with the requirement. Hence, the welfare of Icao is adequately protected.
Having complied with the appeal bond requirement, petitioner's appeal before the NLRC must
therefore be reinstated.
141. Building Care Corp. vs. Macaraeg, G.R. No. 198357, December 10, 2012
Facts:
Petitioner is engaged in the business of providing security services. From 8/25/1986 she was
assigned at the Genato Building in Caloocan City up until March 9, 2008, where she was
relieved of her duty. Following her relief from duty, she was re-assigned to bayview park hotel
from March 9-13, 2008. However, after said date, she was no longer given an assignment.
Because of this, respondent was prompted to file a case for illegal dismissal against petitioner
as well as underpayment of salaries, non-payment of separation pay and refund of cash bond.
Petitioner alleged that respondent Macaraeg was relieved from her duty at the Genato building
for sleeping on the job, as well as continuously borrowing money from the employees and
clients of the Genato Building. Moreover, Bayview Hotel also found that the services of
respondent did not meet their standards, which was the reason why her assignment at bayview
was cut short.
At the Labor Arbiter, the complaint for illegal dismissal was dismissed. However, herein
respondent elevated the case to the NLRC by filing a notice of appeal. The appeal, however,
was dismissed for having been filed out of time.
As a result, the case was elevated by the respondent to the CA via a petition for certiorari, which
was granted in favor of respondent. Hence, this case.
Issue:
1. WON respondent entitled to an appeal, or if the appeal filed before NLRC was filed out
of time.

Ruling: PETITION GRANTED


The Supreme Court held that respondent failed to file a timely appeal before the NLRC, and this
precluded her from availing of other remedies. In other words, the decision of the LA had
become final and executory for her failure to file an appeal within the prescribed period. The
supreme court decided the case in this manner, to wit:
Discussion:

ATHENA M. SALAS | LABOR CASE DIGEST 2015 315

It should be emphasized that the resort to a liberal application, or suspension of


the application of procedural rules, must remain as the exception to the wellsettled principle that rules must be complied with for the orderly administration of
justice. In Marohomsalic v. Cole, the Court stated:

While procedural rules may be relaxed in the interest of justice, it is


well- settled that these are tools designed to facilitate the
adjudication of cases. The relaxation of procedural rules in the
interest of justice was never intended to be a license for erring
litigants to violate the rules with impunity. Liberality in the
interpretation and application of the rules can be invoked only in
proper cases and under justifiable causes and circumstances.
While litigation is not a game of technicalities, every case must be
prosecuted in accordance with the prescribed procedure to
ensure an orderly and speedy administration of justice.

The later case of Daikoku Electronics Phils., Inc. v. Raza, further explained
that:

To be sure, the relaxation of procedural rules cannot be made


without any valid reasons proffered for or underpinning it. To
merit liberality, petitioner must show reasonable cause
justifying its non-compliance with the rules and must
convince the Court that the outright dismissal of the petition
would defeat the administration of substantial justice. . . . The
desired leniency cannot be accorded absent valid and compelling
reasons for such a procedural lapse. . ..
We must stress that the bare invocation of "the interest of
substantial justice" line is not some magic want that will
automatically compel this Court to suspend procedural rules.
Procedural rules are not to be belittled, let alone dismissed
simply because their non-observance may have resulted in
prejudice to a party's substantial rights. Utter disregard of the
rules cannot be justly rationalized by harping on the policy of
liberal construction.
xxxxxxxxx
considering that the belated filing of respondent's appeal before the NLRC was
the fault of respondent's former counsel. Note, however, that neither respondent
nor her former counsel gave any explanation or reason citing extraordinary
circumstances for her lawyer's failure to abide by the rules for filing an appeal.
Respondent merely insisted that she had not been remiss in following up her case
with said lawyer.
It is, however, an oft-repeated ruling that the negligence and mistakes of counsel
bind the client. A departure from this rule would bring about never-ending suits, so

ATHENA M. SALAS | LABOR CASE DIGEST 2015 316

long as lawyers could allege their own fault or negligence to support the client's
case and obtain remedies and reliefs already lost by the operation of law. The
only exception would be, where the lawyer's gross negligence would result in the
grave injustice of depriving his client of the due process of law. In this case, there
was no such deprivation of due process. Respondent was able to fully present
andargue her case before the Labor Arbiter. She was accorded the opportunity to
be heard. Her failure to appeal the Labor Arbiter's Decision cannot, therefore, be
deemed as a deprivation of her right to due process. In Heirs of TeofiloGaudiano
v. Benemerito, the Court ruled, thus:
"The perfection of an appeal within the period and in the manner
prescribed by law is jurisdictional and non-compliance with such
legal requirements is fatal and has the effect of rendering the
judgment final and executory. The limitation on the period of appeal
is not without reason. They must be strictly followed as they are
considered indispensable to forestall or avoid unreasonable delays
in the administration of justice, to ensure an orderly discharge of
judicial business, and to put an end to controversies. . . .

xxxxxxxx
The right to appeal is not a natural right or part of due
process; it is merely a statutory privilege and may be
exercised only in the manner and in accordance with the
provisions of law. Thus, one who seeks to avail of the right to
appeal must strictly comply with the requirements of the
rules, and failure to do so leads to the loss of the right to
appeal."

xxxxxxxxx

Clearly, allowing an appeal, even if belatedly filed, should never be taken lightly.
The judgment attains finality by the lapse of the period for taking an appeal
without such appeal or motion for reconsideration being filed. In Ocampo v. Court
of Appeals (Former Second Division), the Court reiterated the basic rule that
"when a party to an original action fails to question an adverse judgment or
decision by not filing the proper remedy within the period prescribed by law, he
loses the right to do so, and the judgment or decision, as to him, becomes final
and binding." The Decision of the Labor Arbiter, therefore, became final and
executory as to respondent when she failed to file a timely appeal therefrom. The
importance of the concept of finality of judgment cannot be gainsaid. As
elucidated in Pasiona, Jr. v. Court of Appeals, to wit:
xxxxxxxxx

It should also be borne in mind that the right of the winning

ATHENA M. SALAS | LABOR CASE DIGEST 2015 317

party to enjoy the finality of the resolution of the case is also


an essential part of public policy and the orderly
administration of justice. Hence, such right is just as weighty
or equally important as the right of the losing party to appeal
or seek reconsideration within the prescribed period.

When the Labor Arbiter's Decision became final, petitioners attained a vested
right to said judgment. They had the right to fully rely on the immutability of said
Decision. In Sofio v. Valenzuela, it was amply stressed that:

The Court will not override the finality and immutability of a


judgment based only on the negligence of a party's counsel in
timely taking all the proper recourses from the judgment. To
justify an override, the counsel's negligence must not only be
gross but must also be shown to have deprived the party the
right to due process.
In sum, the Court cannot countenance relaxation of the rules absent the
showing of extraordinary circumstances to justify the same. In this case, no
compelling reasons can be found to convince this Court that the CA acted
correctly by according respondent such liberality. CIA
HDT
142. Co Say Coco Products Phils Inc., vs. Baltasar, et al., GR No. 188828, March 5, 2014
Facts:
Petitioner Co Say Coco entered into a contract for cargo handling services with co-petitioner
Tanawan Port to operate and manage the arrastre and stevedoring services of the port.
To commence operations, respondents et al were employed to various positions such as
computer operator, crane operator, crane helper, and fork lift operator. However, the business of
tanawan port failed to take off due to a lack of clientele. As a result, petitioner tanawan port
decided to close its business and cease operations.

Upon approval of its retirement from business by the city treasurer, petitioner tanawan port
called its employees to inform them of their intention to close despite attempts to try to revive its
business.
Because of this, respondents were terminated from employment but were given separation pay
and 13th month pay totaling P86,416.68.

Respondents, however, alleged that petitioners were only feigning business losses to ease out
EEs pointing out the absence of evidence of petitioners to prove business reverses. They had

ATHENA M. SALAS | LABOR CASE DIGEST 2015 318

also raised the failure of petitioners to send notices to the EEs concerned and DOLE one month
before the intended date of closure as required by law.

Petitioner tanawan port, on the other hand, raised the defense that severance from employment
was brought about by cessation of business, an authorized cause of termination for
employment under the labor code. Tanawan port reiterated that cessation of business was
caused due to a lack of clientele, resulting to serious financial drain to the company, leaving the
management with no choice but to shut down operations.

The LA rendered decision in favor of respondents for failure to comply with procedural and
substantial requirements. The LA decision was reversed in the NLRC, stating that dismissal was
not illegal. Respondents moved for the reconsideration of the NLRC decision but was met with
failure.

In the CA, the NLRC decision was reversed due to failure of petitioners to perfect their appeal.
The CA held that petitioners posted the appeal bond after the period to perfect the appeal had
expired, resulting in non-perfection of the appeal.

Instrumental to the issue is the presence of two certificates indicating the posting of a surety
bond in the amount of the monetary award due to the respondents. Petitioner uses this
allegation in imputing error on the ruling of the CA that the appeal was not perfected on time.

The first certificate indicated the receipt of the surety bond on sept. 24, 2003 while the second
certificate indicated the receipt of the surety bond on oct. 28, 2003.

Issues:

1. WON appeal bond was timely posted by the petitioner.

Ruling: Petition DENIED

In this case, the SC ruled that the appeal of the petitioners was not timely filed for failure to post
the appeal bond within the prescribed 10-day period to file an appeal under the NLRC Rules. As
an effect, the decision of the labor arbiter became final and executory between the parties.

ATHENA M. SALAS | LABOR CASE DIGEST 2015 319

Therefore, the decision became binding and the petitioners could no longer contest the decision
of the labor arbiter. The SC further stressed that the appeal period should be strictly applied
since an appeal is not a matter of right. Rather, it is a statutory privilege.

Discussion:

The crucial issue in the resolution of the instant petition concerns the timely posting of the
appeal bond. The pertinent rule on the matter is Article 223 of the Labor Code, as amended,
which sets forth the rules on appeal from the Labor Arbiter's monetary award:
ART. 223. Appeal. Decisions, awards, or orders of the Labor Arbiter are final and executory
unless appealed to the Commission by any or both
parties within ten (10) calendar days from receipt of such decisions, awards, or orders. . . . .
xxxxxxxxx
In case of a judgment involving a monetary award, an appeal by the employer may be perfected
only upon the posting of a cash or surety bond issued by a reputable bonding company duly
accredited by the Commission in the amount equivalent to the monetary award in the judgment
appealed from. (Emphasis ours).
Implementing the aforestated provisions of the Labor Code are the provisions of Rule VI of the
2011 NLRC Rules of Procedure on perfection of appeals which read:

SECTION 1. PERIODS OF APPEAL. Decisions, awards, or orders of the Labor Arbiter shall
be final and executory unless appealed to the Commission by any or both parties within ten (10)
calendar days from receipt thereof; and in case of decisions or resolutions of the Regional
Director of the Department of Labor and Employment pursuant to Article 129 of the Labor Code,
within five (5) calendar days from receipt thereof. If the 10th or 5th day, as the case may be,
falls on a Saturday, Sunday or holiday, the last day to perfect the appeal shall be the first
working day following such Saturday, Sunday or holiday.

xxxxxxxx

These statutory and regulatory provisions explicitly provide that an appeal from theLabor Arbiter
to the NLRC must be perfected within ten calendar days from receipt of such decisions,
awards or orders of the Labor Arbiter. In a judgment involving a monetary award, the appeal
shall be perfected only upon; (1) proof of payment of the required appeal fee; (2) posting of a
cash or surety bond issued by a reputable bonding company; and (3) filing of a
memorandum of appeal.
No appeal was perfected by the petitioners within the 10-day period under Article 223 of the
Labor Code.

ATHENA M. SALAS | LABOR CASE DIGEST 2015 320

xxxxxxxxx

The Court of Appeals therefore, correctly ruled that petitioners failed to perfect their appeal on
time. In holding so, the appellate court only applied the appeal bond requirement as already well
explained in our previous pronouncements that there is legislative and administrative intent to
strictly apply the appeal bond requirement, and the Court should give utmost regard to this
intention. The clear intent of both statutory and procedural law is to require the employer to post
a cash or surety bond securing the full amount of the monetary award within the ten 10-day
reglementary period. Rules on perfection of an appeal, particularly in labor cases,must be
strictly construed because to extend the period of the appeal is to delay the case, a
circumstance which would give the employer a chance to wear out the efforts and meager
resources of the worker to the point that the latter is constrained to give up for less than what is
due him. This is to assure the workers that if they finally prevail in the case the monetary award
will be given to them both upon dismissal of the employer's appeal. It is further meant to
discourage employers from using the appeal to delay or evade payment of their obligations to
the employees. The appeal bond requirement precisely aims to prevent empty or
inconsequential victories secured by laborers in consonance with the protection of labor clause
ensconced and zealously guarded by our Constitution.

It is entrenched in our jurisprudence that perfection of an appeal in a manner and within the
period prescribed by law is not only mandatory but jurisdictional, and failure to perfect an
appeal has the effect of making judgment final and executory. While dismissal of an appeal
on technical grounds is frowned upon, Article 223 of the Labor Code which prescribes the
appeal bond requirement, however, is a rule of jurisdiction and not of procedure. Hence, there is
a little leeway for condoning a liberal interpretation thereof, and certainly none premised on the
ground that its requirements are mere technicalities. It is axiomatic that an appeal is only a
statutory privilege and it may only be exercised in the manner provided by law. The timely
perfection of an appeal is a mandatory requirement, which cannot be trifled with a "mere
technicality" to suit the interest of party. We cannot condone the practice of parties who, either
by their own or their counsel's inadvertence, have allowed the judgment to become final and
executory and, after the same had reached finality, seeks the shield of substantial justice to
assail it.

All considered then, the finding of the Labor Arbiter holding the petitioners liable for illegal
dismissal is binding on them. Not having been timely appealed, this issue is already beyond our
jurisdiction to resolve, and the finding of the Labor Arbiter can no longer be disturbed without
violating the fundamental principle that final judgment is immutable and unalterable and may no
longer be modified in any respect, even if the modification is meant to correct erroneous
conclusion of fact and law.

143. Olores vs. Manila Doctors College et al., GR No. 201663, March 31, 2014
Facts:

ATHENA M. SALAS | LABOR CASE DIGEST 2015 321

Before us is a Petition for Review on Certiorari under Rule 45 of the Rules of Court seekingt he
reversal of the Janary 9, 2012 and April 27, 2012 Resolutions of the Court of Appeals (CA) in
CA- GR. SP No. 122596.

The facts, as found by the NLRC are as follows:

Respondent is a private higher educational institution dedicated to providing academic


degrees and certificate courses related to Allied Medical Services and Liberal Arts and
Sciences.

(Petitioner) was hired as a part-time faculty of respondent on 07 November 2005. He


was assigned at the Humanities Department of the College of Arts and Sciences. Thereafer, he
signed fixed term employment contracts as par-time instructor from 03 November 2008,
(peititioner) signed fixed term employemet, this time as a full time instructor.

(Petitioner) submitted the final grades of his students to Mr. Jacinto Bernanrdo, Jr
(Bernardo), the char of the Humanities Area. On 13 April 2010, Bernando charged (petitioner)
with gross misconduct and gross inefficiency in the performance of duty. (Petitioner) was
accused of employment a grading system not in accordance with the system

On 14 April 2010, (petitioner) submited his answer refuting the allegations of the chair of
Humanites. Also,
on this date, (petitioner) wrote a letter to respondents Human Resource
Manager asking that he should now be granted a permanent status.

Meanwhile, summer classes started on 15 April 2010 without (petitioner) having signed
an employment contract.

Acting on the report of Bernardo, respondent created the Manila Doctors tribunal (MDT)
which was tasked to ascertain the truth. The MDT sent notices of hearing to (petitioner)

On 31 May 2010, The MDT submitted its recoomendation to the president of respondnt.
The culpability of (petitioner) was established, hence dismissal was recoomended. On 07 June
2010, respondent terminated the services of petitioner for grave misconduct and gross
inefficieny and incompetence.

LA: found merit in petitioners charge for illegal dismissal. However it dismissed petitioners
charge for illegal dismissal.

ATHENA M. SALAS | LABOR CASE DIGEST 2015 322

NLRC: denied resolution. NLRC reasoned that respondents appeal was not accompanied by
netiher a cash nor surety bond, thus no appeal was perfected from the deicsion fo the LA.

Issues:
(1) Whether respondents appeal with the NLRC was perfected despite its failure to post a
bond;
(2) Whether the CA erred in dismissing petitioners Rule 65 petition.

Ruling:

There is merit in the petition.

At the outset, it must be emphized that Artcile 223 of the Labor Code states that an
appeal by the employer to the NLRC from a judgment of Labor Arbiter, which involves a
monetary award, may be perfected only upon the posting of a cash or surety bond issued by a
reputablebonding company duly accredited by the NLRC, in an amount equivalent ot he
moentary award in the judgment appealed from.

Section 4 (a) and 6 of Rule VI of the New rules of Procedure of the NLRC, as amended,
reaffirm the explicit jursidictional principle in Article 223.

The posting of a bond is indispensable to the perfection of an appeal in cases involving


monetary awards from the decisions of the Labor Arbiter. The lawmakers clearly intended to
make the bond a mandatory requisite for the perfection of the appeal by the employer as
inferred from the provision that an appealby the employer may be perfected only upon the
posting of a cash or surety bond. The word only makes it clear that the posting of a cash or
surety bond by the employer is the essential and exclusive means by which an employers
appeal may be perfected. Moreover, the filing of the bond is not only mandatory, but a
jurisdictional requirement as well that must be complied with in order to confer jurisdiction upon
the NLRC. Non compliance therewith renders the deicsion of the Labor Arbiter final and
executory. This requirement is intended to assure the workders that if they prevail in the case,
they will receive the money judgment in their upon the dismissal of the employers appeal. It is
intended to discourage employers from using an appeal to delay or evade their obligation to
satisfy their employees just and lawful claims.

Here, it is undisputed that respondents applea was not accompanied by any


appealbond despite the clear monetary obligation to pay petitioner his separation pay in the
amount of Php 100,000.00. Since the posting of a bond for the perfection of an appeal is both
mandatory and jurisdictional, the decision of the Labor Arbiter sought to be appealed before the
NLRC had already become final and executory. Therefore, the NLRC had no authroity to
entertain the appeal, much less to reverse the decision of the Labor Arbiter.

ATHENA M. SALAS | LABOR CASE DIGEST 2015 323

Nevertheless, assuming that the NLRC has jurisdction to take cognizance of the instant
case, this Court is inclined to favor petitioner because the instant case falls under one of the
recognized exception to the rule that a motion for reconsideration is necessary prior to the filing
of a certiorari petition.

The general rule is that a motion for reconsideration is indispensable before resort to
the special civil action for certiorari to afford the court or tribunal the opportunity to correct its
error, if any. The rule is well settled that the filing of a motion for reconsideration is an indispens
able condition to the filing of a special civil action for certiorari.

However, said rule is subject to several recognized exceptions:

(b) Where the questions raised in the certiorari proceedings have been duly raised and
passed upon by the lower court, or are the same as those raised and passed upon in the
lower court

Here. The NLRC had all the opprotunity to review its ruling and correct itself.

All told, the petition is meritorious. However, since this Court is not a trier of facts, we
cannot rule on the substantive issue of the case, i.e., whether petitioner has attained regular
status, inasmuch as the CA has not yet passed upon The factual issues raised by the parties

144. Bergonio et al., vs. South east Asian Airlines, GR No. 195227,.April 21, 2014
Facts:

On April 30, 2004, the petitioners filed before the LA a complaint for illegal dismissal and
illegal suspension with prayer for reinstatement against respondents South East Asian Airlines
(SEAIR) and Irene Dornier as SEAIRs President (collectively, the respondents).

In a decision dated May 31, 2005, the LA found the petitioners illegally dismissed and
ordered the respondents, among others, to immediately reinstate the petitioners with full
backwages. The respondents received their copy of this decision on July 8, 2005.6

On August 20, 2005, the petitioners filed before the LA a Motion for issuance of Writ of
Execution for their immediate reinstatement.

ATHENA M. SALAS | LABOR CASE DIGEST 2015 324

During the scheduled pre-execution conference held on September 14, 2005, the
respondents manifested their option to reinstate the petitioners in the payroll. The payroll
reinstatement, however, did not materialize. Thus, on September 22, 2005, the petitioners filed
before the LA a manifestation for their immediate reinstatement

On October 3, 2005, the respondents filed an opposition to the petitioners motion for
execution.7 They claimed that the relationship between them and the petitioners had already
been strained because of the petitioners threatening text messages, thus precluding the latters
reinstatement.

On October 7, 2005, the LA granted the petitioners motion and issued a writ of
execution.8
The respondents moved to quash the writ of execution with a prayer to hold in abeyance the
implementation of the reinstatement order.9 They maintained that the relationship between them
and the petitioners had been so strained that reinstatement was no longer possible.

In an order dated August 15, 2006,13 the NLRC dismissed the respondents appeal for
non-perfection. The NLRC likewise denied the respondents motion for reconsideration in its
November 29, 2006 resolution, prompting the respondents to file before the CA a petition for
certiorari.

The records show that the petitioners appealed the December 18, 2007 CA decision with
this Court. In a resolution dated August 4, 2008, the Court denied the petition. The Court
likewise denied the petitioners subsequent motion for reconsideration, and thereafter issued an
Entry of Judgment certifying that its August 4, 2008 resolution had become final and executory
on March 9, 2009.

On January 31, 2008, the petitioners filed with the LA an Urgent Ex-Parte Motion for the
Immediate Release of the Garnished Amount.

In its March 13, 2008 order, the LA granted the petitioners motion; it directed MetrobankSan Lorenzo to release the P1,900,000.00 garnished amount. The LA found valid and
meritorious the respondents claim for accrued wages in view of the respondents refusal to
reinstate the petitioners despite the final and executory nature of the reinstatement aspect of its
(LAs) May 31, 2005 decision. The LA noted that as of the December 18, 2007 CA decision (that
reversed the illegal dismissal findings of the LA), the petitioners accrued wages amounted to
P3,078,366.33.

In its July 16, 2008 resolution, the NLRC affirmed in toto the LAs March 13, 2008 order.
The NLRC afterwards denied the respondents motion for reconsideration for lack of merit.

ATHENA M. SALAS | LABOR CASE DIGEST 2015 325

Issue: Whether or not the order of reinstatement is immediately executory

Ruling:

Nature of the reinstatement aspect of the LAs decision on a finding of illegal dismissal

Article 223 (now Article 229) of the Labor Code governs appeals from, and the execution of, the
LAs decision. Pertinently, paragraph 3, Article 223 of the Labor Code provides:
Article 223. APPEAL
xxxx

In any event, the decision of the Labor Arbiter reinstating a dismissed or separated
employee, insofar as the reinstatement aspect is concerned, shall immediately be executory,
pending appeal. The employee shall either be admitted back to work under the same terms and
conditions prevailing prior to his dismissal or separation or, at the option of the employer, merely
reinstated in the payroll. The posting of a bond by the employer shall not stay the execution for
reinstatement provided herein. [Emphasis and underscoring supplied]

Under paragraph 3, Article 223 of the Labor Code, the LAs order for the reinstatement of
an employee found illegally dismissed is immediately executory even during pendency of the
employers appeal from the decision. Under this provision, the employer must reinstate the
employee either by physically admitting him under the conditions prevailing prior to his
dismissal, and paying his wages; or, at the employers option, merely reinstating the employee
in the payroll until the decision is reversed by the higher court. 22 Failure of the employer to
comply with the reinstatement order, by exercising the options in the alternative, renders him
liable to pay the employees salaries.

Otherwise stated, a dismissed employee whose case was favorably decided by the LA is
entitled to receive wages pending appeal upon reinstatement, which reinstatement is
immediately executor. Unless the appellate tribunal issues a restraining order, the LA is duty
bound to implement the order of reinstatement and the employer has no option but to comply
with it.

Moreover, and equally worth emphasizing, is that an order of reinstatement issued by


the LA is self-executory, i.e., the dismissed employee need not even apply for and the LA need
not even issue a writ of execution to trigger the employers duty to reinstate the dismissed
employee.

In short, therefore, with respect to decisions reinstating employees, the law itself has

ATHENA M. SALAS | LABOR CASE DIGEST 2015 326

determined a sufficiently overwhelming reason for its immediate and automatic execution even
pending appeal. The employer is duty-bound to reinstate the employee, failing which, the
employer is liable instead to pay the dismissed employees salary. The Courts consistent and
prevailing treatment and interpretation of the reinstatement order as immediately enforceable, in
fact, merely underscores the right to security of tenure of employees that the Constitution
protects.

145. Arabit et al., vs. Jardine Pacific Finance Inc. GR No. 181719, April 21, 2014
Facts:
Petitioners were former regular employees of respondent Jardine Pacific Finance, Inc. (formerly
MB Finance) (Jardine). The petitioners were also officers and members of MB Finance
Employees Association-FFW Chapter (the Union), a legitimate labor union and the sole
exclusive bargaining agent of the employees of Jardine.
On the claim of financial losses, Jardine decided to reorganize and implement a redundancy
program among its employees. The petitioners were among those affected by the redundancy
program. Jardine thereafter hired contractual employees to undertake the functions these
employees used to perform.
On June 1, 1999, the petitioners and the Union filed a complaint against Jardine with the NLRC
for illegal dismissal and unfair labor practice.
The LA ruled in the petitioners favor. The LA held that the hiring of contractual employees to
replace the petitioners directly contradicts the concept of redundancy which involves the
trimming down of the workforce because a task is being carried out by too many people. The LA
explained that the companys action was a circumvention of the right of the petitioners to
security of tenure.

The LA further held that it was not enough for Jardine to simply focus on its losses. According to
the LA, it was error for Jardine to simply lump together the seven petitioners as employees
whose positions have become redundant without explaining why their respective positions
became superfluous in relation to the other positions and employees of the company. 23 On the
petitioners allegation of unfair labor practice, the LA held that not enough evidence was
presented to prove the claim against Jardine.
Both parties appealed the LAs decision to the NLRC. But the NLRC dismissed the appeals and
affirmed the LAs decision in its entirety. Jardine moved for the reconsideration of the NLRCs
decision, which motion the NLRC also denied. Jardine thereafter sought recourse with the CA
via a petition for certiorari under Rule 65.
The CA reversed the LAs and the NLRCs rulings, and granted Jardines petition for certiorari.
The CA found that Jardines act of hiring contractual employees in replacement of the
petitioners does not run counter to the argument that their positions are already superfluous.
According to the CA, the hiring of contractual employees is a management prerogative that
Jardine has the right to exercise. In the absence of any showing of malice or arbitrariness on the
part of Jardine in implementing its redundancy program, the courts must not interfere with the
companys exercise of a bona fide management decision.

ATHENA M. SALAS | LABOR CASE DIGEST 2015 327

The CA denied the petitioners subsequent motion for reconsideration.

Issue:
whether or not the petitioners were illegally dismissed.
Ruling:
Redundancy in contrast with retrenchment
Jardine, in its petition for certiorari with the CA, posited that the distinction between redundancy
and retrenchment is not material.48 It contended that employers resort to these causes of
dismissal for purely economic considerations.49 Jardine further argued that the immateriality of
the distinction between these two just causes for dismissal is shown by the fact that redundancy
and retrenchment are found and lumped together in just one single provision of the Labor Code
(Article 283 thereof).
We cannot accept Jardines shallow understanding of the concepts of redundancy and
retrenchment in determining the validity of the severance of an employer-employee relationship.
The fact that they are found together in just one provision does not necessarily give rise to the
conclusion that the difference between them is immaterial. This Court has already ruled before
that retrenchment and redundancy are two different concepts; they are not synonymous; thus,
they should not be used interchangeably.50 The clear distinction between these two concepts
was discussed in Andrada, et al., v. NLRC, citing the case of Sebuguero v. NLRC, where this
Court clarified:
Redundancy exists where the services of an employee are in excess of what is reasonably
demanded by the actual requirements of the enterprise. A position is redundant where it is
superfluous, and superfluity of a position or positions may be the outcome of a number of
factors, such as over hiring of workers, decreased volume of business, or dropping of a
particular product line or service activity previously manufactured or undertaken by the
enterprise.
Retrenchment, on the other hand, is used interchangeably with the term "lay-off." It is the
termination of employment initiated by the employer through no fault of the employees and
without prejudice to the latter, resorted to by management during periods of business recession,
industrial depression, or seasonal fluctuations, or during lulls occasioned by lack of orders,
shortage of materials, conversion of the plant for a new production program or the introduction
of new methods or more efficient machinery, or of automation. Simply put, it is an act of the
employer of dismissing employees because of losses in the operation of a business, lack of
work, and considerable reduction on the volume of his business, a right consistently recognized
and affirmed by this Court.
These rulings appropriately clarify that redundancy does not need to be always triggered by a
decline in the business. Primarily, employers resort to redundancy when the functions of an
employee have already become superfluous or in excess of what the business requires. Thus,
even if a business is doing well, an employer can still validly dismiss an employee from the
service due to redundancy if that employees position has already become in excess of what the
employers enterprise requires.
From this perspective, it is illogical for Jardine to terminate the petitioners employment and
replace them with contractual employees. The replacement effectively belies Jardines claim

ATHENA M. SALAS | LABOR CASE DIGEST 2015 328

that the petitioners positions were abolished due to superfluity. Redundancy could have been
justified if the functions of the petitioners were transferred to other existing employees of the
company.
To dismiss the petitioners and hire new contractual employees as replacements necessarily give
rise to the sound conclusion that the petitioners services have not really become in excess of
what Jardines business requires. To replace the petitioners who were all regular employees
with contractual ones would amount to a violation of their right to security of tenure. For this, we
affirm the NLRCs ruling, citing the LAs decision, when it ruled:
In the case at bench, respondents did not dispute that after laying-off complainants herein, they
engaged the services of an agency to perform the tasks used to be done by complainants. This
is [in direct] contradiction to the concept of redundancy which precisely requires the trimming
down of the [workforce] because a task is being carried out by just too many people. The
subsequent contracting out to an agency the functions or duties that used to be the domain of
individual complainants herein is a circumvention of their constitutional rights to security of
tenure, and therefore illegal.
146. Mirant (Phils) Corp., vs. Caro, GR No. 181490, April 23, 2014
Facts:
Respondent was hired by Mirant Pagbilao on January 3, 1994 as its Logistics Officer. In 2002,
when Southern Company was sold to Mirant, respondent was already a Supervisor of the
Logistics and Purchasing Department of petitioner. At the time of the severance of his
employment, respondent was the Procurement Supervisor of Mirant Pagbilao assigned at
petitioner corporation's corporate office. As Procurement Supervisor, his main task was to serve
as the link between the Materials Management Department of petitioner corporation and its
staff, and the suppliers and service contractors in order to ensure that procurement is carried
out in conformity with set policies, procedures and practices. In addition, respondent was put
incharge of ensuring the timely, economical, safe and expeditious delivery of materials at the
right quality and quantity to petitioner corporation's plant. Respondent was also responsible for
guiding and overseeing the welfare and training needs of the staff of the Materials Management
Department. Due to the nature of respondent's functions, petitioner corporation considers his
position as confidential.
Respondent filed a complaint for illegal dismissal and money claims for 13th and 14th month
pay, bonuses and other benefits, as well as the payment of moral and exemplary damages and
attorney's fees.
On November 3, 2004, petitioner corporation conducted a random drug test where respondent
was randomly chosen among its employees who would be tested for illegal drug use. Through
an Intracompany Correspondence, these employees were informed that they were selected for
random drug testing to be conducted on the same day that they received the correspondence.
Respondent was duly notified that he was scheduled to be tested after lunch on that day. His
receipt of the notice was evidenced by his signature on the correspondence.
Respondent avers that at around 11:30 a.m. of the same day, he received a phone call from his
wife's colleague who informed him that a bombing incident occurred near his wife's work station
in Tel Aviv, Israel where his wife was then working as a caregiver. Respondent attached to his
Position Paper a Press Release of the Department of Foreign Affairs (DFA) in Manila to prove
the occurrence of the bombing incident and a letter from the colleague of his wife who allegedly
gave him a phone call from Tel Aviv.

ATHENA M. SALAS | LABOR CASE DIGEST 2015 329

On that same day, at around 6:15 p.m., respondent returned to petitioner corporation's office.
When he was finally able to charge his cellphone at the office, he received a text message from
Tina Cecilia (Cecilia), a member of the Drug Watch Committee that conducted the drug test,
informing him to participate in the said drug test. He immediately called up Cecilia to explain the
reasons for his failure to submit himself to the random drug test that day. He also proposed that
he would submit to a drug test the following day at his own expense. Respondent never heard
from Cecilia again. The Investigating Panel found respondent's explanations as to his
whereabouts on that day to be inconsistent, and recommended that he be suspended for four
weeks without pay.
Issue:
Whether or not there was illegal dismissal
Ruling:
There was illegal dismissal in the case at bar.
While the adoption and enforcement by petitioner corporation of its Anti-Drugs Policy is
recognized as a valid exercise of its management prerogative as an employer, such exercise is
not absolute and unbridled. Managerial prerogatives are subject to limitations provided by law,
collective bargaining agreements, and the general principles of fair play and justice. In the
exercise of its management prerogative, an employer must therefore ensure that the policies,
rules and regulations on work-related activities of the employees must always be fair and
reasonable and the corresponding penalties, when prescribed, commensurate to the offense
involved and to the degree of the infraction. 47 The Anti-Drugs Policy of Mirant fell short of these
requirements.
Petitioner corporation's subject Anti-Drugs Policy fell short of being fair and reasonable.
First. The policy was not clear on what constitutes "unjustified refusal" when the subject drug
policy prescribed that an employee's "unjustified refusal" to submit to a random drug test shall
be punishable by the penalty of termination for the first offense. To be sure, the term "unjustified
refusal" could not possibly cover all forms of "refusal" as the employee's resistance, to be
punishable by termination, must be "unjustified." To the mind of the Court, it is on this area
where petitioner corporation had fallen short of making it clear to its employees as well as to
management as to what types of acts would fall under the purview of "unjustified refusal."
It is not a mere jurisprudential principle, but an enshrined provision of law, that all doubts shall
be resolved in favor of labor. Thus, in Article 4 of the Labor Code, as amended, "[a]ll doubts in
the implementation and interpretation of the provisions of [the Labor] Code, including its
implementing rules and regulations, shall be resolved in favor of labor." In Article 1702 of the
New Civil Code, a similar provision states that "[i]n case of doubt, all labor legislation and all
labor contracts shall be construed in favor of the safety and decent living for the laborer."
Applying these provisions of law to the circumstances in the case at bar, it is not fair for this
Court to allow an ambiguous policy to prejudice the rights of an employee against illegal
dismissal. To hold otherwise and sustain the stance of petitioner corporation would be to adopt
an interpretation that goes against the very grain of labor protection in this jurisdiction. As
correctly stated by the Labor Arbiter, "when a conflicting interest of labor and capital are
weighed on the scales of social justice, the heavier influence of the latter must be counterbalanced by the sympathy and compassion the law must accord the underprivileged worker."
Second. The penalty of termination imposed by petitioner corporation upon respondent fell short
of being reasonable. Company policies and regulations are generally valid and binding between

ATHENA M. SALAS | LABOR CASE DIGEST 2015 330

the employer and the employee unless shown to be grossly oppressive or contrary to law as
in the case at bar.
To be sure, the unreasonableness of the penalty of termination as imposed in this case is further
highlighted by a fact admitted by petitioner corporation itself: that for the ten-year period that
respondent had been employed by petitioner corporation, he did not have any record of a
violation of its company policies.

147. Castro Jr. vs. Ateneo de Naga University et al., GR No. 175293, July 23, 2014
FACTS:

The petitioner started his employment with respondent Ateneo de Naga University (University)
in the first semester of school year 1960-1961. At the time of his dismissal, he was a regular and
full-time faculty member of the University's Accountancy Department in the College of
Commerce with a monthly salary of P29,846.20. Allegedly, he received on February 22, 2000 a
letter from respondent Fr. Joel Tabora, SJ., the University President, informing him that his
contract (which was set to expire on May 31, 2000) would no longer be renewed. After several
attempts to discuss the matter with Fr. Tabora in person, and not having been given any
teaching load or other assignments effective June 2000, he brought his complaint for illegal
dismissal.

The University denied the allegation of illegal dismissal, and maintained that the petitioner was a
participant and regular contributor to the Ateneo de Naga Employees Retirement Plan (Plan);
that upon reaching the age of 60 years on June 26, 1999, he was deemed automatically retired
under the Plan; and that he had been allowed to teach after his retirement only on contractual
basis.

On September 3, 2001, Labor Arbiter (LA) Jesus Orlando M. Quinones ruled in favor of the
petitioner.
Aggrieved, the respondents appealed to the NLRC.Simultaneously, they submitted a
manifestation stating that neither actual nor payroll reinstatement of the petitioner could be
effected because he had meanwhile been employed as a Presidential Assistant for Southern
Luzon Affairs with the position of Undersecretary; and that his reinstatement would result in dual
employment and double compensation which were prohibited by existing civil service rules and
regulations.
On July 12, 2002, the petitioner, citing the executory nature of the order for his
reinstatement, filed his motion to order the respondents to pay his salaries and benefits
accruing in the period from September 3, 2001 until July 3, 2002.
In his order dated October 10, 2002, LA Quinones, explaining that Article 223 of the Labor
Code granted to the employer the option to implement either a physical or a payroll
reinstatement, and that, therefore, the respondents must first exercise the option
regardless of the petitioner's employment with the Government, denied the petitioner's
motion, but ordered the respondents to exercise the option of either actual or payroll
reinstatement of the petitioner.

ATHENA M. SALAS | LABOR CASE DIGEST 2015 331

Dissatisfied, the petitioner filed a notice of partial appeal, but the notice was denied due course
on June 30, 2003.
Upon the denial of his motion for reconsideration, the petitioner elevated the matter to the
CA by petition for certiorari.

In the interim, on June 26, 2004, the petitioner executed a receipt and quitclaim in favor of
the University respecting his claim for the benefits under the Plan, to wit:
RECEIPT and QUITCLAIM
Date: June 26, 2004
This is to acknowledge receipt from ATENEO DE NAGA UNIVERSITY the total sum of SIX
HUNDRED FORTY SIX THOUSAND EIGHT HUNDRED TWENTY EIGHT PESOS & 42/100
(P646,828.42) represen: ing full payment of benefits due me pursuant to the Employees
retirement plan. In view of this payment, I hereby waive all my rights, title, interest in and
over my retirement benefits under said plan which is presently under trusteeship of Bank of
the Philippine Islands. BPI is hereby instructed to reimburse the company for the amount paid
by it to me out of whatever amount due me under the said retirement plan.
(sgd.)
CRISANTO F. CASTRO, JR.
Employee
A few days later, the petitioner sent the following letter to Fr. Tabora, viz:
Dear Fr. fabora,
This is tJ inform you that I am getting my retirement pay as you have approved, together
with the "RECEIPT AND QUIT CLAIM" which your Treasurer forced me to sign upon your
order and/or your lawyer. I will receive pay UNDER PROTEST, and under the following
conditions:
1. That I am getting this retirement pay without prejudice to the case that I have filled
[sic] against Ateneo, Fr. Joel Tabora and Edwin Bernal.
2. That I do not agree nor confirm with your computation as to the number of years of
service I have rendered.
3. That the total amount is still subject to verification.
For your information.
(sgd.)
CRISANTO F. CASTRO, JR.

Meanwhile, the NLRC rendered its decision affirming with modification the ruling of the
LA on the petitioner's illegal dismissal case.

However, upon motion for reconsideration, the NLRC REVERSED its ruling. In justifying its
reversal of its decision, the NLRC held that his execution of the receipt and quitclaim

ATHENA M. SALAS | LABOR CASE DIGEST 2015 332

respecting his benefits under the Plan estopped the petitioner from pursuing other
claims arising from his employer-employee relationship with the University, opining that:

Actually, the execution and signing of the Receipt and Quitclaim by complainantappellee,
in this case, only indicates that he voluntarily waived his rights to his money awards, as
stated in the Labor Arbiter's Decision, as affirmed with modification by the Commission (Second
Division). A person is precluded from maintaining a position inconsistent with one, in which he
has acquiesced x x x. Also, in his signing the said Receipt and Quitclaim, the necessary
implication is that the said document would cover any and all claims arising out of the
employment relationship.

On May 31, 2006, the CA dismissed the petitioner's petition for certiorari on the ground of its
having been rendered moot and academic by the aforecited August 31, 2005 decision of the
NLRC.

ISSUE:
1. WON the petitioner was stopped from pursuing his complaint for illegal dismissal upon
execution of the receipt and quitclaim
2. WON Petitioner has a right to accrued benefits.

RULING:

1. NO. Petitioner was stopped from pursuing his complaint for illegal dismissal upon execution
of the receipt and quitclaim. Execution of the receipt and quitclaim was not a settlement of
the petitioner's claim for accrued salaries.

The NLRC held that the petitioner was estopped from pursuing his complaint for illegal dismissal
upon his receipt of the benefits and his execution of the receipt and quitclaim. He insists,
however, that the payment he had received in protest pertained only to his retirement benefits.
We agree with the petitioner.

The text of the receipt and quitclaim was clear and straightforward, and it was to the effect that
the sum received by the petitioner represented ''full payment of benefits ... pursuant to the
Employee's retirement plan." As such, both the NLRC and the CA should have easily seen that
the quitclaim related only to the settlement of the retirement benefits, which benefits could not
be confused with the reliefs related to the complaint for illegal dismissal.

Worthy to stress is that retirement is of a different species from the reliefs awarded to an illegally
dismissed employee. Retirement is a form of reward for an employee's loyalty and service to the

ATHENA M. SALAS | LABOR CASE DIGEST 2015 333

employer, and is intended to help the employee enjoy the remaining years of his life, and to
lessen the burden of worrying about his financial support or upkeep. In contrast, the reliefs
awarded to an illegally dismissed employee are in recognition of the continuing employeremployee relationship that has been severed by the employer without just or authorized cause,
or without compliance with due process.

2. YES. He is entitled to the benefits. Claim for accrued benefits should be sustained despite
dismissal of the petitioner's complaint

The petitioner argues that according to Roquero v. Philippine Airlines, Inc., the employer is
obliged to reinstate and to pay the wages of the dismissed employee during the period of appeal
until its reversal by the higher Court; and that because he was not reinstated either actually or
by payroll, he should be held entitled to the accrued salaries.
The argument of the petitioner is correct.
Article 279 of the Labor Code, as amended, entitles an illegally dismissed employee to
reinstatement. Article 223 of the Labor Code requires the reinstatement to be immediately
executory even pending appeal. With its intent being ostensibly to promote the benefit of the
employee, Article 223 cannot be the source of any right of the employer to remove the
employee should he fail to immediately comply with the order of reinstatement. In Roquero, the
Comi ruled that the unjustified refusal of the employer to reinstate the dismissed employee
would entitle the latter to the payment of his salaries effective from the time when the employer
failed to reinstate him; thus, it became the ministerial duty of the LA to implement the order of
reinstatement. According to Triad Security & Allied Services v. Ortega, Jr., the law mandates the
prompt reinstatement of the dismissed or separated employee, without need of any writ of
execution.
The provision of Article 223 is clear that an award for reinstatement shall be immediately
executory even pending appeal and the posting of a bond by the employer shall not stay
the execution for reinstatement. The legislative intent is quite obvious, i.e., to make an award
of reinstatement immediately enforceable, even penoing appeal. To require the application for
and issuance of a wit of execution as prerequisites for the execution of a reinstatement award
v10uld certainly betray and run counter to the very object and intent of Article 223, i.e., the
immediate execution of a reinstatement order. The reason is simple. An application for a writ of
execution and its issuance could be delayed for numerous reasons. A mere continuance of
postponement of a scheduled hearing, for instance, or an inaction on the part of the Labor
Arbiter or the NLRC could easily delay the issuance of the writ thereby setting at naught the
strict mandate and noble purpose envisioned by Article 223. In other words, if the requirements
of Article 224 were to govern, as we so declared in Maranaw, then the executory nature of a
reinstatement order or award contemplated by Article 223 will be unduly circumscribed and
rendered ineffectual. In enacting the law, the legislature is presumed to have ordained a valid
and sensible law, one which operates no further than may be necessary to achieve its specific
purpose. Statutes, as a rule, are to be construed in the lights of the purpose to be achieved and
the evil sought to be remedied. And where the statute is fairly susceptible of two or more
constructions, that construction should be adopted which will most tend to give effect to the
manifest intent of the lawmaker and promote the object for which the statute was enacted, and a
construction should be rejected which would tend to render abortive other provisions of the
statute and to defeat the object which the legislator sought to attain by its enactment. In

ATHENA M. SALAS | LABOR CASE DIGEST 2015 334

introducing a new rule on the reinstatement aspect of a labor decision under R.A. No. 6715,
Congress should not be considered to be indulging in mere semantic exercise. On appeal,
however, the apellate tribunal concerned may enjoin or suspend the reinstatement order in the
exercise of its sound discretion.
Furthermore, the rule is that all doubts in the interpretation and implementation of labor laws
should be resolved in favor of labor. In ruling that an order or award for reinstatement does not
require a writ of execution, the Court is simply adhering and giving meaning to this rule.
Henceforth, we rule that an award or order for reinstatement is selfexecutory. After
receipt of the decision or resolution ordering the employee's reinstatement, the employer
has the right to choose whether to re-admit the employee to work under the same terms
and conditions prevailing prior to his dismissal or to reinstate the employee in the
payroll. In either instance, the employer has to inform the employee of his choice. The
notification is based on practical considerations for without notice, the employee has no
way of knowing if he has to report for work or not.
Hence, for as long as the employer continuously fails to actually implement the
reinstatement aspect of the decision of the LA, the employer's obligation to the employee
for his accrued backwages and other benefits continues to accumulate.
The next issue concerns whether or not the petitioner's claim for accrued salaries from the time
of the issuance of the order of reinstatement by LA Quinones until his actual reinstatement in
November 2002 was rendered moot and academic by the reversal of the decision of the LA.

The Court holds that the order of reinstatement of the petitioner was not rendered moot
and academic. He remained entitled to accrued salaries from notice of the LA's order of
reinstatement until reversal thereof. The employee could be barred from claiming accrued
salaries only when the failure to reinstate him was without the fault of the employer.

Considering that the respondents reinstated the petitioner only in November 2002, and that their
inability to reinstate him was without valid ground, they were liable to pay his salaries accruing
from the time of the decision of the LA (i.e., September 3, 2001) until his reinstatement in
November 2002. It did not matter that the respondents had yet to exercise their option to choose
between actual or payroll reinstatement at that point because the order of reinstatement was
immediately executory.

148. Phil. Touristers Inc et al., vs. Mas Transit Workers Union-ANGLO-KMU GR No.
201237, Sept. 3, 2014

Facts:
On June 14, 2000, (the union) filed for certification election before (DOLE). DOLE granted the
unions petition, prompting the client (MTI) to file a motion for reconsideration dated Feb. 7,
2001. Subsequently (MTI) prompted to sell its buses its franchise to another bus operator (PTI).
In view of the sale, MTI informed all its employees of the said sale and transfer its operations to
PTI. Also MTI advised the employees to apply to PTI.

ATHENA M. SALAS | LABOR CASE DIGEST 2015 335

The union filed a complaint for illegal dismissal on the grounds that this scheme was to frustrate
their right to self-organization and there was no actual transfer of ownership of the passenger
buses as the stockholders of MTI and PTI are one and the same. In their defense, MTI claimed
that it was already suffering from serious financial reverses.

LA ruling found MTI guilty of unfair labor practice, i.e illegal lock out. The LA held that MTIs
closure of business and cessation of operations, allegedly due to serious financial reverses,
were actually made to subvert the right of its employees to self-organization. Accordingly, MTI
and petitioners were adjudged jointly and severally liable for the individual respondents
backwages, separation pay, and attorneys fees.

NLRC proceedings Finding merit in petitioners motion for reconsideration, the NLRC, in an
Order45 dated September 30, 2004, reinstated their appeal. It held that there was substantial
compliance with the rules considering the subsequent posting of an additional bond to complete
the full judgment award, adding too that petitioners initial motion to reduce bond was based on
a meritorious ground that is, the inability of PTI to post the full amount due to its liquidity
problems as evidenced by its submitted AFS.46 However, considering that PTIs bonding
company, SSSICI, was not authorized to transact business in all courts all over the country per
the Courts Certification dated August 6, 2004, petitioners were directed to replace the bond,47
which they timely complied with through the posting of Supersedeas Bond No. SS-B-10150,48
in the amount of _12,833,000.00, issued on November 8, 2004 by the Far Eastern Surety &
Insurance Company, Inc.

On January 20, 2006, the NLRC rendered a Decision, modifying its April 19, 2004 Decision by
dismissing the complaint against petitioners. The modification was brought about by the NLRCs
finding that there were no factual and legal bases to hold petitioners jointly and severally liable
with MTI as the two corporations are separate and distinct juridical entities with different
stockholders and owners. To this end, it ruled that the individual respondents were employees of
MTI and not PTI, and that the sale of the passenger buses to PTI was not simulated or fictitious
since the deed evidencing said sale was duly notarized and approved by the LTFRB in a
Decision dated December 28, 2000.

CA Ruling - CA annulled and set aside the modified ruling of the NLRC finding the latter to have
acted with grave abuse of discretion in applying a liberal interpretation of the rules on perfection
of appeal. It held that PTIs alleged liquidity problems cannot be considered as a meritorious
ground to reduce the bond as there was no showing that they were incapable of posting at least
a surety bond equivalent to the full judgment award
ISSUE:
The central issue for the Courts resolution is whether or not the CA erred in ascribing grave
abuse of discretion on the part of the NLRC when the latter gave due course to petitioners
appeal and consequently issued a modified Decision absolving petitioners from liability.
Ruling:

ATHENA M. SALAS | LABOR CASE DIGEST 2015 336

For an appeal from the LAs ruling to the NLRC to be perfected, Article 223 (now Article 229)61
of the Labor Code requires the posting of a cash or surety bond in an amount equivalent to the
monetary award in the judgment appealed from.
In case of a judgment involving a monetary award, an appeal by the employer may be perfected
only upon the posting of a cash or surety bond issued by a reputable bonding company duly
accredited by the Commission in the amount equivalent to the monetary award in the judgment
appealed from.
While it has been settled that the posting of a cash or surety bond is indispensable to the
perfection of an appeal in cases involving monetary awards from the decision of the LA,62 the
Rules of Procedure of the NLRC63 (the Rules), particularly Section 6, Rule VI thereof,
nonetheless allows the reduction of the bond upon a showing of (a) the existence of a
meritorious ground for reduction, and (b) the posting of a bond in a reasonable amount in
relation to the monetary award be reversed and set aside. However, considering that there were
other issues raised in the said petition relating to the substantial merits of the case which were
left undecided, a remand of the case for the CAs resolution of these substantive issues remains
in order, in line with the doctrine of hierarchy of courts as espoused in the St. Martin Funeral
Home v. NLRC ruling.
149. Azuelo vs. Zameco II Electric Cooperative, Inc. GR No. 192573, October 22, 2014

Facts: Petitioner Ricardo N. Azuelo (Azuelo) was employed by the respondent ZAMECO II
Electric Cooperative, Inc. (ZAMECO) as a maintenance worker. a Complaint for illegal dismissal
and non-payment of benefits against ZAMECO. After several mediations, LA Bactin ordered the
parties to submit their respective position papers.
Azuelo, instead of submitting his position paper, moved that the submission of his position paper
be extended. This was granted by the LA. The deadline came, and the LA directed Azuelo to file
his position paper. Azuelo, instead, moved for the issuance of an order to direct ZAMECO to
furnish him with a copy of the investigation report with regard to his dismissal. ZAMECO
opposed the motion on ground that they had already furnished Azuelo with a copy. So the LA
dismissed the complaint on ground of "lack of interest".
Azuelo filed another complaint with the same allegations as his first complaint with the RAB of
the NLRC. ZAMECO filed a motion to dismiss on ground of res judicata. Azuelo countered that
the dismissal was without prejudice. The LA dismissed the 2nd complaint. The NLRC denied
reconsideration, and the CA affirmed.
Issue:
Whether the dismissal of his first complaint for illegal dismissal, on the ground of lack of interest
on his part to prosecute the same, bars the filing of another complaint for illegal dismissal
against ZAMECO based on the same allegations.
Ruling:
The 2005 Revised Rules of Procedure of the NLRC (2005 Revised Rules), the rules applicable
at the time of the controversy, is silent as to the nature of the dismissal of a complaint on the
ground of unreasonable failure to submit a position paper by the complainant. Nevertheless, the
2005 Revised Rules, particularly Section 3, Rule I thereof, provides for the suppletory
application of the Rules of Court to arbitration proceedings before the LAs and the NLRC in the
absence of any applicable provisions therein, viz:

ATHENA M. SALAS | LABOR CASE DIGEST 2015 337

Section 3. Suppletory Application of the Rules of Court. - In the absence of any applicable
provisions in these Rules, and in order to effectuate the objectives of the Labor Code, the
pertinent provisions of the Rules of Court of the Philippines may, in the interest of expeditious
dispensation of labor justice and whenever practicable and convenient, be applied by analogy or
in a suppletory character and effect.
The unjustified failure of a complainant in arbitration proceedings before the LA to submit his
position paper is akin to the case of a complainant's failure to prosecute his action for an
unreasonable length of time in ordinary civil proceedings. In both cases, the complainants are
remiss, sans reasonable cause, to prove the material allegations in their respective complaints.
Accordingly, the Court sees no reason not to apply the rules relative to unreasonable failure to
prosecute an action in ordinary civil proceedings to the unjustified failure of a complainant to
submit his position paper in arbitration proceedings before the LA.
In this regard, Section 3, Rule 17 of the Rules of Court provides that:
Section 3. Dismissal due to fault of plaintiff. If, for no justifiable cause, the plaintiff fails to
appear on the date of the presentation of his evidence in chief on the complaint, or to prosecute
his action for an unreasonable length of time, or to comply with these Rules or any order of the
court, the complaint may be dismissed upon motion of the defendant or upon the court's own
motion, without prejudice to the right of the defendant to prosecute his counterclaim in the same
or in a separate action. This dismissal shall have the effect of an adjudication upon the merits,
unless otherwise declared by the court.

"The dismissal of a case for failure to prosecute has the effect of adjudication on the merits, and
is necessarily understood to be with prejudice to the filing of another action, unless otherwise
provided in the order of dismissal. Stated differently, the general rule is that dismissal of a case
for failure to prosecute is to be regarded as adjudication on the merits and with prejudice to the
filing of another action, and the only exception is when the order of dismissal expressly contains
a qualification that the dismissal is without prejudice."
Thus, in arbitration proceedings before the LA, the dismissal of a complaint on account of the
unreasonable failure of the complainant to submit his position paper is likewise regarded as an
adjudication on the merits and with prejudice to the filing of another complaint, except when the
LA's order of dismissal expressly states otherwise.
The Order which dismissed Azuelo's first complaint due to his unreasonable failure to submit his
position paper is unqualified. It is thus considered as adjudication on the merits and with
prejudice to filing of another complaint. Accordingly, the NLRC did not abuse its discretion when
it affirmed LA Abdon's dismissal of the second complaint for illegal dismissal. Azuelo's filing of a
second complaint for illegal dismissal against ZAMECO based on the same allegations cannot
be permitted lest the rule on res judicata be transgressed.

ATHENA M. SALAS | LABOR CASE DIGEST 2015 338

21. RIGHT TO SELF-ORGANIZATION


150. National Union of bank Employees vs. Philnabank Employees Association G.R. No.
174287, Aug. 12, 2013
FACTS:
PNBs rank-and-file employeeswere represented for collective negotiation by the Philnabank
Employees Association (PEMA). PEMA affiliated with petitioner National Union of Bank
Employees (NUBE), which is a labor federation composed of unions in the banking industry,
adopting the name NUBE-PNB Employees Chapter (NUBE-PEC).NUBE-PEC was certified as
the sole and exclusive bargaining agent of the PNB rank-and-file employees.
Pursuant to Article V on Check-off and Agency Fees of the CBA, PNB shall deduct the monthly
membership fee and other assessments imposed by the union from the salary of each union
member, and agency fee (equivalent to the monthly membership dues) from the salary of the
rank-and-file employees within the bargaining unit who are not union members. Moreover,

ATHENA M. SALAS | LABOR CASE DIGEST 2015 339

during the effectivity of the CBA, NUBE, being the Federation union, agreed that PNB shall
remit P15.00 of the P65.00 union dues per month collected by PNB from every employee, and
that PNB shall directly credit the amount to NUBE's current account with PNB.
Following the expiration of the CBA, a petition for certification election was filed. While the
petition for certification election was still pending, two significant events transpired the
independent union registration of NUBE-PEC and its disaffiliation with NUBE.
With a legal personality derived only from a charter issued by NUBE, NUBE-PEC applied for a
separate registration with the DOLE. It was later registered as an independent labor
organization.
PEMA then sent a letter to the PNB management informing its disaffiliation from NUBE and
requesting to stop, effective immediately, the check-off of the P15.00 due for NUBE.
Acting thereon, PNB informed NUBE of PEMA's letter and its decision to continue the deduction
of the P15.00 fees, but stop its remittance to NUBE effective July 2003. PNB also notified NUBE
that the amounts collected would be held in a trust account pending the resolution of the issue
on PEMA's disaffiliation.
NUBE replied that: it remains as the exclusive bargaining representative of the PNB rank-andfile employees; by signing the Resolution (on disaffiliation), the chapter officers have abandoned
NUBE-PEC and joined another union; in abandoning NUBE-PEC, the chapter officers have
abdicated their respective positions and resigned as such; in joining another union, the chapter
officers committed an act of disloyalty to NUBE-PEC and the general membership. With regard
to the issue of non-remittance of the union dues, NUBE enjoined PNB to comply with the union
check-off provision of the CBA.
ISSUES:
Did PEMA validly disaffiliate itself from NUBE? Consequently, did NUBE still have the right to
check-off, or the right to collect the union dues held in trust by PNB?
RULING:
The right of the local union to exercise the right to disaffiliate from its mother union is well settled
in this jurisdiction. In MSMG-UWP v. Hon. Ramos, We held:
A local union has the right to disaffiliate from its mother union or declare its
autonomy. A local union, being a separate and voluntary association, is free to
serve the interests of all its members including the freedom to disaffiliate or
declare its autonomy from the federation which it belongs when circumstances
warrant, in accordance with the constitutional guarantee of freedom of
association.
The purpose of affiliation by a local union with a mother union [or] a
federation

ATHENA M. SALAS | LABOR CASE DIGEST 2015 340

". . . is to increase by collective action the bargaining power in respect


of the terms and conditions of labor. Yet the locals remained the basic
units of association, free to serve their own and the common interest of
all, subject to the restraints imposed by the Constitution and By-Laws
of the Association, and free also to renounce the affiliation for mutual
welfare upon the terms laid down in the agreement which brought it into
existence."
Thus, a local union which has affiliated itself with a federation is free to sever
such affiliation anytime and such disaffiliation cannot be considered disloyalty.
In the absence of specific provisions in the federation's constitution prohibiting
disaffiliation or the declaration of autonomy of a local union, a local may
dissociate with its parent union.
Likewise, Philippine Skylanders, Inc. v. National Labor Relations Commission restated:
The right of a local union to disaffiliate from its mother federation is not a novel
thesis unillumined by case law. In the landmark case of Liberty Cotton Mills
Workers Union vs. Liberty Cotton Mills, Inc., we upheld the right of local unions
to separate from their mother federation on the ground that as separate and
voluntary associations, local unions do not owe their creation and
existence to the national federation to which they are affiliated but,
instead, to the will of their members. The sole essence of affiliation is to
increase, by collective action, the common bargaining power of local
unions for the effective enhancement and protection of their interests.
Admittedly, there are times when without succor and support local unions may
find it hard, unaided by other support groups, to secure justice for themselves.
Yet the local unions remain the basic units of association, free to serve their
own interests subject to the restraints imposed by the constitution and by-laws
of the national federation, and free also to renounce the affiliation upon the
terms laid down in the agreement which brought such affiliation into existence.
And again, in Coastal Subic Bay Terminal, Inc. v. Department of Labor and Employment
Office of the Secretary, this Court opined:
Under the rules implementing the Labor Code, a chartered local union acquires
legal personality through the charter certificate issued by a duly registered
federation or national union, and reported to the Regional Office in accordance
with the rules implementing the Labor Code. A local union does not owe its
existence to the federation with which it is affiliated. It is a separate and distinct
voluntary association owing its creation to the will of its members. Mere
affiliation does not divest the local union of its own personality, neither
does it give the mother federation the license to act independently of the
local union. It only gives rise to a contract of agency, where the former
acts in representation of the latter. Hence, local unions are considered
principals while the federation is deemed to be merely their agent. As such
principals, the unions are entitled to exercise the rights and privileges of a
legitimate labor organization, including the right to seek certification as the sole
and exclusive bargaining agent in the appropriate employer unit.

ATHENA M. SALAS | LABOR CASE DIGEST 2015 341

Finally, the recent case of Cirtek Employees Labor Union-Federation of Free Workers v. Cirtek
Electronics, Inc. ruled:
. . . [A] local union may disaffiliate at any time from its mother
federation, absent any showing that the same is prohibited under its
constitution or rule. Such, however, does not result in it losing its legal
personality
altogether.
Verily,
Anglo-KMU
v.
Samahan
ng
mgaManggagawangNagkakaisasa Manila Bar Spinning Mills at J.P. Coats
enlightens: ADCSEa
A local labor union is a separate and distinct unit primarily
designed to secure and maintain an equality of bargaining
power between the employer and their employee-members. A
local union does not owe its existence to the federation
with which it is affiliated. It is a separate and distinct
voluntary association owing its creation to the will of its
members. The mere act of affiliation does not divest the
local union of its own personality, neither does it give the
mother federation the license to act independently of the
local union. It only gives rise to a contract of agency
where the former acts in representation of the latter.
In the case at bar, there is nothing shown in the records nor is it claimed by NUBE that PEMA
was expressly forbidden to disaffiliate from the federation nor were there any conditions
imposed for a valid breakaway. This being so, PEMA is not precluded to disaffiliate from NUBE
after acquiring the status of an independent labor organization duly registered before the DOLE.
Also, there is no merit on NUBE's contention that PEMA's disaffiliation is invalid for nonobservance of the procedure that union members should make such determination through
secret ballot and after due deliberation, conformably with Article 241 (d) of the Labor Code, as
amended. Conspicuously, other than citing the opinion of a "recognized labor law authority,"
NUBE failed to quote a specific provision of the law or rule mandating that a local union's
disaffiliation from a federation must comply with Article 241 (d) in order to be valid and effective.
Granting, for argument's sake, that Article 241 (d) is applicable, still, We uphold PEMA's
disaffiliation from NUBE. First, non-compliance with the procedure on disaffiliation, being
premised on purely technical grounds cannot rise above the employees' fundamental right to
self-organization and to form and join labor organizations of their own choosing for the purpose
of collective bargaining. Second, the Article nonetheless provides that when the nature of the
organization renders such secret ballot impractical, the union officers may make the decision in
behalf of the general membership. In this case, NUBE did not even dare to contest PEMA's
representation that "PNB employees, from where [PEMA] [derives] its membership, are
scattered from Aparri to Jolo, manning more than 300 branches in various towns and cities of
the country," hence, "[to] gather the general membership of the union in a general membership
to vote through secret balloting is virtually impossible." It is understandable, therefore, why
PEMA's board of directors merely opted to submit for ratification of the majority their resolution
to disaffiliate from NUBE. Third, and most importantly, NUBE did not dispute the existence of the
persons or their due execution of the document showing their unequivocal support for the
disaffiliation of PEMA from NUBE.

ATHENA M. SALAS | LABOR CASE DIGEST 2015 342

Consequently, by PEMA's valid disaffiliation from NUBE, the vinculum that previously bound the
two entities was completely severed. As NUBE was divested of any and all power to act in
representation of PEMA, any act performed by the former that affects the interests and affairs of
the latter, including the supposed expulsion of Serranaet al., is rendered without force and
effect.
Also, in effect, NUBE loses it right to collect all union dues held in its trust by PNB. The
moment that PEMA separated from and left NUBE and exists as an independent labor
organization with a certificate of registration, the former is no longer obliged to pay dues and
assessments to the latter; naturally, there would be no longer any reason or occasion for PNB to
continue making deductions. As we said inVolkschel Labor Union v. Bureau of Labor Relations:
. . . In other words, ALUMETAL [NUBE in this case] is entitled to receive the
dues from respondent companies as long as petitioner union is affiliated with
it and respondent companies are authorized by their employees (members of
petitioner union) to deduct union dues. Without said affiliation, the employer
has no link to the mother union. The obligation of an employee to pay union
dues is coterminous with his affiliation or membership. "The employees'
check-off authorization, even if declared irrevocable, is good only as
long as they remain members of the union concerned." A contract
between an employer and the parent organization as bargaining agent for the
employees is terminated by the disaffiliation of the local of which the
employees are members. . . .
151. Octavio vs. Phil Long Distance Telephone Company, G.R. No. 175492, Feb. 27, 2013
Facts:
PLDT and Gabay ng Unyon sa Telekominaksyon ng mga Superbisor (GUTS) entered into a
CBA. Article VI, Section I thereof provides: Section 1. The COMPANY agrees to grant the
following across-the-board salary increase during the three years covered by this Agreement to
all employees covered by the bargaining unit as of the given dates:
Effective January 1, 1999 10% of basic wage or P2,000.00 whichever is higher;
Effective January 1, 2000 11% of basic wage or P2,250.00 whichever is higher;
Effective January 1, 2001 12% of basic wage or P2,500.00 whichever is higher.

When Octavio, a Sales System Analyst and member of GUTS was regularized on January 1,
2001, he was receiving a monthly basic salary of P10,000.00. On February 1, 2002, he was
promoted to the position of Sales System Analyst 2 and his salary was increased to P13,730.00.

On May 31, 2002, PLDT and GUTS entered into another CBA covering the period January 1,
2002 to December 31, 2004 (CBA of 2002-2004) which provided for the following salary
increases: 8% of basic wage or P2,000.00 whichever is higher for the first year (2002); 10% of
basic wage or P2,700.00 whichever is higher for the second year (2003); and, 10% of basic
wage or P2,400.00 whichever is higher for the third year (2004).

ATHENA M. SALAS | LABOR CASE DIGEST 2015 343

Claiming that he was not given the salary increases of P2,500.00 effective January 1, 2001 and
P2,000.00 effective January 1, 2002, Octavio wrote the President of GUTS, who in turn wrote
the PLDT Human Resource Head to inform management of the GUTS members claim for
entitlement to the increases. Accordingly, the Union-Management Grievance Committee
convened on October 7, 2002 consisting of representatives from PLDT and GUTS. The
Grievance Committee, however, failed to reach an agreement. In effect, it denied Octavios
demand for salary increases. Aggrieved, Octavio filed before the Arbitration Branch of the NLRC
a Complaint for payment of said salary increases.

Issues:
(1) WON Octavio resorted to the proper procedure by filing a Complaint with the LA;
(2) WON the Greivance Committee Resolution was valid;
(3) WON denial of his claim for salary increases constituted a violation of Article 100;
Ruling:

Issue 1: Under Article 260 of the Labor Code, grievances arising from the interpretation or
implementation of the parties CBA should be resolved in accordance with the grievance
procedure embodied therein. It also provides that all unsettled grievances shall be automatically
referred for voluntary arbitration as prescribed in the CBA.

In its Memorandum, PLDT set forth the grievance machinery and procedure provided under its
CBA of 2002-2004 which in part reads: If the grievance is not settled either because of
deadlock or the failure of the committee (Grievance Committee) to decide the matter, the
grievance shall be transferred to a Board of Arbitrators for the final decision. The Board
shall be composed of three (3) arbitrators, one to be nominated by the Union, another to be
nominated by the Management, and the third to be selected by the management and union
nominees. The decision of the board shall be final and binding both the company and the Union
in accordance with law. Expenses of arbitration shall be divided equally between the Company
and the Union.

Indisputably, the present controversy involves the determination of an employees salary


increases as provided in the CBAs. When Octavios claim for salary increases was referred to
the Union-Management Grievance Committee, the clear intention of the parties was to resolve
their differences on the proper interpretation and implementation of the pertinent provisions of
the CBAs. And in accordance with the procedure prescribed therein, the said committee
convened. Unfortunately, it failed to reach an agreement. Octavios recourse pursuant to the
CBA was to elevate his grievance to the Board of Arbitrators for final decision. Instead, Octavio
filed a Complaint before the NLRC.

It is settled that "when parties have validly agreed on a procedure for resolving grievances and
to submit a dispute to voluntary arbitration then that procedure should be strictly observed."
Before a party is allowed to seek the intervention of the court, it is a precondition that he

ATHENA M. SALAS | LABOR CASE DIGEST 2015 344

should have availed of all the means of administrative processes afforded him (exhaustion of
administrative remedies). Hence, if a remedy within the administrative machinery can still be
resorted to by giving the administrative officer concerned every opportunity to decide on a
matter that comes within his jurisdiction, then such remedy should be exhausted first before the
courts judicial power can be sought. The premature invocation of the courts judicial intervention
is fatal to ones cause of action. By failing to question the Committee Resolution through the
proper procedure prescribed in the CBA, that is, by raising the same before a Board of
Arbitrators, Octavio is deemed to have waived his right to question the same. Clearly, he
departed from the grievance procedure mandated in the CBA and denied the Board of
Arbitrators the opportunity to pass upon a matter over which it has jurisdiction. Hence, Octavios
failure to assail the validity and enforceability of the Committee Resolution makes the same
binding upon him.

Issue 2: Octavio cannot claim that the Committee Resolution is not valid, binding and
conclusive as to him for being a modification of the CBA in violation of Article 253 of the Labor
Code. The said resolution is a product of the grievance procedure outlined in the CBA itself
arrived at after the management and the union through their respective representatives
conducted negotiations in accordance with the CBA. The union representatives are deemed to
have properly bargained on his behalf since "unions are the agent of its members for the
purpose of securing just and fair wages and good working conditions." In fine, it cannot be
gainsaid that the Committee Resolution is a modification of the CBA. Rather, it only provides for
the proper implementation of the CBA provision respecting salary increases.

Issue 3: Octavios argument that the denial of his claim for salary increases constitutes a
violation of Article 100 of the Labor Code is devoid of merit. Even assuming that there has been
a diminution of benefits on his part, Article 100 does not prohibit a union from offering and
agreeing to reduce wages and benefits of the employees as the right to free collective
bargaining includes the right to suspend it. PLDT averred that one of the reasons why
Octavios salary was recomputed as to include in his salary of P13,730.00 the P2,000.00
increase for 2002 is to avoid salary distortion. Bargaining should not be equated to an
"adversarial litigation where rights and obligations are delineated and remedies applied."
Instead, it covers a process of finding a reasonable and acceptable solution to stabilize labormanagement relations to promote stable industrial peace. Clearly, the Committee Resolution
was arrived at after considering the intention of both PLDT and GUTS to foster industrial peace.

152. Takata Phils Corp. vs. Bureau of Labor Relations, et al., GR No. 196276, June 4,
2014

FACTS:
Petioner filed with the DOLE Regional Office a Petitionfor Cancellation of the Certificate of
Union Registration of Respondent SamahangLakasManggagawangTakata (SALAMAT) on the
ground that the latter is guilty of misrepresentation, false statement and fraud with respect to
the number of those who participated in the organizational meeting, the adoption and
ratification of its Constitution and By-Laws, and in the election of its officers. It contended that

ATHENA M. SALAS | LABOR CASE DIGEST 2015 345

in the May 1, 2009 organizational meeting of respondent, only 68 attendees signed the
attendance sheet, and which number comprised only 17% of the total number of the 396
regular rank- and-file employees which respondent sought to represent, and hence,
respondent failed to comply with the 20% minimum membership requirement. Petitioner
insisted that the document PangalanngmgaKasapingUnyon bore no signatures of the
alleged 119 union members; and that employees were not given sufficient information on the
documents they signed; that the 119 union members were actually only 117; and, that the total
number of petitioner's employees as of May 1, 2009 was 470, and not 396 as respondent
claimed.
Respondent denied the charge and claimed that the 119 union members were more than the
20%
requirement
for
union
registration.
The
document
SamaSamangPahayagngPagsapisaUnyon which it presented in its petition for certification
electionsupported their claim of 119 members. Respondent argued that the union members
were informed of the contents of the documents they signed and that the 68 attendees to the
organizational meeting constituted more than 50% of the total union membership, hence, a
quorum
existed
for
the
conduct
of
the
said
meeting.lawred
DOLE Regional Director issued a Decision granting the petition for cancellation of
respondent's certificate of registration.
Respondent filed an Appeal Memorandum with Formal Entry of Appearance to the Office of
the DOLE Secretary, which the latter eventually referred to the BLR.

The BLR rendered its Decision reversing the Order of the Regional Director.
Petitioner went to the CA via a petition for certiorari under Rule 65 but denied the petition and
affirmed the decision of the BLR.

ISSUE:
Whether or not respondents certificate of registration should be cancelled on the grounds of
fraud and misrepresentation.

RULING:

There

is

no

merit

to

cancel

respondents

certificate

of

registration.

Art. 234 of the Labor Code provides:C


ART. 234. Requirements of Registration. - A federation, national union or industry or trade
union center or an independent union shall acquire legal personality and shall be entitled to
the rights and privileges granted by law to legitimate labor organizations upon issuance of the
certificate of registration based on the following requirements:
(a) Fifty pesos (P50.00) registration fee;

ATHENA M. SALAS | LABOR CASE DIGEST 2015 346

(b) The names of its officers, their addresses, the principal address of the labor organization,
the minutes of the organizational meetings and the list of the workers who participated in such
meetings;
(c) In case the applicant is an independent union, the names of all its members comprising at
least twenty percent (20%) of all the employees in the bargaining unit where it seeks to
operate;
(d) If the applicant union has been in existence for one or more years, copies of its annual
financial reports; and
(e) Four copies of the constitution and by-laws of the applicant union, minutes of its adoption
or ratification, and the list of the members who participated in it.
And after the issuance of the certificate of registration, the labor organization's registration
could be assailed directly through cancellation of registration proceedings in accordance with
Articles 238 and 239 of the Labor Code. And the cancellation of union certificate of registration
and the grounds thereof are as follows:Ch
ART. 238. Cancellation of Registration. - The certificate of registration of any legitimate
labor organization, whether national or local, may be cancelled by the Bureau, after due
hearing,
only
on
the
grounds
specified
in
Article
239
hereof.
ART. 239. Grounds for Cancellation of Union Registration. - The following may constitute
grounds for cancellation of union registration:
(a) Misrepresentation, false statement or fraud in connection with the adoption or ratification of
the constitution and by-laws or amendments thereto, the minutes of ratification, and the list of
members who took part in the ratification;
(b) Misrepresentation, false statements or fraud in connection with the election of officers,
minutes of the election of officers, and the list of voters;
(c) Voluntary dissolution by the members.
Petitioner's charge that respondent committed misrepresentation and fraud in securing its
certificate of registration is a serious charge and must be carefully evaluated. Allegations
thereof should be compounded with supporting circumstances and evidence. We find no
evidence
on
record
to
support
petitioner's
accusation.
Petitioner's allegation of misrepresentation and fraud is based on its claim that during the
organizational meeting on May 1, 2009, only 68 employees attended, while respondent
claimed that it has 119 members as shown in the document denominated as
PangalanngmgaKasapingUnyon; hence, respondent misrepresented on the 20%
requirement of the law as to its membership.
We

do

not

agree.

It does not appear in Article 234 (b) of the Labor Code that the attendees in the organizational
meeting must comprise 20% of the employees in the bargaining unit. In fact, even the
Implementing Rules and Regulations of the Labor Code does not so provide. It is only under

ATHENA M. SALAS | LABOR CASE DIGEST 2015 347

Article 234 (c) that requires the names of all its members comprising at least twenty percent
(20%) of all the employees in the bargaining unit where it seeks to operate. Clearly, the 20%
minimum requirement pertains to the employees membership in the union and not to
the list of workers who participated in the organizational meeting. Indeed, Article 234 (b)
and (c) provide for separate requirements, which must be submitted for the union's
registration, and which respondent did submit. Here, the total number of employees in the
bargaining unit was 396, and 20% of which was about 79. Respondent submitted a document
entitled PangalanngMgaKasapingUnyon showing the names of 119 employees as union
members, thus respondent sufficiently complied even beyond the 20% minimum membership
requirement. Respondent also submitted the attendance sheet of the organizational meeting
which contained the names and signatures of the 68 union members who attended the
meeting. Considering that there are 119 union members which are more than 20% of all
the employees of the bargaining unit, and since the law does not provide for the
required number of members to attend the organizational meeting, the 68 attendees
which comprised at least the majority of the 119 union members would already
constitute a quorum for the meeting to proceed and to validly ratify the Constitution and Bylaws of the union. There is, therefore, no basis for petitioner to contend that grounds exist for
the cancellation of respondent's union registration. For fraud and misrepresentation to be
grounds for cancellation of union registration under Article 239 of the Labor Code, the nature
of the fraud and misrepresentation must be grave and compelling enough to vitiate the
consent
of
a
majority
of
union
members.
Petitioner's claim that the alleged union members signed documents without adequate
information is not persuasive. The one who alleges a fact has the burden of proving it and a
mere allegation is not evidence.
Petitioner claims that in the list of members, there was an employee whose name appeared
twice and another employee who was merely a project employee. Such could not be
considered a misrepresentation in the absence of showing that respondent deliberately did so
for the purpose of increasing their union membership. In fact, even if those two names were
not included in the list of union members, there would still be 117 members which was still
more
than
20%
of
the
396
rank-and-file
employees.
As to petitioner's argument that the total number of its employees as of May 1, 2009 was 470,
and not 396 as respondent claimed, still the 117 union members comprised more than the
20%
membership
requirement
for
respondent's
registration.
The bare fact that two signatures appeared twice on the list of those who participated in the
organizational meeting would not, to our mind, provide a valid reason to cancel respondents
certificate of registration. The cancellation of a unions registration doubtless has an
impairing dimension on the right of labor to self-organization. For fraud and
misrepresentation to be grounds for cancellation of union registration under the Labor Code,
the nature of the fraud and misrepresentation must be grave and compelling enough to
vitiate
the
consent
of
a
majority
of
union
members.
The alleged failure of respondent to indicate with mathematical precision the total number of
employees in the bargaining unit is of no moment, especially as it was able to comply with the
20% minimum membership requirement.

ATHENA M. SALAS | LABOR CASE DIGEST 2015 348

22. RIGHTS OF LEGITIMATE LABOR ORGANIZATION

153. Digital Tel. Phils Inc. vs. Digitel Employees Union et al., G.R. No. 184903-04, October
10,
2012
FACTS:
Digitel Employees Union (Union) became the exclusive bargaining agent of all rank and file
employees of Digitel in 1994. The Union and Digitel then commenced collective
bargainingnegotiations which resulted in a bargaining deadlock. The Union threatened to go on
strike, but then Acting Labor Secretary assumed jurisdiction over the dispute and eventually
directed the parties to execute a CBA.2
However, no CBA was forged between Digitel and the Union. Some Union members abandoned
their employment with Digitel. The Union later became dormant.
Ten (10) years thereafter Digitel received from Esplana, who identified himself as President of
the Union, a letter containing the list of officers, CBA proposals and ground rules.
Digitel was reluctant to negotiate with the Union and demanded that the latter show compliance
with the provisions of the Unions Constitution and By-laws on union membership and election
of officers.
Esplana and his group filed a case based on Digitels violation of the duty to bargain. Digitel filed
a petition with the Bureau of Labor Relations (BLR) seeking cancellation of the Unions
registration.
In an Order, the Secretary of Labor directed Digitel to commence the CBA negotiation with the
Union.
Thus:
WHEREFORE, all the foregoing premises considered, this Office hereby orders:
1. DIGITEL to commence collective bargaining negotiation with DEU without further delay.
Digitel moved for reconsideration on the contention that the pendency of the petition for
cancellation of the Unions certificate of registration is a prejudicial question that should first be
settled before the DOLE could order the parties to bargain collectively.
ISSUE:
Whether the Secretary of Labor erred in issuing the assumption order despite the pendency of
the petition for cancellation of union registration.
HELD:
It is well-settled that the pendency of a petition for cancellation of union registration does not
preclude collective bargaining.

The 2005 case of Capitol Medical Center, Inc. v. Hon. Trajano13 is apropos. The respondent
union therein sent a letter to petitioner requesting a negotiation of their CBA. Petitioner refused
to bargain and instead filed a petition for cancellation of the unions certificate of registration.

ATHENA M. SALAS | LABOR CASE DIGEST 2015 349

Petitioners refusal to bargain forced the union to file a notice of strike. They eventually staged a
strike. The Secretary of Labor assumed jurisdiction over the labor dispute and ordered all
striking workers to return to work. Petitioner challenged said order by contending that its petition
for cancellation of unions certificate of registration involves a prejudicial question that should
first be settled before the Secretary of Labor could order the parties to bargain collectively.
When the case eventually reached this Court, we agreed with the Secretary of Labor that the
pendency of a petition for cancellation of union registration does not preclude collective
bargaining, thus:

That there is a pending cancellation proceeding against the respondent Union is not a bar to set
in motion the mechanics of collective bargaining. If a certification election may still be ordered
despite the pendency of a petition to cancel the unions registration certificate, more so should
the collective bargaining process continue despite its pendency. We must emphasize that the
majority status of the respondent Union is not affected by the pendency of the Petition for
Cancellation pending against it. Unless its certificate of registration and its status as the certified
bargaining agent are revoked, the Hospital is, by express provision of the law, duty bound to
collectively bargain with the Union.14
Citing the cases of Association of Court of Appeals Employees v. Ferrer- Calleja 16 and Samahan
ng Manggagawa sa Pacific Plastic v. Hon. Laguesma,17 it was pointed out at the time of the filing
of the petition for certification election or a CBA process as in the instant case the union still
had the personality to file a petition for certification or to ask for a CBA negotiation as in the
present case.
154. Automotive Engine Rebuilders vs. Progresibong Unyon ng mga Manggagawa sa
AER G.R. No. 160138, January 16, 2013 Resolution on the Main decision of July 13, 2011
Facts:
For resolution is the Motion for Partial Reconsideration filed by Progresibong Unyon Ng Mga
Manggagawa Sa AER (Unyon) which questioned the Courts July 13, 2011 Decision insofar as it
failed to award backwages to fourteen (14) of its members. The decretal portion of the decision
reads:
WHEREFORE, the petitions are DENIED. Accordingly, the complaining
employees should be reinstated without backwages. If reinstatement is no
longer feasible, the concerned employees should be given separation pay up to
the date set for their return in lieu of reinstatement.
The precedents of this case as discussed in the 2011 case are as follows:
Both parties filed charges against each other, blaming the other party for violating labor
laws. AER filed a complaint against Unyon and its 18 members for illegal concerted activities. It
likewise suspended 7 union members who tested positive for illegal drugs. On the other hand,
Unyon filed a countercharge accusing AER of unfair labor practice, illegal suspension and illegal
dismissal. In other words, AER claims that Unyon was guilty of staging an illegal strike while
Unyon claims that AER committed an illegal lockout.
AERs fault is obvious from the fact that a day after the union filed a petition for
certification election before the DOLE, it hit back by requiring all its employees to undergo a
compulsory drug test. Although AER argues that the drug test was applied to all its employees, it

ATHENA M. SALAS | LABOR CASE DIGEST 2015 350

was silent as to whether the drug test was a regular company policy and practice in their 35
years in the automotive engine repair and rebuilding business. As the Court sees it, it
was AERs first ever drug test of its employees immediately implemented after the workers
manifested their desire to organize themselves into a union. Indeed, the timing of the drug test
was suspicious.
In awarding reinstatement but denying the award of backwages, the court rejected the
conclusion of waiver by the employee of his right to bring an action against illegal strike and
upheld the in pari delicto doctrine or restoration of conditions prior to the illegal strike and illegal
lockout. Adopting the following rationalization in a relevant jurisprudence, it reasoned out in the
2011 case:
Truly, it is more logical and reasonable for condonation to
apply only to strikers who signified their intention to return and did
return to work. The reason is obvious. These strikers took the
initiative in normalizing relations with their employer and thus
helped promote industrial peace. However, as regards the strikers
who decided to pursue with the case, as in the case of the 114
strikers herein, the employer could not be deemed to have
condoned their strike, because they had not shown any
willingness to normalize relations with it. So, if petitioner really had
any intention to pardon the 114 strikers, it would have included
them in its motion to withdraw on November 17, 1980. The fact
that it did not, but instead continued to pursue the case to the end,
simply means that it did not pardon the 114 strikers.
The findings show that both petitioner and the 114 strikers
are in pari delicto, a situation which warrants the maintenance of
the status quo.This means that the contending parties must be
brought back to their respective positions before the
controversy; that is, before the strike. Therefore, the order
reinstating the 114 employees is proper.
With such restoration of the status quo ante it necessarily follows, as
likewise submitted by the Solicitor General, that the petition must be granted
insofar as it seeks the setting aside of the award of three months' backwages to
the 114 employees ordered reinstated on the basis of the general rule that
strikers are not entitled to backwages (with some exceptions not herein
applicable, such as where the employer is guilty of oppression and union-busting
activities and strikers ordered reinstated are denied such reinstatement and
therefore are declared entitled to backwages from the date of such denial). More
so, is the principle of "no work, no pay" applicable to the case at bar, in view of
the undisputed finding of illegality of the strike.
The union filed a motion for partial consideration from the decision. It argued that backwages
should have been awarded to the 14 employees who were excluded from the complaint filed by
AER and that the latter should have reinstated them immediately because they did not have any
case
at
all.
Issue:

ATHENA M. SALAS | LABOR CASE DIGEST 2015 351

WON the employees are entitled to backwages, aside from reinstatement when both the
employee and employer are at fault or in pari delicto
Ruling:
YES, they are entitled to REINSTATEMENT AND BACKWAGES despite the facts that these
employees also erred their employer. These excluded nine (9) workers, who signed their names
in their petition before the CA, deserve to be reinstated immediately and granted backwages. It
is basic in jurisprudence that illegally dismissed workers are entitled to reinstatement with back
wages pi us interest at the legal rate. Only 9 out of the 14 excluded employees can be granted
the relief they prayed for because the others failed to affix their signatures in the Membership
Resolution of the Union authorizing the filing of the instant Motion for Partial Consideration.
The reinstatement shall be "without prejudice to the right of private respondent AER to subject
them for further medical check-up to determine if subject petitioners are drug dependents.

155. Holy Child Catholic School vs. Hon. Sto Tomas et al., G.R. No. 179146, July 23, 2013
FACTS:
On May 31, 2002, a petition for certification election was filed by private respondent PinagIsangTinig at Lakas ng Anakpawis Holy Child Catholic School Teachers and Employees Labor
Union (HCCS-TELU-PIGLAS), alleging that: PIGLAS is a legitimate labor organization duly
registered with the Department of Labor and Employment (DOLE) representing HCCS-TELUPIGLAS; HCCS is a private educational institution duly registered and operating under
Philippine laws; there are approximately one hundred twenty (120) teachers and employees
comprising the proposed appropriate bargaining unit; and HCCS is unorganized, there is no
collective bargaining agreement or a duly certified bargaining agent or a labor organization
certified as the sole and exclusive bargaining agent of the proposed bargaining unit within one
year prior to the filing of the petition.6 Among the documents attached to the petition were the
certificate of affiliation with Pinag-IsangTinig at Lakas ng AnakpawisKristiyanongAlyansa ng
MakabayangObrero (PIGLAS-KAMAO) issued by the Bureau of Labor Relations (BLR), charter
certificate issued by PIGLAS-KAMAO, and certificate of registration of HCCS-TELU as a
legitimate labor organization issued by the DOLE.7

In its Comment8 and Position Paper,9 petitioner HCCS consistently noted that it is a parochial
school with a total of 156 employees as of June 28, 2002, broken down as follows: ninety-eight
(98) teaching personnel, twenty-five (25) non-teaching academic employees, and thirty-three
(33) non-teaching non-academic workers. It averred that of the employees who signed to
support the petition, fourteen (14) already resigned and six (6) signed twice. Petitioner raised
that members of private respondent do not belong to the same class; it is not only a mixture of
managerial, supervisory, and rank-and-file employees as three (3) are vice-principals, one (1)
is a department head/supervisor, and eleven (11) are coordinators but also a combination of
teaching and non-teaching personnel as twenty-seven (27) are non-teaching personnel. It
insisted that, for not being in accord with Article 24510 of the Labor Code, private respondent is
an illegitimate labor organization lacking in personality to file a petition for certification election,
as held in Toyota Motor Philippines Corporation v. Toyota Motor Philippines Corporation Labor
Union; and an inappropriate bargaining unit for want of community or mutuality of interest, as

ATHENA M. SALAS | LABOR CASE DIGEST 2015 352

ruled in Dunlop Slazenger (Phils.), Inc. v. Secretary of Labor and Employment and De La Salle
University Medical Center and College of Medicine v. Laguesma.
On August 10, 2002, Med-Arbiter Agatha Ann L. Daquigan denied the petition for certification
election on the ground that the unit which private respondent sought to represent is
inappropriate.
Private respondent appealed before the SOLE, who, on December 27, 2002, ruled against the
dismissal of the petition and directed the conduct of two separate certification elections for the
teaching and the non-teaching personnel.
ISSUE:
Whether or not a petition for certification election is dismissible on the ground that the labor
organizations membership allegedly consists of supervisory and rank-and-file employees.
RULING:
It was in R.A. No. 875, under Section 3, that such questioned mingling was first prohibited, to
wit:
Sec. 3. Employees' right to self-organization. - Employees shall have the right to selforganization and to form, join or assist labor organizations of their own choosing for the purpose
of collective bargaining through representatives of their own choosing and to engage in
concerted activities for the purpose of collective bargaining and other mutual aid or protection.
Individuals employed as supervisors shall not be eligible for membership in a labor organization
of employees under their supervision but may form separate organizations of their own.
(Emphasis supplied)
Nothing in R.A. No. 875, however, tells of how the questioned mingling can affect the legitimacy
of the labor organization. Under Section 15, the only instance when a labor organization loses
its legitimacy is when it violates its duty to bargain collectively; but there is no word on whether
such mingling would also result in loss of legitimacy.
Then the Labor Code was enacted in 1974 without reproducing Sec. 3 of R.A. No. 875. The
provision in the Labor Code closest to Sec. 3 is Article 290, which is deafeningly silent on the
prohibition against supervisory employees mingling with rank-and-file employees in one labor
organization.
Effective 1989, R.A. No. 6715 restored the prohibition against the questioned mingling in one
labor organization, viz.:

Sec. 18. Article 245 of the same Code, as amended, is hereby further amended to read as
follows:
Art. 245. Ineligibility of managerial employees to join any labor organization; right of supervisory
employees. Managerial employees are not eligible to join, assist or form any labor organization.
Supervisory employees shall not be eligible for membership in a labor organization of the rankand-file employees but may join, assist or form separate labor organizations of their own
(Emphasis supplied)
Unfortunately, just like R.A. No. 875, R.A. No. 6715 omitted specifying the exact effect any
violation of the prohibition would bring about on the legitimacy of a labor organization.

ATHENA M. SALAS | LABOR CASE DIGEST 2015 353

But then, on June 21, 1997, the 1989 Amended Omnibus Rules was further amended by
Department Order No. 9, series of 1997 (1997 Amended Omnibus Rules). Specifically, the
requirement under Sec. 2(c) of the 1989 Amended Omnibus Rules - that the petition for
certification election indicate that the bargaining unit of rank-and-file employees has not been
mingled with supervisory employees - was removed. Instead, what the 1997 Amended Omnibus
Rules requires is a plain description of the bargaining unit.
When a similar issue confronted this Court close to three years later, the above ruling was
substantially quoted in SamahangManggagawasa Charter Chemical Solidarity of Unions in the
Philippines for Empowerment and Reforms (SMCC-Super) v. Charter Chemical and Coating
Corporation.47 In unequivocal terms, We reiterated that the alleged inclusion of supervisory
employees in a labor organization seeking to represent the bargaining unit of rank-and-file
employees does not divest it of its status as a legitimate labor organization.
Following the doctrine laid down in Kawashima and SMCC-Super, it must be stressed that
petitioner cannot collaterally attack the legitimacy of private respondent by praying for the
dismissal of the petition for certification election:
Except when it is requested to bargain collectively, an employer is a mere bystander to any
petition for certification election; such proceeding is non-adversarial and merely investigative, for
the purpose thereof is to determine which organization will represent the employees in their
collective bargaining with the employer. The choice of their representative is the exclusive
concern of the employees; the employer cannot have any partisan interest therein; it cannot
interfere with, much less oppose, the process by filing a motion to dismiss or an appeal from it;
not even a mere allegation that some employees participating in a petition for certification
election are actually managerial employees will lend an employer legal personality to block the
certification election. The employer's only right in the proceeding is to be notified or informed
thereof.
Further, the determination of whether union membership comprises managerial and/or
supervisory employees is a factual issue that is best left for resolution in the inclusion-exclusion
proceedings, which has not yet happened in this case so still premature to pass upon. We could
only emphasize the rule that factual findings of labor officials, who are deemed to have acquired
expertise in matters within their jurisdiction, are generally accorded not only with respect but
even finality by the courts when supported by substantial evidence.50 Also, the jurisdiction of
this Court in cases brought before it from the CA via Rule 45 is generally limited to reviewing
errors of law or jurisdiction. The findings of fact of the CA are conclusive and binding. Except in
certain recognized instances,51 We do not entertain factual issues as it is not Our function to
analyze or weigh evidence all over again; the evaluation of facts is best left to the lower courts
and administrative agencies/quasi-judicial bodies which are better equipped for the task.
In the same manner, the teaching and non-teaching personnel of [petitioner] school must form
separate bargaining units. Thus, the order for the conduct of two separate certification elections,
one involving teaching personnel and the other involving non-teaching personnel. It should be
stressed that in the subject petition, [private respondent] union sought the conduct of a
certification election among all the rank-and-file personnel of [petitioner] school. Since the
decision of the Supreme Court in the U.P. case prohibits us from commingling teaching and
non-teaching personnel in one bargaining unit, they have to be separated into two separate
bargaining units with two separate certification elections to determine whether the employees in
the respective bargaining units desired to be represented by [private respondent].

ATHENA M. SALAS | LABOR CASE DIGEST 2015 354

Indeed, the purpose of a certification election is precisely to ascertain the majority of the
employees choice of an appropriate bargaining unit to be or not to be represented by a labor
organization and, if in the affirmative case, by which one.

156. Visayas Community Medical Center vs. Yballe, et al., GR No. 196156, January 15,
2014

Facts:
Respondents were hired as staff nurses and midwives by (VCMC), formerly (MCCHI).
(NFL) is the exclusive bargaining representative of the rank-and-file employees of MCCHI.
Under the 1987 and 1991 (CBAs), the signatories were. Pongasi,.for MCCHI, and Atty.. Alforque
(NFL Legal Counsel) and Lumapguid as President of NFL-MCCH Chapter. In the CBA effective
from January 1994 until December 31, 1995, the signatories were Buot as Board of Trustees
Chairman, Iyoy as MCCH Administrator and Atty. Yu as Legal Counsel of NFL, while Nava,
President of NagkahiusangMamumuosa MCCH (NAMA-MCCH-NFL) signed the proof of
Posting.
Nava wrote Rev. Iyoy expressing the unions desire to renew the CBA, signed/endorsed
by 153 union members. Nava subsequently requested that the following employees be allowed
to avail of one-day union leave with pay However, MCCHI returned the CBA proposal for Nava
to secure first the endorsement of the legal counsel of NFL as the official bargaining
representative of MCCHI employees.

Meanwhile, Atty. Alforque informed MCCHI that the proposed CBA submitted by Nava
was never referred to NFL and that NFL has not authorized any other legal counsel or any
person for collective bargaining negotiations.

In his letter addressed to Nava, Atty. Alforque suspended their union membership for
serious violation of the Constitution and By-Laws.

The next day, several union members led by Nava and her group launched a series of
mass actions such as wearing black and red armbands/headbands, marching around the
hospital premises and putting up placards, posters and streamers. Atty. Alforque immediately
disowned the concerted activities being carried out by union members which are not sanctioned
by NFL.

DOLEissued certifications stating that there is nothing in their records which shows that
NAMA-MCCH- NFL is a registered labor organization, and that said union submitted only a copy
of its Charter Certificate on January 31, 1995.

ATHENA M. SALAS | LABOR CASE DIGEST 2015 355

Unfazed, the striking union members held more mass actions. The means of ingress to
and egress from the hospital were blocked so that vehicles carrying patients and employees
were barred from entering the premises.

With the volatile situation adversely affecting hospital operations and the condition of
confined patients, MCCHI filed a petition for injunction in the NLRC. Thereafter, several
complaints for illegal dismissal and unfair labor practice were filed by the terminated employees
against MCCHI,.Iyoy, UCCP and members of the Board of Trustees of MCCHI

Issue:
Whether or not the union employees were illegally dismissed.
Ruling:
Yes. Paragraph 3, Article 264(a) of the Labor Code provides that ". . .any union officer
who knowingly participates in an illegal strike and any worker or union officer who knowingly
participates in the commission of illegal acts during a strike may be declared to have lost his
employment status .
The dismissal of MCCH employees who participated in the illegal strike conducted by
NAMA-MCCH-NFL which is not a legitimate labor organization, was invalid. Since there was no
showing that the complainants committed any illegal act during the strike, they may not be
deemed to have lost their employment status by their mere participation in the illegal strike. On
the other hand, the union leaders (Nava group) who conducted the illegal strike despite
knowledge that NAMA-MCCH-NFL is not a duly registered labor union were declared to have
been validly terminated by petitioner.
We stress that the law makes a distinction between union members and union officers. A
worker merely participating in an illegal strike may not be terminated from employment. It is only
when he commits illegal acts during a strike that he may be declared to have lost employment
status. In contrast, a union officer may be terminated from employment for knowingly
participating in an illegal strike or participates in the commission of illegal acts during a strike.
The law grants the employer the option of declaring a union officer who participated in an illegal
strike as having lost his employment. It possesses the right and prerogative to terminate the
union officers from service.

While there was indeed no evidence of any illegal act committed by respondents during
the strike, the Labor Arbiter and NLRC were one in finding that respondents actively supported
the concerted protest activities, signed the collective reply of union members manifesting that
they launched the mass actions to protest managements refusal to negotiate a new CBA,
refused to appear in the investigations scheduled by petitioner because it was the unions stand
that they would only attend these investigations as a group, and failed to heed petitioners final
directive for them to desist from further taking part in the illegal strike.

Union members who were illegally dismissed for mere participation in an illegal strike are
entitled to separation pay (in lieu of reinstatement) but not to backwages.

ATHENA M. SALAS | LABOR CASE DIGEST 2015 356

The principle of "fair days wage for a fair days labor" remains as the basic factor in
determining the award thereof. If there is no work performed by the employee there can be no
wage or pay unless, of course, the laborer was able, willing and ready to work but was illegally
locked out, suspended or dismissed or otherwise illegally prevented from working. For this
exception to apply, it is required that the strike be legal.

157. Philtranco Service Enterprises Inc. vs. Philtranco Workers Union-Association of


Genuine Labor Organizations, GR No. 180962, February 26, 2014

Facts:
On the ground that it was suffering business losses, petitioner Philtranco Service Enterprises,
Inc., a local land transportation company engaged in the business of carrying passengers and
freight, retrenched 21 of its employees. Consequently, the company union, herein private
respondent Philtranco Workers Union-Association of Genuine Labor Organizations, filed a
Notice of Strike with the Department of Labor and Employment, claiming that petitioner engaged
in unfair labor practices.
Unable to settle their differences at the National Conciliation and Mediation Board, the case was
thereafter referred to the Office of the Secretary of the DOLE.
Acting DOLE Secretary issued a Decision6 dated June 13, 2007,
The PARTIES are enjoined to strictly and fully comply with the provisions of the existing CBA
and the other dispositions of this Decision. Petitioner received a copy of the above Decision on
June 14, 2007. It filed a Motion for Reconsideration on June 25, 2007. Private respondent, on
the other hand, submitted a "Partial Appeal."
In an August 15, 2007 Order8 which petitioner received on August 17, 2007, the Secretary of
Labor declined to rule on petitioners Motion for Reconsideration and private respondents
"Partial Appeal", citing a DOLE Regulation9which provided that voluntary arbitrators decisions,
orders, resolutions or awards shall not be the subject of motions for reconsideration. The
Secretary of Labor held:
WHEREFORE, the complainants and the respondents respective pleadings are hereby
NOTED as pleadings that need not be acted upon for lack of legal basis.
On August 29, 2007, petitioner filed before the CA an original Petition for Certiorari and
Prohibition, and sought injunctive relief. On September 20, 2007, the CA issued the assailed
Resolution which decreed as follows:
WHEREFORE, premises considered, the instant Petition for Certiorari and Prohibition with
Prayer for Temporary Restraining Order and Preliminary Injunction is hereby DISMISSED.

Issues:
THE HONORABLE COURT OF APPEALS ERRED IN RULING THAT THE PETITIONER
AVAILED OF THE ERRONEOUS REMEDY IN FILING A PETITION FOR CERTIORARI UNDER
RULE 65 INSTEAD OF UNDER RULE 43 OF THE RULES OF COURT.

ATHENA M. SALAS | LABOR CASE DIGEST 2015 357

THE HONORABLE COURT OF APPEALS ERRED WHEN IT HELD THAT THE PETITION FOR
CERTIORARI WAS FILED OUT OF TIME.

Ruling:
The Court grants the Petition.
It cannot be said that in taking cognizance of NCMB-NCR CASE No. NS-02-028-07, the
Secretary of Labor did so in a limited capacity, i.e., as a voluntary arbitrator. The fact is
undeniable that by referring the case to the Secretary of Labor, Conciliator-Mediator Aglibut
conceded that the case fell within the coverage of Article 263 of the Labor Code; the impending
strike in Philtranco, a public transportation company whose business is imbued with public
interest, required that the Secretary of Labor assume jurisdiction over the case, which he in fact
did. By assuming jurisdiction over the case, the provisions of Article 263 became applicable, any
representation to the contrary or that he is deciding the case in his capacity as a voluntary
arbitrator notwithstanding.
It has long been settled that the remedy of an aggrieved party in a decision or resolution of the
Secretary of Labor is to timely file a motion for reconsideration as a precondition for any further
or subsequent remedy, and then seasonably file a special civil action for certiorari under Rule 65
of the 1997 Rules on Civil Procedure. There is no distinction: when the Secretary of Labor
assumes jurisdiction over a labor case in an industry indispensable to national interest, "he
exercises great breadth of discretion" in finding a solution to the parties dispute. "The authority
of the Secretary of Labor to assume jurisdiction over a labor dispute causing or likely to cause a
strike or lockout in an industry indispensable to national interest includes and extends to all
questions and controversies arising therefrom. The power is plenary and discretionary in nature
to enable him to effectively and efficiently dispose of the primary dispute." This wide latitude of
discretion given to the Secretary of Labor may not be the subject of appeal.
On the question of whether the Petition for Certiorari was timely filed, the Court agrees with
petitioners submission. Rule 65 states that where a motion for reconsideration or new trial is
timely filed, whether such motion is required or not, the petition shall be filed not later than 60
days counted from the notice of the denial of the motion. This can only mean that even though a
motion for reconsideration is not required or even prohibited by the concerned government
office, and the petitioner files the motion just the same, the 60-day period shall nonetheless be
counted from notice of the denial of the motion. The very nature of certiorari which is an
extraordinary remedy resorted to only in the absence of plain, available, speedy and adequate
remedies in the course of law requires that the office issuing the decision or order be given the
opportunity to correct itself. Quite evidently, this opportunity for rectification does not arise if no
motion for reconsideration has been filed.
So also, considering that a decision of the Secretary of Labor is subject to judicial review only
through a special civil action of certiorari and, as a rule, cannot be resorted to without the
aggrieved party having exhausted administrative remedies through a motion for reconsideration,
the aggrieved party, must be allowed to move for a reconsideration of the same so that he can
bring a special civil action for certiorari before the Supreme Court.

158. Wesleyan University-Phils., vs. Wesleyan University-Phils., Faculty & Staff


Asso., GR No. 181806, March 12, 2014

ATHENA M. SALAS | LABOR CASE DIGEST 2015 358

Principle: A memorandum that imposes a limitation/condition on the employees


entitlement to vacation and sick leave credits, when no such limitation/condition is found
in the Collective Bargaining Agreement, is invalid. Like any other contract, a CBA has the
force of law between the parties and must be complied with in good faith. Unilateral
changes in the provisions of the CBA will therefore be struck down if the consent of both
parties is not obtained.
Facts:
Petitioner Wesleyan University-Philippines is a non-stock, non-profit educational institution duly
organized and existing under the laws of the Philippines. Respondent Wesleyan UniversityPhilippines Faculty and Staff Association, on the other hand, is a duly registered labor
organization7 acting as the sole and exclusive bargaining agent of all rank-and-file faculty and
staff employees of petitioner.
In December 2003, the parties signed a 5-year CBA 9 effective June 1, 2003 until May 31,
2008.On August 16, 2005, petitioner issued a Memorandum providing guidelines on the
implementation of vacation and sick leave credits as well as vacation leave commutation. In the
said memorandum, the vacation and sick leave credits are not automatic and should be earned
on a month-to-month basis. Respondent questioned the memorandum, saying that it was not
amenable to the unilateral changes and contending that said changes violated the existing CBA
between them. The existing CBA does not impose any limitation on the availability of the
vacation and sick leaves.
Issue:
WON the memorandum issued by petitioner is valid
Ruling:
No. It is contrary to the existing CBA between petitioner and respondent.
A Collective Bargaining Agreement (CBA) is a contract entered into by an employer and a
legitimate labor organization concerning the terms and conditions of employment. 1 Like any
other contract, it has the force of law between the parties and, thus, should be complied with in
good faith.2 Unilateral changes or suspensions in the implementation of the provisions of the
CBA, therefore, cannot be allowed without the consent of both parties.
Sections 1 and 2 of Article XII of the CBA provide that all covered employees are entitled to 15
days sick leave and 15 days vacation leave with pay every year and that after the second year
of service, all unused vacation leave shall be converted to cash and paid to the employee at the
end of each school year, not later than August 30 of each year.
The Memorandum dated August 16, 2005, however, states that vacation and sick leave credits
are not automatic as leave credits would be earned on a month-to-month basis. This, in effect,
limits the available leave credits of an employee at the start of the school year. For example, for
the first four months of the school year or from June to September, an employee is only entitled
to five days vacation leave and five days sick leave. Considering that the Memorandum dated
August 16, 2005 imposes a limitation not agreed upon by the parties nor stated in the CBA, we
agree with the CA that it must be struck down.
In closing, it may not be amiss to mention that when the provision of the CBA is clear, leaving no
doubt on the intention of the parties, the literal meaning of the stipulation shall govem.

ATHENA M. SALAS | LABOR CASE DIGEST 2015 359

However, if there is doubt in its interpretation, it should be resolved in favor of labor, as this is
mandated by no less than the Constitution.
159. Tabangao Shell Refinery Employees Association vs. Pilipinas Shell Petroleum Corp.,
GR No. 170007, April 7, 2014
Facts:
In anticipation of the expiration on April 30, 2004 of the 2001-2004 Collective Bargaining
Agreement (CBA) between the petitioner and the respondent Pilipinas Shell Petroleum
Corporation, the parties started negotiations for a new CBA.
After several meetings on the ground rules that would govern the negotiations and on political
items, the parties started their discussion on the economic items on July 27, 2004, their 31st
meeting. The union proposed a 20 percent annual across-the-board basic salary increase for
the next three years that would be covered by the new CBA.
In lieu of the annual salary increases, the company made a counter-proposal to grant all
covered employees a lump sum amount of P80,000.00 yearly for the three-year period of the
new CBA.
The union requested the company to present its counter-proposal in full detail, similar to the
presentation by the union of its economic proposal. The company explained that the lump sum
amount was based on its affordability for the corporation, the then current salary levels of the
members of the union relative to the industry, and the then current total pay and benefits
package of the employees. Not satisfied with the companys explanation, the union asked for
further justification of the lump sum amount offered by the company. When the company
refused to acknowledge any obligation to give further justification, the union rejected the
companys counter-proposal and maintained its proposal for a 20% annual increase in basic pay
for the next three years.
On the 39th meeting of the parties on August 24, 2004, the union lowered its proposal to 12%
annual across-the-board increase for the next three years. For its part, the company increased
its counter-proposal to a yearly lump sum payment of P88,000.00 for the next three years. The
union requested financial data for the manufacturing class of business in the Philippines. It also
requested justification for the companys counter-offer. In response, the company stated that
financial measures for Tabangao were available in the refinery scorecard regularly cascaded by
the management to the employees. The company reiterated that its counter-offer is based on its
affordability for the company, comparison with the then existing wage levels of allied industry,
and the then existing total pay and benefits package of the employees. The company
subsequently provided the union with a copy of the companys audited financial statements.
However, the union remained unconvinced and asked for additional documents to justify the
companys counter-offer. The company invited the attention of the union to the fact that
additional data, such as the refinery performance scorecard, were available from the refinerys
website and shared network drives. The company also declared that the bases of its counteroffer were already presented to the union and contained in the minutes of previous meetings.
The union thereafter requested for a copy of the comparison of the salaries of its members and
those from allied industries. The company denied the request on the ground that the requested
information was entrusted to the company under a confidential agreement. Alleging failure on

ATHENA M. SALAS | LABOR CASE DIGEST 2015 360

the part of the company to justify its offer, the union manifested that the company was
bargaining in bad faith. The company, in turn, expressed its disagreement with the unions
manifestation.
On the parties 41st meeting held on September 2, 2004, the company proposed the declaration
of a deadlock and recommended that the help of a third party be sought. The union replied that
they would formally answer the proposal of the company a day after the signing of the official
minutes of the meeting. On that same day, however, the union filed a Notice of Strike in the
National Conciliation and Mediation Board (NCMB), alleging bad faith bargaining on the part of
the company. The NCMB immediately summoned the parties for the mandatory conciliationmediation proceedings but the parties failed to reach an amicable settlement.
On September 16, 2004, during the cooling off period, the union conducted the necessary strike
vote. The members of the union, who participated in the voting, unanimously voted for the
holding of a strike. Upon being aware of this development, the company filed a Petition for
Assumption of Jurisdiction with the Secretary of Labor and Employment. The petition was filed
pursuant to the first paragraph of Article 263(g) of the Labor Code.
In an Order dated September 20, 2004, the then Secretary of Labor and Employment, Patricia
Sto. Tomas, granted the petition of the company. The Secretary of Labor and Employment took
notice of the Notice of Strike filed by the union in the NCMB which charged the company with
unfair labor practice consisting of bad faith in bargaining negotiations. The Secretary of Labor
and Employment also found that the intended strike would likely affect the companys capacity
to provide petroleum products to the companys various clientele, including the transportation
sector, the energy sector, and the manufacturing and industrial sectors. The Secretary of Labor
and Employment further observed that a strike by the union would certainly have a negative
impact on the price of commodities. Convinced that such a strike would have adverse
consequences on the national economy, the Secretary of Labor and Employment ruled that the
labor dispute between the parties would cause or likely to cause a strike in an industry
indispensable to the national interest. Thus, the Secretary of Labor and Employment assumed
jurisdiction over the dispute of the parties.
The union now comes to this Court to press its contentions. It insists that the corporation is
guilty of unfair labor practice through bad faith bargaining. According to the union, bad faith
bargaining and a CBA deadlock cannot legally co-exist because an impasse in negotiations can
only exist on the premise that both parties are bargaining in good faith. Besides, there could
have been no deadlock between the parties as the union had not given its consent to it,
pursuant to item 8 of the ground rules governing the parties negotiations which required mutual
consent for a declaration of deadlock. The union also posits that its filing of a CBA deadlock
case against the company was a separate and distinct case and not an offshoot of the
companys unfair labor practice through bargaining in bad faith. According to the union, as there
was no deadlock yet when the union filed the unfair labor practice of bargaining in bad faith, the
subsequent deadlock case could neither be an offshoot of, nor an incidental issue in, the unfair
labor practice case. Because there was no deadlock yet at the time of the filing of the unfair
labor practice case, the union claims that deadlock was not an incidental issue but a non-issue.
Issue: WON the Court of Appeals misapplied St. Scholasticas College and the Secretary of
Labor and Employment committed grave abuse of discretion when it presumed deadlock in its
Order dated September 20, 2004 assuming jurisdiction over the labor dispute between the
union and the company

ATHENA M. SALAS | LABOR CASE DIGEST 2015 361

Ruling:
No, the unions petition must fail. The CA and the SOLE are correct. The assumption of
jurisdiction was in order.
As discussed above, there was already an actual existing deadlock between the parties. What
was lacking was the formal recognition of the existence of such a deadlock because the union
refused a declaration of deadlock. Thus, the unions view that, at the time the Secretary of Labor
and Employment exercised her power of assumption of jurisdiction, the issue of deadlock was
neither an incidental issue to the matter of unfair labor practice nor an existing issue is incorrect.
More importantly, however, the unions mistaken theory that the deadlock issue was neither
incidental nor existing is based on its premise that the case is all about the companys alleged
unfair labor practice of bargaining in bad faith, which is the ground stated in its first Notice of
Strike
While the first Notice of Strike is indeed significant in the determination of the existing labor
dispute between the parties, it is not the sole criterion. As this Court explained in Union of Filipro
Employees-Drug, Food and Allied Industries Unions-Kilusang Mayo Uno v. Nestle Philippines,
Inc.:
The Secretary of the DOLE has been explicitly granted by Article 263(g) of the
Labor Code the authority to assume jurisdiction over a labor dispute causing or
likely to cause a strike or lockout in an industry indispensable to the national
interest, and decide the same accordingly. And, as a matter of necessity, it
includes questions incidental to the labor dispute; that is, issues that are
necessarily involved in the dispute itself, and not just to that ascribed in
the Notice of Strike or otherwise submitted to him for resolution. x x x
(Emphasis supplied.)
The totality of the companys Petition for Assumption of Jurisdiction, including every allegation
therein, also guided the Secretary of Labor and Employment in the proper determination of the
labor dispute over which he or she was being asked to assume jurisdiction.
The labor dispute between the union and the company concerned the unresolved matters
between the parties in relation to their negotiations for a new CBA. The power of the Secretary
of Labor and Employment to assume jurisdiction over this dispute includes and extends to all
questions and controversies arising from the said dispute, such as, but not limited to the unions
allegation of bad faith bargaining. It also includes and extends to the various unresolved
provisions of the new CBA such as compensation, particularly the matter of annual wage
increase or yearly lump sum payment in lieu of such wage increase, whether or not there was
deadlock in the negotiations. Indeed, nowhere does the Order dated September 20, 2004 of the
Secretary of Labor and Employment mention a CBA deadlock. What the union viewed as
constituting the inclusion of a CBA deadlock in the assumption of jurisdiction was the inclusion
of the economic issues, particularly the companys stance of yearly lump sum payment in lieu of
annual wage increase, in the directive for the parties to submit their respective position papers.
The unions Motion for Reconsideration (With Urgent Prayer to Compel the Company to Justify
Offer of Wage [Increase] Moratorium) and Second Motion for Reconsideration questioning the
Order dated September 20, 2004 of the Secretary of Labor and Employment actually confirm
that the labor dispute between the parties essentially and necessarily includes the conflicting

ATHENA M. SALAS | LABOR CASE DIGEST 2015 362

positions of the union, which advocates annual wage increase, and of the company, which offers
yearly lump sum payment in lieu of wage increase. In fact, that is the reason behind the unions
prayer that the company be ordered to justify its offer of wage increase moratorium. As there is
already an existing controversy on the matter of wage increase, the Secretary of Labor and
Employment need not wait for a deadlock in the negotiations to take cognizance of the matter.
That is the significance of the power of the Secretary of Labor and Employment under Article
263(g) of the Labor Code to assume jurisdiction over a labor dispute causing or likely to cause a
strike or lockout in an industry indispensable to the national interest. As this Court elucidated in
Bagong Pagkakaisa ng Manggagawa ng Triumph International v. Secretary of the Department
of Labor and Employment:
Article 263(g) is both an extraordinary and a preemptive power to address an
extraordinary situation - a strike or lockout in an industry indispensable to the
national interest. This grant is not limited to the grounds cited in the notice of
strike or lockout that may have preceded the strike or lockout; nor is it limited to
the incidents of the strike or lockout that in the meanwhile may have taken
place. As the term "assume jurisdiction" connotes, the intent of the law is to
give the Labor Secretary full authority to resolve all matters within the dispute
that gave rise to or which arose out of the strike or lockout; it includes and
extends to all questions and controversies arising from or related to the
dispute, including cases over which the labor arbiter has exclusive jurisdiction.

ATHENA M. SALAS | LABOR CASE DIGEST 2015 363

23. REVISED GUIDELINES OF THE NCMB FOR THE CONDUCT OF VOLUNTARY


ARBITRATION PROCEEDINGS

160. (162.) 7K Corp. vs. Albarico, G.R. No. 182295, June 26, 2013
Facts:
Respondent Eddie Albarico (Albarico) was a regular employee of petitioner 7K Corporation, a
company selling water purifiers. He started working for the company in 1990 as a salesman. 4
Because of his good performance, his employment was regularized. He was also promoted
several times: from salesman, he was promoted to senior sales representative and then to
acting team field supervisor.

In 1992, he was awarded the Presidents Trophy for being one of the companys top water
purifier specialist distributors. However, in April of 1993, the chief operating officer of petitioner
7K Corporation terminated Albaricos employment allegedly for his poor sales performance. 5
Respondent had to stop reporting for work, and he subsequently submitted his money claims
against petitioner for arbitration before the National Conciliation and Mediation Board (NCMB).
The issue for voluntary arbitration before the NCMB, according to the parties Submission
Agreement dated 19 April 1993, was whether respondent Albarico was entitled to the payment
of separation pay and the sales commission reserved for him by the corporation.6

Issue:

WON the voluntary arbitrator properly assumed jurisdiction to decide the issue of the legality of
the dismissal of respondent as well as the latters entitlement to backwages, even if neither the
legality nor the entitlement was expressly claimed in the Submission Agreement of the parties.

Ruling:
NO. Petitioner overlooks the proviso in the said article, thus:

Art. 217. Jurisdiction of the Labor Arbiters and the Commission.

ATHENA M. SALAS | LABOR CASE DIGEST 2015 364

a. Except as otherwise provided under this Code, the Labor Arbiters shall have original and
exclusive jurisdiction to hear and decide, within thirty (30) calendar days after the submission of
the case by the parties for decision without extension, even in the absence of stenographic
notes, the following cases involving all workers, whether agricultural or nonagricultural:cralavvonlinelawlibrary

xxxx

2. Termination disputes;

xxxx

6. Except claims for Employees Compensation, Social Security, Medicare and maternity
benefits, all other claims arising from employer-employee relations, including those of
persons in domestic or household service, involving an amount exceeding five thousand pesos
(P5,000.00) regardless of whether accompanied with a claim for reinstatement.
(Emphases supplied)

Thus, although the general rule under the Labor Code gives the labor arbiter exclusive and
original jurisdiction over termination disputes, it also recognizes exceptions. One of the
exceptions is provided in Article 262 of the Labor Code. In San Jose v. NLRC, we said:
The phrase Except as otherwise provided under this Code refers to the following exceptions:

A. Art. 217. Jurisdiction of Labor Arbiters . . .

xxxx

B. Art. 262. Jurisdiction over other labor disputes. The Voluntary Arbitrator or panel of
Voluntary Arbitrators, upon agreement of the parties, shall also hear and decide all other
labor disputes including unfair labor practices and bargaining deadlocks.

From the above discussion, it is clear that voluntary arbitrators may, by agreement of the
parties, assume jurisdiction over a termination dispute such as the present case, contrary to the
assertion of petitioner that they may not.

Petitioner argues that, assuming that the voluntary arbitrator has jurisdiction over the present
termination dispute, the latter should have limited his decision to the issue contained in the

ATHENA M. SALAS | LABOR CASE DIGEST 2015 365

Submission Agreement of the parties the issue of whether respondent Albarico was entitled
to separation pay and to the sales commission the latter earned before being terminated.Thus, it
contends that the voluntary arbitrator exceeded his jurisdiction.

The Court notes that even the NLRC was of the understanding that the NCMB arbitration case
sought to resolve the issue of the legality of the dismissal of the respondent. In fact, the identity
of the issue of the legality of his dismissal, which was previously submitted to the NCMB, and
later submitted to the NLRC, was the basis of the latters finding of forum shopping and the
consequent dismissal of the case before it. In fact, petitioner also implicitly acknowledged this
when it filed before the NLRC its Motion to Dismiss respondents Complaint on the ground of
forum shopping. Thus, it is now estopped from claiming that the issue before the NCMB
does not include the issue of the legality of the dismissal of respondent. Besides, there
has to be a reason for deciding the issue of respondents entitlement to separation pay. To think
otherwise would lead to absurdity, because the voluntary arbitrator would then be deciding that
issue in a vacuum. The arbitrator would have no basis whatsoever for saying that Albarico was
entitled to separation pay or not if the issue of the legality of respondents dismissal was not
resolve first.

Hence, the voluntary arbitrator correctly assumed that the core issue behind the issue of
separation pay is the legality of the dismissal of respondent. Moreover, we have ruled in
Sime Darby Pilipinas, Inc. v. Deputy Administrator Magsalin that a voluntary arbitrator has
plenary jurisdiction and authority to interpret an agreement to arbitrate and to determine the
scope of his own authority when the said agreement is vague subject only, in a proper case,
to the certiorari jurisdiction of this Court.

Having established that the issue of the legality of dismissal of Albarico was in fact
necessarily albeit not explicitly included in the Submission Agreement signed by the
parties, this Court rules that the voluntary arbitrator rightly assumed jurisdiction to
decide the said issue.

Consequently, we also rule that the voluntary arbitrator may award backwages upon a finding of
illegal dismissal, even though the issue of entitlement thereto is not explicitly claimed in the
Submission Agreement. Backwages, in general, are awarded on the ground of equity as a form
of relief that restores the income lost by the terminated employee by reason of his illegal
dismissal.34

In Sime Darby we ruled that although the specific issue presented by the parties to the voluntary
arbitrator was only the issue of performance bonus, the latter had the authority to determine
not only the issue of whether or not a performance bonus was to be granted, but also the
related question of the amount of the bonus, were it to be granted. We explained that there was
no indication at all that the parties to the arbitration agreement had regarded the issue of
performance bonus as a two-tiered issue, of which only one aspect was being submitted to
arbitration. Thus, we held that the failure of the parties to limit the issues specifically to that
which was stated allowed the arbitrator to assume jurisdiction over the related issue.

ATHENA M. SALAS | LABOR CASE DIGEST 2015 366

Similarly, in the present case, there is no indication that the issue of illegal dismissal should
be treated as a two-tiered issue whereupon entitlement to backwages must be
determined separately. Besides, since arbitration is a final resort for the adjudication of
disputes, the voluntary arbitrator in the present case can assume that he has the
necessary power to make a final settlement. Thus, we rule that the voluntary arbitrator
correctly assumed jurisdiction over the issue of entitlement of respondent Albarico to
backwages on the basis of the formers finding of illegal dismissal.

161. Lepanto Consolidated Mining Company vs. The Lepanto Capataz Union, G.R. No.
157086, Feb. 18, 2013
FACTS:
Lepanto is a domestic corporation authorized to engage in large-scale mining. Respondent
Lepanto Capataz Union (Union), a labor organization duly registered with DOLE, filed a petition
for consent election, thereby proposing to represent 139 capatazes of Lepanto.
In due course, Lepanto opposed the petition, contending that the Union was in reality seeking a
certification election, not a consent election, and would be thereby competing with the Lepanto
Employees Union (LEU), the current collective bargaining agent. Lepanto pointed out that
the capatazes were already members of LEU, the exclusive representative of all rank-and-file
employees of its Mine Division.
Med-Arbiter Michaela A. Lontoc of DOLE-CAR issued a ruling to the effect that
the capatazescould form a separate bargaining unit due to their not being rank-and-file
employees.Lepanto appealed to the DOLE Secretary. The DOLE Undersecretary Rosalinda
Dimapilis-Baldoz, acting by authority of the DOLE Secretary, affirmed the ruling of Med-Arbiter
Lontoc.

In the ensuing certification election, the Union garnered 109 of the 111 total valid votes cast.On
the day of the certification election, however, Lepanto presented an opposition/protest. Hence, a
hearing was held on Lepantos opposition/protest. Although the parties were required in that
hearing to submit their respective position papers, Lepanto later opted not to submit its position
paper, and contended that the issues identified during the hearing did not pose any legal issue
to be addressed in a position paper.
Med-Arbiter Florence Marie A. Gacad-Ulep of DOLE-CAR rendered a decision certifying the
Union as the sole and exclusive bargaining agent of all capatazes of Lepanto.Lepanto appealed
the decision of Med-Arbiter Gacad-Ulep to the DOLE Secretary.By her Resolution, DOLE
Secretary Patricia A. Sto. Tomas affirmed the decision.

Appellant accused Med-Arbiter Ulep of grave abuse of discretion amounting to lack of


jurisdiction based on her failure to resolve appellants motion to modify order to submit position
papers and on rendering judgment on the basis only of appellees position paper.

ATHENA M. SALAS | LABOR CASE DIGEST 2015 367

ISSUES: Whether or not Med-Arbiter Ulep is guilty of grave abuse of discretion amounting to
lack of jurisdiction; and, whether the capatazes could form their own union independently of the
rank-and-file employees.
RULING:
I. Med-Arbiter Ulep is NOT guilty of grave abuse of discretion amounting to lack of
jurisdiction.
Section 5, Rule XXV of Department Order No. 9, otherwise known as the New Rules
Implementing Book V of the Labor Code, states that "in all proceedings at all levels, incidental
motions shall not be given due course, but shall remain as part of the records for whatever they
may be worth when the case is decided on the merits".

Section 9, Rule XI of Department Order No. 9, which is applied with equal force in the
disposition of protests on the conduct of election, states that "the Med-Arbiter shall in the same
hearing direct all concerned parties, including the employer, to simultaneously submit their
respective position papers within a non-extendible period of ten days". The parties, including
appellant company were required to submit their respective positions on whether there was
proper challenge of the voters, whether LEU failed to participate in the proceedings, if so,
whether it should be allowed to participate at this belated stage and whether the arguments
raised during the pre-election conferences and in the protests are valid. The parties, including
appellant company were apprised of these issues and they agreed thereto. The minutes of the
hearing even contained the statement that "no order will issue" and that "the parties are
informed accordingly". If there is any matter that had to be clarified, appellant should have
clarified the same during the said hearing and refused to file its position paper simultaneously
with LCU and LEU. It appears that appellant did not do so and acquiesced to the filing of its
position paper within fifteen days from the date of said hearing.

Neither is there merit in appellants contention that the Med- Arbiter resolved the protest based
solely on appellee LCUs position paper. Not only did the Med-Arbiter discuss the demerits of
appellants motion to modify order to submit position papers but likewise the demerits of its
protest. We do not, however, agree with the Med-Arbiter that the protest should be dismissed
due to appellants failure to challenge the individual voters during the election.

II.Capatazes are not rank-and-file employees; hence, they could form their own union.

Thecapatazes were performing functions totally different from those performed by the rank-andfile employees, and that the capatazes were "supervising and instructing the miners, mackers
and other rank-and-file workers under them, assess[ing] and evaluat[ing] their performance,
mak[ing] regular reports and recommend[ing] new systems and procedure of work, as well as
guidelines for the discipline of employees." Hence, Med-Arbiter Lontoc concluded,
the capatazes "differ[ed] from the rank-and-file and [could] by themselves constitute a separate
bargaining unit."

ATHENA M. SALAS | LABOR CASE DIGEST 2015 368

The bargaining unit sought to be represented by the appellee are the capataz employees of the
appellant. There is no other labor organization of capatazes within the employer unit except
herein appellant. Thus, appellant is an unorganized establishment in so far as the bargaining
unit of capatazes is concerned. In accordance with the last paragraph of Section 11, Rule XI,
Department Order No. 9 which provides that "in a petition filed by a legitimate labor organization
involving an unorganized establishment, the Med-Arbiter shall, pursuant to Article 257 of the
Code, automatically order the conduct of certification election after determining that the petition
has complied with all requirements under Section 1, 2 and 4 of the same rules and that none of
the grounds for dismissal thereof exists", the order for the conduct of a certification election is
proper.

We cannot undo the affirmance by the DOLE Secretary of the correct findings of her
subordinates in the DOLE, an office that was undeniably possessed of the requisite expertise on
the matter in issue. In any event, we affirm that capatazes or foremen are not rank-andfile
employees because they are an extension of the management, and as such they may influence
the rank-and-file workers under them to engage in slowdowns or similar activities detrimental to
the policies, interests or business objectives of the employers.

24. UNFAIR LABOR PRACTICE

ATHENA M. SALAS | LABOR CASE DIGEST 2015 369

163. Goya Inc. vs. Goya Inc. Employees Union-FFW, G.R. No. 170054, January 21, 2013
Facts:
Sometime in January 2004, petitioner Goya, Inc. (Company), a domestic corporation engaged in
the manufacture, importation, and wholesale of top quality food products, hired contractual
employees from PESO Resources Development Corporation (PESO) to perform temporary and
occasional services in its factory in Parang, Marikina City. This prompted respondent Goya, Inc.
Employees Union-FFW (Union) to request for a grievance conference on the ground that the
contractual workers do not belong to the categories of employees stipulated in the existing
Collective Bargaining Agreement (CBA). When the matter remained unresolved, the grievance
was referred to the National Conciliation and Mediation Board (NCMB) for voluntary arbitration.
The Union asserted that the hiring of contractual employees from PESO is not a management
prerogative and in gross violation of the CBA tantamount to unfair labor practice (ULP). The
CBA merely provides for three (3) categories of employees in the Company, namely:
probationary employees, regular employees and casual employees.
In countering the Union's allegations, the Company argued that: (a) the law expressly allows
contracting and subcontracting arrangements through Department of Labor and Employment
(DOLE) Order No. 18-02; (b) the engagement of contractual employees did not, in any way,
prejudice the Union, since not a single employee was terminated and neither did it result in a
reduction of working hours nor a reduction or splitting of the bargaining unit; and (c) Section 4,
Article I of the CBA merely provides for the definition of the categories of employees and does
not put a limitation on the Company's right to engage the services of job contractors or its
management prerogative to address temporary/occasional needs in its operation.

Issue:
Whether or not the Company is guilty of unfair labor practice in engaging the services of PESO,
a third party service provider, under existing CBA, laws, and jurisprudence.

Ruling:
A careful reading of the above-enumerated categories of employees reveals that the PESO
contractual employees do not fall within the enumerated categories of employees stated in the
CBA of the parties. Following the said categories, [the Company] should have observed and
complied with the provision of their CBA. Since the Company had admitted that it engaged the
services of PESO to perform temporary or occasional services which is akin to those performed
by casual employees, the Company should have tapped the services of casual employees
instead of engaging PESO.
In justifying its act, the Company posits that its engagement of PESO was a management
prerogative. It bears stressing that a management prerogative refers to the right of the employer
to regulate all aspects of employment, such as the freedom to prescribe work assignments,
working methods, processes to be followed, regulation regarding transfer of employees,
supervision of their work, lay-off and discipline, and dismissal and recall of work, presupposing
the existence of employer-employee relationship. On the basis of the foregoing definition, the
Company's engagement of PESO was indeed a management prerogative. This is in
consonance with the pronouncement of the Supreme Court in the case of Manila Electric

ATHENA M. SALAS | LABOR CASE DIGEST 2015 370

Company vs. Quisumbing where it ruled that contracting out of services is an exercise of
business judgment or management prerogative.
This management prerogative of contracting out services, however, is not without
limitation. In contracting out services, the management must be motivated by good faith and
the contracting out should not be resorted to circumvent the law or must not have been the
result of malicious arbitrary actions. In the case at bench, the CBA of the parties has already
provided for the categories of the employees in the Company's establishment. These categories
of employees particularly with respect to casual employees serve as limitation to [the
Company's] prerogative to outsource parts of its operations especially when hiring contractual
employees. As stated earlier, the work to be performed by PESO was similar to that of the
casual employees. With the provision on casual employees, the hiring of PESO contractual
employees, therefore, is not in keeping with the spirit and intent of their CBA.

164. Park Hotel et al., vs. Soriano et al., G.R. No. 171118, September 10, 2012

Principle: The Labor Code considers it an unfair labor practice when an employer interferes,
restrains or coerces employees in the exercise of their right to self-organization or the right to
form an association.

Facts:
Harbutt (Harbutt) and Bill Percy (Percy) are the General Manager and owner,
respectively, of Park Hotel. Percy, Harbutt and Atty. Roberto Enriquez are also the officers and
stockholders of Burgos Corporation (Burgos), a sister company of Park Hotel.

Respondent Manolo Soriano (Soriano) was hired by Park Hotel in July 1990 as
Maintenance Electrician, and then transferred to Burgos in 1992. Respondent Lester Gonzales
(Gonzales) was employed by Burgos as Doorman, and later promoted as Supervisor.
Respondent Yolanda Badilla (Badilla) was a bartender of J's Playhouse operated by Burgos.

In October of 1997, Soriano, Gonzales and Badilla were dismissed from work for
allegedly stealing company properties. As a result, respondents filed complaints for illegal
dismissal, unfair labor practice, and payment of moral and exemplary damages and attorney's
fees, before the Labor Arbiter (LA). In their complaints, respondents alleged that the real reason
for their dismissal was that they were organizing a union for the company's employees.

In his Affidavit, respondents claimed that on October 4, 1997, he was barred from
entering the company premises and that the following day, Harbutt shouted at him for having
participated in the formation of a union.

Issue: Whether or not petitioners committed unfair labor practice.

ATHENA M. SALAS | LABOR CASE DIGEST 2015 371

Ruling: Yes
Anent the unfair labor practice, Article 248 (a) of the Labor Code 27 considers it an unfair
labor practice when an employer interferes, restrains or coerces employees in the exercise of
their right to self-organization or the right to form an association. 28 In order to show that the
employer committed unfair labor practice under the Labor Code, substantial evidence is
required to support the claim. In the case at bar, respondents were indeed unceremoniously
dismissed from work by reason of their intent to form and organize a union. As found by the LA:
The immediate impulse of petitioners, as in the case at bar, was to terminate the
organizers. Petitioners have to cripple the union at sight, to frustrate attempts of employees
from joining or supporting it, preventing them, at all cost and to frustrate the employees bid to
exercise their right to self-organization.

Thus, respondents were illegally dismissed.

[***Although relevant, the rulings below are no longer part of the issue emphasized by JMM.]

Requisites for a valid dismissal

The requisites for a valid dismissal are: (a) the employee must be afforded due
process, i.e., he must be given an opportunity to be heard and defend himself; and (b) the
dismissal must be for a valid cause as provided in Article 282 of the Labor Code, or for any of
the authorized causes under Articles 283 and 284 of the same Code. 25 In the case before us,
both elements are completely lacking. Respondents were dismissed without any just or
authorized cause and without being given the opportunity to be heard and defend themselves.
The law mandates that the burden of proving the validity of the termination of employment rests
with the employer. Failure to discharge this evidentiary burden would necessarily mean that the
dismissal was not justified and, therefore, illegal

The doctrine of piercing the veil of corporate fiction in a Labor case

As to whether Park Hotel may be held solidarily liable with Burgos, the Court rules that
before a corporation can be held accountable for the corporate liabilities of another, the veil of
corporate fiction must first be pierced. Thus, before Park Hotel can be held answerable for the
obligations of Burgos to its employees, it must be sufficiently established that the two
companies are actually a single corporate entity, such that the liability of one is the liability of the
other.

In the case at bar, respondents utterly failed to prove by competent evidence that Park
Hotel was a mere instrumentality, agency, conduit or adjunct of Burgos, or that its separate

ATHENA M. SALAS | LABOR CASE DIGEST 2015 372

corporate veil had been used to cover any fraud or illegality committed by Burgos against the
respondents.

Verily, a corporation, being a juridical entity, may act only through its directors, officers
and employees. Obligations incurred by them, while acting as corporate agents, are not their
personal liability but the direct accountability of the corporation they represent. 38 However,
corporate officers may be deemed solidarily liable with the corporation for the termination of
employees if they acted with malice or bad faith. In the present case, the lower tribunals
unanimously found that Percy and Harbutt, in their capacity as corporate officers of Burgos,
acted maliciously in terminating the services of respondents without any valid ground and in
order to suppress their right to self-organization.

165. Baptista et al., vs. Villanueva et al., G,.R. No. 194709, July 31, 2013
FACTS:
Petitioners were former union members of Radio Philippines Network Employees Union
(RPNEU), a legitimate labor organization and the sole and exclusive bargaining agent of the
rank and file employees of Radio Philippines Network, while the respondents were the union's
elected
officers
and
members.
On suspicion of union mismanagement, petitioners filed a complaint for impeachment against all
the union officers and members of the union before the DOLE. Thereafter, written complaints
were filed against petitioners for the alleged violation of the union's Constitution and By-Laws,
specifically for urging that a member start an action in any court of justice or external
investigative body against the Union or its officer without first exhausting all internal remedies
open to him or available in accordance with the CBL. Petitioners denied the charges imputed
against
them.
After the investigation, petitioners were infromed of their expulsion from the union and their
termination from employment in compliance with their CBA's union security clause. Aggrieved,
petitioners filed complaints for ULP against the respondents pursuant to Article 249 (a) and (b)
of the Labor Code. The LA adjudged the respondents guilty of ULP. The NLRC set aside the LA
decision and dismissed the complaint for ULP for lack of merit. The CA sustained the NLRC
decision.
ISSUE:
Whether or not the respondents committed ULP under Article 249 (a) and (b) of the Labor Code.
RULING: NO. There is no ULP.
The primary concept of ULP is embodied in Article 247 of the Labor Code. In essence, ULP

ATHENA M. SALAS | LABOR CASE DIGEST 2015 373

relates to the commission of acts that transgress the workers' right to organize. As specified in
Articles 248 and 249 of the Labor Code, the prohibited acts must necessarily relate to the
workers' right to self-organization and to the observance of a CBA. Absent the said vital
elements, the acts complained, although seemingly unjust, would not constitute ULP.
In this case, petitioners claim that the respondents, as union officers, are guilty of ULP for
violating paragraphs (a) and (b) of Article 249 of the Labor Code, to wit:
ART. 249.UNFAIR LABOR PRACTICES OF LABOR ORGANIZATIONS. It shall be unfair
labor practice for a labor organization, its officers, agents or representatives:
(a)To restrain or coerce employees in the exercise of their rights to self-organization. However, a
labor organization shall have the right to prescribe its own rules with respect to the acquisition or
retention
of
membership;
(b)To cause or attempt to cause an employer to discriminate against an employee, including
discrimination against an employee with respect to whom membership in such organization has
been denied or to terminate an employee on any ground other than the usual terms and
conditions under which membership or continuation of membership is made available to other
members.
It is well-settled that workers' and employers' organizations shall have the right to draw up their
constitutions and rules. In this case, RPNEU's Constitution and By-Laws expressly mandate that
before a party is allowed to seek the intervention of the court, it is a pre-condition that he should
have availed of all the internal remedies within the organization. Petitioners were found to have
violated the provisions of the union's Constitution and By-Laws when they filed petitions for
impeachment against their union officers and for audit before the DOLE without first exhausting
all internal remedies available within their organization. This act is a ground for expulsion from
union membership. Thus, petitioners' expulsion from the union was not a deliberate attempt to
curtail or restrict their right to organize, but was triggered by the commission of an act, expressly
sanctioned
by
the
union's
Constitution
and
By-Laws.
For a charge of ULP against a labor organization to prosper, the onus probandi rests upon the
party alleging it. In labor cases, substantial evidence is required. Moreover, all the prohibited
acts constituting unfair labor practice should materially relate to the workers' right to selforganization.
Unfortunately, petitioners failed to discharge the burden required to prove the charge of ULP
against the respondents. Aside from their self-serving allegations, petitioners were not able to
establish how they were restrained or coerced by their union in a way that curtailed their right to
self-organization. The records likewise failed to sufficiently show that the respondents unduly
persuaded management into discriminating against petitioners other than to bring to its attention
their expulsion from the union, which in turn, resulted in the implementation of their CBA's union
security
clause.

166. TH Shopfitters Corp., et al., vs. T&H Shopfitters Corp., Union, GR No. 191714, Feb 26,
2014
FACTS:

ATHENA M. SALAS | LABOR CASE DIGEST 2015 374

On September 7, 2004, Respondents filed their Complaint for Unfair Labor Practice (ULP) by
way of union busting, and Illegal Lockout, with moral and exemplary damages and attorney's
fees, against Petitioners, T&H Shopfitters Corporation (T&H Shopfitters) and Gin Queen
Corporation (Gin Queen).
Respondents treated T&H Shopfitters and Gin Queen as a single entity and their sole employer.
In their desire to improve their working conditions, respondents and other employees of
petitioners held their first formal meeting on November 23, 2003 to discuss the formation of a
union. The following day or on November 24, 2003, seventeen (17) employees were barred
from entering petitioners' factory premises and ordered to transfer to T&H Shopfitters'
warehouse purportedly because of its expansion. Afterwards, the said employees were
repeatedly ordered to go on forced leave due to the unavailability of work.
Respondents contended that the affected employees were not given regular work assignments,
while subcontractors were continuously hired to perform their functions. This development
prompted respondents to seek the assistance of the National Conciliation and Mediation Board.
Subsequently, an agreement between petitioners and THS-GQ Union was reached. Petitioners
agreed to give priority to regular employees in the distribution of work assignments.
Respondents averred, however, that petitioners never complied with its commitment but instead
hired contractual workers.
Petitioners stress that T&H Shopfitters and Gin Queen are corporations separate and distinct
from each other. Consequently, T&H Shopfitters and Stinnes Huang, an officer of T&H
Shopfitters, cannot be held liable for ULP for the reason that there is no employer-employee
relationship between the former and respondents. Further, Gin Queen avers that its decision to
implement an enforced rotation of work assignments for respondents was a management
prerogative permitted by law, justified by the decrease in the orders it received from its
customers. It explains that its failure to present concrete proof of its decreasing orders was due
to the impossibility of proving a negative assertion. It also asserts that the transfer from
Castillejos to Cabangan was made in good faith and solely because of the expiration of its lease
contract in Castillejos.
In its Decision, dated December 21, 2005, the LA dismissed respondents' complaint and all their
money claims for lack of merit.Aggrieved, respondents appealed to the NLRC. In its July 24,
2007 Decision, the NLRC reversed the LA decision and ruled in favor of respondents.
Petitioners filed a motion for reconsideration but the NLRC denied the same. Dissatisfied with
the adverse ruling, petitioners instituted a petition for certiorari under Rule 65 of the Rules of
Court before the CA arguing grave abuse of discretion on the part of the NLRC in reversing the
LA decision.
In its Decision, dated November 12, 2009, the CA sustained the NLRC ruling.
ISSUE:
Whether or not Unfair Labor Practice acts were committed by petitioners against respondents.
RULING:
As to the issue of ULP, petitioners' argument is utterly without merit.

ATHENA M. SALAS | LABOR CASE DIGEST 2015 375

In the case at bench, petitioners are being accused of violations of paragraphs (a), (c), and (e)
of Article 257 (formerly Article 248) of the Labor Code, to wit:
Article 257. Unfair labor practices of employers. It shall be unlawful for an employer
to commit any of the following unfair labor practices:
(a) To interfere with, restrain or coerce employees in the exercise of their right to
self-organization;
xxxxxxxxx
(c) To contract out services or functions being performed by union members
when such will interfere with, restrain, or coerce employees in the exercise of
their right to self-organization;
xxxxxxxxx
(e) To discriminate in regard to wages, hours of work, and other terms and
conditions of employment in order to encourage or discourage membership in
any labor organization. . . .
In essence, ULP relates to the commission of acts that transgress the workers' right to
organize. As specified in Articles 248 [now Article 257] and 249 [now Article 258] of the Labor
Code, the prohibited acts must necessarily relate to the workers' right to self-organization.
The test of whether an employer has interfered with and coerced employees in the
exercise of their right to self-organization, that is, whether the employer has engaged in conduct
which, it may reasonably be said, tends to interfere with the free exercise of employees' rights;
and that it is not necessary that there be direct evidence that any employee was in fact
intimidated or coerced by statements of threats of the employer if there is a reasonable
inference that anti-union conduct of the employer does have an adverse effect on selforganization and collective bargaining.
The questioned acts of petitioners, namely: 1) sponsoring a field trip to Zambales for its
employees, to the exclusion of union members, before the scheduled certification election; 2)
the active campaign by the sales officer of petitioners against the union prevailing as a
bargaining agent during the field trip; 3) escorting its employees after the field trip to the polling
center; 4) the continuous hiring of subcontractors performing respondents' functions; 5)
assigning union members to the Cabangan site to work as grass cutters; and 6) the
enforcement of work on a rotational basis for union members, all reek of interference on the part
of petitioners.
Indubitably, the various acts of petitioners, taken together, reasonably support an
inference that, indeed, such were all orchestrated to restrict respondents' free exercise of their
right to self-organization.
The Court is of the considered view that petitioners' undisputed actions prior and
immediately before the scheduled certification election, while seemingly innocuous, unduly
meddled in the affairs of its employees in selecting their exclusive bargaining representative.
Petitioners had no business persuading and/or assisting its employees in their legally protected
independent process of selecting their exclusive bargaining representative. The fact and
peculiar timing of the field trip sponsored by petitioners for its employees not affiliated with THSGQ Union, although a positive enticement, was undoubtedly extraneous influence designed to
impede respondents in their quest to be certified. This cannot be countenanced.

ATHENA M. SALAS | LABOR CASE DIGEST 2015 376

Not content with achieving a "no union" vote in the certification election, petitioners
launched a vindictive campaign against union members by assigning work on a rotational basis
while subcontractors performed the latter's functions regularly. Worse, some of the respondents
were made to work as grass cutters in an effort to dissuade them from further collective action.
Again, this cannot be countenanced.
More importantly, petitioners' bare denial of some of the complained acts and
unacceptable explanations, a mere afterthought at best, cannot prevail over respondents'
detailed narration of the events that transpired. At this juncture, it bears to emphasize that in
labor cases, the quantum of proof necessary is substantial evidence, or that amount of relevant
evidence as a reasonable mind might accept as adequate to support a conclusion, even if other
minds, equally reasonable, might conceivably opine otherwise.
In fine, mindful of the nature of the charge of ULP, including its civil and/or criminal
consequences, the Court finds that the NLRC, as correctly sustained by the CA, had sufficient
factual and legal bases to support its finding of ULP.

ATHENA M. SALAS | LABOR CASE DIGEST 2015 377

You might also like